A Posteriori Justification for Necessary Truths
필연적 참에 대한 후험적 정당성

1) The second reason D3 fails is that it implies the following: Whenever you believe a necessary truth, your justification for it is a priori. However, isn't it possible to be justified a posteriori in believing a necessary truth? If a famous and eminent logician were to tell you that a certain theorem is true, wouldn't you be justified in believing it to be true? It's plausible to say that you would be. But if you are justified in believing a proposition on the basis of authority, then your justification for believing that proposition is a posteriori. So if trusting an authority can justify you in accepting a necessary proposition, then it's possible to be justified a posteriori in believing necessary truths. D3 does not allow for this possibility, and thus must be rejected.<5>
1) D3가 실패하는 두 번째 이유는 D3가 다음을 시사한다는 점이다. 즉 당신이 필연적인 참을 믿는 어느 때에든, 당신의 그 믿음에 대한 정당성은 선험적이라는 것이다. 그렇지만, 필연적 참을 믿음에 있어서 후험적으로 정당화되는 것은 가능하지 않은가? 만일 유명하고 저명한 논리학자가 당신에게 어떤 정리가 참이라고 말하게 된다면, 당신은 그 정리가 참임을 믿음에 있어서 정당화되지 않겠는가? 당신이 정당화되리라 말하는 것은 그럴 성싶다. 그러나 만일 당신이 권위에 기초하여 한 명제를 믿음에 있어서 정당화된다면, 당신의 그 명제를 믿음에 대한 정당성은 후험적이다. 그래서 만일 권위에 대한 신뢰가 당신을 한 필연적 명제를 수용함에 있어서 정당화시킬 수 있다면, 필연적 참들을 믿음에 있어서 후험적으로 정당화되는 것은 가능하다. D3는 이러한 가능성을 허용하지 않고, 그래서 반드시 거부되어야만 한다.<5>

2) Against this argument, the following objection could be raised. When we consider a particular proposition, we must distinguish between the sentence expressing that proposition and the proposition it expresses. Let's call a sentence that expresses a logical theorem a "formula." When you learn from an authority on logic that a certain formula is true, then what you are justified in believing - so the objection goes - is that the formula expresses a truth. You are not, however, justified in believing that truth itself. According to the objection, you are not justified in believing the truth itself because you don't really understand it.
2) 이러한 논증에 반대하여, 다음 반박이 제기될 수 있다. 우리가 특수한 명제를 고찰할 때, 우리는 반드시 명제를 표현하는 문장과 그 문장이 표현하는 명제를 구별해야만 한다. 논리적 정리를 표현하는 문장을 "식"이라 부르도록 하자. 당신이 한 논리학의 권위자로부터 특정한 식이 참임을 배울 때, 당신이 믿음에 있어서 정당화되는 것은 - 그 반박이 그렇게 이야기하듯 - 그 식이 참을 표현한다는 것이다. 그렇지만 당신은 그 참 자체에 대한 믿음에 있어서 정당화되지는 않는다. 그 반박에 따르면, 당신은 그 참 자체를 믿음에 있어서는 정당화되지 않는데 왜냐하면 당신이 그 참 자체를 실제로 이해하지는 못하기 때문이다.

3) Now, it must certainly be admitted that there are logical formulate whose truth master logicians can see, whereas ordinary mortals can't even begin to understand them. And if we were to take a master logician's word that such a formula is true, then indeed we would be justified, not in believing the truth that formula expresses, but only in believing that a certain formula expresses a truth. However, it is implausible that this is so whenever we believe a formula to be true on the basis of authority. The objection does not, therefore, rule out the possibility of a posteriori justification for believing a necessary truth.
3) 이제, 그 참을 일류 논리학자가 알 수 있는 반면, 일상의 필부들은 그것들을 이해하려고 시작조차 할 수 없는 논리적 식이 있다는 것이 확실히 인정되어야만 한다. 그리고 만일 우리가 그러한 식이 참이라는 일류 논리학자의 말을 받아들이게 된다면, 더 나아가 우리는 정당화될 것인데, 그 식이 표현하는 참을 믿음에 있어서가 아니라, 오로지 한 특정한 식이 참을 표현한다는 것에 대한 믿음에 있어서만 그러할 것이다. 그렇지만, 이것이 우리가 한 식이 참이라는 것을 권위에 기초하여 믿는 어느 때이든 그렇다는 것은 그럴 성싶지 않다. 그러므로 그 반박은 필연적 참을 믿는 일에 대한 후험적 정당성의 가능성을 배제시키지 않는다.

4) Consider, for example, the Goldbach conjecture: Every even number greater than two is the sum of two prime numbers.<6> If the Goldbach conjecture is true, it is necessarily true, and if it is false, it is necessarily false.<7> As a matter of fact, however, we don't know whether the Goldbach conjecture is true or false; no one has proven or disproven it. Suppose a famous mathematical genius proves that the Goldbach conjecture is true, his proof is well publicized, and the community of mathemtical experts agrees the proof is sound. The question we must answer is whether under these circumstances we would be justified in believing the Goldbach conjecture itself - or only in believing that

(1) The sentence "Every even number greater than two is the sum of two primes" expresses a truth.

It could reasonably be argued that someone who knows what an even number, a prime number, and a sum of two numbers are, understands perfectly well what the Goldbach conjecture means. If so, then we would (in the case we are imagining) be justified in believing not only (1) but the Goldbach conjecture itself. Thus, if we were to learn from an authority that the Goldbach conjecture is true, then we would be justified a posteriori in believing a necessary truth. D3 does not allow for this possibility, and thus must be rejected.
4) 예를 들어 골드바흐 추측을 고찰해 보도록 하자. 그것은 모든 2보다 큰 짝수는 두 소수들의 합이라는 것이다.<6> 만일 골드바흐 추측이 참이라면, 그것은 필연적으로 참이고, 만일 거짓이라면 필연적으로 거짓이다.<7> 그렇지만 사실상 우리는 골드바흐 추측이 참인지 거짓인지 알지 못한다. 왜냐하면 누구도 그것을 증명하거나 반증하지 않았기 때문이다. 유명한 수학 천재가 골드바흐의 추측이 참이라고 증명했고, 그의 증명은 잘 알려졌으며, 수학 전문가 집단은 그 증명이 건전하다고 동의하였다고 가정해 보자. 우리가 대답해야만 하는 물음은 이러한 상황들 아래에서 우리가 골드바흐 추측 자체를 믿음에 있어서 정당화될는지 - 혹은 오로지 다음에 대해서 믿음에 있어서만 정당화될는지 말이다.

(1) "2보다 큰 모든 짝수는 두 소수의 합이다"라는 문장이 참을 표현한다.

짝수, 소수, 그리고 두 수들의 합이 무엇인지 아는 누군가는 골드바흐 추측이 의미하는 바를 완전히 잘 이해한다는 것은 합리적으로 주장될 수 있다. 만일 그렇다면, 우리는 (우리가 상상하는 사례에 있어서) 오로지 (1)을 믿음에 있어서만 정당화될 것이고 그러나 골드바흐 추측 자체에 대해서는 그렇지 않을 것이다. 그래서, 만일 우리가 한 권위자로부터 골드바흐 추측이 참이라고 배우게 된다면, 우리는 필연적 참을 믿음에 있어서 후험적으로 정당화될 것이다. D3는 이러한 가능성을 허용하지 않으며, 그래서 반드시 부정되어야만 한다.

5) D3, then, fails for two reasons: first, it is too liberal regarding the manner in which p is believed, and second, it doesn't allow for the possibility of being justified a posteriori in believing a necessary truth. The following definition suffers from neither of these problems:

D4    S is justified a priori in believing that p if and only if S grasps that p is necessarily true.

D4 requires that S grasp that p is necesarily true, and thus places a constraint on the manner in which p is believed. Furthermore, it permits a posteriori justification for believing necessary truths, for if one believes a necessary truth on the basis of authority, then one believes that truth without grasping that it is necessarily true. If you were to believe the Goldbach conjeture because an authority told you that it was a necessary truth, then you would believe a necessary truth without grasping its necessity. Hence, according to D4, your justification for believing the Goldbach conjecture would be a posteriori.
5) 그래서 D3는 두 가지 이유들 때문에 실패한다. 첫째로, 그것은 p가 믿어지는 방식을 지나치게 엄밀하지 못하게 간주하고 있고, 둘째, 그것은 필연적 참을 믿음에 있어서 후험적으로 정당화됨의 가능성에 대해 허용하지 않는다. 다음 정의는 이러한 문제들 중 어느 것으로부터도 곤란을 겪지 않는다.

D4    S는 p를 믿음에 있어서 선험적으로 정당화된다 iff S는 p가 필연적으로 참임을 파악한다.

D4는 S가 p가 필연적으로 참임을 파악할 것을 요구하며, 그래서 p가 믿어지는 방식에 하나의 제한을 둔다. 더 나아가서, 그것은 필연적 참들을 믿음에 대한 후험적 정당성을 허용하는데, 왜냐하면 만일 권위에 기초하여 필연적 참을 믿는다면, 그 참이 필연적으로 참임을 파악함이 없이 믿는 것이기 때문이다. 만일 당신이 골드바흐 추측을 한 권위자가 당신에게 그 추측이 필연적 참이라고 말해주었기 때문에 믿게 된다면, 당신은 필연적 참의 필연성을 이해함이 없이 필연적 참을 믿을 것이다. 따라서, D4에 따르자면, 골드바흐 추측을 믿음에 대한 당신의 정당성은 후험적일 것이다.


The Truth Value and Modal Status of Propositions
명제들의 진리치와 양상

1) Unfortunately, the proposal to define a priori justification in terms of the grasp of necessity raises another kind of problem. Consider again the proposition

(1) Whatever is red is colored.

Probably you have formed the belief that (1) is necessarily true. However, we must distinguish between two different beliefs:

B1    (1) is true.
B2    (1) is necessarily true.

Since B1 is a belief about the truth value of (1) and B2 a belief about the modal status of (1), B1 and B2 are two very different beliefs.<8> It is possible for a person to believe a proposition to be true without giving any thought to the question of what the proposition's modal status is. This means it's possible to believe that p is true without simultaneously believing that it is necessarily true.
1) 불행히도, 필연성의 파악으로 선험적 정당성을 정의하자는 제안은 또 다른 종류의 문제를 일으킨다. 다시 다음 명제를 고찰해 보자.

(1) 붉은 것은 무엇이든 색이 있다.

아마도 당신은 (1)이 필연적 참이라는 믿음을 형성하였을 것이다. 그렇지만, 우리는 다음 두 가지 상이한 믿음들을 구분해야만 한다.

B1    (1) 은 참이다.
B2    (1) 은 필연적으로 참이다.

B1은 (1)의 진리치에 대한 믿음이고 B2는 (1)의 양상에 대한 믿음이므로, B1과 B2는 두 가지 매우 상이한 믿음들이다.<8> 한 사람이 그 명제의 양상이 무엇인지에 대한 물음에 대해 어떤 생각도 함이 없이 그 명제가 참이라고 믿는 것은 가능하다. 이것은 p가 참이라는 것을 동시에 p가 필연적으로 참이라고 믿지 않으면서도 믿는 것이 가능하다는 뜻이다.

2) Suppose you believe that (1) is true - say, because you experience a feeling of "intellectual compulsion" - without, however, having considered the question of whether (1) is necessarily true. You have grasped that (1) is true, but you have not grasped that it is necessarily true because you didn't even ask yourself whether (1) is necessarily true. D4 implies that, in this case, you would not be justified a priori in believing (1). Hence, if it is true that you would be, then D4 is false.
2) 당신이 (1)은 참이라고 믿는다고 가정해 보자 - 말하자면, 당신은 "지적 강요"의 감정을 경험하였기 때문에 - 그렇지만, (1)이 필연적으로 참인지에 대한 물음은 고려하지 않고서 말이다. 당신은 (1)이 참임을 파악했지만, (1)이 필연적으로 참이라는 것은 파악하지 않았는데 왜냐하면 당신이 (1)이 필연적으로 참인지 당신 자신에게조차 묻지 않았기 때문이다. D4는 이 사례에서 당신이 (1)을 믿음에 있어서 선험적으로 정당화되지 않을 것임을 시사한다. 따라서, 만일 당신이 정당화되지 않을 것임이 사실이라면, D4는 거짓이다.

3) The point of this objection is this: Grasping that p is necessarily true involves, minimally, believing that p is necessarily true. However, it seems possible to be justified a priori in believing a proposition without believing that that proposition is necessarily true. Consequently, if we define a priori justification in terms of grasping a proposition's necessity, our definition would appear to be too narrow.
3) 이 반박의 요지는 이것이다. 즉 p가 필연적으로 참이라는 파악은 최소한 p가 필연적으로 참이라는 믿음을 포함한다. 그렇지만, 한 명제를 믿음에 있어서 그 명제가 필연적으로 참이라는 믿음 없이 선험적으로 정당화되는 것은 가능해 보인다. 결론적으로, 만일 우리가 선험적 정당성을 한 명제의 필연성을 파악함으로 정의한다면, 우리의 정의는 지나치게 협소한 것으로 드러날 것이다.


The Fallibility of A Priori Justification
선험적 정당성의 오류가능성

1) A second problem afflicting D4 arises from the possibility of having a priori justification for believing a proposition to be necessarily true although that proposition is in fact false. If there is such a possibility - that is, if a priori justification is fallible - then D4 is false, for according to D4, a priori justification for p involves grasping that p is necessarily true. Can you grasp that a proposition p is necessarily true when p is in fact false? You cannot, for if p is false, then you mistakenly believe, but do not grasp, that p is necessarily true. So, grasping that p is necessarily true entails that p is true. Thus D4 tells us that one can't be justified a priori in believing a false proposition.
1) D4를 괴롭히는 두 번째 문제는 한 명제가 필연적으로 참이라는 믿음에 대한 선험적 정당성을 비록 그 명제가 사실은 거짓이라 할지라도 가질 가능성으로부터 발생한다. 만일 그러한 가능성이 있다면 - 즉, 만일 선험적 정당성이 오류가능적이라면 - D4는 거짓인데, 왜냐하면 D4에 따르자면, p에 대한 선험적 정당성은 p가 필연적으로 참이라는 파악을 포함하기 때문이다. 당신은 한 명제 p가 실제로는 거짓일 때 필연적으로 참이라고 파악할 수 있는가? 당신은 그럴 수 없는데, 왜냐하면 만일 p가 거짓이라면, 당신은 p가 필연적으로 참이라 잘못 믿는 것이지 그것을 파악하는 것은 아니기 때문이다. 그래서, p가 필연적으로 참이라는 파악은 p가 참임을 논리적으로 함축한다. 그래서 D4는 우리에게 거짓 명제를 믿음에 있어서는 선험적으로 정당화될 수 없다고 말해준다.

2) Is it possible, however, to be justified a priori in believing a falsehood to be true? Here is an argument for answering this question with "yes": Ask yourself whether you believe that when you take away from a heap of sand a single grain, what's left is still a heap of sand. If you believe that, then, on reflection, you should believe as well the following assumption:

(A) If two collections of grains of sand differ in number by just one grain, then either both collections are heaps of sand or neitehr is.

Is your justification for (A) a priori? It's plausible to say it is. The issue of whether (A) is ture or false is not one that calls for an empirical investigation of heaps of sand. Rather, it is a conceptual issue, raising problems having to do with "heapness." Unfortunately, (A) leads to the paradoxical result that there are heaps of sand consisting of one grain only.<9> Once you become aware of the paradox, it is questionable whether you are any longer justified in believing (A).<10> However, it could be argued that before you become aware of the paradox, your justification for believing (A) was untarnished. And if this is right, then prior to discovering the paradox you were justified a priori in believing (A) - a proposition that is arguably false.<11>
2) 그렇지만, 하나의 거짓을 참이라고 믿음에 있어서 선험적으로 정당화되는 것이 가능한가? 여기 이 물음에 "그렇다"라고 대답하는 하나의 논증이 있다. 당신 자신에게 당신이 모래 더미에서 모래 한 알을 깍아 낼 때, 남겨진 것이 여전히 모래더미라고 믿는지 물어 보자. 만일 당신이 그렇게 믿는다면, 그래서, 숙고해 보자면, 당신은 다음 추정도 마찬가지로 믿어야 할 것이다.

(A) 만일 모래 더미들 두 더미들이 수에 있어서 단지 모래 한 알만큼 다르다면, 그 두 더미들은 모두 모래 더미이거나 둘 다 모래 더미가 아니다.

(A)에 대한 당신의 정당성은 선험적인가? 그렇다고 말하는 것이 그럴 성싶다. (A)가 참인지 거짓인지의 문제는 모래 더미들에 대한 경험적 조사를 필요로 하는 것은 아니다. 오히려, 그것은 개념적 문제, "더미임"과 관련된 문제들을 불러 일으키는 그러한 개념적 문제이다. 불행히도, (A)는 단 한 알의 모래만으로 이루어진 모래 더미들이 있다는 역설적 결론으로 이끈다.<9> 당신이 그 역설을 알아차리자 마자, 당신이 (A)를 믿음에 있어서 더 이상 정당화되는지 의문스러워진다.<10> 그렇지만, 당신이 그 역설을 알아차리기 전에, 당신의 (A)를 믿음에 대한 정당성이 불분명하지 않았다는 것은 주장될 수 있다. 그리고 만일 이것이 옳다면, 그 역설을 발견하기에 앞서 당신은 (A)를 믿음에 있어서 선험적으로 정당화되었다 - 거의 틀림없이 거짓인 명제에 대해서 말이다.<11>

3) Thus far, we have examined two approaches to the analysis of a priori justification: according to the first, a priori justification can be analyzed in terms of necessity; according to the second, it can be analyzed in terms of grasping necessity. We have seen that there are obstacles to both approaches. We should not, however, completely dismiss the possibility that, with appropriate refinements, one of these approaches might succeed. In the next section, we shall examine yet another approach.
3) 여기까지, 우리는 선험적 정당성을 분석하는 두 가지 접근방식들을 검토하였다. 첫 번째 접근에 따르자면, 선험적 정당성은 필연성이라는 말로 분석될 수 있다. 그리고 두 번째 접근에 따르자면, 그것은 필연성의 파악이라는 말로 분석될 수 있다. 우리는 두 접근 방식들 모두에 장애물들이 있음을 보았다. 그렇지만 우리는 적절한 개선들을 가지고 이러한 접근법들 중 하나가 성공할 수도 있다는 가능성을 완전히 부정하지는 않아야 할 것이다. 다음 절에서, 그럼에도 우리는 또 다른 접근법을 검토할 것이다.

4) Before moving on, however, let's briefly summarize what has emerged from our discussion. An adequate definition of a priori justification for a proposition p should: (i) place constraints on the manner in which p is believed; (ii) permit a posteriori justification for necessary truths; (iii) allow for the possibility of being justified a priori in believing a proposition to be true without believing it to be necessarily true; (iv) make a priori justification fallible.
4) 그렇지만 다음으로 넘어가기에 앞서, 우리의 논의에서 드러났던 것을 간략히 정리해 보도록 하자. p라는 명제에 대한 선험적 정당성의 적절한 정의는 (i) p가 믿어지는 방식에 제한들을 두어야 할 것이고, (ii) 필연적 참들에 대한 후험적 정당성을 허용해야 할 것이며, (iii) 한 명제를 참이라고 믿음에 있어서 그 명제가 필연적으로 참이라는 믿음 없이 선험적으로 정당화될 가능성을 허용해야 할 것이고, (iv) 선험적 정당성을 오류가능적으로 만들어야 할 것이다.


A Third Way of Defining Apriority
선험성을 정의하는 세 번째 방법

1) Some philosophers attempt to define apriority by saying the following: The reason justification for a priori propositions does not require experience is that in order to recognize an a priori proposition's truth, understanding it is all that is necessary. For example, the proposition "Whatever is red is colored" is such that, once you understand it, you are justified in believing it. Can this idea be turned into a satisfactory definition of a priori justification? Consider the following try:

D5    S is justified a priori in believing that p if and only if, necessarily, if S understands p, then S is justified in believing that p is true.

The problem with D5 is that it is too narrow. Suppose you are justified a priori in believing a proposition p on the basis of a simple proof.<12> Now, if in order to see that p is true you need to prove p, then we can't say that, necessarily, if you understand p, then you are justified in believing that p. For prior to proving p, you understand it without being justified in believing it. So, D5 implies that if your proof for p is a priori itself - if experience is necessary neither for knowing its premises nor for knowing that the premises imply the conclusion - then, so the objection to D5 goes, you are justified a priori in believing that p.
1) 일부 철학자들은 다음과 같이 말함으로써 선험성을 정의하고자 시도한다. 선험적 명제들에 대한 정당성이 경험을 요청하지 않는 까닭은 선험적 명제가 그만큼 필연적이라는 것을 이해하면서 진리를 인식하기 위해서이다. 예를 들어 "붉은 것은 무엇이든 색이 있다"라는 명제는 당신이 그 명제를 이해하기만 하면 곧 당신이 그 명제를 믿음에 있어서 정당화되는 그러한 것이다. 이러한 생각이 선험적 정당성에 대한 만족스러운 정의가 될 수 있는가? 다음 시도를 고찰해 보자:

D5    S는 p를 믿음에 있어서 선험적으로 정당화된다 iff 필연적으로 만일 S가 p를 이해한다면 S는 p가 참임을 믿음에 있어서 정당화된다.

D5가 가진 문제는 지나치게 협소하다는 것이다. 당신이 하나의 단순한 증명에 기초하여 명제 p를 믿음에 있어서 정당화된다고 생각해 보자.<12> 이제, 만일 p가 참임을 알기 위해서 당신이 p를 증명할 필요가 있다면, 우리는 필연적으로 당신이 p를 이해한다면 p를 믿음에 있어서 정당화된다고 말할 수 없을 것이다. 왜냐하면 p를 증명하기에 앞서서 당신은 p를 믿음에 있어서 정당화됨이 없이 그 p를 이해하기 때문이다. 그래서, D5는 만일 P에 대한 당신의 증명이 그 자체로 선험적이라면 - 만일 경험이 그 증명의 전제들을 알기 위해서도 그 전제들이 시사하는 결론을 알기 위해서도 필연적이지 않다면 - D5에 대한 반박이 그렇게 진행하듯 당신은 p를 믿음에 있어서 선험적으로 정당화된다.

2) We must distinguish, then, between a priori justification in a narrow and a broad sense. In the narrow sense, a priori propositions are restricted to propositions that are such that understanding them is sufficient for being justified in believing them. Let us call propositions that are a priori in the first sense "axioms." An axiom is a necessary truth that is incapable of proof in the sense that there is no other proposition that is "better known" than it is.<13> Recognition of an axiom's truth is derived not from other propositions, but simply from understanding it.
2) 그래서 우리는 선험적 정당성을 협의와 광의로 구분해야만 한다. 협의에서 선험적 명제들은 그 명제들을 이해함이 그 명제들을 믿음에 있어서 정당화됨의 충분조건인 그러한 명제들이다. 첫 번째 의미에서 선험적인 명제들을 우리는 "공리들"이라 부르도록 하자. 공리는 그보다 "더 잘 알려진" 어떠한 다른 명제도 없다는 의미에서 증명 불가능한 필연적 참이다.<13> 한 공리가 참이라는 인지는 다른 명제들로부터 유도되지 않고, 단순히 그 공리를 이해함으로부터 유도된다.

3) In the broad sense of apriority, a proposition that is known a priori is either an axiom or known on the basis of a proof. Though D5 fails as a definition of a priori justification in this broad sense, it succeeds as a definition of an axiom. We can, therefore, use the definiens (i.e., the defining condition after the "if and only if") of D5 to define the concept of an axiom.

D6    p is an axiom for S if and only if, necessarily, if S understands p, then S is justified in believing that p is true.

The proposition "Whatever is red is colored" is, according to D6, an axiom. It is not possible to understand it without being justified in believing it. In contrast, D6 implies, as it should, that the Goldbach conjecture is not an axiom, for understanding this conjecture is not sufficient for being justified in believing it to be true. D6 implies the same about the sand-heap assumption:

(A) If two collections of grains of sand differ in number by just one grain, then either both collections are heaps of sand or neither is.

Although it is possible to be justified in believing (A), it is false that, necessarily, if one understands (A), one is justified in believing it. For as soon as one notices the paradox it involves, one's initial justification for it is undermined.
3) 광의의 선험성에 있어서, 선험적이라 알려진 한 명제는 공리이거나 증명에 기초해 알려진 것이다. D5가 이러한 광의에서 선험적 정당성에 대한 정의로서 실패한다 할지라도 공리에 대한 정의로서는 성공한다. 그러므로 우리는 D5의 정의항(즉, "if and only if" 이후의 한정 조건)을 공리 개념을 정의하기 위해 사용할 수 있다.

D6    p는 S에 대해 공리이다 iff 필연적으로 만일 S가 p를 이해한다면 S는 p가 참이라고 믿음에 있어서 정당화된다.

"붉은 것은 무엇이든 색이 있다"라는 명제는, D6에 따르자면, 공리이다. 그 명제를 믿음에 있어서 정당화됨이 없이 그 명제를 이해하기란 가능하지 않다. 반대로, D6는 기대되는 것처럼 골드바흐 추측이 공리가 아님을 시사하는데, 왜냐하면 이 추측을 이해한다는 것이 그 추측이 참이라고 믿음에 있어서 정당화되기에 충분하지 않기 때문이다. D6은 모래 더미 가정에 대해서도 똑같은 바를 시사한다:

(A) 만일 모래 알갱이들로 된 두 더미들이 수에 있어서 단지 모래 한 알로 차이가 난다면, 두 더미 모두 모래 더미이거나 둘 다 모래 더미가 아니다.

(A)를 믿음에 있어서 정당화됨이 가능하다 할지라도, 필연적으로 만일 (A)를 이해한다면 (A)를 믿음에 있어서 정당화된다는 것은 거짓이다. (A)가 포함하는 그 역설에 주목하자 마자 (A)에 대한 최초의 정당성은 약화되기 때문이다.

4) D6, however, defines merely axiomatic a priori justification. Thus we still need a definition of a priori justification in the broad sense - that is, of the kind of justification one can have for nonaxiomatic a priori propositions. The general idea for such a definition is expressed by D7.

D7    S is justified a priori in believing that p if and only if either p is an axiom for S or S believes that p on the basis of a proof that is axiomatic for S.<14>

For D7 to be satisfactory, we would have to give an account of what it means, first, to believe a proposition on the basis of a proof and, second, for a proof to be axiomatic. However, even assuming that these phrases could be defined in turn, it is still far from clear that D7 is satisfactory. The problem is that there are many examples of propositions that are not a priori according to D7, but could reasonably be claimed to be a priori. For example, many epistemologists would agree that the proposition

(1) If there is an object that looks red to S, then S is prima facie justified in believing that that object is red

is a necessary truth that is knowable a priori. However, (1) is not axiomatic, for skeptics understand perfectly well what (1) means, but they would say that (1) is false. Perhaps, then, our justification for either accepting or rejecting (1) is a function of our reasons either for or against (1). But whatever the form of the reasoning in favor or against (1), philosophers hardly ever claim that it takes the form of an axiomatic proof. In fact, typically the arguments for or against principles such as (1) are long and complicated. It follows that, according to D7, philosophers who believe (1) are not justified a priori in believing it, and that philosophers who reject (1) are not justified a priori in rejecting it. Now, this wouldn't be a problem for D7 if we could simply conclude that when philosophers are justified in accepting, or rejecting, (1) their justification is a posteriori. Many philosophers, however, would refuse to draw that conclusion. They would say that if one is justified in accepting (or rejecting) a proposition such as (1), then one's justification is a priori.<15>
4) 그렇지만 D6은 단지 공리적으로 선험적인 정당성만을 정의한다. 그래서 우리는 여전히 광의에서의 선험적 정당성에 대한 정의가 필요하다. - 즉, 비공리적 선험적 명제들에 대해 가질 수 있는 그러한 종류의 정당성에 대한 정의가 필요하다. 그러한 정의에 대한 일반적인 발상은 D7에 의해 표현된다.

D7    S는 p를 믿음에 있어서 선험적으로 정당화된다 iff p가 S에 대해 공리이다, 또는 S가 p를 S에 대해 공리적인 증명에 기초하여 믿는다.<14>

D7이 만족스러운 것이 되기 위해서, 우리는 우선, 증명에 기초하여 명제를 믿는다는 것, 둘째로, 증명이 공리적이라는 것이 의미하는 바에 대해 설명을 제시해야 할 것이다. 그렇지만, 이러한 구절들이 정의될 수 있다고 가정해도, D7이 만족스럽다는 것은 여전히 전혀 분명치 못하다. 문제는 D7에 따르자면 선험적이지 않지만 선험적인 것이라고 합리적으로 주장될 수 있을 그러한 명제들의 많은 예시들이 있다는 점이다. 예를 들어, 많은 인식론자들은 다음 명제,

(1) 만일 S에게 붉게 보이는 대상이 있다면, S는 그 대상이 붉다고 믿음에 있어서 직견적으로 정당화된다

이러한 명제가 선험적으로 알 수 있는 필연적 참이라는 데에 동의할 것이다. 그렇지만, (1)은 공리적이지 않은데, 왜냐하면 회의주의는 (1)이 의미하는 바를 완전히 잘 이해하지만 (1)이 거짓이라고 말할 것이기 때문이다. 그래서 아마도 (1)을 수용하거나 거부함에 대한 우리의 정당성은 (1)에 대한 추론 혹은 (1)에 반대한 추론의 기능일 것이다. 그러나 (1)에 대해 지지함에 있어서나 반대함에 있어서 추론의 형식이 무엇이든, 철학자들은 그 추론 형식이 공리적 증명의 형식을 취한다고는 주장하기 무척 어려울 것이다. 사실상, 전형적으로 (1)과 같은 그러한 원칙들에 대해서나 혹은 반대하는 논증들은 길고 복잡하다. D7에 따르면 (1)을 믿는 철학자들은 그 명제를 믿음에 있어서 선험적으로 정당화되지 않고, (1)을 거부하는 철학자들은 그것을 거부함에 있어서 선험적으로 정당화되지 않는다는 결과가 된다. 이제, 우리가 만일 단순히 철학자들이 (1)을 받아들이거나 거부함에 있어서 정당화될 때 그들의 정당성이 후험적이라고 결론내려 버릴 수 있다면 D7에 대해 이것은 문제가 되지 않을 것이다. 그렇지만 여러 철학자들은 그러한 결론을 이끌어내는 것을 거절할 것이다. 그들은 만일 한 사람이 (1)과 같은 그런 하나의 명제를 받아들임(또는 거부함)에 있어서 정당화된다면 그 사람의 정당성은 선험적이라고 말할 것이다.<15>

5) It could be objected, then, that D7 is too narrow. However, not every philosopher would agree with the objection's premises. Also, it might be possible to modify D7 in such a way that it captures those examples of apriority that it arguably leaves out. In any case, it seems safe to say that D7 provides us with a sufficient condition of apriority: if a proposition is either an axiom or such that it can be established on the basis of an axiomatic proof, then that proposition is knowable a priori.
5) 그래서 그것은 D7이 지나치게 좁다고 반박될 수 있을 것이다. 그렇지만, 모든 철학자가 그 반박의 전제들에 동의하지는 않을 것이다. 또한, D7이 거의 틀림없이 배제시키는 그러한 선험성의 사례들을 포착하는 그런 방식으로 수정하는 것이 가능할 수 있다. 어떤 경우든, D7이 우리에게 선험성의 충분조건을 제공한다고 말하는 편이 안정해 보인다. 즉 만일 한 명제가 공리이거나 공리적 증명에 기초해 확립될 수 있는 그러한 것이라면, 그 명제는 선험적으로 알 수 있다는 것이다.

-蟲-
Chapter Three

A Priori Knowledge
선험적인 앎

Kant's definition of Apriority
선험성에 대한 칸트의 정의

1) In this chapter, we shall concern ourselves with the following topics: (i) the concept of apriority, (ii) the distinction between analytic and synthetic propositions, and (iii) skepticism about a priori knowledge.
1) 이 장에서, 우리는 다음 주제들에 관심을 둘 것이다: (i) 선험성의 개념, (ii) 분석적 명제와 종합적 명제 사이의 구별, 그리고 (iii) 선험적인 앎에 대한 회의주의.

2) The terms a priori and a posteriori were introduced into philosophy by the German philosopher Immanuel Kant, marking a distinction that had already been drawn by Descrates, Leibniz, and Locke. In his Critique of Pure Reason[de. Kritik der reinen Vernunft], Kant defined a priori knowledge in terms of independence of experience.<1> According to Kant, a priori knowledge is knowledge that is prior to, or gained independently of, experience, whereas a posteriori knowledge is knowledge gained through experience. The point of his definition may be illustrated by the following two propositions:

(1)    If Yuri lives in Alma-Ata and Alma-Ata is the capital of Kazakhstan, then Yuri lives in the capital of Kazakhstan.
(2)    Yuri lives in Alma-Ata.

In order to see that (1) is true, you need only think about what it is that (1) asserts. Thought alone, without the aid of any information drawn from experience, is sufficient to determine whether (1) is true or false. In contrast, thought alone is not sufficient to determine whether (2) is true or false. Rather, in order to find out the truth value of (2), you must have the kind of experience needed to acquire the relevant information. For example, Yuri might tell you he lives in Alma-Ata, or you might hire a private investigator to find out where Yuri lives, or you might watch a report about Yuri on 60 Minutes. Obviously, no such experiences are necessary in order to see that (1) is true. Thus Kant would say that while you know a priori that (1) is true, knowledge of (2) would have to be a posteriori.
2) a priori(선험)와 a posteriori(후험)라는 용어들은 데카르트, 라이프니츠, 로크에 의해 이미 도입된 용어들과 구별을 짓기 위해 독일 철학자 임마누엘 칸트에 의해 철학에 소개되었다. 그의 책 『순수이성비판』에서, 칸트는 선험적 앎을 경험 독립성이란 말로 정의하였다.<1> 칸트에 따르면, 선험적 앎은 경험에 앞서는, 또는 경험으로부터 독립적으로 획득되는 앎인 반면, 후험적 앎은 경험을 통해 획득되는 앎이다. 그의 정의에 있어서 핵심은 아마도 다음 두 가지 명제들에 의해 설명될 것이다:

(1)    만일 유리가 알마-아타에 살고 알마-아타가 카타키스탄의 수토라면, 유리는 카자키스탄의 수도에 산다.
(2)    유리는 알마-아타에 산다.

(1)이 참이라는 것을 알기 위해, 당신은 단지 (1)이 주장하는 것이 무엇인지에 대해서만 생각하면 된다. 단지 사유만으로, 경험으로부터 도출된 어떠한 정보의 도움 없이도, (1)이 참인지 거짓인지 결정하기에는 충분하다. 반대로, 오로지 사유만으로 (2)가 참인지 거짓인지 결정하기에는 불충분하다. 오히려, (2)의 진리치를 알아내기 위해서, 당신은 반드시 관련 정보를 얻기 위해 필요한 그런 종류의 경험을 지녀야만 한다. 예를 들어, 유리가 당신에게 그 자신은 알마-아타에 산다 말할 수도 있고, 또는 당신이 유리가 어디에 사는지 찾아내도록 사설 탐정을 고용할 수도 있으며, 아니면 당신은 '추적 60분'에서 유리에 대한 보고를 시청할 수도 있을 것이다. 분명히, (1)이 참임을 알기 위해 그런 어떠한 경험들도 필요하지 않다. 그래서 칸트는 (1)이 참임을 당신은 선험적으로 아는 반면, (2)에 대한 앎은 후험적이어야할 것이라고 말할 것이다.

3) The notion of a priori knowledge is closely related to that of a priori justification because what makes an instance of knowledge a priori is the way in which it is justified. Justification through experience is one mode of justification; justification that is independent of experience is another. A priori knowledge, then, can be defined as the kind of knowledge whose justification is a priori. Of course, this definition is not very illuminating, for it doesn't tell us what makes justification a priori. If we were to apply Kant's suggestion, we could say that a belief is justified a priori if and only if its justification is independent of experience. This definition raises two issues. First, precisely what do we mean by the concept of experience? Second, what do we mean by the concept of independence of experience?
3) 선험적 앎의 관념은 선험적 정당성의 관념과 밀접하게 관련되는데 왜냐하면 앎의 일례를 선험적으로 만드는 것은 그 예가 정당화되는 방법이기 때문이다. 경험을 통한 정당화는 정당화의 한 형태이다; 경험 독립적 정당화는 또 다른 형태이다. 선험적 앎은, 그래서, 그 앎의 정당성이 선험적인 그러한 종류의 앎으로 정의될 수 있다. 물론, 이러한 정의는 그렇게 분명하진 않은데, 왜냐하면 그 정의가 우리에게 무엇이 정당화를 선험적으로 만드는지 말해주지 못하기 때문이다. 만일 우리가 칸트의 제안을 적용할 것이라면, 우리는 하나의 믿음이 정당화된다 iff 그 믿음의 정당성이 경험독립적이다 라고 말할 수 있을 것이다. 이 정의는 두 가지 문제들을 일으킨다. 우선, 우리가 경험의 개념을 가지고 뜻하는 것이 정확하게 무엇인가? 다음으로, 경험 독립성의 개념으로 우리는 무엇을 의미하는가?


Sensory and Nonsensory Experience
감각적 경험과 비감각적 경험

1) Suppose by "experience" we mean sense experience: experience that is either visual, auditory, olfactory, tactile, or gustatory. In that case, we would have to formulate Kant's definition thus:

D1    S is justified a priori in believing that p if and only if S's justification for believing that p does not depend on sense experience.

But this definition is not satisfactory, for there are beliefs such that their justification does not depend on sense experience, but they are not justified a priori. Suppose I believe "This morning I was thinking about having steak for dinner." This belief is not a priori, for its justification depends on the memorial experience of clearly recalling what dinner plans I was making this morning. But a memorial experience is not a form of sense experience, and thus my justification for believing that I was thinking about having steak for dinner does not depend on sense experience. Hence D1 yields the wrong result that I am justified a priori in believing that I was thinking about having steak for dinner.
1) "경험"으로 우리가 감각 경험을 의미한다고 가정해 보자: 시각, 청각, 후각, 촉각, 아니면 미각 중 하나일 그러한 경험 말이다. 그 경우, 우리는 칸트의 정의를 다음과 같이 정식화해야 할 것이다.

D1    S는 p를 믿음에 있어서 선험적으로 정당하다 iff S의 p라는 믿음에 대한 정당성은 감각 경험에 의존하지 않는다.

그러나 이러한 정의는 만족스럽지 못한데, 왜냐하면 그 믿음들의 정당성이 감각 경험에 의존하지는 않지만, 그 믿음들이 선험적으로 정당화되지도 않는 그러한 믿음들이 있기 때문이다. 내가 "오늘 아침에 나는 저녁으로 스테이크를 먹는 일에 대해서 생각하고 있었다"라고 믿는다고 생각해 보자. 이러한 믿음은 선험적이지 않은데, 왜냐하면 그 믿음의 정당성이 내가 오늘 아침에 구성하고 있었던 저녁으로 무엇을 먹을지 계획한 일을 명백하게 떠올리는 기억 경험에 의존하기 때문이다. 그러나 기억 경험은 감각 경험의 형식이 아니고, 그래서 내가 저녁으로 스테이크를 먹는 일에 대해 생각하고 있었다는 믿음에 대한 나의 정당성은 감각 경험에 의존하지 않는다. 따라서 D1은 내가 저녁으로 스테이크를 먹는 일에 대해 생각하고 있었다는 믿음에 있어서 내가 선험적으로 정당화된다는 잘못된 결론을 내놓는다.

2) A parallel argument can be made about introspection. My justification for believing that I would like to eat a bar of chocolate right now does not depend on any sense experience. Thus D1 implies that my belief is justified a priori. But this is the wrong result; my belief is not justified a priori because it depends for its justification on my experiencing desire for a piece of chocolate. If I did not have such an experience, I could not justifiably believe that I would like a piece of chocolate. So, D1 proves wanting again.
2) 똑같은 논증이 내성에 대해서도 구성될 수 있다. 내가 초콜렛 바 하나를 당장 먹고 싶다는 믿음에 대한 나의 정당성은 어떠한 감각 경험에도 의존하지 않는다. 따라서 D1은 나의 믿음이 선험적으로 정당화된다는 것을 암시한다. 그러나 이것은 잘못된 결론이다. 왜냐하면 나의 믿음은 선험적으로 정당화되지 않는데 그 까닭은 그 믿음이 그 믿음의 정당화를 한 조각의 초콜렛에 대한 내가 느끼는 욕망에 의존하기 때문이다. 만일 내가 그러한 경험을 가지지 않았다면, 나는 내가 초콜렛 한 조각을 먹고 싶다고 정당화할 수 있도록 믿을 수 없었을 것이다. 그래서, D1은 부족하다는 것을 다시 한 번 입증한다.

3) These two examples show that when a priori justification is defined as justification that is independent of experience, the relevant concept of experience includes not only sense experience but memorial and introspective experience as well. However, just as we must take care to note what the relevant concept of experience includes, we must also consider what it excludes. Now in order to see what kind of experience a priori justification need not be independent of, ask what it intellectually feels like to consider the following two propositions:

(2) 2 + 2 = 4
(3) 2 + 2 = 5

Alvin Plantinga would say that while (2) feels right, (3) feels wrong. (2), he suggests, feels "compelling," while (3) "off-putting and eminently rejectable."<2> Following Plantinga, let us say that there are experiences such as intellectual compulsion and repulsion. Many a priori beliefs are accompanied by an experience of intellectual compulsion. Some philosophers would even say that a priori beliefs can be justified by such an experience.
3) 이러한 두 가지 예시들은 언제 선험적 정당성이 경험으로부터, 감각 경험만이 아니라 기억과 내성의 경험도 마찬가지로 포함하는 경험의 관련 개념으로부터 독립적인 정당성으로 정의되는지 보여준다. 그렇지만, 경험에 관련한 개념이 포함하는 것이 무엇인지 주의 깊게 관심을 기울여야만 하는 것과 마찬가지로, 우리는 그 개념이 또한 무엇을 배제시키는지에 대해서도 관심을 기울여야만 한다. 이제 선험적 정당성이 그로부터 독립적일 필요가 없는 경험의 종류가 무엇인지 보기 위해, 다음 두 가지 명제들을 고찰하는 것이 지적으로 느껴질 것 같은 것은 무엇인지 묻도록 하자.

(2) 2 + 2 = 4
(3) 2 + 2 = 5

앨빈 플랜팅가는 (2) 옳게 여겨지는 반면, (3)은 그른 것으로 여겨진다고 말할 것이다. 그가 제안하기로, (2)는 "강력한" 반면 (3)은 "받아들이기 어렵고 대단히 거부할 만하다"고 여겨진다.<2> 플랜팅가에 따라, 우리는 지적 강요와 반발과 같은 경험들이 있다고 말하도록 하자. 여러 선험적인 믿음들은 지적 강요의 경험에 의해 동반된다. 일부 철학자들은 심지어 선험적 믿음들이 그러한 경험에 의해 정당화될 수 있다고까지 말할 것이다.

4) When a priori justification is said to be independent of experience, what is meant, then, is not that it must be independent of an experience such as intellectual compulsion. Hence in defining a priori justification as justification that is independent of experience, the relevant concept of experience includes memorial and introspective experience, but excludes the experience of intellectual compulsion. We shall stipulate, therefore, that in the following definition the term "experience" is to be understood in precisely this sense.

D2    S is justified a priori in believing that p if and only if S's justification for believing that p does not depend on any experience.

Does D2 imply that if you believe

(1) If Yuri lives in Alma-Ata and Alma-Ata is the capital of Kazakhstan, then Yuri lives in the capital of Kazakhstan

you are justified a priori? Since your justification for believing (1) depends neither on sense nor on memorial or introspective experience, the answer is that it does.
4) 선험적 정당성이 경험 독립적이라 이야기될 때, 그것이 의미하는 바는, 그래서, 그 정당성이 지적 강요와 같은 그러한 경험으로부터 독립적이어야만 한다는 것을 말하는 것이 아니다. 따라서 선험적 정당성을 경험 독립적인 정당성으로 정의함에 있어서, 경험에 관련된 개념은 기억과 내성의 경험을 포함하지만, 지적 강요의 경험은 배제한다. 그러므로 우리는 다음 정의에 있어서 "경험"이라는 용어가 정확히 이러한 의미에서 이해되는 것이라고 규정해야 할 것이다.

D2    S는 p를 믿음에 있어서 선험적으로 정당화된다 iff S의 p라는 믿음에 대한 정당성은 어떠한 경험에도 의존하지 않는다.

D2는 만일 당신이 다음과 같이 믿는다면,

(1) 만일 유리가 알마-아타에 살고 알마-아타가 카자키스탄의 수도라면, 유리는 카자키스탄의 수도에 산다

당신이 선험적으로 정당화된다는 것을 시사하는가? (1)을 믿음에 대한 당신의 정당성이 감각에도 기억이나 내성 경험에도 의존하지 않으므로, 대답은 그렇다는 것이다.


A Priori Jutification and Concept Learning
선험적 정당성과 개념 학습

1) It could be objected, however, that your justification for believing (1) depends on experience after all. You couldn't be justified in believing (1) unless you understood what the proposition means, and you couldn't understand what the proposition means unless you knew what the relevant concepts mean. So in order for you to be justified in believing (1), you must understand the concepts of city, nation, and capital. It would be impossible to learn what these concepts mean without undergoing a vast array of experiences. Typically, learning such concepts involves complex social interactions between parents and children, and between teachers and students, and thus a succession of innumerable experiences. Indeed, for any proposition p, the justification you might have for believing p could not possibly be independent of the experiences necessary for learning the concepts out of which p is composed. So it would appear that your justification for believing (1) does depend on a vast number of experiences, and thus is not a priori.
1) 그렇지만 그것은 당신의 (1)을 믿음에 대한 정당성이 어쨌든 경험에 의존한다고 반박될 수 있을 것이다. 당신은 (1)을 믿음에 있어서 그 명제가 의미하는 것이 무엇인지 이해하지 않는 한 정당화될 수 없고, 당신은 그 명제가 의미하는 바가 무엇인지 그 관련 개념들이 의미하는 바를 알지 못하는 한 이해할 수 없을 것이다. 그래서 당신이 (1)을 믿음에 있어서 정당화되기 위해, 당신은 도시, 국가, 수도의 개념들을 이해해야만 한다. 이러한 개념들이 의미하는 바는 경험들의 막대한 집합체를 거침이 없이는 배우는 것이 불가능할 것이다. 전형적으로, 그러한 개념들을 배우는 것은 부모와 자녀들, 교사들과 학생들 사이의 복잡한 사회적 상호작용들을 포함하고, 그래서 헤아릴 수 없이 많은 경험들의 연쇄를 포함한다. 덧붙여서, 어떠한 명제 p에 대해서든, 당신이 p를 믿음에 대해서 가질 정당화는 p가 그로부터 구성되는 그러한 개념들을 배우는 데에 필요한 경험들로부터 독립적으로는 가능할 수 없을 것이다. 그래서 (1)을 믿음에 대한 당신의 정당성이 막대한 수의 경험들에 의존한다는 것, 그래서 선험적이지 않다는 것이 드러날 것이다.

2) In reply to this objection, we would have to admit that the experiences you had when you learned the concepts occurring in (1) were necessary for the jutification you have for believing (1). Had you not had those experiences, you would not understand (1), and thus could not be justified in believing (1). However, when you are now justified in believing (1), experiences that you had many years ago when you learned certain words surely do not now play a justificatory role. That is, although you could not be justified in believing (1) without having had those experiences, they are not what generates your justification for believing (1) at the present time. And this is precisely the point when the expression "S is justified a priori in believing that p" is defined by appeal to the concept "independence of experience": that which generates S's justification for believing p, or makes S's belief that p justified, must not be experience. When you are justified a priori in believing a proposition p, then whatever it is that generates your justification must be something other than experience - which is entirely compatible with the fact that had you not had certain experiences, you could not be justified in believing p at all.
2) 이러한 반대에 답함에 있어서, 우리는 (1)에서 나타나는 개념들을 당신이 배웠을 때 당신이 지녔던 경험들이 당신이 (1)을 믿는 정당성에 필요했다는 점을 이정해야 할 것이다. 당신이 그러한 경험들을 가지고 있지 않았더라면, 당신은 (1)을 이해하지 못하였을 터이고, 그래서 (1)을 믿음에 있어서 정당화될 수 없었을 것이다. 그렇지만, 당신이 지금 (1)을 믿음에 있어서 정당화되고 있을 때, 당신이 특정 단어들을 배웠을 때 수 년 전에 지녔던 경험들은 확실히 지금은 정당화의 역할을 수행하지 않는다. 즉, 비록 당신이 그러한 경험들을 가지고 있지 않았더라면 (1)을 믿음에 있어서 정당화될 수 없었다 할지라도, 그러한 경험들은 바로 지금 순간 (1)을 믿음에 대한 당신의 정당성을 산출하는 것은 아니다. 그리고 이것이 "S가 p를 믿음에 있어서 선험적으로 정당화된다"라는 표현이 "경험 독립성" 개념에 호소함으로써 정의될 때의 정확한 요지이다. 즉 p를 믿음에 대한 S의 정당성을 산출하거나, S의 p라는 믿음을 정당화되도록 만드는 것은 반드시 경험이 아니어야만 한다. 당신이 명제 p를 믿음에 있어서 선험적으로 정당화될 때, 당신의 정당성을 산출하는 그것이 무엇이 되었든지간에 그것은 반드시 경험과 다른 어떤 것이어야만 한다. - 그것은 당신이 특정한 경험들을 가지고 있지 않았다면, 당신이 p를 믿음에 있어서 전혀 정당화될 수 없었으리라는 사실과 전적으로 양립가능하다.

3) Let us distinguish between two ways in which something can depend on something else. First, an event x can depend on an event y because had y not happend, x would not have happened. Second, an event x can depend on an event y because y makes x the kind of event it is. For example, my thinking about tonight's dinner depends on my breathing a sufficient amount to oxygen. Without breathing a sufficient amount of oxygen, I couldn't be thinking about tonight's dinner. My thinking about tonight's dinner also depends on my hunger. If I were not hungry, then I wouldn't be thinking about tonight's dinner. There is, however, a big difference between these two examples of dependence. For certainly it is not my breathing oxygen, but rather my hunger, that makes me think about tonight's dinner. My justification for believing (1) depends on my having learned certain concepts in the same way in which my thinking about tonight's dinner depends on my breathing oxygen. This is not the kind of dependence that matters in D2; rather, the kind of dependence that matters is the second kind: what makes you justified in believing (1) - whatever it is - must be something other than experience. Hence it is true after all that, according to D2, your justification for believing (1) is a priori.
3) 어떤 것이 다른 어떤 것에 의존할 수 있는 두 가지 방식을 구별하도록 하자. 첫째로, 하나의 사건 x는 또 하나의 사건 y에 의존할 수 있는데 왜냐하면 y가 발생하지 않았더라면, x가 발생하지도 않았을 것이기 때문이다. 두 번째, 한 사건 x는 또 하나의 사건 y에 의존할 수 있는데 왜냐하면 y가 x를 그 x가 그렇게 일어난 그러한 종류의 사건으로 만들기 때문이다. 예를 들어, 오늘 밤 저녁 식사에 대한 나의 생각은 충분한 양의 산소를 공급하는 나의 호흡에 의존한다. 충분한 양의 산소를 공급하는 호흡이 없이, 나는 오늘 저녁 식사에 대해 생각하고 있을 수 없을 것이다. 오늘 저녁 식사에 대한 나의 생각은 또한 나의 허기에도 의존한다. 만일 내가 배고프지 않았더라면, 나는 오늘 저녁 식사에 대해 생각하지 않았을 것이다. 그렇지만, 이러한 의존의 두 예시들 사이에는 커다란 차이가 있다. 확실히, 오늘 저녁 식사에 대해 내가 생각하도록 만드는 것은 산소 호흡이 아니라 오히려 나의 배고픔이다. 이것은 D2에서 문제시되는 그런 종류의 의존이 아니다. 오히려, 문제시되는 그런 종류의 의존은 두 번째 종류의 것이다. 즉 (1)을 믿음에 있어서 당신을 정당화되도록 만드는 것 - 그것이 무엇이 되었든지 간에 - 그것은 반드시 경험과는 다른 어떤 것이어야만 한다. 따라서 D2에 따르자면 (1)을 믿음에 대한 당신의 정당성이 선험적이라는 것은 어찌 되었든 참이다.

4) In the next two sections, we shall consider two alternative ways of characterizing a priori justification. In discussing them, we shall sometimes use the phrase "a priori propositions" instead of the cumbersome expression "propositions that are knowable or justifiable a priori." Similarly, we shall substitute the term "apriority" for the "the property of being knowable or justifiable a priori."
4) 다음 두 절에 걸쳐서, 우리는 선험적 정당성을 규정하는 두 가지 대안적 방식들을 고찰할 것이다. 그러한 방식들을 논의함에 있어서, 우리는 때때로 "선험적 명제들"이라는 구를 "선험적으로 알 수 있거나 정당화될 수 있는 명제들"이라는 다루기 어려운 표현 대신으로 사용할 것이다. 비슷하게, 우리는 "선험적으로 알 수 있거나 정당화될 수 있음의 속성"을 "선험성"이라는 용어로 대신할 것이다.


Apriority and Necessity
선험성과 필연성

1) Our first approach to the analysis of apriority - what makes justification a priori is independence of experience - tells us what does not generate a priori justification, but it doesn't tell us what does. Thus we are still left with the question of how it can it be possible to determine a proposition's truth value without relying on any experience. One answer to this question appeals to the concept of necessity. Consider the following examples of propositions that, according to a long-standing philosophical tradition, are examples of a priori knowledge.

(a)  The sum of two and two is four
(b)  Two is an even number
(c)  Whatever is square is rectangular
(d)  Whatever is red is colored
(e)  Whatever is blue all over doesn't have green spots
(f)  For any two propositions p and q, if p is true and q false, then the conjunction of p and q is false.
(g) Either it's raining or it's not raining

Proposition (a) and (b) are truths of mathematics, (c) is a truth of geometry, (d) and (e) are examples of what we might call "conceptual" truths, and (f) and (g) are logical truths. What all of these propositions have in common is that they are necessary. According to a second conception of a priori justification, that is precisely what makes a priori justification possible. If a proposition is necessarily true, then simply thinking about it will be sufficient for recognizing its truth - assuming, of course, the proposition isn't so complicated that it cannot be known at all.
1) 선험성 분석에 대한 우리의 첫 번째 접근은 - 정당성을 선험적으로 만드는 것은 경험독립적이다 - 우리에게 선험적 정당성을 산출하지 못하는 것은 말해주지만, 그러한 정당성을 산출하는 것은 말해주지 못한다. 따라서 우리는 여전히 어떻게 한 명제의 진리치를 어떠한 경험에도 의존함이 없이 결정하는 것이 가능할 수 있는지에 대한 물음을 남겨두게 된다. 이 물음에 대한 한 가지 대답은 필연성의 개념에 호소하는 것이다. 오래 지속되는 철학적 전통에 따라 선험적 앎의 예시들인 그렇나 명제들의 다음 예시들을 고찰해 보도록 하자.

(a) 2와 2의 합은 4이다
(b) 2는 짝수이다
(c) 정사각형인 것은 무엇이든 직사각형이다
(d) 붉은 것은 무엇이든 색이 있다.
(e) 푸른 것은 무엇이든 그 어디에서도 녹색 반점들을 지니지 않는다
(f) 어떤 명제 p와 q에 대해서든, 만일 p가 참이고 q가 거짓이라면, p와 q의 결합은 거짓이다.
(g) 비가 오고 있거나 비가 오지 않고 있다.

명제 (a)와 (b)는 수학의 참들이고, (c)는 기하학의 참이며, (d)와 (e)는 우리가 "개념적" 참들이라 부를 것의 예시들이고, (f)와 (g)는 논리적 참들이다. 이러한 명제들이 공통으로 가지고 있는 것은 그것들이 필연적이라는 점이다. 선험적 정당성의 두 번째 개념에 따르면, 그것이 정확히 선험적 정당성을 가능하게 만드는 것이다. 만일 한 명제가 필연적으로 참이라면, 단순히 그 명제에 대한 생각만으로 그 명제의 참을 아는 데에 충분할 것이다 - 물론, 그 명제가 전혀 알려질 수 없을 만큼 그렇게 복잡하지 않다고 가정하면.

2) Consider the proposition "Whatever is red is colored." If you understand what it is for an object to be red and what it is for an object to be colored, then you can understand as well that there couldn't possibly be an object that is red without being colored. Put differently, you can come to see that the property of being red includes the property of being colored, and thus that the proposition "Whatever is red is colored" is necessarily true.<3> Thus one might propose to explain apriority by saying the following: What makes the proposition "Whatever is red is colored" a priori is that you can grasp its necessity. Consider a second example: Nothing that is square is circular. If you know what a square is (a rectangle with four equal sides), then you know that the property of being square excludes the property of being circular: whatever is square cannot also be circular. This is why thought alone enables you to see that there couldn't possibly be a nonrectangular square - why you don't need experience in order to know that all squares are rectangular. Again, so the proposal goes, what makes your justification for believing this proposition a priori is that when you consider that proposition and reflect on it, you can grasp its necessity.
2) "붉은 것은 무엇이든 색이 있다"라는 명제를 고찰해 보자. 만일 당신이 한 대상이 붉다는 것과 한 대상이 색이 있다는 것이 무엇인지 이해한다면, 당신은 마찬가지로 색이 없으면서 붉은 대상이 있는 게 가능할 수 없다는 것도 이해할 수 있다. 달리 말하면, 당신은 색 있음의 속성을 포함하는 붉음의 속성을 알게 될 수 있고, 그래서 "붉은 것은 무엇이든 색이 있다"라는 명제는 필연적으로 참이라는 것을 알게 될 수 있다.<3> 따라서 다음과 같이 말함으로써 선험성을 설명하고자 제안할 수 있을지도 모른다. 즉 "붉은 것은 무엇이든 색이 있다"라는 명제를 선험적으로 만드는 것은 당신이 그 명제의 필연성을 파악할 수 있음이다. 두 번째 예를 고찰해 보자. 그 예는 정사각형인 그 어떤 것도 원형이 아니라는 것이다. 만일 당신이 정사각형이 무엇인지 안다면(동일한 네 변을 지닌 직사각형), 당신은 원형임의 속성을 배제하는 정사각형임의 속성을 안다. 즉 정사각형인 무엇이든 또한 원형일 수는 없다는 것을 안다. 이것은 생각만으로 당신이 비직사각 정사각형이 있는 게 가능할 수 없는지 알도록 할 수 있는 이유 - 당신이 모든 정사각형이 직사각형임을 알기 위해 경험을 필요로 하지 않는 이유 - 이다. 다시, 그 제안이 진행되는 그대로, 이 명제를 믿음에 대한 당신의 정당성을 선험적으로 만드는 것은 당신이 그 명제를 고찰하고 그 명제에 대해 숙고할 때, 당신이 그 명제의 필연성을 파악할 수 있다는 것이다.

3) In order to evaluate this proposal, it is important to examine what relation exists between apriority and necessity. To begin with, let us consider the following definition:

D3    S is jutified a priori in believing that p if and only if S believes that p and p is a necessary truth.

This definition fails for two reasons. First, it fails because it does not place any constraints on the manner in which p is believed; it does not demand that S form the belief that p in a way that makes S's belief justified.<4> Suppose you believe a complicated logical theorem (i.e., a necessary truth) because you think you proved it. In constructing this proof, though, you were quite sloppy. You were preoccupied with other matters, and thus didn't concentrate very well. Your proof, therefore, is of dubious value. Its validity would be no more than a lucky accident. Surely, such a proof does not justify you in believing the theorem. However, since you believe a proposition that, ex hypothesi, is a necessary truth, D3 yields the wrong result that you are justified in believing it. The fact of the matter is that you are not at all justified in believing it. The general lesson to be drawn from this case is that since it is possible to believe a necessary proposition without any justification at all, it is not sufficient for a priori justification that the proposition believed be necessary.
3) 이 제안을 평가하기 위해, 선험성과 필연성 사이에 어떤 관계가 존재하는지 검토하는 것이 중요하다. 시작함에 있어서, 다음 정의를 살펴 보도록 하자:

D3    S는 p를 믿음에 있어서 선험적으로 정당화된다 iff S가 p를 믿고 p는 필연적으로 참이다.

이 정의는 두 가지 이유들로 실패한다. 첫째로, 그 정의는 실패하는데 왜냐하면 그 정의가 p가 믿어지는 방식에 대해 어떠한 제한들도 두지 않기 때문이다. 즉 그 정의는 S가 p에 대한 믿음을 S의 믿음이 정당화되도록 만드는 방식으로 형성하도록 요구하지 않는다는 것이다.<4> 당신이 복잡한 논리적 정리 (즉, 필연적 참)을 믿는데 왜냐하면 당신이 그것을 증명했다고 생각하기 때문이라고 가정해 보자. 이러한 증명을 구성함에 있어서, 그렇다 할지라도, 당신은 꽤나 설렁설렁했었다. 당신은 다른 문제들에 정신이 팔려 있었고, 그래서 아주 잘 집중하지는 못했다. 그러므로 당신의 증명은 불확실하다. 그 증명의 유효성은 우연한 일치 이상의 아무것도 아닐 것이다. 확실히, 그러한 증명은 그 정리를 믿음에 있어서 당신을 정당화하지 않는다. 그렇지만, 당신이 전제에 따라 필연적으로 참인 명제를 믿으므로, 3D는 당신이 그 증명을 믿음에 있어서 정당화된다는 잘못된 결론을 내린다. 그 문제에 대한 사실은 당신이 그 증명을 믿음에 있어서 전혀 정당화되지 않는다는 것이다. 이 사례로부터 도출되는 일반적인 교훈은 필연적 명제를 어떠한 정당성도 전혀 없이 믿는다는 것이 가능하므로, 명제가 필연적으로 것으로 믿어진다는 것은 선험적 정당성에 충분치 못하다는 점이다.


-蟲-
Principles of Prima Facie Justification
직견적 정당화의 원칙들

1) The doctrine that the property of being justified strongly supervenes on nonnormative grounds tells us this: whenever a belief is justified, there is a nonnormative property P (where this property may be a conjunction of several distinct properties) such that, necessarily, any belief having P is justified. This is a doctrine of metaphysics: a doctrine about how the property of justification is related to certain other properties. It entails nothing with respect to our ability to pin down what these other properties are. It is, therefore, entirely consistent to maintain that epistemic status strongly supervenes on nonnormative base properties and at the same time to doubt that the project of criteriological analysis can meet with success.
1) 정당화됨의 속성이 비규범적 토대들에 강력하게 수반한다는 원칙은 우리에게 이와 같은 것을 말해준다. 즉 한 믿음이 정당화될 때라면 언제든, (이 속성이 아마도 몇몇 구별되는 속성들의 결합일 곳에서) P를 가지는 어떠한 믿음이든 필연적으로 정당화되는 그러한 비규범적 속성 P가 있다는 것이다. 이것은 형이상학의 원칙이다. 즉 어떻게 정당화의 속성이 특정한 다른 속성들에 관계되는지에 대한 원칙이다. 그 원칙은 이러한 다른 특성들이 무엇인지 확정짓는 우리의 능력에 대한 측면에 관해서는 아무것도 함의하지 않는다. 그러므로 그 원칙은 전적으로 한결같이 인식적 상태는 강력하게 비규범적 기본 속성들에 수반한다는 것을 주장함과 동시에 규준적 분석의 기획이 성공할 수 있다는 것을 의심하는 것이다.

2) Since reliabilism will be discussed in a separate chapter, let us discuss the prospects of criteriological analysis focusing on the Chisholmian project of listing epistemic principles. Consider the second of the epistemic principles we stated above:

P2    If a belief is grounded in perception, then it is justified.

It is easy to see that this principle is false. Recall the standard example of a defeated belief that was explained in Chapter 1: You are looking at a white sheet of paper that appears blue to you because it is illuminated by blue light. Although you have evidence for suspecting that the sheet of paper is thus illuminated, you still believe it to be blue. Your perceptual evidence for your belief is defeated, and thus, though grounded in perception, your belief is unjustified.
2) 신빙론은 독립된 장에서 논의될 것이므로, 우리는 인식 원칙들을 열거함에 대한 치셜주의 기획에 촛점을 맞춘 규준적 분석을 논의하도록 하자. 우리가 위에서 진술하였던 인식 원칙들 중 두 번째 것을 고찰해 보자.

P2    만일 하나의 믿음이 지각에 근거된다면, 그 믿음은 정당화된다.

이 원칙이 거짓임을 알아보기는 쉽다. 1장에서 설명되었던 상쇄된 믿음의 표준 사례를 기억해 보자. 즉 당신이 파란 조명에 비춰지고 있기 때문에 당신에게 파랗게 보이는 한 장의 흰 종이를 보고 있는 경우를 기억해 보자. 비록 당신이 그 한 장의 종이가 그렇게 비춰지고 있다고 의심할 증거를 가지고 있다 할지라도, 당신은 여전히 그 종이가 파랗다고 믿는다. 당신의 믿음에 대한 당신의 지각적 증거는 상쇄되고, 그래서, 비록 지각에 근거될지라도, 당신의 믿음은 정당화되지 않는다.

3) In order to repair P2, we could add a clause to the effect that, for a perceptual belief to be justified, one may not be in possession of further evidence that defeats one's perceptual evidence. This would give us the following modified principle:

P2a   If a belief is grounded in perception and is not defeated by further evidence, then it is justified.

To be sure, P2a is a good candidate for a true epistemic principle. It tells us that undefeated perceptual evidence is a source of justified belief. Indeed, it provides us with a criterion of justification: the property of being grounded in undefeated perceptual evidence is arguably a property that entails the property of being justified. However, P2a is not the kind of principle cirteriological analysis is supposed to establish, for its antecedent makes use of the concept of evidential defeat, which is an epistemically evaluative term.
3) P2를 수정하기 위해, 우리는 지각적 믿음이 정당화된다는 그 결과에 하나의 절을 추가시킬 수 있을 것이다. 그 절은 한 사람의 지각적 증거를 상쇄시키는 추가 증거를 소유하지 않을 것이라는 절이다. 이는 다음의 수정된 원칙을 우리에게 제시한다.

P2a  만일 하나의 믿음이 지각에 근거하고 추가 증거에 의해 상쇄되지 않는다면, 그 믿음은 정당화된다.

확실히, P2a는 참된 인식 원칙의 훌륭한 후보이다. 그 원칙은 우리에게 상쇄되지 않는 지각적 증거가 정당화된 믿음의 원천이라 말해준다. 덧붙여서, 그 원칙은 우리에게 정당화의 기준을 제공한다. 즉 상쇄되지 않는 지각적 증거에 근거됨이라는 속성은 거의 틀림없이 정당화됨의 속성을 함의하는 속성이다. 그렇지만, P2a는 확립하기로 되어 있는 그러한 종류의 규준적 분석이 아닌데, 왜냐하면 그 원칙의 전제가 인식적으로 평가적인 용어인 증거적 상쇄의 개념을 활용하기 때문이다.

4) To see why the notion of evidential defeat is epistemically evaluative, we need only consider its definition:

d defeats e as evidence for p if and only if e is evidence for believing that p, but e in conjunction with d is not evidence for believing that p.

The right-hand side of this definition makes use of the term "is evidence for," which is a term of epistemic evaluation. Hence the concept of defeat is itself epistemically evaluative.
4) 어째서 증거적 상쇄의 관념이 인식적으로 평가적인지 보기 위해, 우리는 단지 그 개념의 정의를 고찰해 보면 족하다.

d는 p에 대한 증거로서의 e를 상쇄시킨다 iff e는 p라는 믿음에 대한 증거이지만, d와 결합한 e는 p라는 믿음에 대한 증거가 아니다.

이 정의의 오른편은 "~에 대한 증거이다"라는 용어를 활용한다. 그 용어는 인식적 평가의 용어이다. 따라서 상쇄의 개념은 그 자체로 인식적으로 평가적이다.

5) P2a, then, contains an epistemic concept in its antecedent and thus falls short of satisfying the objective of criteriological analysis. This does not mean, though, that it is worthless. Indeed, as mentioned above, it tells us something interesting: undefeated perceptual evidence is an entailing ground of epistemic justification. There are no possible worlds in which beliefs grounded in such evidence fail to be justified. But since we could render P2a true only by adding a normative qualifier in its antecedent, it must be said that it is not the kind of principle we seek when we are trying to set forth a criteriological analysis of epistemic justification.
5) 그래서 P2a는 인식적 개념을 그 전제에 포함하고 그래서 규준적 분석의 목적을 만족시키기에는 불충분하다. 그럴지라도 이것은 이 원칙이 무가치하다는 말은 아니다. 덧붙여서, 위에서 언급되었듯, 그 원칙은 우리에게 어떤 흥미로운 것을 말해준다. 즉 상쇄되지 않는 지각적 증거는 인식적 정당성을 수반하는 근거이다. 그러한 증거에 근거된 믿음들이 정당화되지 않는 그러한 어떠한 가능 세계도 없다. 그러나 우리는 오직 규범적 수식어구를 그 원칙의 전제에 추가시킴으로써만 P2a를 참으로 만들 수 있을 것이므로, 그 원칙은 우리가 인식적 정당성의 규준적 분석을 제시하고자 시도하고 있을 때 추구하는 그러한 종류의 원칙은 아니라고 이야기되어야만 한다.

6) Is there any way of modifying P2 so that it remains true, yet its antecedent stays nonnormative? There is indeed: we can weaken the principle's consequent. Instead of saying that beliefs grounded in perception are justified, we can say that they are prima facie justified. When we call perceptual beliefs prima facie justified, we mean that such beliefs are justified in the absence of defeating evidence. Compare perceptual beliefs with beliefs that are instances of wishful thinking. Such beliefs are not prima facie justified, for we can't say that they are justified in the absence of further evidence. Suppose you take yourself to be perceiving a dog in front of you. Unless there is further evidence undermining your perceptual evidence, you are justified in believing there is a dog in front of you. On the other hand, if wishful thinking induces you to believe that tomorrow's weather will be good, we can't say that, unless there is further undermining evidence, you are justified in believing that tomorrow's weather will be good. The difference between the two beliefs is that the first is based on evidence that, in the absence of further countervailing evidence, justifies your belief.
6) P2를 수정함으로써 그것이 참으로 남지만, 그럼에도 그 전제는 비규범적으로 머무르도록 할 어떤 방법이 있는가? 실제로 있다. 즉 우리는 그 원칙의 결론을 약화시킬 수 있다. 지각에 근거하는 믿음들이 정당화된다고 말하는 대신, 우리는 그러한 믿음들이 직견적으로 정당화된다고 말할 수 있다. 우리가 지각적 믿음들을 직견적으로 정당화된다고 할 때, 우리는 그러한 믿음들이 상쇄하는 증거의 부재 중에 정당화된다고 말하는 것이다. 지각적 믿음들과 낙관적 생각의 사례들인 믿음들을 비교해 보자. 그러한 믿음들은 직견적으로 정당화되지 않는데, 왜냐하면 우리는 그 믿음들이 추가적인 증거의 부재 중에 정당화된다고도 말할 수 없기 때문이다. 당신이 당신 앞에 있는 개 한 마리를 지각하고 있다고 가정해 보자. 당신의 지각적 증거 기반을 약화시키는 추가적 증거가 없는 한, 당신은 당신 앞에 개 한 마리가 있다고 믿음에 있어서 정당화된다. 다른 한편, 만일 낙관적 생각이 당신으로 하여금 내일 날씨가 좋을 것이라 믿도록 만든다면, 우리가 약화시키는 추가적 증거가 없는 한 당신은 내일 날씨가 좋을 것이라 믿음에 있어서 정당화된다고, 그렇게 말할 수는 없다. 이 두 가지 믿음들 사이의 차이는 첫 번째 믿음이 무효화하는 추가 증거가 부재하는 중에 당신의 믿음을 정당화하는 증거에 기초된다는 것이다.

7) A prima facie justified belief might or might not be ultima facie justified. An ultima facie justified belief is justified all things considered. Consider again the counterexample to P2. Since the sheet of paper appears blue to you, we may say that although you are prima facie justified in believing that it is blue, you are not ultima facie justified in believing this because you are in possession of defeating evidence. So, de facto, your belief is unjustified. Of course, beliefs that are prima facie justified need not be unjustified. Typically, prima facie beliefs are also ultima facie justified beliefs.
7) 직견적으로 정당화된 믿음은 궁극적으로 정당화되거나 그렇지 않거나 할 것이다. 궁극적으로 정당화된 믿음은 어느 모로 보나 정당화된다. 다시 한 번 P2의 반례를 고찰해 보자. 그 한 장의 종이가 당신에게 파랗게 보이므로, 우리는 아마도 비록 당신이 그 종이가 파랗다는 믿음에 있어서 직견적으로 정당화된다 할지라도, 당신은 이것을 믿음에 있어서 궁극적으로 정당화되지는 않는데 왜냐하면 당신이 상쇄 증거를 소유하고 있기 때문이라고 말할 것이다. 그래서 사실상 당신의 믿음은 정당화되지 않는다. 물론, 직견적으로 정당화되는 믿음들은 정당화되지 않은 것일 필요는 없다. 전형적으로, 직견적 믿음들은 또한 궁극적으로 정당화된 믿음들이기도 하다.

8) Let us compare the notion of prima facie justification in epistemology with that of a prima facie duty in ethics. According to W. D. Ross, telling the truth is merely a prima facie duty: a duty that can be defeated by other duties. For example, if you could save a person's life by telling a lie, then the duty to save that person's life would defeat your duty to tell the truth. In this particular situation, telling the truth is your prima facie duty, but not your ultima facie duty. All things considered, it is your duty to tell a lie. Typically, however, telling the truth is not only your prima facie duty but also your ultima facie duty. Truth telling, we may say, tends to be one of our duties, which is to say that typically it is our duty to tell the truth. Likewise, perceptual evidence tends to justify our beliefs, which is to say that, neglecting exceptions, beliefs grounded in perception are justified.
8) 인식론에 있어서의 직견적 정당성을 윤리학에 있어서의 직견적 의무와 비교해 보도록 하자. W. D. 로스에 따르면, 진실을 말하는 것은 단지 직견적 의무이다. 즉 그 의무는 다른 의무들에 의해 상쇄될 수 있는 의무이다. 예를 들어, 만일 당신이 거짓말을 함으로써 한 사람의 생명을 구할 수 있다면, 한 사람의 생명을 구할 의무는 진실을 말할 당신의 의무를 상쇄시킬 것이다. 이 특수한 상황에서, 진실을 말하는 것은 당신의 직견적 의무이지만, 당신의 궁극적 의무는 아니다. 모든 면에서, 거짓을 말하는 것이 당신의 의무이다. 그렇지만 전형적으로 진실을 말하는 것은 당신의 직견적 의무일 뿐만 아니라 당신의 궁극적 의무이기도 하다. 진실을 말하는 것은 우리의 의무들 중 하나이기 쉽다고 말할 만하다. 그것은 전형적으로 진실을 말하는 것이 우리의 의무라고 말하는 것이다. 같은 식으로, 지각적 증거는 우리의 믿음을 정당화하는 경향이 있다. 그 말은 즉, 예외들을 무시하면, 지각에 근거된 믿음들은 정당하다는 말이다.

9) The maneuver of weakening the consequent of P2 by truning it into a principle about prima facie justification gives us the following result:

P2b  If a belief is grounded in perception, then it is prima facie justified.

Unlike P2, P2b is an excellent candidate for a true epistemic principle. However, just like P2a, it is not the kind of principle we seek when we are interested in producing a criteriological analysis of epistemic justification. For what we want to find out in pursuing such an analysis is what nonnormative base properties make beliefs ultima facie, not prima facie, justified. Thus, if all we can achieve is to formulate principles such as P2b, our analysis would fall short of accomplishing its objective.
9) P2를 직견적 정당성에 대한 원칙으로 전환시킴으로써 그 결론을 약화시키는 책략은 우리에게 다음과 같은 결과를 제공한다:

P2b  만일 하나의 믿음이 지각에 근거한다면, 그 믿음은 직견적으로 정당화된다.

P2와 달리 P2b는 참된 인식 원칙의 훌륭한 후보이다. 그렇지만, P2a와 마찬가지로, P2b는 우리가 인식적 정당성에 대한 규준적 분석을 산출하는 데에 관심을 가질 때 우리가 찾고 있는 그러한 종류의 원칙은 아니다. 우리가 그러한 분석을 추구함에 있어서 찾아내기를 바라는 것은 믿음들을 직견적으로가 아니라 궁극적으로 정당화되도록 만드는 어떤 비규범적 기본 속성들이기 때문이다. 그래서, 만일 우리가 성취할 수 있는 것은 전부 P2b와 같은 그러한 원칙들을 만들어 내는 것이라면, 우리의 분석은 그 목적을 완수하기에 부족할 것이다.

10) In order to see why this is so, note that P2b marks no real improvement over P2a because it is logically equivalent to it. Given our understanding of prima facie justification, we may reformulate P2b thus:

P2b* If a belief is grounded in perception, then it is justified unless it is defeated by further evidence.

P2b tells us the same as P2b*, which tells us the same as P2a. Consequently, if P2a is not what we aspire to in analyzing the concept of epistemic justification, then neither is P2b. So whichever way we go - modifying the antecedent of P2 by adding a "no defeat" clause or weakening the consequent by turning P2 into a principle about prima facie justification - we don't get the result we intended to get: a principle that specifies a nonnormative base property on which justification supervenes.
10) 어째서 이러한지 보기 위해, P2b가 P2a에 대해 아무런 실질적인 진전도 나타내지 않는데 왜냐하면 P2b가 논리적으로 P2a에 동치이기 때문이라는 점을 주의하도록 하자. 직견적 정당성에 대한 우리의 이해를 고려해 보면, 우리는 P2b를 다음과 같이 재정식화할 수 있을지도 모른다.

P2b* 만일 하나의 믿음이 지각에 근거한다면, 그 믿음은 추가적인 증거에 의해 상쇄되지 않는 한 정당화된다.

P2b는 우리에게 P2b*와 똑같은 것을 말해준다. P2b*는 우리에게 P2a와 동일한 것을 말해준다. 결론적으로, 만일 P2a가 인식적 정당성의 개념을 분석함에 있어서 우리가 열망하는 것이 아니라면, P2b도 그러한 것이 아니다. 그래서 우리가 가는 어떤 경로로도 - "무(無)상쇄" 절을 추가함으로써 P2b의 전제를 수정하는 것이나 P2를 직견적 정당성에 대한 원칙으로 전환시켜 결론을 약화시키는 것이나 - 우리는 우리가 얻으려는 결과를 얻지 못한다. 즉 정당성이 그에 수반하는 비규범적 기본 속성을 상술하는 원칙을 얻을 수 없다.

11) Parallel reasoning can be applied to epistemic principles about the other two nonnormative base properties we introduced: being grounded in introspection and being grounded in memory. The question arises, therefore, whether it is at all possible to find the kind of principles criteriological analysis seeks.
11) 동일한 추론이 우리가 소개하였던 다른 두 가지 비규범적 기본 속성들에 대한 인식 원칙들에도 적용될 수 있다. 즉 내성에 기초됨과 기억에 기초됨에 대한 인식 원칙들 말이다. 그러므로 규준적 분석이 추구하는 그러한 종류의 원칙들을 찾는 것이 도대체 가능하기는 한 건지 의구심이 든다.


Epistemological Pessimism
인식론적 비관주의

1) Let us call epistemological pessimism the view that although there are nonnormative properties on which epistemic justification supervenes, we can't pin down what these properties are. Ernest Sosa refers to this view as "skepticism about our ability to comprehend the principles that underlie [the supervenience of epistemic on descriptive properties], perhaps because they are infinite in number or degree of complexity."<24> If epistemological pessimism is correct, we must give up the hope of identifying which nonnormative base properties entail the property of justification. Instead, we must be content with formulating principles of prima facie justification: principles telling us which nonnormative properties tend to make beliefs justified.
1) 인식론적 비관주의를 비록 인식적 정당성이 그에 수반하는 비규범적 속성들이 있다할지라도 우리는 그 속성들이 무엇인지 확정지을 수 없다는 관점이라 말하도록 하자. 어니스트 소사는 이 관점을 "아마도 그러한 원칙들이 수에 있어서나 복잡성의 정도에 있어서 무한정하다는 이유에 의한, [기술적 속성들에 대한 인식적 속성들의 수반성의] 기저를 이루는 원칙들을 파악할 우리의 능력에 대한 회의주의"라 언급한다.<24> 만일 인식론적 비관주의가 맞다면, 우리는 어떤 비규범적 기본 속성들이 정당성의 속성을 함의하는지 규정하려는 희망을 포기하여야만 한다. 대신에, 우리는 직견적 정당화의 원칙들, 즉 우리에게 어떤 비규범적 속성들이 믿음들을 정당화되도록 만드는 경향이 있는지를 말해주는 원칙들을 정식화하는 데에 만족해야만 한다.

2) According to a theory of prima facie justification, there are several sources of justification - for example, introspection, perception, and memory. Beliefs grounded in any of these sources are prima facie justified, or justified unless defeated by further evidence. Consider again perceptual beliefs. Whether a particular perceptual belief is justified depends on the specifics of the situation: does the subject, or does she not, have further evidence that defeats her perceptual evidence for the belief? Epistemological pessimists would say that we cannot find general criteria that, when applied to particular situations, tell us whether the subject has further defeating evidence, and thus whether the belief in question is justified or not. Rather, they would say that we must, for each individual case, consider all the relevant data that are available and then judge the belief's epistemic merits.
2) 직견적 정당화 이론에 따르면, 정당성의 여러 원천들이 있다. 예를 들어 내성, 지각, 기억이 그러한 원천들이다. 이러한 원천들 중 어느 것에든 근거하는 믿음들은 직견적으로 정당화되거나, 추가 증거에 의해 상쇄되지 않는 한 정당화된다. 다시 지각적 믿음들을 고찰해 보도록 하자. 특수한 지각적 믿음이 정당화되는지 여부는 그 상황의 특성들에 의존한다. 즉 그 주체가 그 믿음에 대한 주체의 지각적 증거를 상쇄시키는 추가적 증거를 가지는가, 갖지 않는가에 의존한다. 인식론적 비관주의자들은 우리가 특수한 상황들에 적용되었을 때 우리에게 그 주체가 추가 상쇄 증거를 가지는지, 그래서 문제가 되는 믿음이 정당화되는지 아닌지를 말해주는 일반적인 기준들을 발견할 수 없다고 말할 것이다. 오히려, 그들은 우리가 각각의 개별적인 경우에 대해 관련된 모든 가용한 자료들을 고찰하고 그래서 그 믿음의 인식적 장점들을 판단해야만 한다고 말할 것이다.

3) Let us compare epistemological pessimism with its counterpart in ethics: W. D. Ross's theory of prima facie duties. According to Ross, our moral duties spring from sources such as fidelity, reparation, gratitude, justice, beneficence, self-improvement, and nonmaleficence. Each of these sources provides us with a sufficient ground of prima facie duty. Thus Ross would say that if doing x were an act of gratitude, then doing x would be a prima facie duty; and if doing x were an act of injustice, then refraining from doing x would be a prima facie duty. But he does not propose what we have called a criteriological analysis. For, first of all, some of the items on Ross's list are normative: justice and beneficence, for example. And second, he does not claim that the properties on his list entail that acts instantiating these properties are acts one has a duty to perform (or refrain from performing).
3) 인식론적 비관주의를 윤리학에 있어서의 그 대응물과 비교해 보도록 하자. 그것은 W. D. 로스의 직견적 의무들에 대한 이론이다. 로스에 따르면, 우리의 도덕적 의무들은 성실성, 보상, 감사, 정의, 이익, 자기-증진, 비악(非惡)과 같은 그러한 원천들로부터 나온다. 이러한 원천들의 각각은 우리에게 직견적 의무의 충분한 근거를 제공한다. 그래서 로스는 만일 x를 행함이 감사의 행위라면, x를 행함은 직견적 의무일 것이라고 말할 것이다. 그리고 만일 x를 행함이 부정의에 속하는 행위라면, x를 행함을 삼가는 것은 직견적 의무일 것이다. 그러나 그는 우리가 규준적 분석이라 불렀던 것을 제안하지 않는다. 왜냐하면 무엇보다도 로스의 목록에 있는 항목들 중 일부는 규범적이기 때문이다. 예를 들어 정의와 유익이 그러하다. 그리고 다음으로, 그는 그의 목록에 있는 속성들이 이러한 속성들을 예시하는 행위들이 수행해야 할(또는 수행을 삼갈) 의무를 지니는 행위들임을 함의한다고 주장하지 않는다.

4) Obviously, different duties can come into conflict with one another. For example, justice and beneficence notoriously conflict with each other. In a situation in which they do, there is a prima facie duty to be just and a prima facie duty to be beneficent. The problem is deciding which of these duties would take precedence all things considered. Ross does not believe that there are general principles that tell us when considerations of justice outweigh considerations of beneficence, or vice versa. Rather, he believes that each case in which these considerations come into conflict must be judged individually. Ross, then, is a pessimist about criteriological analysis in ethics. Though his view is compatible with the supervenience doctrine - in each case of a particular act's being a moral duty, there is a base property on which the act's moral status supervenes - it entails that we cannot pin these properties down by formulating general principles of ultima facie duties.
4) 분명히, 상이한 의무들은 서로 갈등을 일으킬 수 있다. 예를 들어, 정의와 이익은 서로 반목하기로 악명이 높다. 그 두 의무들이 반목하는 상황에서, 정의로우라는 직견적 의무와 유익하라는 직견적 의무가 있다. 이러한 의무들 중 어느 한 쪽을 결정하는 문제는 모든 것을 고려하여 우선순위를 정할 것이다. 로스는 우리에게 언제 유익에 대한 고려들이 정의에 대한 고려들보다 더욱 클지, 혹은 그 역은 어떠한지 말해주는 일반적 원칙들이 있다고 믿지 않는다. 오히려, 그는 이러한 고려들이 갈등을 일으키는 각각의 경우가 반드시 개별적으로 판단되어야만 한다고 믿는다. 그래서 로스는 윤리학에 있어서의 규준적 분석에 대해 비관주의자이다. 그의 관점이 수반 원칙 - 특수한 행위가 도덕적 의무인 각 경우에 있어서, 그 행위의 도덕적 상태가 그에 수반하는 기본 속성이 있다는 - 과 양립가능할지라도 그의 관점은 우리가 이러한 속성들을 궁극적 의무들에 대한 일반 원칙들을 정식화함으로써 확정지을 수는 없다는 것을 함의한다.

5) In epistemology, conflicting bodies of evidence are analogous to the phenomenon of conflicting duties. For example, perceptual evidence about an object's color can be undermined by further evidence about the conditions of observation. If the object appears blue, then there is prima facie justification for believing it to be blue. But if we know that blue light is shining on the object, there is also prima facie justification for refraining from believing that it is blue. The question about such cases is this: Which body of evidence is stronger, the evidence that supports believing the object is blue or the evidence that supports refraining from believing this? In other words, what would be our ultima facie duty: to believe, or not to believe, that the object is blue? The answer to this question depends, of course, on the specific features of the situation. Perhaps there is further evidence suggesting that the sheet of paper is actually blue or further evidence supporting the belief that it is actually white. Epistemological pessimists would say that because there are too many different ways in which evidence can be defeated, there are no general principles that tell us, for all such cases, what we are ultima facie justified in believing. All we can know, in the way of general principles, is that an object's looking blue provides us with prima facie justification for believing it to be blue, and that evidence to the effect that blue light is shining on the object provides us with prima facie justification for refraining from believing that the object is blue.
5) 인식론에 있어서 갈등을 일으키는 증거의 실체들은 의무들이 갈등하는 현상과 유사하다. 예를 들어, 한 대상의 색에 대한 지각적 증거는 관찰 조건들에 대한 추가적 증거에 의해 약화될 수 있다. 만일 그 대상이 푸르게 보인다면, 그 대상이 푸르다는 믿음에 대한 직견적 정당성이 있다. 그러나 만일 우리가 푸른 조명이 그 대상을 비추고 있다는 것을 안다면, 그 대상이 푸르다는 믿음을 삼가는 데에 대한 직견적 정당성또한 있다. 그러한 사례들에 대한 물음은 이것이다. 어떠한 증거체가 더욱 강한지, 그 대상이 푸르다는 믿음을 뒷받침하는 증거인지 아니면 이렇게 믿는 것을 삼가는 일을 뒷받침하는 증거인지? 달리 말해서, 그 대상이 파랗다는 것을 믿는 것, 또는 믿지 않는 것, 무엇이 우리의 궁극적 의무일 것인가? 이 물음에 대한 대답은 물론 그 상황의 구체적인 특성들에 달려있다. 아마도 그 한 장의 종이가 실제로 파랗다는 것을 시사하는 추가 증거나 또는 그 종이가 실제로 하얗다는 믿음을 뒷받침하는 추가 증거가 있을지도 모른다. 인식론적 비관주의자들은 증거가 상쇄될 수 있는 너무나 많은 상이한 방식들이 있기 때문에, 우리에게 그러한 모든 경우들에 대해서 우리가 믿음에 있어서 궁극적으로 정당화된다는 것이 무엇인지 말해주는 일반적 원칙들은 아무것도 없다고 말할 것이다. 일반적 원칙들의 방식으로 우리가 알 수 있는 모든 것은 한 대상의 푸르게 보임이 우리에게 그 대상이 푸르다는 믿음에 대한 직견적 정당화를 제공한다는 것과, 푸른 조명이 그 대상을 비추고 있다는 영향에 대한 증거가 우리에게 그 대상이 푸르다는 믿음을 삼가는 데에 대한 직견적 정당성을 제공한다는 것이다.

6) Is epistemological pessimism a plausible view? Of course we should not dogmatically rule out the possibility that an ingenious epistemologist could generate a list of epistemic principles with nonnormative antecedents or even an equivalence with a nonnormative analysans. However, advocates of epistemological pessimism could argue that as long as there is neither a monistic nor a pluralistic theory that is clearly acceptable, epistemological pessimism is at least indirectly confirmed.<25>
6) 인식론적 비관주의는 타당한 관점인가? 물론 우리는 기발한 인식론자가 비규범적 전제들을 가지고 인식 원칙들의 일람표를 산출할 수 있을 것이라거나 혹은 비규범적 가분석항을 가지고 동치까지도 만들어 낼 수 있을 것이라는 가능성을 독단적으로 배제시키지 않아야 할 것이다. 그렇지만, 인식론적 비관주의의 지지자들은 분명하게 허용가능한 일원론적 이론도 다원론적 이론도 없는 한, 인식론적 비관주의가 최소한 간접적으로나마 확정된다고 주장할 수 있을 것이다.

<2> In order to generate such series, we don't have to restrict ourselves to arithmetic. For example, there is an infinite number of things that are not in my office: elephants, vending machines, nuclear missiles, aircraft carriers, and so on. Thus we can easily form extremely long and cumbersome truths about my office that are certainly not matched by any existing sentence tokens.
<2> 그러한 연쇄를 산출하기 위해, 우리가 우리 자신을 산술에 제한해야 하는 것은 아니다. 예를 들어, 내 사무실에 없는 무한한 수의 것들이 있다. 즉 코끼리들, 자동판매기들, 핵 미사일들, 항공모함들 등등. 그래서 우리는 나의 사무실에 대해서 확실히 어떠한 현존하는 문장 표시들에 의해서도 일치되지 않는 극도로 길고 복잡한 진리들을 쉽사리 구성할 수 있다.

<4> In order to avoid a counterexample provided by modern medical technology, the concepts in question should be taken to refer to animals whose proper genetic design involves a heart and animals whose proper genetic design involves a liver.
<4> 현대 의학 기술에 의해 제공되는 반례를 피하기 위해서, 문제가 되는 개념들은 심장을 포함하는 온전한 유전자 지도의 동물들과 간을 포함하는 온전한 유전자 지도의 동물들을 가리키는 것으로 받아들여져야 한다.

<8> It would be a mistake to think that the project of defining epistemic concepts and the project of providing a criteriological analysis are mutually exclusive. In fact, the former is necessary for the latter. After all, how can we determine what the criteria for a given epistemic concept are if we are not clear on what we mean by that concept? …[the rest omitted]
<8> 인식적 개념들의 정의 기획과 규준적 분석 제공의 기획이 상호 배타적이라 생각하는 것은 잘못일 것이다. 사실상, 전자는 후자에 대해 필요조건이다. 무엇보다도, 만일 우리가 그 개념으로 의미하는 바가 명확하지 않다면 어떻게 우리가 주어진 인식적 개념에 대한 기준들이 무엇인지 결정할 수 있겠는가? [후략]

<11> The locus classicus for the weak supervenience thesis in ethics is the following statement by Richard Hare (1952): "Suppose that we say 'St. Francis was a good man.' It is logically impossible to say this and to maintain at the same time that there might have been another man placed exactly in the same circumstances as St. Francis, and who behaved in exactly the same way, but who differed from St. Francis in this respect only, that he was not a good man."(p.145).
<11> 윤리학에서의 수반 명제에 대한 표준구는 리처드 해어에 의한 다음 진술이다(1952). "성 프란체스코는 좋은 사람이었다'라고 우리가 말한다고 생각해 보자. 이렇게 말하면서 동시에 성 프란체스코와 정확히 똑같은 상황들에 위치된, 그리고 성 프란체스코와 정확히 같은 방식으로 행동하였던, 그러나 오직 좋은 사람이 아니었다는 점에서만 달랐던 그런 또 다른 사람이 있어 왔다고 주장하는 것은 논리적으로 불가능하다."(p.145).

<14> This is not to say that this reason is decisive. As we shall see later in this chapter, there are also good reason to doubt that the attempt to set forth a criteriological analysis of epistemic concepts can succeed.
<14> 이것은 이 이유가 결정적이라고 말하는 것이 아니다. 우리가 이 장에서 나중에 볼 것과 같이, 인식적 개념들에 대한 규준적 분석을 내놓고자 하는 시도가 성공할 수 있다는 것을 의심할 훌륭한 이유도 있다.

<17> Another example of a monistic analysis is ethical egoism, which asserts that an act is right if and only if it maximizes the agent's self-interest. Yet another example is the divine command theory, according to which an act is right if and only if it is commanded by God.
<17> 일원론적 분석의 또 다른 사례는 윤리적 유아주의이다. 윤리적 유아주의는 하나의 행위가 옳기 위한 필충조건은 그 행위가 행위자의 자기이익을 극대화시키는 것이라고 단언한다. 또 다른 예는 신성한 명령 이론이다. 그 이론에 따르면 하나의 행위가 옳기 위한 필충조건은 그 행위가 신에 의해 명령된다는 것이다.

<18> Reliable cognitive processes are those whose belief output enjoys a high ratio of true beliefs. …[The rest omitted].
<18> 신빙성 있는 인지 과정들은 그 과정들의 믿음 산출이 참인 믿음들에 대해 높은 비율을 갖는 과정들이다. [후략].

<20> The underlying metaphysical phenomenon here is this: if the property of being justified strongly supervenes on a set of normative properties, then there is a necessary coextension between that property and a set of normative properties. … In case of monistic theories, necessary coextension is obvious. According to the SS thesis, in every instance of a justified belief, the belief's justification is entailed by a nonnormative property N. Now, if there is one, and only one, such property N, it is clear that the property of being justified is equivalent to the property N. In the case of a Chisholm-type pluralistic theory, there is a necessary coextension as well. If there is a finite number of nonnormative base properties {N1, N2, Nn}, the property of being justified will be necessarily coextensive with the disjunctive property "N1 or N2 or Nn." If the number of base properties is infinite, the property of being justified will be necessarily coextensive with an infinite disjunction of base properties.
<20> 여기에서 기저를 이루는 형이상학적 현상은 이것이다. 즉 만일 정당화됨의 속성이 일련의 규범적 속성들에 강력하게 수반한다면, 그 속성과 일련의 규범적 속성들 사이에는 필연적 동연이 있을 것이다. …일원론적 이론들의 경우에, 필연적 동연은 명백하다. SS 명제에 따르자면, 정당화된 믿음의 모든 사례들에서, 그 믿음의 정당성은 비규범적 속성 N에 의해 함의된다. 이제, 만일 하나의 유일한 그러한 속성 N이 있다면, 정당화됨의 속성이 그 속성 N과 동치임을 분명하다. 치셜-유형 다원론적 이론의 경우, 마찬가지로 필연적 동연이 있다. 만일 한정된 수의 비규범적 기본 속성들 {N1, N2, Nn}이 있다면, 정당화됨의 속성은 필연적으로 선언적 속성 "N1 또는 N2 또는 Nn"과 동연일 것이다. 만일 기본 속성들의 수가 무한하다면, 정당화됨의 속성은 필연적으로 기본 속성들의 무한한 선언 명제와 동연일 것이다.

<22> In the current literature, reliabilism is often viewed as an example of the naturalistic approach to epistemology, whereas the Chisholmian approach is considered the epitome of nonnaturalism. See, for example, Maffie (1990). Such views are mistaken. Both reliabilism and the Chisholmia approach are examples of what we have called "criteriological analysis," and thus share significant metaphysical assumptions. In fact, reliabilism, as an instance of analytical monism, is committed to a more daring metaphysical claim than the endeavour of listing a variety of different nonnormative base properties.
<22> 현대적 용어로, 신빙론은 종종 인식론에 대한 자연주의적 접근방식으로 조망되곤 한다. 반면에 치셜주의 접근법은 비자연주의의 축도로 간주된다. 예를 들어 Maffie(1990)을 보라. 그러한 관점들은 오류이다. 신빙론과 치셜주의적 접근 모두 우리가 "규준적 분석"이라 부른 것의 사례들이고, 그래서 중대한 형이상학적 전제들을 공유한다. 사실상, 분석적 일원론의 사례로서 신빙론은 상이한 비규범적 기본 속성들의 다양을 열거하려는 노력보다 더욱 대담한 형이상학적 주장에 전념한다.

<25> Above, we referred to Chisholm's project of listing epistemic principles as an example of pluralistic criteriological analysis. Interestingly, though, Chisholm's principles, with one exception, are all prima facie principles: they all have epistemic qualifiers in their antecedents. In Theory of Knowledge (1989), Chisholm begins his list of ten epistemic principles (which he calls "material epistemic principles") with this one:

MP1    If the property of being F is self-presenting, if S is F, and if S believes himself to be F, then it is certain for S that he is F.

Assuming that the concepts of self-presentation and belief are nonnormative, MP1 would be an example of criteriological analysis, having a descriptive antecendent and an epistemic term in its consequent. However, Chisholm's second principle,

MP2    Accepting h tends to make h probable.

does not tell us what is probable, but only what is prima facie probable. And his third principle

MP3    If S accepts h and if h is not disconfirmed by S's total evidence, then h is probable for S

is similar to our principle P2a. Although it does provide us with a condition that entails the epistemic status of being probable, it specifies this condition using a concept of epistemic evaluation: that of a proposition's being disconfirmed by S's total evidence. The same holds for the remaining seven principles: they all contain epistemic concepts in their antecedents. It would appear, therefore, that, with the exception of MP1, Chisholm's principles are a version of epistemological pessimism.
<25> 위에서, 우리는 다원주의적인 규준적 분석의 일례로 인식 원칙들을 열거하려는 치셜의 기획을 언급하였다. 흥미롭게도, 그렇다 할지라도 치셜의 원칙들은 하나의 예외만을 두고 모두 직견적 원칙들이다. 그 원칙들은 모두 그것들의 전제들에 인식적 수식어구들을 가지고 있다. 『앎의 이론』(1989)에서 치셜은 열 가지 인식적 원칙들(그가 "질료적 인식 원칙들"이라 부르는)에 대한 그의 목록을 이와 같은 원칙을 가지고 시작한다.

MP1    만일 F임의 속성이 자기현전적이고, 만일 S가 F이며, 또한 만일 S가 그 자신을 F라고 믿는다면, S에게 그가 F임은 확실하다.

자기현전에 대한 개념과 믿음이 비규범적이라는 전제에서, MP1은 기술적 전제를 가지고 그 결론에서 인식적 용어를 가지는 규준적 분석의 일례일 것이다. 그렇지만 치셜의 두 번째 원칙,

MP2    h를 허용하는 것은 h를 개연적으로 만드는 경향이 있다.

이것은 우리에게 개연적인 것이 무엇인지 말해주지 않고, 오로지 무엇이 직견적으로 개연적인지만을 말해준다. 그리고 그의 세 번째 원칙,

MP3    만일 S가 h를 허용하고 또 만일 h가 S의 믿음 총체에 의해 거부되지 않는다면, h는 S에 대해 개연적이다

이것은 우리의 P2a 원칙과 유사하다. 비록 MP3가 우리에게 개연적임의 인식적 상태를 함의하는 조건을 제공한다 할지라도, 그것은 이 조건을 인식적 평가 개념을 사용하여 상술한다. 즉 S의 증거 총체에 의한 한 명제의 거부됨에 대한 개념을 사용한다. 동일한 점이 남은 일곱 가지 원칙들에 대해서도 지속된다. 즉 그것들은 모두 그 전제들에 인식적 개념들을 포함한다. 그러므로 MP1을 예외로 하여 치셜의 원칙들은 인식론적 비관주의의 한 형태임이 드러날 것이다.

-蟲-
Weak and Strong Supervenience
약한 수반성과 강한 수반성

1) To understand the significance of supervenience in epistemology, it is necessary to distinguish between weak and strong supervenience. According to the weak version of the supervenience doctrine, it would be incoherent to say that your belief is justified, but another belief just like yours is not. Suppose a friend of yours joins you on your walk. She sees the dog, too, and thus believes, just as you do, that there is a dog over there. Let us suppose that her belief and your belief share all the relevant nonnormative characteristics. Given this assumption, it would be capricious to say that your belief is justified while hers is not, or vice versa. Given that the two beliefs are alike in all their relevant nonnormative features, either both are justified or neither is.
1) 인식론에서 수반성의 의미를 이해하기 위해서, 약한 수반성과 강한 수반성 사이에 구분을 지을 필요가 있다. 약한 형태의 수반 원칙에 따르자면, 당신의 믿음이 정당화되지만 당신의 믿음과 꼭 같은 다른 믿음은 정당화되지 않는다고 말하는 것은 일관성이 없는 것일 터이다. 당신의 친구가 당신이 걷는 길에서 당신과 함께 한다고 생각해 보자. 그녀도 마찬가지로 그 개를 보고, 그래서 바로 당신이 믿는 것처럼 저기 개 한 마리가 있다고 믿는다. 그녀의 믿음과 당신의 믿음이 모든 적절한 비규범적 특징들을 공유한다고 가정해 보자. 이러한 전제를 고려하면, 당신의 믿음은 정당화되는 반면에 그녀의 믿음은 그렇지 않다거나 혹은 그 역이 성립한다고 말하는 건 변덕스러운 일이 될 것이다. 그 두 가지 믿음들이 그 믿음들의 적절한 비규범적 특징들에 있어서 유사하다는 것을 고려해 보면, 둘 다 정당화되든지 혹은 둘 다 정당화되지 않는다.

2) This point can be stated by saying that epistemic judgments must be universalizable, which means that, when we judge a belief B to be justified, there must be a reason for doing so that is equally applicable to all beliefs that are just like B. In short, we must judge like beliefs alike; they must be either all justified or all unjustified. The doctrine of weak supervenience, then, expresses the requirement that epistemic judgments must be universalizable.<11> It asserts that if two beliefs are alike in their nonnormative properties, they must also be alike in epistemic satus - there can't be an epistemic difference without a corresponding nonepistemic difference.
2) 이 점은 인식적 판단들이 반드시 보편화가능적이어야만 한다고 말함으로써 진술될 수 있다. 그것은 즉 우리가 하나의 믿음 B를 정당화되는 것으로 판단할 때, 반드시 그렇게 하는 데에 대한 이유가 있어야만 하고 그 이유는 똑같이 B와 꼭 같은 모든 믿음들에 적용가능해야 한다는 뜻이다. 간단히 말해서, 우리는 닮은 믿음들을 똑같이 판단해야만 한다; 그러한 믿음들은 반드시 모두 정당화되든지 혹은 모두 정당화되지 않아야만 한다. 약한 수반 원칙은 그래서 인식적 판단들이 반드시 보편화가능적이어야만 한다는 조건을 표현한다.<11> 그 조건은 만일 두 믿음들이 그 믿음들의 비규범적 특성들에 있어서 유사하다면, 그 믿음들은 또한 반드시 인식적 상태에 있어서도 유사해야만 한다고 - 상응하는 비인식적 차이와 무관하게 인식적 차이는 있을 수 없다고 - 주장한다.

3) We will define the concept of weak supervenience as follows (letting J stand for the property of being justified, and N for the family of nonnormative base properties on which J supervenes):<12>

Weak Supervenience
J weakly supervenes on N if and only if, necessarily, whenever a belief has property J, there is property P in N such that whenever a belief has P, it has J.

The concept of weak supervenience must be distinguished from the assertion of weak supervenience. Let the term "WS thesis" refer to the assertion that the property of being justified weakly supervenes on nonnormative properties. When we apply the WS thesis to our example of a justified belief, it tells us this: Given that your belief that there is a dog over there is justified, there must be a nonnormative property (or set of properties) such that all beliefs having it are justified. Consequently, it can't be that two beliefs share this property, and yet one of them is justified while the other one is not. Put differently: There can't be a difference in epistemic status without a corresponding nonepistemic difference.
3) 우리는 약한 수반성의 개념을 다음과 같이 정의할 것이다 (J를 정당화된 것의 특성을 의미하도록 두고, N을 J가 그에 수반하는 비규범적 기본 특성들의 집단을 의미하도록 두면서):<12>

약한 수반성
J는 N에 약하게 수반한다 iff 필연적으로 한 믿음이 특성 J를 지닐 때에는 언제든, N 안에는 한 믿음이 P를 가지는 그 어느 때에든 그 믿음이 J를 가지는 특성 P가 있다.

약한 수반성의 개념은 반드시 약한 수반성의 주장과 구별되어야만 한다. "WS 명제"라는 용어를 정당화된 것의 특성이 비규범적 특성들에 약하게 수반한다는 주장을 말한다고 하자. 우리가 WS 명제를 정당화된 믿음에 대한 우리의 예시에 적용시킬 때, 그 명제는 우리에게 이와 같은 것을 말해준다. 즉 저기에 개가 있다는 당신의 믿음이 정당화된다는 것을 고려하면, 반드시 비규범적 특징 (또는 일련의 특징들) 이, 그 특징을 가지는 그러한 모든 믿음들이 정당화되는 그러한 비규범적 특징이 있어야만 한다. 결론적으로, 두 믿음들이 이러한 특징을 공유하지만, 그런데도 그 믿음들 중 하나는 정당화되는 반면 나머지 하나는 그렇지 않을 수는 없다. 달리 말하자면, 상응하는 비인식적 차이 없이 인식적 상태의 차이는 있을 수 없다.

4) Some philosophers would argue that the WS thesis is too weak. They would maintain that although the WS thesis goes a long way toward providing us with an adequate understanding of epistemic normativity, it falls short of satisfying a further important intuition about the nature of epistemic evaluation. This intuition will become clear if we digress into ethics. The ethical equivalent to the WS thesis asserts that, necessarily, whenever an act has a moral property M, there is a nonnormative property P such that any act having P has M. Let us apply this principle to Jeffrey Dahmer's horrible crimes: acts of kidnapping and killing boys and young men for the sake of gratifying a perverse lust for murder and domination. The ethical version of the WS thesis, expressing the requirement of universalizability, asserts that we must judge any act like that, irrespective of who commits it, in the same way we judge Jeffrey Dahmer's acts. However, it does not tell us how we must judge Jeffrey Dahmer's acts. Thus, if we were perverted enough to call his acts noble, and if we were to call noble all acts like his, then we would meet the requirement of universalizability. Philosophers who consider the ethical version of the WS thesis too weak would say, however, that surely we believe acts like Dahmer's are necessarily wicked, and thus could not possibly be noble. They would deny that there is a possible world in which someone commits acts like Dahmer's and does something noble. The WS thesis, however, allows for such a world. It merely tells us that if in any world W in which Dahmer's acts are noble, all like acts in W are noble as well.
4) 일부 철학자들은 WS 명제가 지나치게 약하다고 주장할 것이다. 그들은 WS 명제가 우리에게 인식적 규범성에 대한 충분한 이해를 제공하는 데에 많은 도움이 된다 할지라도, 그 명제는 인식적 평가의 본성에 대한 더욱 중요한 직관을 만족시키기에는 부족하다고 주장할 것이다. 이러한 직관은 만일 우리가 윤리학으로 말을 돌린다면 분명해질 것이다. WS 명제의 윤리학적 대응 명제는 다음과 같이 주장할 것이다. 필연적으로, 하나의 행위가 도덕적 특성 M을 지닐 때에는 언제라도, P를 가지는 어떤 행위이든 M을 가지는 그러한 비규범적 특성 P가 있다는 것이다. 이 원칙을 제프리 다머의 끔찍한 범죄들에 적용시켜 보도록 하자. 그 범죄들은 곧 살인과 지배에 대한 이상성욕의 충족을 위한 아동 및 청년 납치 살해 행위들이다. 보편화가능성 조건을 표현하는 WS 명제의 윤리학적 변형은 우리가 반드시 그와 같은 어떤 행위이든, 그 행위를 누가 저질렀든 상관 없이, 우리가 제프리 다머의 행위들을 판단하는 방식과 동일한 방식으로 판단해야만 한다고 단언한다. 그렇지만, 그 명제는 우리에게 우리가 제프리 다머의 행위들을 어떻게 판단해야만 하는지 말해주지 않는다. 그래서, 만일 우리가 그의 행위를 고결하다고 할 만큼 충분히 비뚤어졌다면, 그리고 우리가 그의 행위들과 유사한 모든 행위들을 고결하다고 부르게 된다면, 우리는 보편화가능성 조건을 충족시킬 것이다. 그렇지만 WS 명제의 윤리학적 변형이 지나치게 약하다고 고찰하는 철학자들은 확실히 우리가 다머의 행위들과 같은 행위들이 필연적으로 사악하다고 믿고, 그래서 고귀하다고 믿을 가능성은 있을 수 없다고 말할 것이다. 그들은 누군가 다머와 비슷한 일들을 저지르면서 어떤 고귀한 일을 하는 그러한 가능세계가 있다는 것을 부정할 것이다. 그렇지만 WS 명제는 그러한 세계를 허용한다. 그 명제는 단지 우리에게 만일 다머의 행위들이 고귀한 어떤 세계 W에서라면, W에서 그와 유사한 모든 행위들도 마찬가지로 고귀하다고 말해줄 따름이다.

5) In epistemology, the WS thesis raises a parallel problem. Consider again your and your friend's belief that there is a dog over there. The WS thesis tells us that if we call your belief justified, then we must call your friend's belief - and indeed all beliefs that are like your belief in their relevant features - justified. Likewise, it tells us that if we call your belief unjustified, then we must call your friend's belief unjustified, and indeed all beliefs that share their relevant nonnormative properties with your belief. The WS thesis does not tell us, however, that the beliefs in question, given their nonepistemic properties, must be justified. Rather, it allows for possible worlds in which such beliefs are unjustified. According to those who think the WS thesis is too weak, this is objectionable, for they take it that given the descriptive properties of these beliefs, they could not possibly be unjustified.
5) 인식론에서, WS 명제는 병렬문제를 일으킨다. 다시 한 번 저기 개 한 마리가 있다는 당신의 믿음과 당신 친구의 믿음을 고찰해 보자. WS 명제는 우리에게 만일 우리가 당신의 믿음을 정당화된다 한다면, 우리는 반드시 당신 친구의 믿음 - 덧붙여서 그 관계된 특징들에 있어서 당신의 믿음과 유사한 모든 믿음들 - 도 정당화된다고 해야만 한다. 같은 방식으로, 그 명제는 우리에게 만일 우리가 당신의 믿음을 정당화되지 않는다고 말한다면, 우리는 반드시 당신 친구의 믿음도, 그리고 그 관계된 비규범적 특징들을 당신의 믿음과 공유하는 모든 믿음들도 정당화도지 않는다고 해야만 한다. 그렇지만 WS 명제는 우리에게 문제가 되는 믿음들이, 그 믿음들의 비인식적 특징들을 고려해 보면, 반드시 정당화되어야 한다는 것을 말해주지는 않는다. 오히려, 그 명제는 그러한 믿음들이 정당화되지 않는 가능세계들을 허용한다. WS 명제가 지나치게 약하다고 생각하는 사람들에 따르면, 이 점은 반박가능한데, 왜냐하면 그들은 그 명제를 이러한 믿음들의 기술적 특징들을 고려해 보면, 그 믿음들이 정당화되지 않을 가능성은 있을 수 없다고 받아들이기 때문이다.

6) If we wish to modify the definition of weak supervenience so as to ensure this result, we must add the modal operator "necessarily" before the last clause. Thus we get a special case of what Kim calls "strong supervenience":

Strong Supervenience
J strongly supervenes on N: necesarily, whenever a belief has property J, there is property P in N such that, necessarily, whenever a belief has P, it has J.

Let the term "SS thesis" stand for the claim that the property of being justified strongly supervenes on nonnormative properties. The SS thesis tells us this: Whenever a belief is justified, there must be a nonnormative property that entails the property of justification. A belief that has this property could not possibly fail to be justified. Thus, if the SS thesis is true, there is no possible world in which a belief has the same nonnormative features as your belief that there is a dog over there and yet is unjutified.
6) 만일 우리가 이러한 결과를 보증하도록 약한 수반성에 대한 정의를 수정하고자 바란다면, 우리는 반드시 서법 기능어 "필연적으로"를 마지막 절 앞에 덧붙여야만 한다. 그래서 우리는 김재권이 "강한 수반성"이라 부르는 것에 대한 특수한 사례를 얻는다.

강한 수반성
J는 강력하게 N에 수반한다. 즉 필연적으로, 하나의 믿음이 특성 J를 가지는 때라면 언제든, N 안에는, 필연적으로, 한 믿음이 P를 가진다면 언제든 그 믿음이 J를 가지는 그러한 속성 P가 있다.

"SS 명제"가 정당화된 것의 속성은 강력하게 비규범적 속성들에 수반한다는 주장을 뜻하도록 하자. SS 명제는 우리에게 이와 같은 것을 말해준다. 즉 한 믿음이 정당화되는 때라면 언제든, 반드시 정당화의 속성을 함의하는 비규범적 속성이 있어야만 한다. 이러한 속성을 지니는 한 믿음은 정당화되지 않을 가능성이 있을 수 없다. 그래서, 만일 SS 명제가 참이라면, 한 믿음이 저기에 개 한 마리가 있다는 당신의 믿음처럼 동일한 비규범적 특성들 을 가지는데에도 정당화되지는 않는 어떠한 가능세계도 없다.

7) Consider again the ethical analogy. The ethical equivalent to the SS thesis, according to which moral properties strongly supervene on nonmoral properties, asserts that whenever an action has a moral property M, there must be a nonnormative property P that entails M. When we apply this thesis to the Jeffrey Dahmer example, it tells us that the wickendness of his acts supervenes on a nonnormative property that is a necessitating ground of wickedness. Put differently, the thesis that moral properties strongly supervene on natural properties tells us that there are no possible worlds in which acts like those committed by Jeffrey Dahmer fail to be wicked.
7) 다시 윤리학적 유추를 고찰해 보도록 하자. SS 명제의 윤리학적 대응 명제, 그에 따르면 도덕적 특성들은 강력하게 무도덕적 특성들에 수반하는 그 대응 명제는 하나의 행위가 도덕적 특성 M을 가지는 때라면 언제든, 반드시 M을 함의하는 비규범적 특성 P가 있어야만 한다. 우리가 이 명제를 제프리 다머 사례에 적용할 때, 그 명제는 우리에게 그의 행위에 속하는 악함은 악함의 근거를 필연적으로 동반하는 것인 하나의 비규범적 특성에 수반한다고 말해준다. 달리 말하면, 도덕적 특성들이 강력하게 자연적 특성들에 수반한다는 그 명제는 우리에게 제프리 다머에 의해 저질러진 것과 같은 그러한 행위들이 사악하지 않은 어떠한 가능세계도 없다고 말해준다.

8) Back to epistemology: Let's consider the example involving your justified belief that there is a dog over there. Regarding that belief (let's call it B), the SS thesis asserts that the conjunction of its nonnormative properties entails that it is justified. There is no possible world in which a belief B* shares its descriptive characteristics with B and fails to be justified. Thus the SS thesis is considerably stronger than the WS thesis. Whereas the WS thesis merely asserts that in any possible world in which B is justified, all beliefs like it are justified, the SS thesis tells us that beliefs like B are justified in all possible worlds.
8) 인식론으로 되돌아 가서, 저기 개 한 마리가 있다는 당신의 정당화된 믿음을 포함하는 사례를 고찰해 보도록 하자. 그 믿음(그 믿음을 B라고 부르도록 하자)과 관련하여, SS 명제는 그 믿음의 비규범적 특성들의 결합은 그 믿음이 정당화된다는 것을 함의한다고 단언한다. 믿음 B*가 그 믿음의 기술적 특징들을 B와 공유하면서 정당화되지 않는 어떠한 가능 세계도 없다. 그래서 SS 명제는 WS 명제보다 상당히 더 강력하다. WS 명제가 단지 B가 정당화되는 어떠한 가능세계에서든, B와 비슷한 모든 믿음들은 정당화된다고만 주장하는 반면에, SS 명제는 우리에게 B와 같은 명제들은 모든 가능세계들에서 정당화된다고 말한다.


The Case for Strong Supervenience
강력한 수반성에 대한 사례

1) Should we accept the doctrine that a belief's justification supervenes on its nonnormative properties? If we were to reject this doctrine, then we would have to deny either the WS thesis or the SS thesis. Let us see what such denials would involve.
1) 우리는 한 믿음의 정당성이 그 믿음의 비규범적 특성들에 수반한다는 원칙을 허용하여야 하는가? 만일 우리가 이 원칙을 거절하게 된다면, 우리는 WS 명제나 SS 명제를 부정해야 할 것이다. 그러한 부정들이 수반할 것을 보도록 하자.

2) In order to discuss what it means to deny the WS thesis, consider again the example in which you and your friend see a dog run across the lawn, and thus both of you believe that there is a dog over there. We stipulated that this belief is a paradigm example of what we would consider a justified belief. We also stipulated that there is no relevant difference between your believing that there is a dog over there and your friend's believing that there is a dog over there. According to the WS thesis, your belief's epistemic status of being justified must be accompanied by a nonnormative property P such that any belief having P is justified.
2) WS 명제를 부정한다는 것이 의미하는 바를 논의하기 위해, 다시 당신과 당신 친구가 개 한 마리가 잔디밭을 가로질러 달려가는 것을 보고, 그래서 당신과 친구 모두 저기 개 한 마리가 있다고 믿는 그 사례를 고찰해 보도록 하자. 우리는 이 믿음이 우리가 정당화된 믿음으로 여길 믿음의 전형적인 예라고 규정하였다. 또한 우리는 개 한 마리가 저기 있다는 당신의 믿음과 개 한 마리가 저기 있다는 당신 친구의 믿음 사이에 어떠한 관련된 차이점도 없다고 규정하였다. WS 명제에 따르면, 정당화됨에 대한 당신 믿음의 인식적 상태는 반드시 P를 가진 어떠한 믿음이든 정당화되는 그러한 비규범적 특성 P에 의해 동반되어야만 한다.

3) This calim can be attacked in two ways. First, it might be asserted that it's possible that your belief is justified while your friend's is not, and vice versa. But if that were possible, then it would be possible for two beliefs to differ in epistemic status for no reason whatever. If we were to ask, Why do they differ in epistemic status? there would be no other answer than: Well, they just do. Certainly this would not be a very appealing view of epistemic normativity.
3) 이러한 주장은 두 가지 방식으로 반격받을 수 있다. 첫째로, 아마도 당신의 믿음이 정당화되는 반면 당신 친구의 믿음은 정당화되지 않는, 그리고 그 역이 성립하는 가능성이 주장될 수 있을 것이다. 그러나 만일 그것이 가능하다면, 그 두 믿음들이 무엇이 되었든 아무런 이유도 없이 인식적 상태에 있어서 상이하다는 것이 가능할 것이다. 만일 우리가 어째서 그 믿음들이 인식적 상태에 있어서 상이한지 묻게 된다면 "네, 그것들이 그냥 그렇네요."라는 대답 외에 다른 어떠한 대답도 없을 것이다. 확실히 이것은 그렇게 호소력 있는 인식적 규범성에 대한 관점은 아닐 것이다.

4) Second, the WS thesis might be attacked by asserting that while a difference in epistemic status requires a nonepitsemic difference, this difference need not be nonnormative. For example, it could be argued that differences in epistemic status are grounded in differences in moral status. The problem with this suggetion is that a difference in moral status, being a normative difference itself, is just as much in need of an explanation as the difference in epistemic status it is supposed to explain. Ultimately, we are seeking a nonnormative difference that explains the difference in epistemic status. We must conclude, therefore, that there are good reasons for endorsing a supervenience thesis that is at least as strong as the WS thesis.
4) 둘째로, WS 명제는 인식적 상태에서의 상이성이 비인식적 상이성을 요청하고, 이 상이성은 비규범적일 필요가 없다고 주장함으로써 반격받을 수도 있다. 예를 들어, 인식적 상태에 있어서의 상이성들이 도덕적 상태에 있어서의 상이성들에 기초된다고 주장될 수 있다. 이러한 제안과 관련된 문제는 도덕적 상태에 있어서의 차이, 그 자체로 규범적 차이인 그것이 바로 설명해야 할 것으로 생각되는 인식적 상태에 있어서의 차이만큼이나 설명이 필요하다는 점이다. 궁극적으로, 우리는 인식적 상태에 있어서의 차이를 설명하는 비규범적 차이를 추구하고 있다. 우리는 그러므로 최소한 WS 명제 만큼은 강한 수반성 명제를 지지하기 위한 훌륭한 이유들이 있다고 결론내려야만 한다.

5) Let us now see what is involved in denying the SS thesis. If we were to accept the WS thesis, but not the SS thesis, then we would have to accept that there are pairs of possible worlds W1 and W2 such that although there are no relevant nonnormative differences between them,

(i) all beliefs that are justified in W1 are unjustified in W2;
(ii) in W1, all beliefs are jutified, while in W2 all beliefs are unjustified;
(iii) in W1, all beliefs formed in the hour of 12:00 to 1:00 p.m., and no other beliefs, are justified, while in W2 all beliefs formed in the hour of 12:00 to 1:00 p.m., and no other beliefs, are unjustified.

Advocates of the SS thesis would say that such possibilities are absurd: there are no possible wordls that correspond to these descriptions. The WS thesis, However, allows for such worlds, for the WS thesis does not demand that the nonepistemic property on which a belief's justification supervenes be such that it entails the property of being justified. Consequently, the WS thesis permits possible worlds in which epistemic properties are linked to nonepistemic properties in the most bizarre ways.<13>
5) 이제 SS 명제를 부정함에 있어서 수반되는 것을 살펴 보도록 하자. 만일 우리가 WS 명제를 허용하지만, SS 명제는 허용하지 않게 된다면, 우리는 다음과 같은 가능 세계들 W1과 W2의 한 쌍이 있다는 것을 허용해야 할 것이다. 그 두 세계들은 비록 관련된 비규범적 차이들이 그 세계들 사이에 없다 할지라도,

(i) W1에서 정당화되는 모든 믿음들은 W2에서 정당화되지 않고,
(ii) W1에서 모든 믿음들이 정당화되는 반면 W2에서 모든 믿음들은 정당화되지 않으며,
(iii) W1에서는 다른 어떠한 믿음들도 아니라 정오부터 오후 1시 안에 형성된 모든 믿음들만 정당화되는 반면, W2에서는 다른 어떠한 믿음들도 아니라 정오부터 오후 1시 안에 형성된 모든 믿음들만 정당화되지 않는다.

SS 명제의 지지자들은 그러한 가능성들이 부조리하다고 말할 것이다. 즉 이러한 기술들에 부합하는 어떠한 가능세계들도 없다고 말할 것이다. 그렇지만 WS 명제는 그러한 세계들을 허용하는데, 왜냐하면 WS 명제는 한 믿음의 정당성이 그에 수반하는 그러한 비인식적 특성이 정당화됨의 특성을 함의하는 그러한 것이기를 요구하지 않기 때문이다. 결론적으로, WS 명제는 인식적 특성들이 가장 기이한 방식들로 비인식적 특성들과 결부되는 그러한 가능세계들을 허용한다.<13>

6) Opponents of the SS thesis might wonder whether we shouldn't concede that there are possible wordls as bizarre as those mentioned above. Such worlds are certainly crazy, but why should that mean they don't exist? The weakness of this objection becomes apparent as soon as we expand our view to include morality. If we were to assert that moral properties supervene only weakly on nonnormative properties, then we would have to accept that there are pairs of possible wordls W1 and W2 such that although there are no relevant nonnormative differences between them,

(i) all acts that are right in W1 are wrong in W2;
(ii) in W1, all acts (no matter how cruel) are right, while in W2, all acts (no matter how beneficent) are wrong;
(iii) in W1, all acts performed in the hour of 12:00 to 1:00 p.m., and no other acts, are right, while in W2 all acts performed in the hour of 12:00 to 1:00 p.m., and no other acts, are wrong.

Anyone who denies strong supervenience for moral properties must be prepared to assert that it is possible for an act of unspeakable cruelty to be right simply because it was performed during the hour between 12:00 and 1:00 p.m. Such an assertion, however, seems absurd. The property of being performed in the hour between 12:00 and 1:00 p.m. is not a right-making characteristic: it is not possible for such an act to be right simply because it was performed at that time. If, however, we are prepared to accept, on the basis of such considerations, that moral properties strongly supervene on nonnormative properties, it would seem to follow that we should accept the strong supervenience thesis for epistemic properties as well.
6) SS 명제의 반대자들은 우리가 위에서 언급한 것들 만큼이나 기괴한 가능 세계들이 있다는 것을 인정해야 하는지 궁금해할 수 있을 것이다. 그러한 세계들은 확실히 말도 안 되지만, 어째서 그게 그 세계들이 존재하지 않는다는 뜻이 되는가? 이 반박의 약점은 우리가 우리의 관점을 도덕성을 포함하는 데에까지 확장시키자마자 명백해진다. 만일 우리가 도덕적 특성들이 오로지 약하게만 비규범적 특성들에 수반한다고 주장하게 된다면, 우리는 우리는 다음과 같은 가능 세계들 W1과 W2의 한 쌍이 있다는 것을 허용해야 할 것이다. 그 두 세계들은 비록 관련된 비규범적 차이들이 그 세계들 사이에 없다 할지라도,

(i) W1에서 옳은 모든 행위들은 W2에서 그르고,
(ii) W1에서 모든 행위들은(얼마나 잔인한지는 상관 없이) 옳은 반면, W2에서 모든 행위들은(얼마나 유익한지와 무관하게) 그르며,
(iii) W1에서는 다른 어떠한 행위들도 아니라 정오부터 오후 1시 안에 수행된 모든 행위들만 옳은 반면, W2에서는 다른 어떠한 행위들도 아니라 정오부터 오후 1시 안에 수행된 모든 행위들만 그르다.

도덕적 특성들에 대한 강한 수반성을 부정하는 누구든지 이루 말할 수 없는 잔학행위에 속하는 행위가 단순히 정오와 오후 1시 사이 시간 안에 수행되었기 때문에 옳은 것일 가능성이 있다고 주장할 준비가 되어 있어야만 한다. 그렇지만 그러한 단언은 부조리한 듯하다. 정오와 오후 1시 사이 시간 안에 수행되었음의 속성은 옳게 만드는 특징이 아니다. 즉 단순히 그 행위가 그 시간에 수행되었기 때문만으로 그러한 한 행위가 옳은 것일 가능성은 없다. 그렇지만 만일 그러한 고찰들을 기초로 하여 도덕적 속성들이 비규범적 속성들에 강력하게 수반한다는 것을 허용할 준비가 되어 있다면, 우리가 인식적 속성들에 대한 강한 수반성 명제 또한 허용해야 할 것이라는 점이 뒤따를 것으로 보일 것이다.


Analytical Monism and Pluralism
분석적 일원론과 다원론

1) If we agree that a belief's epistemic status supervenes on certain of its nonnormative properties, obviously we would like to know what these properties are. Thus if we accept the supervenience doctrine, we have a good reason to conceive of the analysis of epistemic justification in the way Alvin Godman suggests in the passage cited earlier: as the task of specifying the nonnormative properties that are both necessary and sufficient for epistemic justification.<14> Such a project can be carried out in two ways. First, it might be argued that there is one, and only one, nonevaluative property on which the property of being justified supervenes. Second, it might be argued that there are many such properties. We shall call examples of the first approach "monistic" theories, and examples of the second "pluralistic" theories.
1) 만일 우리가 한 믿음의 인식적 상태는 그 믿음의 비규범적 속성들 중 어떤 것들에 수반한다는 데에 동의한다면, 분명히 우리는 이러한 속성들이 무엇인지 알고 싶어할 것이다. 따라서 만일 우리가 수반 원칙을 허용한다면, 우리는 앞서 인용된 구절에서 앨빈 골드만이 제안한 방식으로 인식적 정당성에 대한 분석을 염두에 둘 훌륭한 이유를 가진다. 즉 인식적 정당성의 필요충분조건인 비규범적 특성들을 상술하는 과업으로서 말이다.<14> 그러한 하나의 기획은 두 가지 방식으로 완수될 수 있다. 첫째로, 정당화됨의 속성이 그에 수반하는 한 가지의 유일한 비평가적 속성이 있다고 주장될 수 있을 것이다. 둘째로, 그러한 여러 특성들이 있다고 주장될 수도 있을 것이다. 우리는 처 번째 접근방식의 예들을 "일원론적" 이론들이라 부를 것이고, 두 번째에 속하는 예들을 "다원론적" 이론들이라 부를 것이다.

2) In ethics, the difference between these two types of theories is nicely illustrated by utilitarianism, on the one hand, and W.D. Ross's theory of prima facie duties, on the other.<15> As G.E. Moore aptly puts it, utilitarianism is an answer to the following question: "What charateristic is there which belongs to all voluntary actions which are right, and only to those among them which are right?"<16> According to utilitarianism, the characteristic in question is the maximization of utility. Utilitarians believe that there is one unique property all right acts have in common: that of maximizing utility.<17> W.E. Ross, however, rejects the idea that there is just one "right-making" characteristic; he believes that there are many diffrenct ones - gratitude, justice, and beneficence, for example.
2) 윤리학에 있어서, 이러한 두 가지 유형의 이론들 사이에서의 차이는 한편으로 실용주의에 의해, 다른 한편으로는 W.D. 로스의 직견적 의무들에 대한 이론에 의해 훌륭하게 설명된다.<15> G.E. 무어가 적절히 풀이하였듯, 실용주의는 다음 물음에 대한 하나의 대답이다. "모든 옳은 자발적 행위들에 속하는, 그리고 오로지 그 행위들 중에서 옳은 그러한 행위들에만 속하는 어떤 특징이 있는가?"<16> 실용주의에 따르자면, 문제시되는 특징은 공리의 최대화이다. 실용주의자들은 모든 옳은 행위들이 공통으로 지니는 하나의 유일한 특성이 있다고 믿는다.<17> 그렇지만 W.E. 로스는 단지 하나의 "옳게 만드는" 특징만이 있다는 생각을 거부한다. 그는 여러 상이한 특징들이 - 예를 들어 감사, 정의, 그리고 유익성 - 있다고 믿는다.

3) In epistemology, a prominent example of the monistic approach is reliabilism, according to which there is one unique nonnormative property that makes beliefs justified: the property of being produced by a reliable cognitive process.<18> Reliabilism, in its simplest form, asserts the following:

SR      S is justified in believing that p if and only if S's belief that p is produced by a reliable cognitive process.

Roderick Chisholm champions the pluralistic approach. According to him, the project of analyzing the concept of justification must be carried out by producing a list of epistemic principles.<19> An epistemic principle is a conditional that (i) is supposed to express an entailment and (ii) may not contain any normative terms in the antecedent. Thus the antecedent of an epistemic principle is meant to state a nonnormative base property that entails the property of being justified. Any belief having such a property would be a justified belief.
3) 인식론에 있어서, 일원론적 접근법의 중요한 예는 신빙론이다. 그에 따르면 믿음들을 정당화되도록 만드는 하나의 유일한 비규범적 특성이 있다. 그것은 즉 신뢰할 만한 인식 과정에 의해 산출되었음의 속성이다.<18> 가장 단순한 형태로서 보자면 신빙론은 다음과 같이 단언한다:

SR      S는 p라고 믿음에 있어서 정당화된다 iff S의 p라는 믿음은 믿을 만한 인식 과정에 의해 산출된다.

Roderick Chisholm은 다원론적 접근법을 옹호한다. 그에 따르면, 정당화의 개념을 분석하는 기획은 인식 원칙들의 목록을 산출함으로써 수행되어야만 한다.<19> 인식 원칙은 (i) 수반을 설명하는 것으로 생각되고 (ii) 전제 안에 어떠한 규범적 용어들도 포함하지 않을 하나의 조건문이다. 그래서 인식 원칙의 전제는 정당화됨의 속성을 함의하는 비규범적 기본 속성을 진술하는 것을 의미한다. 그러한 속성을 지닌 어떠한 믿음이든 정당화된 믿음일 것이다.

4) Consider the following three nonnormative base properties - being grounded in (i) introspection, (ii) perception, and (iii) memory - and assume that these are just some among many properties on which epistemic justification supervenes. The following is a simplified version of a criteriological analysis of epistemic justification:

P1    If a belief is grounded in introspection, then it is justified.
P2    If a belief is grounded in perception, then it is justified.
P3    If a belief is grounded in memory, then it is justified.

We shall refer to this analysis as a "Chisholm-type" analysis because it is a simplified model of the kind of theory Chisholm has proposed. Now since none of these three base properties on our list entails the property of justification, P1-P3 are all false. However, here we are not concerned with which principles are correct, but only with how the monistic and the pluralistic approaches differ from each other, and what they have in common.
4) 다음 세 가지 비규범적 기본 속성들을 - (i) 내성, (ii) 지각, 그리고 (iii) 기억에 기초된 - 고찰하고 이러한 속성들이 인식적 정당성이 그에 수반하는 여러 속성들 중 단지 일부일 뿐이라고 생각하도록 하자. 다음은 인식적 정당성에 대한 규준적 분석의 단순화된 형태이다.

P1    만일 한 믿음이 내성에 기초된다면, 그 믿음은 정당화된다.
P2    만일 한 믿음이 지각에 기초된다면, 그 믿음은 정당화된다.
P3    만일 한 믿음이 기억에 기초된다면, 그 믿음은 정당화된다.

우리는 이러한 분석을 "치셜-유형" 분석으로 언명할 것인데 왜냐하면 그 분석이 치셜이 제안하였던 그러한 종류의 이론의 단순화된 모형이기 때문이다. 이제 우리의 목록에서 이러한 세 가지 기본 속성들 중 어느 것도 정당화의 속성을 함의하지 않으므로, P1에서 P3까지는 모두 거짓이다. 그렇지만, 여기에서 우리는 어떤 원칙들이 맞는지가 아니라, 오로지 어떻게 일원론적 접근법과 다원론적 접근법이 서로 다른지, 그리고 그 접근법들이 무엇을 공통으로 지니는지에만 관심을 가진다.

5) The chief difference between our simplified Chisholm-type list of epistemic principles and a monistic analysis such as SR is this: According to SR, there is one, and only one, nonevaluative property. According to the Chisholm-type list, however, there are three. Furthermore, SR tells us both what is sufficient and what is necessary for justification. Our Chisholm-type list provides us with three properties that are sufficient for justification, but it doesn't tell us what is necessary for it. After all, a belief might be justified because it has a property that is not on the list. It does not, therefore, amount to an equivalence.
5) 인식 원칙들에 대한 우리의 단순화된 치셜-유형 목록과 SR과 같은 그런 일원론적 분석 사이의 주된 차이는 이것이다. 즉 SR에 따르면 하나의 유일한 비평가적 속성이 있다. 그렇지만 치셜-유형 목록에 따르면 세 가지가 있다. 더 나아가서, SR은 우리에게 정당성에 대해 무엇이 필요조건이고 무엇이 충분조건인지 둘 모두를 말해준다. 우리의 치셜-유형 목록은 우리에게 정당성에 대한 충분조건인 세 가지 속성들을 제공하지만, 무엇이 정당성에 대한 필요조건인지는 말해주지 않는다. 무엇보다도, 하나의 믿음은 그 믿음이 그 목록에 없는 속성을 지니기 때문에 정당화될 것이다. 그러므로 그것은 동치가 되지 않는다.

6) Suppose, however, that besides the three base properties we introduced above, there are no other properties on which justification supervenes. In that case, our list would tell us both what is sufficient and what is necessary for justification, and we could state the criteriological analysis of justification expressed by P1-P3 in the form of a biconditional:<20>

CA    S is justified in believing that p if and only if S's belief that p is grounded in either introspection, perception, or memory.

As a matter of fact, Chisholm does not aspire to state a complete list; he does not intend to make a claim about what is necessary for justification.<21> His analysis, therefore, falls short of asserting what epistemic justification is equivalent to.<22>
6) 그렇지만 우리가 위에서 소개하였던 세 가지 기본 속성들 이외에, 정당화가 그에 수반하는 다른 어떠한 속성들도 없다고 생각해 보자. 그 경우, 우리의 목록은 우리에게 정당성의 필요충분조건을 말해줄 것이고, 우리는 P-1에서 P3까지에 의해 쌍조건문 형식으로 표현된 정당성에 대한 규준적 분석을 진술할 수 있을 것이다.<20>

CA    S는 p라고 믿음에 있어서 정당화된다 iff S의 p라는 믿음은 내성, 지각, 또는 기억에 기초된다.

사실상, 치셜이 완전한 목록을 진술하고자 열망하지는 않는다. 그는 무엇이 정당성의 필요조건인지에 대한 주장을 형성하고자 의도하지 않는다.<21> 그러므로 그의 분석은 무엇에 대해 인식적 정당성이 동치인지 단언하기에는 부족하다.<22>

7) Some philosophers object to the project of criteriological analysis because they think that the analysis of epistemic justification ought to avoid what has been called the "scatter problem." Suppose CA is true: being grounded in introspection, perception, or memory are the three nonnormative properties that makes beliefs justified. Critics of analytical pluralism would argue that it is unsatisfactory to list several distinct properties as entailing grounds of epistemic justification without being able to say what these three properties have in common or what unites them.<23>
7) 일부 철학자들은 규준적 분석의 기획에 반대하는데 왜냐하면 그들이 인식적 정당성에 대한 분석은 "산란 문제"라 불려온 것을 피해야 한다고 생각하기 때문이다. CA가 참이라고 생각해 보자. 내성, 지각, 혹은 기억에 기초됨은 믿음들을 정당화되도록 만드는 세 가지 비규범적 속성들이다. 분석적 다원주의에 대한 비판자들은 각각의 구별되는 속성들을 이러한 세 가지 속성들이 공통으로 지니는 어떤 것 또는 그것들을 통합시키는 어떤 것을 말할 수 있도록 함이 없이 인식적 정당성에 대한 수반하는 근거들로서 열거하는 것은 불충분하다고 주장할 것이다.<23>

8) Why, however, should we assume that there is a unique characteristic that is shared by all instances of justified belief? There is no obvious reason in support of this assumption. Of course, the existence of such a property could be established by demonstraing the truth of a particular monistic analysis. For example, if reliabilism could be proven true, then we could know that perceptual, introspection, and memorial beliefs are ultimately justified, not because they are grounded in perception, introspection, or memory, but rather because they are reliably produced.
8) 그렇지만, 어째서 우리가 정당화된 믿음의 모든 사례들에 의해 공유되는 하나의 유일한 특징이 있다고 생각해야 하는가? 이러한 추정을 뒷받침하는 아무런 명백한 근거도 없다. 물론, 그러한 속성의 현존은 특수한 일원론적 분석의 진리를 증명함으로써 확립될 수 있을 것이다. 예를 들어, 만일 신빙론이 참으로 입증될 수 있다면, 우리는 지각적, 내성적, 그리고 기억에 의한 믿음들이 궁극적으로 정당화되며, 그 까닭은 그 믿음들이 지각, 내성, 또는 기억에 기초되기 때문이 아니라, 오히려 그 믿음들이 믿을 만하게 산출되기 때문임을 알 수 있을 것이다.

9) What, however, if no monistic theory from among the various competing analyses proposed emerges as the clear winner? In that case, we may suspect that the advocates of analytical monism have set their expectations too high. And as a matter of fact, in contemporary epistemology, no undisputed monistic theory has emerged. Perhaps, then, to assume that there is one unique base property, without having at hand a theory to demonstrate the existence of such a property, is to be unduly optimistic about what criteriological analysis can achieve. Indeed, it is quite optimistic even to assume that a Chisholm-type pluralistic analysis of justification can be found, as we shall see in the next two sections.
9) 그렇지만 제안된 다양한 경쟁적 분석들 중으로부터 만일 어떠한 일원론적 이론도 명백한 승리자로 등장하지 않는다면 어찌 되는가? 그 경우, 우리는 분석적 일원론의 지지자들이 그들의 기대를 지나치게 높게 가진 것은 아닌지 의심할 것이다. 그래서 아마도 하나의 유일한 기본 속성이, 수중에 그러한 속성의 현존을 증명할 이론도 없이, 있다고 상정하는 것은 규준적 분석이 성취할 수 있는 것에 대해 지나치게 낙관적인 것일 터이다. 덧붙여서, 정당성에 대한 치셜-유형의 다원론적 분석이 발견될 수 있으리라는 추정조차 꽤나 낙관적인 것이다. 우리가 다음 두 절들에서 보게 될 것처럼 말이다.

-蟲-
Conceptual Analysis


1) In many cases, when we attempt to solve a philosophical problem, we try to analyze what a certain concept means. For example, the question

What is a person?
What is a cause?
Waht is an action?
What is knowledge?

call for the analysis of concepts. Concepts, however, can be simple or complex. While a complex concept can be analyzed by uncovering its constituent elements, no such thing can be done with a simple concept. Rather, in order to convey what we mean by a simple concept, we can do no more than give a definition by relating it to other concepts that we take to mean the same thing. We shall, therefore, distinguish between (i) defining a simple concept by stating synonyms and (ii) defining (or analyzing) a complex concept by breaking it down into its constituent elements.
1) 많은 경우들에서, 우리가 철학적 문제들을 해결하고자 시도할 때, 우리는 특정한 개념들이 무엇을 의미하는지 분석하고자 노력한다. 예를 들어

인간이란 무엇인가?
원인이란 무엇인가?
행위란 무엇인가?
앎이란 무엇인가?

이런 물음들을 개념들에 대한 분석이라 부른다. 그렇지만 개념들은 단순하거나 복잡할 수 있다. 복잡한 개념은 그 개념의 구성 요소들을 밝혀냄으로써 분석될 수 있는 반면, 단순한 개념을 가지고는 그러한 어떤 것도 수행될 수 없다. 오히려, 우리가 단순한 개념으로 의미하는 것을 전달하기 위해, 우리는 그 개념을 우리가 똑같은 것을 의미하는 것으로 치는 다른 개념들에 관련시킴으로써 정의를 내리는 것 이외에 다른 무엇도 할 수 없다. 그러므로 우리는 (i) 동의어들을 진술함으로써 다순한 개념을 정의하는 일과 (ii) 구성 요소들로 분해함으로써 복잡한 개념을 정의(또는 분석)하는 일 사이에 구분을 지을 것이다.

2) The concept of necessity, for example, is a simple one. A conceptual analysis of it, in the sense of identifying its constituent elements, cannot be given. However, we can define what we mean by the concept of necessity by relating it to the concept of possiblity. We can define the concept of a necessary proposition by saying this: A proposition is necessary if and only if it is not possible for it to be false. A parallel point can be made regarding the concept of possiblity. Since it cannot be broken down into constituent elements, we cannot give an analysis of it. We can, however, define a possible proposition by saying that a proposition is possibly true if and only if it is not necessarily false.
2) 예를 들어 필연성의 개념은 단순한 개념이다. 그 구성요소들을 규정한다는 의미에서 그 개념에 대한 개념적 분석은 제시될 수 없다. 그렇지만, 우리는 우리가 필연성이라는 개념으로 의미하는 것을 그 개념을 가능성의 개념에 관련시킴으로써 정의할 수 있다. 우리는 필연 명제의 개념을 이와 같이 말함으로써 정의할 수 있다. 한 명제는 필연적이다 iff 그 명제가 거짓이기는 불가능하다. 가능성의 개념을 고려하는 데에 같은 점이 지적될 수 있다. 그 개념은 구성 요소들로 분해될 수 없으므로, 우리는 그 개념에 대한 분석을 제시할 수 없다. 그렇지만 우리는 가능 명제를 한 명제가 가능적으로 참이다 iff 그 명제가 필연적으로 거짓은 아니다라고 말함으로써 정의할 수 있다.

3) Another example of a simple concept is that of moral rightness. Rightness does not have any constituent parts, and therefore is not accessible to conceptual analysis. However, we can define the concept of rightness by relating it to other normative concepts. For example, we may say this: Doing x is morally right if and only if it is not morally obligatory to refrain from doing x. And the concept of obligation can in turn be defined by the concept of permissibility: Doing x is obligatory if and only if refraining from doing x is not permissible.
3) 단순한 개념에 대한 또 다른 예는 도덕적 옳음에 대한 개념이다. 옳음은 어떠한 구성 부분들도 가지지 않으며, 그러므로 개념적 분석이 허용될 수 없다. 그렇지만, 우리는 옮음의 개념을 그 개념을 다른 규범적인 개념들과 관련시킴으로써 정의할 수 있다. 예를 들어 우리는 이렇게 말할 것이다. x를 하는 것은 도덕적으로 옳다 iff x를 하는 것을 그만두는 것이 도덕적 의무가 아니다. 그리고 의무의 개념은 다시 허용가능성의 개념에 의해 정의될 수 있다. x를 하는 것은 의무이다 iff x를 하는 것을 그만두는 것이 허용가능하지 않다.

4) Unlike simple concepts, complex concepts have parts that are constitutive of their meaning. Thus, in order to convey what we mean by a complex concept, we can analyze its constituent elements. For example, the concept of a mother is complex: it contains the two constituents "being female" and "having at least one child." In order to formulate an analysis of that concept, we can put down a biconditional of the following form:

Sample Analysis #1
For every x, x is a mother if and only if (i) x is female and (ii) x has at least one child.

We shall call the left-hand side of an analysis the analysandum (that which needs to be analyzed) and the right-hand side the analysans (that which does the analyzing). For an analysis to be correct, the analysans must specify conditions that are individually necessary and jointly sufficient for the analysandum. Our sample analysis does just that. The two conditions it lists are individually necessary: a person can't be a mother without being female, and a person can't be a mother without having at least one child. Furthermore, the two conditions it lists are jointly sufficient: whoever satisfies both (i) and (ii) is a mother.
4) 단순 개념들과 달리, 복합 개념들은 그 개념들의 의미에 대해 구성적인 부분들을 가진다. 그래서, 우리가 한 복합 개념으로 의미하는 바를 전달하기 위해, 우리는 그 개념의 구성요소들을 분석할 수 있다. 예를 들어, 어머니의 개념은 복합적이다. 즉 그 개념은 두 가지 구성요소들인 "여성임"과 "최소한 하나 이상의 자식을 가짐"을 함의한다. 그 개념에 대한 분석을 정식화하기 위해, 우리는 다음과 같은 형식의 쌍조건문을 내놓을 수 있다.

단순 분석 #1
모든 x에 대해서, x는 어머니이다 iff (i) x는 여성이고 (ii) x는 최소한 하나 이상의 자식을 가진다.

우리는 왼편의 분석을 피분석항(분석될 필요가 있는 것)이라 부르고 오른편을 가분석항(분석을 행하는 것)이라 부를 것이다. 옳은 분석에 대해서, 가분석항은 반드시 피분석항에 대해 개별적으로 필요조건이며 공통적으로 충분조건인 조건들을 상술하여야만 한다. 우리의 분석 사례는 바로 그것을 수행한다. 그 분석 사례가 열거한 두 가지 조건들은 개별적으로 필요조건이다. 즉 한 사람이 여성이지 않고 어머니일 수 없으며, 한 사람이 하나 이상의 아이를 가지지 않고 어머니일 수 없다. 더 나아가, 그 분석이 열거하는 두 가지 조건들은 공통적으로 충분조건이다. 즉 (i)과 (ii)를 모두 만족시키는 그 누구든지 어머니이다.

5) The same point can be made by saying that for an analysis to be correct, the analysandum must entail each of the conditions in the analysans, and the analysans must entail the analysandum. Again, our sample analysis satisfies this requirement. The concept of a mother entails "being female" as well as "having at least one child," and these two concepts taken together entail "being a mother." In short, then, a correct analysis is one in which the analysandum and analysans entail each other - that is, are equivalent, or necessarily coextensive. Thus if an analysis is correct, there is no possible world in which an object is an instance of the analysans but fails to satisfy the conditions stated in the analysans, and vice versa. The following two analyses do not satisfy this requirement.

Sample Analysis #2
For all x, x is a mother if and only if (i) x has at least one child.

Sample Analysis #3
For all x, x is a mother if and only if (i) x is female and (ii) x has at least one son.

Our second sample analysis fails because the analysandum does not entail the analysans, for there is a counterexmple: a person who has at least one child but is not a mother - that is, a person who is a father. This means the analysis is too broad; it includes too much. In contrast, the third sample analysis is too narrow; it excludes too much. Although its analysans implies the analysandum, it is not the case that the analysandum implies the analysans. That this is so can again be illustrated with a counterexample: a mother who does not satisfy condition (ii) - that is, a mother who has one or several daughters, but no sons. A general requirement for an analysis of a concept, then, is that there be no counterexamples to it, which is the case if and only if it is neither too broad nor too narrow.
5) 동일한 점이 분석이 옳다고 말함으로써 지적될 수 있다, 피분석항은 반드시 가분석항에서 각각의 조건들을 함의해야만 하고, 가분석항은 반드시 피분석항을 함의해야만 한다. 다시, 우리의 분석 사례는 이 요구조건을 만족시킨다. 어머니의 개념은 "여성임"뿐만 아니라 "최소한 하나 이상의 아이를 가짐"도 함의하고, 이러한 두 개념들은 합쳐서 "어머니임"을 함의한다. 그래서 짧게 말해 옳은 분석은 그 안에서 피분석항과 가분석항이 상호 함의하는 것이다 - 즉, 동치, 또는 필연적 동연인 것이다. 그래서 만일 하나의 분석이 옳다면, 그 안에서 한 대상이 가분석항의 사례이지만 가분석항에서 진술된 조건들을 만족시키는 데에는 실패하는, 또 그 역이 성립하는 그러한 가능세계는 아무것도 없다. 다음 두 가지 분석들은 이 요구조건을 만족시키지 않는다.

분석 사례 #2
모든 x에 대해서, x는 어머니이다 iff (i) x는 최소한 하나 이상의 자식을 가진다.

분석 사례 #3
모든 x에 대해서, x는 어머니이다 iff (i) x는 여성이고 (ii) x는 최소한 하나 이상의 아들을 가진다.

우리의 두 번째 분석 사례는 실패하는데 왜냐하면 피분석항이 가분석항을 함의하지 않기 때문이며, 왜냐하면 반례가 있기 때문이다. 즉 최소한 하나 이상의 자식을 가지지만 어머니가 아닌 사람 - 즉, 아버지라는 반례가 있다. 이것은 그 분석이 너무 넓다는 것을 의미한다. 다시 말해 그것은 너무 많은 것을 포함한다. 반대로, 세 번째 분석 사례는 너무 좁다. 즉 너무 많은 것을 배제시킨다. 그 사례의 가분석항이 피분석항을 함축한다 할지라도, 그 사례는 피분석항이 가분석항을 함축하는 경우가 아니다. 이것이 그렇다는 것은 다시 반례를 가지고 설명될 수 있다. 그 반례는 조건 (ii)를 만족시키지 않는 어머니 - 즉, 하나 혹은 여러 딸들을 가졌지만 아들들은 전혀 없는 어머니이다. 개념 분석에 대한 일반적 요구조건은 그래서 그 분석에 대한 어떠한 반례들도 없다는 것이며, 그 분석은 너무 넓지도 너무 좁지도 않다면 또한 오직 그러한 때에만 그러한 경우이다.

6) The point of the last paragraph can be restated as follows: For an analysis to be correct, the analysandum and the analysans must be equivalent, or necessarily coextensive. It would be a mistake to think, however, that any biconditional relating two equivalent concepts amounts to an analysis. For example, the following analysis

Sample Analysis #4
For all x, x is a mother if and only if x is a mother of at least one child

meets the equivalence requirement. Nevertheless, it fails because it is circular: the concept that needs to be analyzed occurs in the analysans. For an analysis to be successful, it must be illuminating, which it cannot be if it is circular. An analysis also fails to be illuminating if the concept used in the analysans is as much in need of analysis as the analysandum itself. For example, the following analysis

Sample Analysis #5
For all x, x is a triangle if and only if x is three-sided

does not admit of any counterexamples, for triangularity and three-sidedness are necessarily coextensive concepts. Nevertheless, it fails to be illuminating because when we are told that all triangles are three-sided and vice versa, we are not really being told a triangle is.(?)
6) 마지막 문단의 요점은 다음과 같이 재진술될 수 있다. 분석이 옳은 것이기 위해서, 피분석항과 가분석항은 반드시 동치 또는 필연적 동연이어야만 한다. 그렇지만 동치 개념들과 관련된 어떠한 쌍조건문이든 분석이 된다고 생각하는 것은 실수일 것이다. 예를 들어 다음과 같은 분석,

분석 사례 #4
모든 x에 대해서, x는 어머니이다 iff x가 최소한 하나 이상의 아이의 어머니이다

같은 것은 동치 필요조건을 충족시킨다. 그럼에도 불구하고, 그것은 실패하는데 왜냐하면 그것이 순환적이기 때문이다. 즉 분석될 필요가 있는 개념이 가분석항에서 발생한다. 분석이 성공하기 위해, 그 분석은 반드시 이해를 돕는 것이어야만 한다. 그 분석이 순환적이라면 이해를 도울 수 없다. 또한 하나의 분석은 만일 가분석항에서 사용된 개념이 피분석항 자체만큼 분석을 필요로 한다면 이해를 도울 수 없다. 예를 들어, 다음과 같은 분석

분석 사례 #5
모든 x에 대해서, x는 삼각형이다 iff x는 세 변을 가진 도형이다

이런 분석은 어떤 반례들도 허용하지 않는데, 왜냐하면 삼각형과 세 변 도형임은 필연적으로 동연 개념들이기 때문이다. 그럼에도 불구하고, 그 분석은 해명에 실패하는데 왜냐하면 우리가 모든 삼각형은 세변도형이고 그 역도 성립한다고 말할 때, 우리는 실제로 삼각형을 말하고 있지 않기 때문이다.


Criteriological Analysis
규준적 분석

1) A definition of a simple concept conveys that concept's meaning by relating it to other concepts. An analysis of a complex concept conveys that concept's meaning by beaking it down into its constitutive elements. In this section, we shall see that when we are concerned with evaluative (or normative) concepts, there is yet another type of analysis.
1) 단순 개념에 대한 정의는 그 개념의 의미를 그 개념을 다른 개념들에 관련시킴으로써 전달한다. 복합 개념에 대한 분석은 그 개념의 의미를 그 개념을 그 개념의 구성 요소들로 분해함으로써 전달한다. 이 절에서, 우리는 우리가 평가적 (혹은 규범적) 개념들과 관련하게 될 것이지만, 또 다른 유형의 분석이 있지는 않다.

2) When nonevaluative concepts are applied to an object, nothing is asserted to the effect that there is some way in which that object is good or bad, worthy of our praise, or just the way it ought to be. In contrast, when evaluative concepts are applied to an object, the object is being evaluated as in some respect good or bad, as being, or not being, the way it ought to be. In ethics, some central evaluative concepts, to be applied to human acts, are: obligatory, permissible, right, forbidden. And in epistemology, some central evaluative concepts, to be applied to either beliefs or propositions, are: justified, certain, probable, reasonable, evident, unreasonable, improbable, doubtful, unjustified.<5>
2) 평가적이지 않은 개념들이 대상에 적용될 때, 그 방식에서 대상이 좋거나 나쁜, 혹은 우리의 칭찬을 받을 가치가 있는, 또는 그 대상이 따라야만 하는 방식대로인 어떤 방식이 있다는 결과를 단언하게 되는 아무것도 없다. 반대로, 평가적 개념들이 대상에 적용될 때에는, 그 대상은 어떤 면에서 좋거나 나쁜 것으로, 그 대상이 따라야 하는 방식대로인 것, 또는 그렇지 않은 것으로서 평가되는 것이다. 윤리학에서, 인간 행위들에 적용될 일부 중심적인 평가적 개념들은 다음과 같다. 의무, 허용가능, 옳음, 금지. 그리고 인식론에서 믿음들이나 명제들에 적용될 일부 중심적인 평가적 개념들은 다음과 같다. 정당화됨, 확실함, 가능함, 타당함, 증거, 부당함, 불가능함, 의심스러움, 정당화되지 않음.<5>

3) What matters with regard to the analysis of these concepts is this: they are all simple. They can be defined in terms of other evaluative concepts belonging to the same family, but they cannot be broken down into constituent parts. From this we should not, however, draw the conclusion that philosophical investigations of these concepts are restricted to offering definitions. Rather, philosophers have traditionally been engaged in two projects concerning the evaluative concepts of ethics and epistemology: conveying what these concepts mean by giving definitions of them; and stating criteria for the application of these concepts. In addition to definition and conceptual analysis, there is, then, a third philosophical project: criteriological analysis.
3) 이러한 개념들의 분석에 관계되는 문제들은 이것, 즉 그 개념들이 모두 단순하다는 것이다. 그 개념들은 다른 평가적 개념들, 동류에 속하지만 구성 부분들로 분해될 수는 없는 다른 평가적 개념들로 정의될 수 있다. 그렇지만 우리는 이로부터 이러한 개념들에 대한 철학적 탐구들이 정의들을 제공하는 것으로 제한되어 있다는 결론을 이끌어 내서는 안 될 것이다. 오히려, 철학자들은 전통적으로 윤리학과 인식론의 평가적 개념들에 관련한 두 가지 기획들에 착수하여 왔다. 즉 이러한 개념들이 의미하는 것을 그 개념들에 정의들을 내림으로써 전달하는 일과 이러한 개념들의 적용을 위한 규준들을 진술하는 일이 그 두 가지 기획들이다. 정의와 개념적 분석에 덧붙여서, 그래서 세 번째 철학적 기획, 규준적 분석이 있다.

4) To see what criteriological analysis amounts to, consider hedonistic utilitarianism, according to which an action's moral status is determined by the effect it has on the overall balance of pleasure over pain. According to hedonistic utilitarianism, moral rightness is to be analyzed thus:

Hedonistic Utilitarianism
An action x is right if and only if x maximizes the balance of pleasure over pain.

The right-hand side of this biconditional does not tell us what we mean by rightness. Rather, it provides us with a criterion for the application of the term right: it tells us when an action is morally right by specifying a condition that is supposed to be both necessary and sufficient for moral rightness. It asserts, then, that the concept of maximizing the balance of pleasure over pain entails, and is entailed by, the concept of moral rightness.
4) 규준적 분석이 무엇에 해당하는지 보기 위해, 쾌락주의적 실용주의를 고찰해 보도록 하자. 그에 따르면 행위의 도덕적 상태는 고통에 대한 쾌락의 종합적 우세에 대해 그 행위가 가지는 영향(결과)에 의해 결정된다. 쾌락주의적 실용주의에 따르자면, 도덕적 옳음은 그래서 다음과 같이 분석된다:

쾌락주의적 실용주의
하나의 행위 x는 옳다 iff x가 고통에 대한 쾌락의 우세를 최대화한다.

이 쌍조건문의 오른편은 우리에게 우리가 옳음을 가지고 의미하는 것을 말해주지 않는다. 오히려, 그것은 우리에게 그 옳음이란 용어의 적용을 위한 기준을 제공한다. 즉 그것은 우리에게 언제 하나의 행위가 도덕적으로 옳은지를 도덕적 옳음에 대한 필요충분조건으로 생각되는 조건을 상술함으로써 말해준다. 그래서 그것은 고통에 대한 쾌락의 우위를 최대화함의 개념이 도덕적 옳음의 개념을 함의하며 또 그에 의해 함의된다고 단언한다.

5) Of course, we might have objections to hedonistic utilitarianism. However, the point here is not to debate which analysis of moral rightness is correct, but rather to illustrate that when philosophers set forth an analysis of rightness - be it a version of utilitarianism or some other theory - they attempt to specify what are sometimes called "right-making characteristics." According to hedonistic utilitarianism, there is one, and only one, right-making characteristic: the maximization of the balance of pleasure over pain. Other ethical theories offer alternative right-making characteristics. But however these theories differ, they typically share a common objective: to pin down what it is that makes an action right and, most importantly, to do so without using evaluative terms.
5) 물론, 우리는 쾌락주의적 실용주의에 반론들을 가질지도 모른다. 그렇지만, 여기에서 요점은 도덕적 옳음에 대한 어떤 분석이 옳은지 논쟁하는 것이 아니라, 오히려 철학자들이 옳음의 분석을 내놓을 때 - 그 분석이 실용주의의 견해이든 어떤 다른 이론의 견해이든 - 그들이 종종 "옳게 만드는 특징들"이라 불리는 것을 상술하고자 시도한다는 것을 설명하는 것이다. 쾌락주의적 실용주의에 따르자면, 옳게 만드는 특징이 한 가지 있고 그 한 가지 특징이 유일한 것이다. 그것은 고통에 대한 쾌락의 우위를 최대화하는 것이다. 다른 윤리학 이론들은 대안적인 옳게 만드는 특징들을 제시한다. 그러나 이러한 이론들이 상이하기는 하지만, 그 이론들은 전형적으로 공통된 목적을 공유한다. 그 목적은 하나의 행위를 옳은 것으로 만드는 것이 무엇인지 확정짓는 것이며, 가장 중요한 것은, 평가적 용어들을 사용하지 않고 그렇게 하는 것이다.

6) A striking feature of hedonistic utilitarianism is this: while its analysandum - rightness - is a normative term, no such term occurs in the analysans. The key notions of the analysans are those of pleasure and pain. But these concepts are nonnormative - or, as we might say, naturalistic, or descriptive. When we state, for example, that Jones is in pain, we are not evaluating Jones in any way whatever, but merely dscribing a condition Jonse is in. If we were to state that it is a bad thing for Jones to be in pain, or that Jones is deserving of the pain he is experiencing, or that we are obligated to alleviate Jone's pain, then we would be making an evaluative statement. But if we state simply that Jones is in pain and say no more than that, we are not at all making a normative judgment.
6) 쾌락주의적 실용주의의 가장 강력한 특징은 이와 같다. 즉 그것의 피분석항 - 옳음 - 은 규범적 용어인 반면에, 가분석항에서는 그러한 용어가 전혀 나타나지 않는다는 것이다. 그 가분석항의 핵심 관념들은 쾌락과 고통에 대한 관념들이다. 그러나 이러한 개념들은 규범적이지 않다 - 또는, 우리가 아마도 그렇게 말할 것처럼, 중립적이거나, 기술적(서술적)이다. 예를 들어 우리가 존스는 고통받고 있다고 진술할 때, 우리는 존스를 무엇이 되었든 어떤 방식으로도 평가하고 있지 않고, 단지 존스가 처한 상황을 기술하고 있다. 만일 우리가 고통 속에 있는 것이 존스에게 나쁜 일이라고, 또는 존스가 그가 경험하고 있는 고통을 받을 만하다고, 혹은 우리가 존스의 고통을 경감시킬 의무가 있다고 진술할 것이라면, 우리는 하나의 평가적 진술을 만들고 있는 것일 터이다. 그러나 만일 우리가 단순히 존스가 고통 속에 있다고 진술하고 그 이상의 다른 어떤 것도 말하지 않는다면, 우리는 전혀 규범적 판단을 형성하고 있지 않다.

7) Hedonistic utilitarianism, then, offers us a descriptive criterion for an evaluative term; it states the necessary and sufficient conditions of moral rightness in nonnormative terms. In epistemology, criteriological analyses of normative concepts are supposed to do the same: they are expected to state criteria for the application of such terms without using any evaluative language.<6> With regard to the concept of justification, Alvin Goldman puts this point thus:

The term "justified" … is an evaluative term, a term of appraisal. Any correct definition or synoym of it would also feature evaluative terms. I assume that such definitions or synoyms might be given, but I am not interested in them. I want a set of substantive conditions that specify when a belief is justified. [Consider] the term "right." … Normative ethics tries to specify non-ethical conditions that determine when an action is right …. Analogously, I want a theory of justified belief to specify in non-epistemic terms when a belief is justified.<7>

Notice that Goldman distinguishes here between defining the concept of epistemic justification and offering what we are calling a criteriological analysis of that concept. Both are, of course, legitimate projects of epistemology.<8> Note further that Goldman demands of his analysis that it be carried out in nonepistemic terms. By this he means terms that do not share the evaluative character of the concepts of epistemic appraisal we mentioned earlier, concepts such as certainty, probability, reasonableness, justification, and the like.
7) 그래서 쾌락주의적 실용주의는 우리에게 평가적 용어에 대한 기술적 규준을 제공한다. 그 규준은 비규범적 용어들로 도덕적 옳음의 필요충분조건을 진술한다. 인식론에서, 평가적 개념들에 대한 규준적 분석들도 동일한 역할을 하는 것으로 간주된다. 즉 그 규준적 분석들은 그러한 용어들의 적용을 위한 기준들을 어떤 평가적 언어도 사용함이 없이 진술할 것으로 기대된다.<6> 정당화의 개념과 관계하여, 알빈 골드만은 이 점을 이와 같이 내놓았다.

"정당화된"이란 용어는 … 평가적인 용어, 평가에 속하는 용어이다. 그 용어의 어떤 정확한 정의나 동의어도 또한 평가적 용어들을 특징으로 삼을 것이다. 나는 그러한 정의들이나 동의어들이 주어질지도 모른다고 추정하지만, 그러한 것들에는 관심이 없다. 나는 언제 하나의 믿음이 정당화되는지를 상술하는 일련의 본질적 조건들을 원한다. "옳음"이라는 용어.[를 고려해 보자] … 규범적 윤리학은 언제 하나의 행위가 옳은지 결정하는 비-윤리학적 조건들을 상술하고자 노력한다…. 유사하게, 나는 비-인식론적 용어들로 언제 하나의 믿음이 정당화되는지 상술하는 정당화된 믿음에 대한 이론을 원한다.

골드만이 여기에서 인식론적 정당화에 대한 개념 정의와 우리가 그 개념에 대한 규준적 분석이라 부르는 것 사이에 구별을 짓는다는 점에 주의하도록 하자. 물론 둘 다 인식론의 타당한 기획들이다.<8> 더 나아가 골드만이 그의 분석에 대해 그 분석이 비인식론적 용어들로 완수되기를 요구한다는 점에 주의하도록 하자. 이로써 그는 우리가 앞서 언급한, 확실성, 가능성, 타당성, 정당성, 그리고 그와 같은 그러한 개념들, 인식론적 평가에 속하는 개념들의 평가적 성격을 공유하지 않는 용어들을 의미한다.


Supervenience in Epstemology
인식론에 있어서의 수반

1) Why is it the aim of a criteriological analysis of epistemic justification to specify when a belief is jutified in nonevaluative terms? One answer to this question has been proposed by Jaegwon Kim. He writes:

A justified belief cannot be a brute fundamental fact unrelated to the kind of belief it is. There must a reason for it, and this reason must be grounded in the factual descriptive properties of that particular belief.<9>

The same can be said about the moral status of an action. If a particular action is the right thing to do, then there must be a reason for its rightness - that is, there must be something about this action that makes it a right action. Its rightness, Kim would say, cannot be a "brute" fact that is inaccessible to an explanation in terms of nonevaluative right-making characteristics. Likewise, a belief's epistemic status - the degree to which it is justified or unjustified - must be explainable by reference to certain characteristics of the belief, characteristics that confer on it its epistemic status.
1) 어째서 언제 하나의 믿음이 정당화되는지를 비평가적 용어들로 상술하는 것이 인식론적 정당화의 규준적 분석의 목표인가? 이 물음에 대한 한 가지 대답은 김재권에 의해 제안되었다. 그는 다음과 같이 썼다:

정당화된 믿음은 그 믿음이 속하는 그러한 종류의 믿음과 무관한 맹목적인 기초적 사실일 수 없다. 그 믿음에 대한 이유가 반드시 있어야만 하고, 이러한 이유는 그 특수한 믿음에 대한 사실 기술적 특성들에 근거되어야만 한다.

동일한 내용이 한 행위에 대한 도덕적 상태에 대해서도 이야기될 수 있다. 만일 특수한 한 행위가 해야 할 옳은 일이라면, 그 행위의 옳음에 대한 이유가 있어야만 한다 - 즉, 이 행위에 대해 그것을 하나의 옳은 행위로 만드는 어떤 것이 반드시 있어야만 한다. 김재권은 그 행위의 옳음이 비규범적인 옳게 만드는 특징들에 속하는 용어들로 이루어진 설명에 수용불가능한 "맹목적" 사실일 수 없다고 말할 것이다. 비슷한 방식으로, 한 믿음의 인식적 상태 - 그 믿음이 정당화되거나 정당화되지 않은 정도 - 는 반드시 그 믿음에 대한 특정 특징들, 그 믿음에 그 믿음의 인식적 상태를 부여하는 특징들을 언급함으로써 설명가능해야만 한다.

2) This thougt motivates a doctrine that has in recent years gained much prominence: the doctrine that normative properties supervene on nonnormative properties.<10> To say that normative properties supervene on nonnormative properties means, roughly, that whether an object does or does not have a certain normative property depends on, or is determined by, its nonnormative properties. Another way of making the same point is to say that things have their normative properties by virtue of their nonnormative properties. Yet another way to state the same point is to say that what makes an object have the normative properties it has are its nonnormative properties. Applied to ethics, the supervenience doctrine tells us that an action's moral status is determined by its nonnormative properties; applied to epistemology, the doctrine tells us that a belief's epistemic status is determined by its nonnormative properties. Our ground for making these claims is the belief that things don't just happen to have the properties they have; rather, there are reasons they have certain properties and lack others.
2) 이러한 사유는 최근 수년간 많은 명성을 얻어온 한 주의를 유발시킨다. 그 주의는 규범적 특징들이 비규범적 특징들에 수반한다는 주의이다.<10> 규범적 특징들이 비규범적 특징들에 수반한다고 말하는 것은, 거칠게 말해서 하나의 대상이 특정한 규범적 특징을 가지는지 여부가 그 대상의 비규범적 특징들에 의존하거나 혹은 비규범적 특징들에 의해 결정된다는 뜻이다. 똑같은 점을 다른 방식으로 말하자면 그것들의 비규범적 특징들 덕분으로 그것들이 규범적 특징들을 가진다고 말하는 것이다. 하지만 똑같은 점을 다른 식으로 말하자면 한 대상이 그 대상이 가진 규범적 특징들을 가지도록 만드는 것은 그 대상의 비규범적 특징들이라는 것이다. 윤리학에 적용해 보면, 수반원칙은 우리에게 한 행위의 도덕적 상태는 그 행위의 비규범적 특징들에 의해 결정된다고 말해준다. 인식론에 적용해 보면 그 원칙은 우리에게 한 믿음의 인식적 상태는 그 믿음의 비규범적 특징들에 의해 결정된다고 말해준다. 이러한 주장들을 만들기 위한 우리의 근거는 사물들이 그것들이 가진 특징들을 단지 우연히 가지는 것은 아니라는 믿음이다. 오히려, 그것들이 특정한 특징들을 지니고 다른 특징들은 가지지 않는 이유들이 있다.

3) Let us apply the supervenience doctrine to the concept of justification as that concept is used in epistemology. Suppose that, while taking a walk in a park, you see a dog running across the lawn, and thus you believe "There is a dog over there." Let us stipulate that the conditions of observation are excellent: it is board daylight, your vision is excellent, and the dog is close enough to you to be clearly discernible. We may safely assume, your belief's epistemic status - its being justified - supervenes on your belief's epistemically relevant nonnormative properties. In this case, some of these properties are: its being a perceptual belief, its being a belief formed during daylight, and its being a belief about a medium-sized object that is not very far away.
3) 수반 원칙을 인식론에서 사용되는 개념으로서 정당화의 개념에 적용해 보도록 하자. 공원을 걷는 동안, 당신이 잔디밭을 달리고 있는 한 마리 개를 보고, 그래서 당신이 "저기 개 한 마리가 있다"고 믿는다고 가정해 보자. 관찰 조건들이 훌륭하다고, 즉 널리 햇살이 비추고, 당신의 시야가 탁월하며, 그 개는 명확하게 인식할 수 있도록 충분히 당신에게 가깝다고 규정하도록 하자. 우리는 안전하게(확실하게), 당신의 믿음의 인식적 상태 - 그 믿음의 정당화되어 있음 - 가 당신의 믿음의 인식적으로 관련있는 비규범적 특징들에 수반한다고 추정할 것이다. 이 경우, 이러한 특징들 중 일부는 그 믿음이 지각적 믿음임, 낮동안에 형성된 믿음임, 그리고 너무 멀지 않은 중간 크기의 대상에 대한 믿음임일 것이다.


-蟲-
Chapter Two


Epistemology and Philosophical Analysis
인식론과 철학적 분석

Concepts and Propositions
개념들과 명제들

1) In this chapter, we shall distinguish between two ways of analyzing concepts. However, before we take up the topic of conceptual analysis, a few remains about how we use the term "concept" are in order. Concepts, as we shall use the term, are properties.<1> Thus we shall take the concept of knowledge to be the property of knowing, pertaining to persons, and the concept of justification to be the property of being justified, pertaining to the attitudes persons take toward propositions.
1) 이 장에서, 우리는 개념들을 분석하는 두 가지 방법들을 구분할 것이다. 그렇지만, 우리가 개념적 분석을 화제로 택하기에 앞서서, 어떻게 우리가 "개념"이란 용어를 적합하게 사용할 것인지에 대한 약간의 문제가 남아 있다. 우리가 사용할 용어로서의 개념들은 속성들이다.<1> 그래서 우리는 앎의 개념을 안다는 것의 속성일 것으로, 개인들에 관련될 것으로, 그리고 정당화의 개념을 정당화된 것의 속성일 것으로, 개인들이 명제들에 대해 취하는 태도들에 관련될 것으로 칠 것이다.

2) Concepts, in the sense in which we shall use the term, must not be confused with either words or ideas in the mind. When in Chapter 1 we discussed the analysis of knowledge, our concern was not with the English word "knowledge", but rather with the concept that this word denotes (and that synonymous words in countless other languages denote). Words are particulars, concrete items belonging to specific languages. Concepts, however, are universals: properties that can be exemplified by many individual objects. Thus we may speak of concepts as having "instances", which are the individual objects that exemplify them. For example, my neighbor's cat is an instance of the concept of catness, and your knowing that cats have four legs is an instance of the concept of knowledge.
2) 개념들은, 우리가 그 용어를 사용하는 의미에서, 마음 속의 말들이나 생각들과 혼동되지 않아야만 한다. 1장에서 우리가 앎의 분석을 논의했을 때, 우리의 관심사는 영단어 "앎"이 아니라, 오히려 이 단어가 나타내는 (그리고 무수한 다른 언어들에서 동의어들이 의미하는) 개념이었다. 단어들은 특칭들이고, 특수한 언어들에 속하는 구체적인 항목들이다. 그렇지만 개념들은 보편자들이다. 즉 많은 개별적 객체들에 의해 예시될 수 있는 속성들이다. 그래서 우리는 개념들에 대해 그 개념들을 예시할 수 있는 개별적 객체들인 "사례들"을 가진 것이라 이야기할 것이다. 예를 들어, 내 이웃의 고양이는 고양이임에 대한 개념의 한 사례이고, 고양이들이 네 다리를 가진다는 당신의 앎은 앎에 대한 개념의 일례이다.

3) Since we view concepts as universals, we must not confuse them with ideas in people's minds, which are particulars. When we engage in a philosophical examination of such things as knowledge and justification, then, what we are interested in is not what ideas of knowledge and justification people carry in their heads, but rather what people have in common when they know something and when they are justified in believing something.
3) 우리가 개념들을 보편자들로 보므로, 우리는 그 개념들을 특칭들인 사람들의 마음 속 생각들과 혼동하지 않아야만 한다. 우리가 앎과 정당화 같은 것들에 대한 철학적 검토에 착수할 때, 그래서, 우리가 관심을 가지게 되는 것은 앎과 정당화에 대해 사람들이 그들의 머릿속에 가져오는 어떤 생각들이 아니라, 오히려 사람들이 어떤 것을 알 때 그리고 그들이 어떤 것을 믿음에 있어서 정당화될 때 그들이 공통적으로 갖는 어떤 것이다.

4) Parallel to the distinction between concepts and words is the distinction between propositions and sentences. The constituents of sentences are words; therefore, just like words, sentences are particular items belonging to particular languages. In contrast, propositions are the things that are expressed by sentences (that is, by sentences that are either true or false). For example, the two sentences

(1) Two and Two is four
(2) Zwei und zwei ist vier

belong to different languages: (1) is English and (2) is German. But what they express is one and the same thing: the proposition that two and two equal four. Sentences are particulars; they have a location in space and time, and since they are physical in nature, they can be perceived. Sentences (1) and (2), for example, are located wherever your copy of this book is located. They came into existence when this book was printed, and if, after reading it, you decide to burn it, they will cease to exist. However, the proposition (1) and (2) express is an abstract object; it does not have a location in space and time, and since it is not physical in nature, it will not cease to exist if you burn this book.
4) 개념들과 말들 사이의 구별과 유사한 것은 명제들과 문장들 사이의 구별이다. 문장의 구성성분들은 단어들이다. 그러므로, 바로 단어들과 마찬가지로, 문장들은 특수한 언어들에 속하는 특수한 항목들이다. 반대로, 명제들은 문장들에 의해(다시 말해, 참 또는 거짓인 문장들에 의해) 표현되는 것들이다. 예를 들어, 다음 두 명제들

(1) (en) 2 더하기 2는 4이다
(2) (de) 2 더하기 2는 4이다

는 상이한 언어들에 속한다. 즉 (1)은 영어이고 (2)는 독일어이다. 그러나 그 문장들이 표현하는 것은 한 가지이고 동일한 것이다. 즉 2 더하기 2는 4와 같다는 명제이다. 문장들은 특수자들이다. 문장들은 공간과 시간에서 위치를 지니고, 그것들이 본성상 물리적이므로, 그것들은 지각될 수 있다. 예를 들어 문장 (1)과 (2)는 당신이 이 책을 복사한 것이 위치된 어디에든 위치한다. 그 문장들은 이 책이 인쇄되었을 때 존재하게 되었고, 만일, 그 책을 읽은 이후에, 당신이 그 책을 태워 버리기로 결심했다면, 그 문장들은 존재하기를 중단할 것이다. 그렇지만, 명제 (1)과 (2)가 표현하는 것은 추상적 객체이다; 그 객체는 공간과 시간 중에 위치를 지니지 않고, 본성상 물리적이지 않으므로, 만일 당신이 이 책을 불태워 버리더라도 존재하기를 그치지 않을 것이다.

5) If we view the relation between sentences and propositions in this way, we may say that a sentence is true or false depending on the truth value of the proposition it expresses. Propositions, then, are the primary bearers of truth, whereas sentences have their truth values only derivatively: by expressing propositions. Note that this view is not uncontroversial. Some philosophers object that, since propositions are supposed to be abstract objects, they are inaccessible to human experience. Sentences, on the other hand, are concrete physical entities, and thus are accessible through perception. Printed sentences can be seen, and spoken sentences heard. It might be argued, therefore, that we ought to prefer sentences over propositions for the role of truth bearers.
5) 만일 우리가 문장들과 명제들의 관계를 이런 식으로 바라본다면, 우리는 문장이 그 문장이 표현하는 명제의 진리치에 의존하는 참 또는 거짓이라고 말할지도 모른다. 그래서 명제들은 우선적으로 진리의 전달자들이고, 반면에 문장들은 오로지 파생적으로만, 즉 명제들을 표현함으로써만 그 문장들의 진리치들을 가진다. 이러한 관점이 논란의 여지가 없지는 않다는 점을 주의해야 한다. 일부 철학자들은 그 관점에 반대하는데, 명제들은 추상적 객체들인 것으로 생각되므로, 명제들은 인간 경험에 허용하기 어렵다는 것이다. 다른 한편 문장들은 구체적인 물리적 실체들이고, 그래서 지각을 통해 수용가능하다. 인쇄된 문장들은 보일 수 있고, 발화된 문장들은 들릴 수 있다. 그러므로 우리가 진리 전달자들의 역할을 위해 명제들보다 문장들을 선호해야 한다고 주장될지 모른다.

6) Let us briefly consider two replies to this argument. First, friends of propositions would argue that while it is true that propositions cannot be perceived, it is false that they are altogether inaccessible to the human mind. After all, we can understand propositions and consider whether they are true or false. Second, friends of propositions would point out that if we were to prefer sentences to propositions as the bearers of truth, we would encounter a serious difficulty. Consider this box:

Caesar was killed by Brutus.
Caesar was killed by Brutus.

How many sentences are in the box? in order to answer this question, we must distinguish between sentence types and sentence tokens, which enables us to say this: There are two tokens of the same sentence type in this box. Sentence tokens are particular physical inscriptions: patterns of ink on paper or chalk on a board. Sentence types, in contrast, are abstract forms that are instantiated by sentence tokens. Now, when critics of propositions maintain that we ought to take sentences as the bearers of truth, what they have in mind can't be sentence types, for sentence types are abstract objects, just like propositions. When you look at the box, what you see (and thus what is accessible to your experience) is two sentence tokens. The sentence type of which these two tokens are instances is something you can comprehend, or grasp, but you can't perceive it, just as you can't perceive the proposition that is expressed by the two sentence tokens in the box.
6) 이 논증에 대한 두 가지 응답들을 간략하게 고찰해 보도록 하자. 우선, 명제들의 지지자들은 명제들이 지각될 수 없다는 사실이 참인 반면, 그 명제들이 모두 인간 정신에 수용되기 어렵다는 것은 거짓이라고 주장할 것이다. 무엇보다도, 우리는 명제들을 이해할 수 있고 그 명제들이 참인지 거짓인지 고찰할 수 있다. 둘째로, 명제들의 지지자들은 만일 우리가 진리의 전달자들로서 명제들보다 문장들을 선호하게 된다면, 심각한 어려움에 직면하게 될 것이라 지적할 것이다. 이 상자를 고찰해 보자.

카이사르는 브루투스에 의해 죽임을 당했다.
카이사르는 부루투스에 의해 죽임을 당했다.

이 상자에는 몇 개의 문장들이 있는가? 이 물음에 대답하기 위해서, 우리는 문장 유형들과 문장 표시들 사이의 구분, 우리가 이 상자에는 동일한 문장 유형에 속하는 두 개의 표시들이 있다고 말할 수 있도록 하는 구분을 해야만 한다. 문장 표시들은 특수한 물리적으로 적힌 글들이다. 즉 종이 위 잉크의 무늬들이나 칠판 위에 분필의 무늬들이다. 문장 유형들은 반대로 문장 표시들에 의해 예시되는 추상적 형식들이다. 이제, 명제들에 대한 비판자들이 우리가 문장들을 진리의 전달자들로 취해야 한다고 주장할 때, 그들이 생각하고 있는 것은 문장 유형들일 수 없는데, 왜냐하면 문장 유형들은 바로 명제들과 같이 추상적 객체들이기 때문이다. 당신이 그 상자를 바라볼 때, 당신이 보는 것 (그리고 따라서 당신의 경험에 수용될 수 있는 것) 은 두 개의 문장 표시들이다. 이러한 두 표시들이 그에 대한 사례들인 문장 유형은, 당신이 상자 안의 두 문장 표시들에 의해 표현되는 명제를 지각할 수 없는 것과 마찬가지로, 당신이 이해할 수 있거나 파악할 수 있지만, 당신이 그것을 지각할 수 없는 어떤 것이다.

7) The critics of propositions, then, claim that the sorts of things that are true or false are sentence tokens. But now we may wonder whether there are enough true sentence tokens to express all the truths there are. Just consider the infinite series 2 + 1 = 3, 2 + 2 = 4, 2 + 3 = 5, 2 + 4 = 6, and so on. At some point, this series will reach truths that, if printed, would be several miles long and, if spoken, would take days to formulate. There is an infinite number of corresponding sentence tokens.<2> It would appear, therefore, that a theory that identifies the bearers of truth with sentence tokens is not very plausible.<3>
7) 명제들에 대한 비판자들은 그래서 참 또는 거짓인 그러한 종류의 것들은 문장 표시들이라고 주장한다. 그러나 이제 우리는 존재하는 모든 진리들을 표현할 만큼 참인 문장 표시들이 충분한지 궁금해할 수 있다. 당장 2 + 1 = 3, 2 + 2 = 4, 2 + 3 = 5, 2 + 4 = 6 등등으로 이어지는 무한한 연쇄를 고찰해 보도록 하자. 어떤 지점에서, 이 연쇄는 만일 그것이 인쇄된다면 그 길이가 수 마일에 이르는 진리들에 당도할 것이고, 만일 발화된다면, 진술하는 데에 수일이 걸릴 것이다. 무한한 수의 상응하는 문장 표시들이 있다.<2> 그러므로 진리의 전달자들을 문장 표시들과 동일시하는 이론은 그렇게 타당하지는 않다는 것이 드러날 것이다.<3>


Necessity and Possibility
필연성과 가능성

1) In this section, we will distinguish among four different types of propositions. To begin with, there are propositions that are necessarily true. We may refer to them as "necessary truths." Here are some examples:

All bachelors are unmarried.
Whatever is red is colored.
If there are more than three quarters in my wallet, then there are more than two quarters in my wallet.
If Royce is as tall as Gabelli and Gabelli is as tall as Berger, then Royce is as tall as Berger.

Each of these propositions is such that it could not be false. The sense of "could not" here is logical: it is logically impossible for any of these propositions to be false. There are also propositions that are necessarily false. Examples of such propositions are:

Smith has a colorless red car.
There are more than three but fewer than two quarters in my wallet.
Royce is as tall as Gabelli and Gabelli is as tall as Berger, but Royce is shorter than Berger.

Each of these propositions is a necessary falsehood: such that it could not possibly be true.
1) 이 절에서, 우리는 네 가지 상이한 유형의 명제들을 구별할 것이다. 우선, 필연적으로 참인 명제들이 있다. 우리는 그러한 명제들을 "필연적 진리들"로 언급할 것이다. 여기 몇 가지 예시들이 있다:

모든 총각들은 미혼이다.
붉은 것은 무엇이든 색이 있다.
만일 내 주머니에 25센트짜리 동전들이 세 개 이상 있다면, 내 주머니에는 25센트짜리 동전들이 두 개 이상 있다.
만일 로이스가 가벨리만 하고 가벨리가 베거만 하다면, 로이스는 베거만 하다.

이러한 명제들 각각은 거짓일 수 없을 그러한 것이다. "~일 수 없을"이란 뜻은 여기에서 논리적인 의미이다. 즉 이러한 명제들 중 어느 것도 거짓이기는 논리적으로 불가능하다는 것이다. 또한 필연적으로 거짓인 명제들도 있다. 그러한 명제들의 예시들은 다음과 같다.

스미스는 색 없는 붉은 차를 가지고 있다.
내 주머니 안에는 25센트짜리 동전들이 세 개 보다는 적지만 두 개 보다는 많이 있다.
로이스는 가벨리만 하고 가벨리는 베거만 하지만 로이스는 베거보다 작다.

이러한 명제들의 각각은 필연적으로 거짓이다. 즉 참이기가 가능할 수 없을 그러한 것이다.

2) Third, there are contingent propositions: propositions that are neither necessarily true nor necessarily false. Consider the following three examples:

(1) The forty-second president of the United States is George Bush.
(2) The forty-second president of the United States is Bill Clinton.
(3) If Bill Clinton is married to Hilary Clinton, then Hilary Clinton is married to Bill Clinton.

(1) is contingently false, and (2) is contingently true. (3), however, is not a contingent proposition, for it is necesarily true.
2) 셋째로, 우연적인(경험적인) 명제들이 있다. 즉 필연적으로 참인 것도 아니고 필연적으로 거짓인 것도 아닌 명제들이다. 다음 세 가지 예시들을 고찰해 보도록 하자.

(1) 미합중국의 마흔 두 번째 대통령은 조지 부시이다.
(2) 미합중국의 마흔 두 번째 대통령은 빌 클린턴이다.
(3) 만일 빌 클린턴이 힐러리 클린턴과 결혼한다면, 힐러리 클린턴은 빌 클린턴과 결혼한다.

3) Fourth, there are propositions that are not necessarily false. Such propositions are possibly true. The three propositions displayed above all belong to this category. (1) is actually false, but possibly true. (2), on the other hand, is actually true, and hence is possibly true. Finally, (3) is necessarily true, and thus possibly true. To generalize: Any false proposition that isn't necessarily false is possibly true, and every true proposition, be it necessarily or contingently true, is possibly true.
3) 넷째로, 필연적으로 거짓이지는 않은 명제들이 있다. 그러한 명제들은 가능적으로 참이다. 위에 제시된 세 명제들은 모두 이 범주에 속한다. (1)은 실제로 거짓이지만, 가능적으로 참이다. 다른 한편 (2)는 실제로 참이고 따라서 가능적으로 참이다. 끝으로, (3)은 필연적으로 참이고, 그래서 가능적으로 참이다. 일반화시키자면 필연적으로 거짓이지는 않은 어떠한 거짓 명제든 가능적으로 참이고, 필연적으로 참이든 우연적으로 참이든 모든 각각의 참 명제는 가능적으로 참이다.

4) In thinking about matters of necessity and possiblity, it is important not to confuse logical and physical necessity. The laws of nature tell us what is physically possible and what is not. For example, it is physically impossible for humans to cross the Atlantic by flapping their arms or to stay alive for ten years without eating any food. However, as fas as logic is concerned, if something is not outright contradictory - as, for example, a round square or a married bachelor - then it is possible, however far-fetched it may strike us. Hence we shall consider propositions such as

Jones crossed the Atlantic by falpping his arms;
Gabelli didn't eat anything for ten years and didn't lose a pound;
Thorugh an act of sheer will, Berger lifted himself from his chair and floated in midair;

as merely contingently false. Though as far as the laws of nature are concerned, they are bound to be false, from the point of view of logic, they are not. Although actually false, they are possibly true.
4) 필연성과 가능성의 문제들에 대해 생각함에 있어서, 논리적 필연성과 물리적 필연성을 혼동하지 않는 것이 중요하다. 자연법칙은 우리에게 무엇이 물리적으로 가능하고 무엇이 가능하지 않은지를 말해준다. 예를 들어, 인간들이 대서양을 그들의 두 팔로 헤엄쳐서 건너는 것이나 10년 동안 어떠한 식량도 먹지 않고 살아남는 것은 물리적으로 불가능하다. 그렇지만, 논리와 관계된 한에서, 만일 어떤 것이 완전히 모순만 아니라면 - 예를 들어 둥근 사각형이나 결혼한 총각 같이 - 그것은 가능하지만, 그것은 우리에게 설득력 없는 인상을 줄 것이다. 따라서 우리는 다음과 같은 명제들을 고찰해 볼 것이다.

존스는 그의 두 팔로 헤엄쳐서 대서양을 건넜다.
가벨리는 10년 동안 아무것도 먹지 않았고 1파운드도 빠지지 않았다.
순전히 의지만으로, 베거는 그의 의자에서 자신을 들어 올렸고 허공 중에 그 자신을 띄웠다.

이 명제들은 순전히 우연적으로 거짓이다. 자연법칙에 관계되는 한, 그 명제들은 거짓일 수밖에 없지만, 논리적 관점에서 보자면, 그렇지는 않다. 실제로 거짓이라 할지라도, 그 명제들은 가능적으로는 참이다.


Entailment and Necessary coextension.
계사 한정과 필연적 동연.

1) In the next section, we shall concern ourselves with the nature of philosophical analysis. But first we need to consider two ways in which concepts and propositions can be related to each other. We shall define these relations as follows:

Entailment
A concept A entails a concept B if and only if, necessarily, whatever is an instance of A is also an instance of B.

Equivalence
Two concepts A and B are equivalent if and only if, necessarily, whatever is an instance of A is also an instance of B, and vice versa.
Two propositions p and q are equivalent if and only if it is impossible that p and q have different truth values.

Here are some examples to illustrate these definitions. The concept of being a mother entails the concept of being female, and is equivalent to the concept of being a female who has at least one child. The concept of triangularity entails, and is equivalent to, the concept of three-sidedness. The proposition "Smith and Brown know that Jones owns a Ford" entails the proposition "It is true that Jones owns a Ford" and is equivalent to the proposition "Brown and Smith know that Jones owns a Ford." Finally, the proposition "It is true that Smith has three quarters in his wallet" entails the proposition "it is possible that Smith has three quarters in his wallet," which in turn is equivalent to the proposition "It is not necessarily false that Smith has three quarters in his wallet."
1) 다음 절에서, 우리는 철학적 분석의 본성에 관심을 가질 것이다. 그러나 우선 우리는 개념들과 명제들이 서로에게 관련될 수 있는 두 가지 방식들을 고찰할 필요가 있다. 우리는 이러한 관계들을 다음과 같이 정의할 것이다.

함의
개념 A가 개념 B를 함의한다. iff 필연적으로 A의 사례인 어떤 것이든 B의 사례이기도 하다.

등가(동치)
두 개념들 A와 B는 등가(동치)이다. iff A의 사례인 무엇이든 B의 사례이기도 하며, 그 역도 성립한다.
두 명제들 p와 q는 등가(동치)이다. iff p와 q가 상이한 진리치들을 가진다는 것이 불가능하다.

이러한 정의들을 설명하기 위한 몇몇 예시들이 있다. 어머니라는 개념은 여성이라는 개념을 함의하고, 최소한 하나 이상의 자식을 가진 여성이라는 개념에 동치이다. 삼각형의 개념은 세-변임의 개념을 함의하고 그에 동치이다. "스미스와 브라운은 존스가 Ford차를 소유하고 있다는 것을 안다"라는 명제는 "존스가 Ford차를 소유하고 있다"라는 명제를 함의하고 "브라운과 스미스는 존스가 Ford차를 소유하고 있다는 것을 안다"라는 명제와 동치이다. 끝으로, "스미스가 그의 주머니에 25센트 짜리 동전 세 개를 가지고 있다는 것은 참이다"라는 명제는 "스미스가 그의 주머니 안에 25센트 짜리 동전 세 개를 가지고 있다는 것은 가능하다"라는 명제, 결국 "스미스가 그의 주머니에 25센트 짜리 동전 세 개를 가지고 있다는 것이 필연적으로 거짓이지는 않다"라는 명제와 동치인 그 명제를 함의한다.

2) If two concepts are equivalent to each other, they are necessarily coextensive. A concept's extension is the collection of all instances of that concept. For example, the extension of concept "goat" is the collection of all goats. The extension of the concept "being a member of the Supreme Court of the United States" is a collection of exactly nine justices, and the extension of the concept "being the president of the United States" is (in 1994) Bill Clinton. For two concepts to be necessarily coextensive, they must mutually entail each other: it must be logically impossible for there to be an object that is an instance of one concept without being an instance of the other. The two concepts we considered above, "being a mother" and "being a female who has at least one child," are an example of such a pair of concepts. Concepts that are necessarily coextensive have the same instances in all possible worlds. For example, there is no possible world in which an object that is three-sided fails to be an object with three angles, and vice versa; and there is no possible world in which a person is a brother without being a male sibling, and vice versa.
2) 만일 두 개념들이 서로 동치라면, 그 개념들은 필연적으로 동연적이다. 개념의 범위는 그 개념의 모든 사례들에 대한 총합이다. 예를 들어, "염소"라는 개념의 범위는 모든 염소들의 총합이다. "미합중국 대법원의 구성원 중 하나임"이라는 개념의 범위는 정확히 아홉 판단들의 총합이고, "미합중국의 대통령임"이라는 개념의 범위는 (1994년에) 빌 클린턴이다. 필연적으로 동연일 두 개념들에 대해서, 그 개념들은 반드시 공통으로 상호 함의하여야만 한다. 다른 하나의 개념에 대한 사례이지 않고 남은 하나의 개념의 사례인 객체가 있기는 논리적으로 불가능해야만 한다. 우리가 위에서 고찰하였던 두 개념들, "어머니임"과 "최소한 하나 이상의 자식을 가진 여성임"은 그러한 개념쌍의 일례이다. 필연적으로 동연인 개념들은 모든 가능세계들에서 동일한 사례들을 가진다. 예를 들어, 세 변인 객체가 세 각들을 가진 객체이지 않은, 그리고 그 역이 성립하는 그러한 가능세계는 없으며, 한 사람이 형(또는 동생)이면서 남자 형제이지는 않은, 그리고 그 역이 성립하는 그러한 가능세계도 없다.

3) For an example of two concepts that are coextensive, but not necessarily coextensive, consider the two concepts "being president of the United States" and "being the commander-in-chief." The U.S Constitution stipulates that the president and the commander-in-chief are one and the same person. But there is nothing necessary about what the Constitution stpulates. An amendment putting an end to this tradition might get passed, which means that it is logically possible for the president and the commander-in-chief to be two different people. Another example of two concepts that are coextensive without being necessarily coextensive are "x is an animal with a heart" and "x is an animal with a liver."<4> As far as the laws of nature go, whatever is an instance of one must also be an instance of the other concept; logically, however, there is nothing necessary about that. Nature might change, which is to say that, although this is difficult to conceive, in the future creatures might evolve that have hearts but not livers.
3) 동연이지만 필연적으로 동연이지는 않은 두 개념들에 대한 일례에 대해 "미합중국의 대통령임"과 "(미합중국의)총사령관임"이라는 두 개념들에 대해 고찰해 보자. 미합중국 헌법은 대통령과 총사령관이 한 사람의 동일인물이라고 규정한다. 그러나 헌법이 규정하는 것에 대해서는 아무런 필연성도 없다. 이 전통을 종결시키는 수정안이 통과될지 모르고, 그것은 대통령과 총사령관이 상이한 두 사람들일 것이 논리적으로는 가능하다는 뜻이다. 필연적으로 동연이지는 않으나 동연인 두 개념들에 대한 또 다른 사례는 "x는 심장을 가진 동물이다"와 "x는 간을 가진 동물이다"이다.<4> 자연법칙이 유효한 한, 이 중 한 가지 개념의 사례인 어떤 것이든 또한 반드시 나머지 다른 개념의 사례이기도 해야만 한다. 그렇지만 논리적으로 그에 대한 아무런 필연성도 없다. 자연이 변할지 모르고, 그것은 비록 이걸 이해하기는 어렵다 할지라도 미래에 생물들이 심장은 가지지만 간들은 가지지 않게끔 진화할지도 모른다는 말이다.

-蟲-
60) Suppose, therefore, that you pay no heed to Anytus, but are prepared to let me go. He said I need never have been brought to court in the first place; but that once I had been, your only option was to put me to death. He declared before you that, if I got away from you this time, your sons would all be utterly corrupted by practising Socrates' teachings. Suppose, in the face of that, you were to say to me:
60) 그러므로 가정해 보십시오, 여러분께서 아뉘토스에게 전혀 주의를 기울이시지 않고, 저를 놓아주실 준비가 되셨다고 말씀입니다. 그는 제가 애초부터 결코 법정으로 끌려오지 않았어야 한다고, 그러나 일단 제가 나온 이상은, 여러분들의 유일한 선택은 저에게 사형을 선고하는 것뿐이라고 말했습니다. 그는 여러분들 앞에 공언하였습니다, 만일 제가 이번에 여러분들로부터 빠져나온다면, 여러분들의 자제분들께서 소크라테스의 가르침들을 행함으로써 모두가 완전히 타락되어 버릴 것이라고 말입니다. 생각해 보십시오, 그 이야기에 직면하여서, 여러분들께서 저에게 말씀하시는 것을 말입니다:

61) "Socrates, we will not listen to Anytus this time. We are prepared to let you go - but only on this condition: you are to pursue that quest of yours and practise philosophy no longer; and if you are caught doing it any more, you shall be put to death."
61) "소크라테스, 이번은 우리가 아뉘토스의 말을 듣지 않을 것입니다. 우리는 당신을 놓아줄 준비가 되었습니다 - 그러나 오직 이러한 조건 하에서만 입니다. 당신은 더 이상 당신의 저 탐구를 추구하는 것도 지혜를 사랑하는 일을 행하는 것도 그만둬야 합니다. 그리고 만일 당신이 그 이상 그 일을 행하여 잡힌다면, 당신은 사형을 선고받을 것입니다."

62) Well, as I just said, if you were prepared to let me go on those terms, I should reply to you as follows:
62) 좋습니다, 제가 방금 말했듯이, 만일 여러분들께서 그러한 조건들로 저를 풀어주실 준비가 되어 계시다면, 저는 여러분들께 다음과 같이 대답해 드릴 것입니다:

63) "I have the greatest fondness and affection for you, fellow Athenians, but I will obey my god rather than you; and so long as I draw breath and am able, I shall never give up practising philosophy, or exhorting and showing the way to any of you whom I ever encounter, by giving my usual sort of message. 'Excellent friend,' I shall say; 'You are an Athenian. Your city is the most important and renowned for its wisdom and power; so are you not ashamed that, while you take care to acquire as much wealth as possible, with honour and glory as well, yet you take no care or thought for understanding or truth, or for the best possible state of your soul?'
63) "저는 여러분들께 대단한 친애와 감사를 품고 있습니다, 아테네인 여러분, 그러나 저는 여러분들 보다는 저의 신께 순종할 것입니다. 그리고 제가 숨이 붙어있고 가능한 동안에는, 전 결코 지혜를 사랑하기를, 또는 여러분들 중 제가 마주치는 그 어떤 분께든 그 길을 권하고 보여주기를 포기하지 않을 것입니다, 저의 늘상 하던데로의 뜻을 전함으로써 말씀입니다. '훌륭한 친구여,' 저는 말할 테지요; '당신은 아테네인이오. 당신의 도시는 그 지혜와 힘으로 가장 중요하고 또 이름난 도시라오. 그래서 당신은 가능한 한 많은 부를 얻는 일에 주의를 기울이면서도 명성과 영광도 또한 얻고자 하지만, 그런데도 지혜나 진리, 또는 당신의 영혼에 있어서 가능한한 가장 훌륭한 상태에는 주의를 기울이지 않는다는 것이 부끄럽지 않소?'

64) "And should any of you dispute that, and claim that he does take such care, I will not let him go straight away nor leave him, but I will question and examine and put him to the test; and if I do not think he has acquired goodness, though he says he has, I shall say, 'Shame on you, for setting the lowest inferior ones more highly?' That I shall do for anyone I encounter, young or old, alien or fellow citizen; but all the more for the latter, since your kinship with me is closer."
64) "그리고 여러분들 중 누구시든 그것에 반발하시고, 그분께서 그러한 주의를 기울이셨다 주장하신다면, 저는 그분을 곧장 놓아드리거나 제가 떠나지 않을 것이고, 묻고 또 검토하며 그분을 검증받도록 할 것입니다. 그리고 만일 제가 그분이 탁월함을 얻은 것으로 생각되지 않는다면, 그분이 자신은 가졌다고 말씀하실지라도, 저는 말할 것입니다, '당신에 대해 가장 최하의 열등한 것을 더욱 높게 치는 것이 부끄럽지 않습니까?' 그것이 젊은이든 노인이든, 외지인이든 동료 시민들이든 누구라도 제가 마주치는 사람에게 제가 행할 일입니다. 그러나 동료 시민 여러분께 훨씬 더 그러할 것인즉, 여러분들의 혈통이 저와 더욱 가깝기 때문입니다."

65) Those are my orders from my god, I do assure you. Indeed, I believe that no greater good has ever befallen you in our city than my service to my god; because all I do is to go about presuading you, young and old alike, not to care for your bodies or for your wealth so intensely as for the greatest possible wellbeing of your souls. "It is not wealth," I tell you, "that produces goodness; rather, it is from goodness that wealth, and all other benefits for human beings, accrue to them in their private and public life."
65) 그러한 것들이 저의 신으로부터 제게 내려진 명령이라고, 저는 여러분께 장담합니다. 더욱이, 저는 우리의 폴리스에서 여러분께 내려졌던 그 어떤 것도 저의 신께 올리는 저의 봉사보다 더 대단히 좋은 것은 없다고 믿습니다. 왜냐하면 제가 하는 모든 일은 여러분들을 젊은이도 노인들도 마찬가지로 여러분들의 일신이나 부를 돌보는 것이 아니라 매우 강력하게, 말하자면 여러분의 영혼들이 가능한 한 가장 좋은 상태이도록 하는 일에 주의를 기울이도록 설득하는 일을 계속하는 것이기 때문입니다. 저는 여러분께 말씀드립니다. "탁월함을 낳는 것, 그것은 부가 아닙니다. 오히려, 부유함, 그리고 인간에게 유익한 다른 모든 것들이, 인간들의 사적인 삶과 공적인 삶 모두에 있어서 그들에게 축적되는 것은 탁월함으로부터입니다."

66) If, in fact, I am corrupting the young by those assertions, you may call them harmful. But if anyone claims that I say anything different, he is talking nonsense. In the face of that I should like to say: "Fellow Athenians, you may listen to Anytus or not, as you please; and you may let me go or not, as you please, because there is no chance of my acting otherwise, even if I have to die many times over-"
66) 만일, 사실로 제가 그러한 주장들로 젊은 사람들을 타락시키고 있다면, 여러분들께서는 그것들을 해롭다고 말씀하실 테지요. 그러나 만일 누구라도 제가 이와 다른 것들을 말한다고 주장한다면, 그는 말도 안 되는 소리를 떠들고 있는 것입니다. 그러한 이야기들을 마주하여 저는 이야기할 것 같습니다. "아테네 시민여러분, 여러분들께서는 아뉘토스의 말을 들으시든 그렇지 않으시든, 여러분들의 뜻대로 하십시오. 그리고 여러분들께서 저를 놓아주시든 그렇지 않든, 또한 여러분 뜻대로 하십시오. 왜냐하면 제가 다른 식으로 행동할 어떠한 경우도 없을 것이기 때문입니다. 설령 제가 여러 번을 죽어야 한다 할지라도 말씀입니다-"

67) Stop protesting, fellow Athenians! Please abide by my request that you not protest against what I say, but hear me out; in fact, it will be in your interest, so I believe, to do so. You see, I am going to say some further things to you which may make you shout out - although I beg you not to.
67) 소란을 멈춰 주십시오, 아테네인 여러분! 여러분들께서 제가 하는 말에 반대하여 항의하지는 말아 주십사 하였던 제 청에 따라 기다려 주시고, 그러나 제 이야기를 끝까지 들어 주십시오. 사실, 제가 믿기로, 그렇게 하시는 것은 여러분들의 관심에 달려있을 것입니다. 아시다시피, 저는 여러분들을 더욱 고함치시게끔 만들 얼마간 더한 것들을 계속해서 말씀드릴 것입니다. - 비록 제가 그러지 말아 주십사 간청하였을지라도 말입니다.

68) You may be assured that if you put to death the sort of man I just said I was, you will not harm me more than you harm yourselves. Meletus or Anytus would not harm me at all; nor, in fact, could they do so, since I believe it is out of the question for a better man to be harmed by his inferior. The latter may, of course, inflict death or banishment or disenfranchisement; and my accuser here, along with others no doubt, believes those to be great evils. But I do not. Rather, I believe it a far greater evil to try to kill a man unjustly, as he does now.
68) 만일 여러분들께서 제가 방금 저 자신이 그러하다 말씀드린 그런 종류의 사람에게 사형을 선고하신다면, 여러분들께서 여러분들 자신에게 해를 끼치는 것보다 더욱 저에게 해를 입히시지 못하시리란 것을 여러분들께서는 확신하시게 될 것입니다. 멜레토스나 아뉘토스는 전혀 저를 해할 수 없을 것입니다. 사실 그들이 그러할 수도 없을 것인데, 더 나은 사람이 그보다 열등한 자에 의해 해를 입는다는 것은 전혀 불가능한 일이라 제가 믿기 때문입니다. 뒤엣 사람들(멜레토스와 아뉘토스)은 물론, 아마도 사형이나 추방형 또는 권리박탈을 부과할 것입니다. 그리고 다른 사람들은 의심할 것도 없이 마찬가지로 여기 나를 고발한 자도 그런 것들이 대단히 안 좋은 것들이라 믿습니다. 그러나 저는 아닙니다. 오히려, 저는 한 사람을 부정의하게 죽이려고 드는 것이 훨씬 더 대단히 사악한 짓이라 믿습니다, 그가 지금 하고 있는 일처럼 말씀입니다.

69) At this point, fellow Athenians, so far from pleading on my own behalf, as might be supposed, I am pleading on yours, in case by condemning me you should mistreat the gift which God has bestowed upon you - because if you put me to death, you will not easily find another like me. The fact is, if I may put the point in a somewhat comical way, that I have been literally attached by God to our city, as if to a horse - a large thoroughbred, which is a bit sluggish because of its size, and needs to be aroused by some sort of gadfly. Yes, in me, I believe, God has attached to our city just such a creature - the kind which is constantly alighting everywhere on you, all day long, arousing, cjoling, or reproaching each and every one of you. You will not easily acquire another such gadfly, gentlemen; rather, if you take my advice, you will spare my life. I dare say, though, that you will get angry, like people who are awakened from their doze. Perhaps you will heed Anytus, and give me a swat: you could happily finish me off, and then spend the rest of your life asleep - unless God, in his compassion for you, were to send you someone else.
69) 이 시점에서, 아테네 시민여러분, 저는 제 자신의 이익에 대해서라기 보다는, 추측되건데, 여러분들의 이익에 대해서 변호하고 있는데, 여러분들께서 저를 비난하심으로써 신께서 여러분들께 내리신 선물을 잘못 다룰 것을 대비하여서 - 왜냐하면 만일 여러분들께서 저에게 사형을 내리신다면, 여러분들께서는 저와 같은 다른 사람을 쉽사리 찾지 못하실 것이기 때문입니다. 사실은, 만일 제가 그 점을 어떤 우스운 방식으로 내놓자면, 저는 말 그대로 신에 의해 우리의 폴리스에 들러붙여졌습니다. 마치 한 마리 말, 커다란 순종 말, 그 덩치 때문에 다소 둔한, 그리고 일종의 쇠파리에 의해 자극받는 것이 필요한 그러한 말에 붙여지듯 말씀입니다. 네, 제게 있어서는, 저는 믿습니다, 신께서 우리의 폴리스에 바로 그러한 벌레를 - 끊임없이 여러분들의 어디에나 내려 앉아, 온종일, 여러분 각기 모두를 자극하고, 회유하거나, 또는 힐난하고 있는 그런 종류의 벌레를 붙여 놓으셨습니다. 여러분들께서는 그러한 또 다른 쇠파리를 쉽사리 얻지 못하실 것입니다, 여러분; 오히려, 만일 여러분들께서 저의 충고를 받아들이신다면, 여러분들께서는 제 삶을 아껴두시게 될 것입니다. 그렇다 할지라도 저는 여러분들께서 마치 졸다가 잠이 깬 사람들마냥 화를 내시리란 것을 장담합니다. 아마도 여러분들께서는 아뉘토스의 말을 들으실 것이고, 파리 잡듯 저를 때리실 것입니다: 여러분들께서는 저를 행복하게 끝장내실 수 있을 것이고, 여러분들의 남은 여생을 잠든 채로 허비하실 겁니다 - 신께서 여러분들에 대한 연민에서 다른 누군가를 여러분께 보내주시지 않는 한 말씀입니다.

70) That I am, in fact, just the sort of gift that God would send to our city, you may recognize from this: it would not seem to be in human nature for me to have neglected all my own affairs, and put up with the neglect of my family for all these years, but constantly minded your interests, by visiting each of you in private like a father or an elder brother, urging you to be concerned about goodness. Of course, if I were gaining anything from that, or were being paid to urge that course upon you, my actions could be explained. But in fact you can see for yourselves that my accusers, who so shamelessly level all those other charges against me, could not muster the impudence to call evidence that I ever once obtained payment, or asked for any. It is I who can call evidence sufficient, I think, to show that I am speaking the truth - namely, my poverty.
70) 사실, 제가 신께서 우리의 폴리스에 보내주신 바로 그러한 종류의 선물이라는 것을, 여러분들께서는 이로부터 알게 되실 겁니다: 제가 저 자신의 모든 일들을 내팽개쳤다는 것, 제 가족에 대해 이 모든 세월 동안 소홀함을 감수하였다는 것, 그러나, 여러분들께 탁월함에 대해 관심을 가지시라 주장하면서, 여러분 각자를 아버지나 형처럼 개인적으로 방문함으로써, 끊임없이 여러분들의 이익을 신경썼다는 것은 평범한 사람의 일로 보이지 않을 것이기 때문입니다. 물론, 만일 제가 그로부터 뭔가를 얻고 있다면, 또는 여러분들께 그러한 가르침을 권하기 우해 값을 받고 있다면, 제 행동들이 설명될 수 있을 것입니다.

71) Now it may perhaps seem peculiar that, as some say, I give this counsel by going around and dealing with others' concerns in private, yet do not venture to appear before the Assembly, and counsel the city about your business in public. But the reason for that is one you have frequently heard me give in many places: it is a certain divine or spiritual sign which comes to me, the very thing to which Meletus made mocking allusion in his indictment. It has been happening to me ever since childhood: a voice of some sort which comes, and which always - whenever it does come - restrains me from what I am about to do, yet never gives positive direction. That is what opposes my engaging in politics - and its opposition is an excellent thing, to my mind; because you may be quite sure, fellow Athenians, that if I had tried to engage in politics, I should have perished long since, and should have been of no use either to you or to myself.

72) And please do not get angry if I tell you the truth. The fact is that there is no person on earth whose life will be spared by you or by any other majority, if he is genuinely opposed to many injustices and unlawful acts, and tries to prevent their occurrence in our city. Rather, anyone who truly fights for what is just, if he is going to survive for even a short time, must act in a private capacity rather than a public one.

73) I will offer you conclusive eivdence of that - not just words, but the sort of evidence that you respect, namely, actions. Just hear me tell my experiences, so that you may know that I would not submit to a single person for fear of death, contrary to what is just; nor would I do, even if I were to lose my life on the spot. I shall mention things to you which are vulgar commonplaces of the courts; yet they are true.

74) Although I have never held any other public office in our city, fellow Athenians, I have served on its Council. My own tribe, Antiochis, happend to be the presiding commission on the occasion when you wanted a collective trial for the ten generals who had failed to rescue the survivors from the naval battle. That was illegal, as you all later recognized. At the time I was the only commissioner opposed to your acting illegally, and I voted against the motion. And though its advocates were prepared to lay information against me and have me arrested, while you were urging them on by shouting, I believed that I should face danger in siding with law and justice, rather than take your side for fear of imprisonment or death, when your proposals were contrary to justice.

75) Those events took place while our city was still under democratic rule. But on a subsequent occasion, after the oligarchy had come to power, the Thirty summoned me and four others to the round chamber, with orders to arrest Leon the Salaminian, and fetch him from Salamis for execution; they were constantly issuing such orders, of course, to many others, in their wish to implicate as many as possible in their crimes. On that occasion, however, I showed, once again not just by words, but by my actions, that I couldn't care less about death - if that would not be putting it rather crudely - but that my one and only care was to avoid doing anything sinful or unjust. Thus, powerful as it was, that regime did not frighten me into unjust action: when we emerged from the round chamber, the other four went off to Salamis and arrested Leon, whereas I left them and went off home. For that I might easily have been put to death, had the regime not colapsed shortly afterwards. There are many witnesses who will testify before you about those events.

76) Do you imagine, then, that I would have survived all these years if I had been regularly active in public life, and had championed what was right in a manner worthy of a brave man, and valued that above all else, as was my duty? Far from it, fellow Athenians: I would not, and nor would any other of person that I, for my part, shall prove to have been throughout my life; and likewise in my private life, because I have never been guilty of unjust association with anyone, including those whom my slanderers allege to have been my students.

77) I never, in fact, was anyone's instructor at any time. But if a person wanted to hear me talking, while I was engaging in my own business, I never grudged that to anyone, young or old; nor do I hold conversation only when I receive payment, and not otherwise. Rather, I offer myself for questioning to wealthy and poor alike, and to anyone who may wish to answer in response to questions from me. Whether any of those people acquires a good character or not, I cannot fairly be held responsible, when I never at any time promised any of them that they would learn anything from me, nor gave them instruction. And if anyone claims that he ever learnt anything from me, or has heard privately something that everyone else did not hear as well, you may be sure that what he says is untrue.

78) Why then, you may ask, do some people enjoy spending so much time in my company? You have already heard, fellow Athenians: I have told you the whole truth - which is that my listeners enjoy the examination of those who think themselves wise but are not, since the process is not unamusing. But for me, I must tell you, it is a mission which I have been bidden to undertake by the god, through oracles and dreams, and through every means whereby a divine injunction to perform any task has ever been laid upon a human being.

79) That is not only true, fellow Athenians, but is easily verified - because if I do corrupt any of our young people, or have corrupted others in the past, then presumably, when they grew older, should any of them have realized that I had at any time given them bad advice in their youth, they ought now to have appeared here themselves to accuse me and obtain redress. Or else, if they were unwilling to come in person, members of their families - fathers, brothers, or other relations - had their relatives suffered any harm at my hands, ought now to put it on record and obtain redress.

80) In any case, many of those people are present, whom I can see: first there is Crito, my contemporary and fellow demesman, father of Critobulus here; then Lysanias of Sphettus, father of Aeschines here; next, Epigenes' father, Antiphon from Cephisia, is present; then again, there are others here whose brothers have spent time with me in these studies: Nicostratus, son of Theozotides, brother of Theodotus - Theodotus himself, incidentally, is deceased, so Nicostratus could not have come at his brother's urging; and Paralius here, son of Demodocus, whose brother was Theages; also present is Ariston's son, Adimantus, whose brother is Plato here, and Aeantodorus, whose brother is Apollodorus here.

81) There are many others I could mention to you, from whom Meletus should surely have called some testimony during his own speech. However, if he forgot to do so then, let him call it now - I yield the floor to him - and if he has any such evidence, let him produce it. But quite the opposite is true, gentlemen: you will find that they are all prepared to support me, their corruptor, the one who is, according to Meletus and Anytus, doing their relatives mischief. Support for me from the actual victims of corruption might perhaps be explained; but what of the uncorrupted - older men by now, and relatives of my victims? What reason would they  have to support me, apart from their right and proper one, which is that they know very well that Meletus is lying, whereas I am telling the truth?


-작성중-
33) So much for my defence before you against the charges brought by my first group of accusers. Next, I shall try to defend myself against Meletus, good patriot that he claims to be, and against my more recent critics. So once again, as if they were a fresh set of accusers, let me in turn review their deposition. It runs something like this: "Socrates is guilty of corrupting the young, and of failing to acknowledge the gods acknowledged by the city, but introducing new spiritual beings instead." Such is the charge: let us examine each item within it.
33) 저에 대한 첫 번째 고발자들의 무리에 의해 제시된 죄목들을 반박하여 여러분들 앞에서의 저의 변론은 그 정도로 합시다. 다음으로, 저는 그가 주장하기로는 훌륭한 애국자인 멜레토스와 저의 더욱 최근 비판자들에 대항하여 저 자신을 변호하고자 노력할 것입니다. 그래서 다시 한 번, 마치 그들이 새로운 고발자들의 무리인 것처럼, 저는 이번엔 그들의 증언을 되짚어 보겠습니다. 그 증언은 이와 같이 되어 있습니다. "소크라테스는 젊은이들을 타락시키는 죄가 있고, 폴리스가 믿는 신들을 인정하지 않으나 새로운 정령들(신들)을 그 대신 도입하는 죄가 있다." 그 죄목은 그런 것입니다. 그 안에서 각각의 항목을 검토해 봅시다. 검토해 봅시다.
   
34) Meletus says, then, that I am guilty of corrupting the young. Well I reply, fellow Athenians, that Meletus is guilty of trifling in a serious matter, in that he brings people to trial on frivolous grounds, and professes grave concern about matters for which he has never cared at all. I shall now try to prove to you too that that is so.
34) 그러니까 멜레토스는 제가 젊은이들을 타락시키는 죄를 짓는다고 말합니다. 좋아요, 아테네인 여러분, 저는 멜레토스가 심각한 문제를 하찮게 여기는 죄를 짓고 있다고 맞받아치겠습니다. 그가 경솔한 근거들을 가지고 사람들을 법정으로 데려왔고, 그가 전혀 단 한 번도 주의한 적 없는 문제들에 대해 심각하게 염려한다고 주장한다는 점에서 말씀입니다. 이제 저는 여러분들께도 그것이 그러하다는 것을 마찬가지로 입증해 드리고자 애쓸 것입니다.

35) Step forward, Meletus, and answer me. It is your chief concern, is it not, that our younger people shall be as good as possible?
     -It is.
    Very well, will you please tell the judges who influences them for the better - because you must obviously know, seeing that you care? Having discovered me, as you allege, to be the one who is corrupting them, you bring me before the judges here and accuse me. So speak up, and tell the court who has an improving influence.
35) 앞으로 나오세요, 멜레토스, 그리고 저에게 대답하세요. 우리의 젊은 사람들이 가능한 한 훌륭하게 되는 것이 무엇보다도 당신의 주된 관심사이지요, 그렇지 않나요?
     -그건 그렇죠.
     좋아요, 당신은 심판인들께 누가 그 젊은이들에게 보다 나아지도록 영향을 주는지 말해 주세요 - 왜냐하면 당신은 분명하게 알 것이 틀림없으니까요, 당신이 관심을 가지는 것으로 보아서는 말이죠? 당신이 혐의를 두는 것처럼, 제가 그 젊은이들을 타락시키는 한 사람일 것이라 밝혀냈기 때문에, 당신은 저를 여기 심판인들 앞에 데려오고 저를 고발하지요. 그러니 거리낌 없이 털어 놓으세요, 그리고 누가 유익한 영향력을 지닌 사람인지 공판정에 말하세요.

36) You see, Meletus, you remain silent, and have no answer. Yet doesn't that strike you as shameful, and as proof in itself of exactly what I say - that you have never cared about these matters at all? Come then, good fellow, tell us who influences them for the better.
     -The laws.
     Yes, but that is not what I'm asking, excellent fellow. I mean, which person, who already knows the laws to begin with?
     -These gentlemen, the judges, Socrates.
     What are you saying, Meletus? Can these people educate the young, and do they an improving influence?
     -Most certainly.
     All of them, or some but not others?
     -All of them.
     My goodness, what welcome news, and what a generous supply of benefactors you speak of! And how about the audience here in court? Do they too have an improving influence, or not?
     -Yes, they do too.
     And how about members of the Council?
     -Yes, the Councilors too.
     But in that case, how about people in the Assembly, its individual members, Meletus? They won't be corrupting their youngers, will they? Won't they all be good influences as well?
     -Yes, they will too.
     So every person in Athens, it would appear, has an excellent influence on them except for me, whereas I alone am corrupting them. Is that what You're saying?
     -That is emphatically what I'm saying.
     보시다시피, 멜레토스, 당신은 침묵을 지키고 있고, 아무런 대답도 갖고 있지 않아요. 그런데도 그게 당신을 부끄럽게 만들지 않고, 그 자체로 정확하게 제가 말한 것의 증거로 여겨지지도 않는 건가요 - 당신이 전혀 단 한 번도 이러한 문제들에 마음 쓴 일이 없었다는 게? 자, 훌륭한 친구, 누가 그 젊은이들을 더 나아지도록 영향을 끼치는지 우리에게 이야기해 주세요.
      -법률입니다.
      그렇죠, 하지만 제가 묻고 있는 건 그게 아니에요, 뛰어난 친구. 무슨 말인가 하면, 우선 어떤 사람이, 그 법률을 이미 알고 있는 사람이 누군가요?
      -이분들, 심판인들이십니다, 소크라테스.
       지금 무슨 소릴 하고 있는 건가요, 멜레토스? 이분들이 젊은 사람들을 가르칠 수 있고, 그들이 유익한 영향력을 행사한다고요?
       -거의 확실히요.
       그들 모두가 그런가요, 아니면 몇몇이 그러하고 다른 이들은 아닌가요?
      -그들 모두가요.
       맙소사, 얼마나 반가운 소식인가요, 또 당신 말대로 이 무슨 넘치는 은인들인가요!(...에이, 씨발. 의역을 하려면 제대로 하든지. 왜 여기서 감탄이 나와, 빌어먹을.) 그럼 여기 법정의 청중들은 어떤가요? 그들도 또 유익한 영향력을 가지나요, 혹은 아닌가요?
       -네, 그들도 가지죠.
       평의회의 구성원들은 어떤가요?
       -네, 평의원들도 마찬가지입니다.
       하지만 그렇다면, 민회의 사람들은 어떤가요, 그 민회의 개인적인 구성원들은 어떻지요, 멜레토스? 그들은 그들의 젊은이들을 타락시키지 않을까요, 아니면 타락시킬까요? 그들도 모두가 좋은 영향들을 미칠까요?
       -네, 그들 또한 그럴 것입니다.
       그러면 저를 제외한 아테네의 모든 사람들이 훌륭한 영향력을 젊은이들에게 훌륭한 영향을 주는 것으로, 반면에 저는 혼자서만 그들을 타락시키고 있는 것으로 보이는군요. 그게 당신이 말하고 있는 건가요?
       -그것이 제가 단호하게 말하고 있는 바입니다.

37) Then I find myself, if we are to believe you, in a most awkward predicament. Now answer me this. Do you think the same is true of horses? Is it everybody who improves them, while a single person spoils them? Or isn't the opposite true: a single person, or at least very few people, namely the horsetrainers, can improve them; while lay people spoil them, don't they, if they have to do with horses and make use of them? Isn't that true of horses as of all other animals, Meletus? Of course it is, whether you and Anytus deny it or not. In fact, I dare say our young people are extremely lucky if only one person is corrupting them, while everyone else is doing them good.
37) 그러면 저는 제 자신이, 우리가 당신 말을 믿는다면, 대단히 처치 곤란한 곤경에 처했음을 깨닫는군요.이제 나에게 이걸 대답해 주세요. 당신은 말들에 대해서도 똑같은 것이 사실이라고 생각하나요? 각기 모두가 말들을 낫게 만드는 반면, 단 한 사람만이 그것들을 망치나요? 아니면 그 반대가, 단 한 사람 또는 적어도 아주 드문 사람들만이, 즉 말사육자들만이 그것들을 더 낫게 만들 수 있나요? 사람들이 말들을 망치는 동안에, 설령 그들이 말들을 가지고 일을 해야 하거나 그 말들을 사용한다면 그들은 말들을 망치지 않나요? 다른 모든 동물들에 대해서와 같이 말들에 대해서도 그것이 사실 아닌가요, 멜레토스? 물론 그렇지요, 당신과 아뉘토스가 그걸 부정하든 하지 않든 말이에요. 사실, 저는 감히 말하건데 우리의 젊은 사람들이 엄청난게 운이 좋다고 할 겁니다, 만일 오직 단 한 사람만 그들을 타락시키는 반면 다른 모든 이들은 각자가 그들을 좋게 만들고 있다면 말이에요.

38) All right, Meletus. Enough has been said to prove that you never were concerned about the young. You betray your irresponsibility plainly, because you have not cared at all about the charges on which you bring me before this court.
38) 좋아요, 멜레토스. 당신이 젊은 사람들에 대해 단 한 번도 염려한 일이 없다는 것을 증명하는 일은 충분하 이야기되었네요. 당신은 당신의 무책임함을 넘치도록 드러냈어요, 왜냐하면 당신이 나를 이 법정으로 데려온 죄목들에 대해 전혀 주의를 기울이지 않았으니 말이에요.

39) Furthemore, Meletus, tell us, in God's name, whether it is better to live among good fellow citizens or bad ones. Come sir, answer: I am not asking a hard question. Bad people have a harmful impact upon their closest companions at any given time, don't they, whereas good people have a good one?
     -Yes.
     Well, is there anyone who wants to be harmed by his companions rather than benefited? - Be a good fellow and keep on answering, as the law requires you to. Is there anyone who wants to be harmed?
     -Of course not.
     Now tell me this. In bringing me here, do you claim that I am corrupting and depraving the young intentionally or unintentionally?
     -Intentionally, so I maintain.
     Really, Meletus? Are you so much smarter at your age than I at mine as to realize that the bad have a harmful impact upon their closets companions at any given time, whereas the good have a beneficial effect? Am I, by contrast, so far gone in my stupidity as not to realize that if I make one of my companions vicious, I risk incurring harm at his hands? And am I, therefore, as you allege, doing so much damage intentionally?
39) 더 나아가서, 멜레토스, 우리에게 신들의 이름에 걸고 말해 주세요, 좋은 시민 친구들 사이에서 사는 것과 나쁜 친구들 사이에서 사는 것 중 어는 것이 더 나은지. 이봐요, 대답을 하세요. 제가 어려운 질문을 하는 게 아니잖아요. 나쁜 사람들은 그들의 가장 가까운 친구들에게 언제든지 해를 끼치는 반면에, 훌륭한 사람들은 이로운 영향을 끼치죠?
     -네.
     좋아요, 그의 동료들로부터 이익을 얻기보다도 차라리 피해를 받길 바라는 그런 사람이 누구라도 있나요? - 바람직한 시민으로서 계속해서 대답하도록 하세요, 법률이 당신에게 요구하고 있으니까요. 누구라도 해를 입기를 바라는 사람이 있나요?
     -물론 없습니다.
     이제 저에게 이걸 말해주세요. 저를 여기로 데려 오면서, 당신은 제가 젊은이들을 고의로 타락시키고 망쳐놓고 있다고 주장하나요, 아니면 무심코 그런다고 하는 건가요?
     -고의로 그러고 있다고, 저는 그렇게 주장합니다.
     정말로요, 멜레토스? 당신은, 나쁜 자들이 그들의 가까운 동료들에게 언제든지 해로운 영향을 끼치는 반면에 좋은 사람들은 유익한 영향을 끼친다는 것을 알 만큼, 당신 나이에서 이 나이에 이른 저보다도 그렇게나 훨신 더 영리하군요? 반대로 저는 만일 제가 저의 동료들 중 누군가를 사악하게 만든다면, 그의 손으로 해를 초래할 위험이 있다는 것을 깨닫지 못할 만큼 제 자신의 어리석음이 극에 달했고요? 그리고 그러니까 저는 당신이 주장하듯 그렇게나 엄청난 피해를 고의로 저지르고 있고요?

40) That I cannot accept from you, Meletus, and neither could anyone else, I imagine. Either I am not corrupting them - or if I am, I am doing so unintentionally; so either way your charge is false. But if I am corrupting them unintentionally, the law does not require me to be brought to court for such mistakes, but rather to be taken aside for private instruction and admonition - since I shall obviously stop doing unintentional damage, if I learn better. But you avoided association with me and were unwilling to instruct me. Instead you bring me to court, where the law requires you to bring people who need punishment rather than enlightenment.
40) 저는 당신에게서 그런 것은 받아들일 수가 없어요, 멜레토스, 그리고 제가 생각하기로는 다른 누구라도 그럴 수 없을 겁니다. 게다가 제가 젊은이들을 타락시키고 있지도 않고, 혹여 내가 그랬다 하더라도, 그건 무심코 저지르고 있는 것이죠. 그래서 당신이 비난하는 방식은 잘못된 겁니다. 만일 제가 그들을 무의식적으로 타락시키고 있다면, 법은 저에게 그런 실수 때문에 법정으로 이끌려 나오도록 요구하지 않고, 차라리 사적인 가르침과 주의를 위해 한쪽으로 불려가도록 요구합니다. - 제가 확실히 무의식적으로 해악을 저지르는 것을 관둘 테니까요, 만일 제가 더 낫게 배운다면 말입니다. 그러나 당신은 저와의 만남을 피했고 저에게 가르침을 주기를 바라지 않고 있었습니다. 대신에 당신은 저를 법정으로 데려왔죠. 일깨움보다는 처벌이 필요한 사람들을 데려오도록 법률이 당신에게 요구하는 그 곳으로 말이에요.

41) Very well, fellow Athenians. That part of my case is now proven: Meletus never cared about these matters, either a lot or a little. Nevertheless, Meletus, please tell us in what way you claim that I am corrupting our younger people. That is quite obvious, isn't it, from the indictment you drew up? It is by teaching them not to acknowledge the gods acknowledged by the city, but to accept new spiritual beings instead? You mean, don't you, that I am corrupting them by teaching them that?
     -I most emphatically do.
     좋습니다, 아테네인 여러분. 제 주장에 대한 그 부분은 이제 증명되었습니다. 멜레토스는 결코 이러한 문제들에 대해서 많든 적든 주의를 기울인 일이 없습니다. 그럼에도 불구하고, 멜레토스, 제가 우리의 젊은이들을 타락시키고 있다는 걸 당신이 어떤 방식으로 주장하고 있는 건지 말해 주세요. 그건 당신이 작성한 고소장에 아주 명백한 것이니까요, 그렇지 않나요? 그건 젊은이들에게 폴리스가 믿는 신들을 인정치 않도록, 그러나 대신에 새로운 영적 존재자들을 받아들이도록 가르침으로써 그러한 것이지요? 당신은 제가 그들을 그렇게 가르침으로써 망쳐놓고 있다고 말하는 거죠, 그렇지 않나요?
     -저는 단호하게 그렇다고 말합니다.

42) Then, Meletus, in the name of those very gods we are now discussing, please clarify the matter further for me, and for the jury here. You see, I cannot make out what you mean. Is it that I am teaching people to acknowledge that some gods exist - in which case it follows that I do acknowledge their existence myself as well, and am not a complete atheist, hence am not guilty on that count - and yet that those gods are not the ones acknowledged by the city, but different ones? Is that your charge against me - namely, that they are different? Or are you saying that I ackonwledge no gods at all myself, and teach the same to others?
    -I am saying the latter; you acknowledge no gods at all.
     그러면, 멜레토스, 바로 그러한 신들의 이름을 걸고 우리가 지금 논의하면서, 저에게, 그리고 여기 배심원들에게 그 점을 좀 더 명확하게 밝혀주세요. 보시다시피, 저는 당신이 무슨 말을 하는지 알아들을 수가 없어요. 그러니까 제가 사람들에게 몇몇 신들은 현존한다는 것을 인정하도록 - 그런 경우 제가 그 신들의 현존을 제 자신이 있다는 것만큼이나 인정하고 있다는 것, 그리고 제 자신이 완전한 무신론자가 아니라는 것, 따라서 저는 그 죄목에 대해서 죄가 없다는 것이 뒤따르지요 - 그리고 하지만 폴리스가 믿는 그런 신들은 전혀 아니고, 다른 어떤 신들이 있다고 가르치고 있다는 건가요? 그게 저에 대한 당신의 고소인가요 - 말하자면, 그 신들이 다른 신들이라는 것? 아니면 당신은 제가 제 자신은 어떠한 신들도 전혀 인정하지 않고, 그와 같은 것을 다른 이들에게 가르친다고 말하고 있는 건가요?
     -저는 뒤엣 것을 말하고 있습니다. 당신은 어떤 신들도 전혀 인정하지 않습니다.

43) What ever makes you say that, Meletus, you strange fellow? Do I not even acknowledge, then, with the rest of mankind, that the sun and the moon are gods?
     -By God, he does not, mambers of the jury, since he claims that the sun is made of rock, and the moon of earth!
     도대체 무엇이 당신에게 그런 얘기를 하도록 만드는 건가요, 멜레토스, 당신은 이상한 친구로군요? 그러니까 제가 모든 인류 이외의 것들과 더불어, 해와 달이 신들이라는 것도 인정하지 않는다는 건가요?
     -신들에 맹세코, 그는 믿지 않습니다, 배심원 여러분, 왜냐하면 그는 해가 바위로 이루어졌고 달은 땅으로 이루어졌다고 주장하니까요!

44) My dear Meletus, do you imagine that it is Anaxagoras you are accusing? Do you have such contempt for the jury, and imagine them so illiterate as not to know that books by Anaxagoras of Clazomenae are crammed with such assertions? What's more, are the young learning those things from me when they can acquire them at the bookstalls, now and then, for a drachma at most, and so ridicule Socrates if he claims those ideas for his own, especially when they are so bizarre? In God's name, do you really think me as crazy as that? Do I acknowledge the existence of no god at all?
     -By God no, none whatever.
44) 나의 친애하는 멜레토스, 당신은 지금 아낙사고라스를 고소하고 있다고 상상하는 건가요? 당신은 배심원분들을 그렇게 얕잡아 보고, 그분들께서 클라조메나이의 아낙사고라스가 쓴 그 책들이 그런 주장들로 가득 채워져 있다는 걸 알지 못할 만큼 글줄도 모른다고 생각해요? 더군다나, 나로부터 그런 것들을 배운 젊은 사람들이 서점들에서 그것들을 언제든지 기껏해야 1 드라크메로 구할 수 있을 때, 소크라테스를 두고 만일 그가 그러한 생각들을 그 자신의 것이라고, 특히나 그 생각들이 그렇게나 기이할 때 그렇게 주장한다면 얼마나 조롱하겠어요? 신들의 이름에 걸고, 당신은 정말로 제가 그만큼이나 제정신이 아니라고 생각하나요? 제가 어떤 신도 전혀 존재하지 않는다고 인정한다고요?
      -신들에 걸고 전혀, 아무것도요.

45) I can't believe you, Meletus - nor, I think, can you believe yourself. To my mind, fellow Athenians, this fellow is an impudent scoundrel who has framed this indictment out of sheer wanton impudence and insolence. He seems to have devised a sort of riddle in order to try me out: "Will Socrates the Wise tumble to my nice self-contradiction? Or shall I fool him along with my other listeners?" You see, he seems to me to be contradicting himself in the indictment. It's as if he were saying: "Socrates is guilty of not acknowledging gods, but of acknowledging gods"; and yet that is sheer tomfoolery.
     저는 당신을 믿을 수가 없군요, 멜레토스 - 제 생각엔 당신도 당신 자신을 믿을 수 없을 거에요. 제 생각에, 에테네인 여러분, 이 친구는 순전히 악의적인 몰염치와 거만함으로 이러한 기소를 계획한 버르장머리 없는 악당입니다. 그는 저를 시험해 보려고 일종의 수수께끼를 고안한 듯합니다. "저 현자 소크라테스는 내 교묘한 모순을 단박에 알아차릴까? 아니면 내가 저 사람을 나의 다른 청중들과 마찬가지로 속여넘길까?" 보시다시피, 그는 제게 기소에 있어서 그 자신에 모순되는 것으로 보입니다. 그것은 마치 그가 이렇게 말하고 있는 것 같습니다. "소크라테스는 신들을 인정하지 않는 죄를 지었지만, 신들을 인정하는 죄를 짓고 있다", 그런데 그건 완전히 바보 같은 짓입니다.

46) I ask you to examine with me, gentlemen, just how that appears to be his meaning. Answer for us, Meletus; and the rest of you, please remember my initial request not to protest if I conduct the argument in my usual manner.
46) 여러분, 저는 여러분들께 저와 함께 그의 말뜻이 바로 어떤 것으로 드러날지 검토해주시길 청합니다. 우리에게 대답하세요, 멜레토스, 그리고 다른 여러분들께서는, 만일 제가 제 일상적인 방법으로 논변을 이끌지라도 반발치 말아주십사 처음 부탁드렸던 것을 기억해 주시기 바랍니다.

47) Is there anyone in the world, Meletus, who acknowledges that human phenomena exist, yet does not acknowledge human beings? - Require him to answer, gentlemen, and not to raise all kinds of confused objections. Is there anyone who does not acknowledge horses, yet does acknowledge equestrian phenomena? Or who does not acknowledge that musicians exist, yet does acknowledge musical phenomena?
47) 세상 그 누구가, 멜레토스, 인간들이 벌이는 일들이 있다는 걸 인정하지만, 인간인 자들이 있다는 건 인정하지 않습니까? - 여러분, 그가 대답하도록 요구하십시오, 그리고 모든 종류의 혼란된 반박들을 일으키지 않도록 요구하십시오. 말들은 인정하지 않지만, 그런데도 말타기와 관련된 일들은 인정하는 사람이 대체 누가 있죠? 또는 악사들이 있다는 걸 인정치 않으면서, 그런데도 음악을 연주하는 일들은 인정하는 사람이 누군가요?

48) There is no one, excellent fellow: if you don't wish to answer, I must answer for you, and for the jurors here. But at least answer my next question yourself. Is there anyone who acknowledges that spiritual phenomena exist, yet does not acknowledge spirits?
     -No.
48) 대단한 친구, 아무도 없습니다. 만일 당신이 대답하고 싶지 않다면, 제가 당신에게, 그리고 이 자리의 배심원 여러분께 대답해야만 하겠네요. 그러나 최소한 나의 이 다음 질문은 당신 자신이 대답하도록 하세요. 영적인 일들이 있다는 건 인정하지만, 그런데도 정령들은 인정치 않는 그런 누구라도 있나요?
     -아니오.

49) How good of you to answer - albeit reluctantly and under compulsion from the jury. Well now, you say that I acknowledge spiritual beings and teach other to do so. Whether they actually be new or old is no matter: I do at any rate, by your account, acknowledge spiritual beings, which you have also mentioned in your sworn deposition. But if I acknowledge spiritual beings, then surely it follows quite inevitably that I must acknowledge spirits. Is that not so? -Yes, it is so: I assume your agreement, since you don't answer. But we regard spirits, don't we, as either gods or children of gods? Yes or no?
     -Yes.
     당신은 어찌나 대답을 잘 하는지 - 비록 마지못해 배심원들로부터 강요받아서라고 할지라도 말이죠. 자, 이제 당신은 제가 영적인 존재들을 인정하고 다른 이들에게도 그렇게 가르친다고 말합니다. 그 존재들이 새로운 것이든 오래된 것이든 문제가 아니고요. 당신 설명에 의하면, 저는 어떤 식으로든 영적인 존재자들을 인정하는군요, 당신이 또한 당신의 그 서약한 증언에서 언급했던 그런 존재자들을요. 하지만 만일 영적인 존재자들을 제가 인정한다면, 확실히 아주 필연적으로 저는 정령들을 인정하는 게 틀림없다는 것이 뒤따르네요. 그렇지 않나요? -네, 그렇습니다, 라고 당신이 동의한다고 칠게요, 왜냐하면 당신이 대답을 않으니까요. 하지만 우리는 정령들을 신들이나 신들의 자식들로 생각하죠, 그렇지 않나요? 그런가요, 아닌가요?
     -그렇습니다.

50) Then given that I do believe in spirits, as you say, if spirits are gods of some sort, this is precisely what I claim when I say that you are presenting us with a riddle and making fun of us: you are saying that I do not believe in gods, and yet again that I do believe in gods, seeing that I believe in spirits.
     그러면 당신이 말한 것처럼 만일 정령들이 어떤 종류의 신들이라면 제가 정령들을 믿는다는 것이 제시되었으니, 이것이 제가 당신은 우리에게 하나의 수수께끼를 보여주면서 우리를 웃겨주고 있다고 말할 때 주장하는 바로 그것이네요: 당신은 제가 신들을 믿지 않는다고 말하면서, 그런데도 또 다시 제가 정령들을 믿는다는 점으로 보아 제가 신들을 믿는다고도 말하고 있다는 것 말이에요.

51) On the other hand, if spirits are children of gods, some sort of bastard offspring from nymphs - or from whomever they are traditionally said, in each case, to be born - then who in the world could ever believe that there were children of gods, yet no gods? That would be just as absurd as accepting the existence of children of horses and asses - namely, mules - yet rejecting the existence of horses or asses!
51) 다른 한편, 만일 정령들이 신들의 자식들이라면, 요정들로부터의 어떤 종류의 좀 못한 자손이라면 - 또는 전통적으로 각각의 경우 그런 어떤 요정들로부터든 태어난 것으로 이야기되는 - 세상의 그 누가 도대체 신들의 자식들이 있으리라 믿으면서도, 신들은 아무도 없다고 여길 수 있겠어요? 그건 말들과 나귀들의 새끼들이 있음을 인정하면서도 - 그러니까, 노새들 - 말들과 나귀들이 있다는 건 부정하는 바로 그것만큼이나 터무니없는 짓일 거에요!

52) In short, Meletus, you can only have drafted this either by way of trying us out, or because you were at a loss how to charge me with a genuine offence. How could you possibly persuade anyone with even the slightest intelligence that someone who accepts spiritual beings does not also accept divine ones, and again that the same person also accepts neither spirits nor gods nor heroes? There is no conceivable way.
52) 간단히 말해서, 멜레토스, 당신은 이걸 우리를 시험해 보려고 작성하게 되었거나, 아니면 당신이 어떻게 나를 진짜 위법행위로 고소할지 당황에 빠졌기 때문에 작성하게 되었을 수밖에 없어요. 어떻게 당신이 영적인 존재들을 허용하는 누군가가 또한 신성한 존재들을 인정하지 않는다고, 그리고 다시 똑같은 사람이 정령들도 신들도 영웅들도 아무도 인정하지 않는다고, 조금이라도 머리가 있는 사람이라면 그 누구에게든 설득하는 게 가당키나 할 수 있겠어요? 아무런 방법도 생각할 수 없어요.

53) But enough, fellow Athenians. It needs no long defence, I think, to show that I am not guilty of the charges in Meletus' indictment; the foregoing will suffice. You may be sure, though, that what I was saying earlier is true: I have earned great hostility among many people. And that is what will convict me, if I am convicted: not Meletus or Anytus, but the slander and malice of the crowd. They have certainly convicted many other good men as well, and I imagine they will do so again; there is no risk of their stopping with me.
53) 하지만 이걸로 충분합니다, 아테네인 여러분. 제 생각엔 제가 멜레토스의 기소문에 적힌 죄목들에 속하는 죄를 지었다는 걸 보여주는 데에 반론할 필요가 없습니다. 앞서 말한 것들로 충분할 겁니다. 비록 여러분들께서 제가 더 일찍이 이야기하고 있었던 것이 참이라는 걸 확실하실 테지만 말씀입니다. 그건 제가 많은 사람들로부터 엄청난 적대심을 얻었다는 것입니다. 그리고 제가 유죄라면 그것이 제 죄일 것입니다. 멜레토스나 아뉘토스가 아니라, 군중들의 그 모략과 적의 말입니다. 그것들은 확실히 다른 많은 훌륭한 이들도 유죄로 몰았고, 저는 그것들이 또 다시 그리하리라 생각합니다. 그것들이 제게서 그칠 위험은 없으니 말입니다.

54) Now someone may perhaps say: "Well then, are you not ashamed, Socrates, to have pursued a way of life which has now put you at risk of death?"
54) 이제 누군가 말할는지 모르겠습니다: "좋습니다만, 당신은 부끄럽지 않나요, 소크라테스, 지금 당신을 죽음의 위험에 처하도록 만든 그런 삶의 방식을 추구하였다는 게?"

55) But it may be fair for me to answer him as follows: "You are sadly mistaken, fellow, if you suppose that a man with even a grain of self-respect should reckon up the risks of living or dying, rather than simply consider, whenever he does something, whether his actions are just or unjust, the deeds of a good man or a bad one. By your principles, presumably, all those demigods who died in the plain of Troy were inferior creatures - yes, even the son of Thetis, who showed so much scorn for danger, when the alternative was to endure dishonour. Thus, when he was eager to slay Hector, his mother, goddess that she was, spoke to him - something like this, I fancy:

     My child, if thou dost avenge the murder of thy friend, Patroclus,
    And dost slay Hector, then straightway [so runs the poem]
     Shalt thou die thyself, since doom is prepared for thee
     Next after Hector's.

55) 하지만 저는 그에게 다음과 같이 답하는 것이 온당할 것 같습니다. "당신은 안타깝게도 착각하고 있어요, 친구여, 만일 당신이 아주 조금이라도 자존심을 가진 사람이, 단순히 그가 무엇을 하는 순간이든, 그의 행위들이 올바른지 정의로운지, 훌륭한 사람의 행실인지 나쁜 사람의 것인지 고려하지는 않고, 오히려 살고 죽는 위험들을 모두 낱낱이 헤아려 보아야 한다고 생각한다면 말이에요. 생각컨데 당신의 원칙들에 따르자면 트로이의 평원에서 죽어갔던 그 모든 신들과 같은 이들은 하등한 피조물들이었을 거에요 - 네, 불명예를 감수할 대안이 있었을 때 위험에 대해 그렇게나 큰 경멸을 보여주었던 테티스의 아들조차 말이지요. 그래서, 그가 헥토를 죽이고자 열망하였을 때, 그의 어머니, 여신이었던 그분께서는 그에게 말씀하셨었지요 - 제가 믿기로는 이와 같은 어떤 것을 말입니다.

      나의 아이야, 만일 네가 너의 친구 파트로클로스를 살해한 자에게 보복을 한다면,
      그리고 헥토의 목을 벤다면, 그리하여 당장에 [그 시는 그렇게 계속되지요]
      너는 너 자신을 죽음에 이르게 하리라, 너를 위한 숙명이 예비되어 있는 까닭에
      헥토르의 운명 다음으로.

56) But though he heard that, he made light of death and danger, since he feared far more to live as a base man, and to fail to avenge his dear ones. The poem goes on:

     Then straightway let me die, once I have given the wrongdoer
     His deserts, less I remain here by the beak-prowed ships,
     An object of derision, and a burden upon the earth.

     Can you suppose that he gave any thought to death or danger?
56) 그러나 그는 그것을 들었음에도, 죽음과 위험을 가소로이 여겼습니다, 그가 야비한 자로 살아갈 것을, 그리고 그의 귀애하는 자를 복수해주는 일에 실패할 것을 훨씬 더 두려워하였기 때문이지요. 시는 계속됩니다.

     그렇게 제가 곧장 죽도록 두소서, 제가 부정한 짓을 행한 자에게 내린 다음에는
     그의 죗값을, 제가 여기 부리처럼 휜 함선들을 곁에,
     웃음거리로, 대지 위에 짐처럼 남겨지지 않기 위해서라면.

     당신은 그가 죽음이나 위험을 안중에나 두었다고 생각할 수 있나요?

57) You see, here is the truth of the matter, fellow Athenians. Wherever a man has taken up a position because he considers it best, or has been posted there by his commander, that is where I believe he should remain, steadfast in danger, taking no account at all of death or of anything else rather than dishonour. I would therefore have been acting absurdly, fellow Athenians, if when assigned to a post at Potidaea, Amphipolis, or Delium by the superiors you had elected to command me, I remained where I was posted on those occasions at the risk of death, if ever any man did; whereas now that the god assigns me, as I became completely convinced, to the duty of leading the philosophical life by examining myself and others, I desert that post from fear of death or anything else. Yes, that would be unthingkable; and then I truly should deserve to be brought to court for failing to acknowledge the gods' existence, in that I was disobedient to the oracle, was afraid of death, and thought I was wise when I was not.
57) 보시다시피, 그 문제에 대한 진실이 여기 있습니다, 아테네인 여러분. 한 사람이, 그가 그것을 최선으로 여겨서이든, 아니면 그의 지휘자에 의해 그 곳에 배치되었던 때문이든, 한 자리를 지키고 있을 때에는 언제라도, 그곳이 제가 믿기로는, 변함없이 위험 속에, 불명예가 아닌 차라리 죽음이나 다른 어떤 것에 대해서든 전혀 조금도 고려하지 않으면서 그가 머물러야 할 자리입니다. 그러므로 저는 부조리한 짓을 저지르고 있었던 것일 터입니다, 아테네인 여러분, 만일 포테이다이아에서, 암피폴리스에서, 또는 델리온에서, 여러분들께서 저를 명령하도록 선출하였던 지휘관들에 의해 한 자리에 배치되었을 때, 그 때마다 죽음의 위험을 무릅쓰고 제가 배치된 그 곳에 다른 누구라도 그러했을 것처럼 머물러 있었던 반면에 그 신께서 제가 완전히 이해하게 되었던 것처럼, 저 자신과 다른 사람들을 검토함으로써 지혜를 사랑하는 삶을 살아가는 책임을 저에게 부과하신 지금은, 그 자리를 죽음이나 다른 어떤 것에 대한 두려움으로 져버린다면 말씀입니다. 네, 그런 것은 생각할 수조차 없을 것입니다. 그리고 그래서 저는 진정으로 신들의 현존을 인정치 않는다고 법정에 이끌려올 만할 것입니다, 제가 그 신탁을 거슬렀다는 점에서, 죽음을 두려워하였고, 제 자신이 지혜롭지 않은데도 지혜롭다고 여겼다는 점에서 말입니다.

58) After all, gentlemen, the fear of death amounts simply to thinking one is wise when one is not: it is thinking one knows something one does not know. No one knows, you see, whether death may not in fact prove the greatest of all blessings for mankind; but people fear it as if they knew it for certain to be the greatest of evils. And yet to think that one knows what one does not know must surely be the kind of folly which is reprehensible.
58) 무엇보다도, 여러분, 죽음에 대한 두려움은 단순히 현명하지 않으면서도 현명하다고 생각하는 것이 됩니다. 즉 그것은 어떤 것을 모르는 사람이 그것을 안다고 생각하는 것입니다. 아시다시피 그 누구도 죽음이 사실은 인류의 모축복들 중에서도 가장 대단한 것으로 증명될는지 알지 못합니다. 그러나 사람들은 마치 죽음이 안 좋은 것들 중에서 가장 큰 것임이 틀림없다는 것을 아는 것마냥 죽음을 두려워합니다. 하지만 누군가 모르는 것을 안다고 생각하는 것은 부끄러울 만한 바보스러운 종류의 것입니다.

59) On this especially, gentlemen, that may be the nature of my own advantage over most people. If I really were to claim to be wiser than anyone in any respect, it would consist simply in this: just as I do not possess adequate knowledge of life in Hades, so I also realize that I do not possess it; whereas acting unjustly in disobedience to one's betters, whether god or human being, is something I know to be evil and shameful. Hence I shall never fear or flee from something which may indeed be a good for all I know, rather than from things I know to be evils.
59) 특히 이에 대해서, 여러분, 그것이 아마도 모든 사람들에 대해 제 자신의 나은 점의 본성일 것입니다. 만일 제가 정말로 어떤 점에서 누구보다 더 지혜롭다고 주장한다면, 그것은 단순히 이 점에 있을 것입니다. 그러니까 제가 하데스(저승)에서의 삶에 대해 충분히 알지 못하고, 그래서 또한 제가 그런 앎을 지니고 있지 않다는 걸 실감하지만, 반면에 올바르지 못하게 더 나은 이를, 그가 신이든 인간이든 거슬러 행동한다는 것은 제가 나쁘고 부끄러운 것임을 아는 그런 것임과 꼭 같이 말입니다. 따라서 저는 절대로, 더욱이 의외로 좋은 것일지도 모를 어떤 것을, 되레 제가 나쁜 것들임을 아는 그러한 것들로부터보다도, 두려워하거나 피하지는 않을 것입니다.

-蟲-
A Failed Attempt at Solving The Gettier Problem
게티어 문제 해결에 실패한 시도

1) In the next section, we shall discuss how the concept of defeasibility can be exploited for the purpose of solving the Gettier problem. In this section, we shall consider a different solution, one that has turned out to be a dead end.<20> The gist of this solution is expressed by the following condition:

C1    S's justification for p does not depend on any falsehood.

This condition gives us the right result for Gettier's original case about Smith and his ruminations about Jones's Ford. When Smith deduces (2), he uses (1) as a premise. But (1) is false, and thus his justification for believing (2) depends on a falsehood. So S does not meet condition C1, and hence does not know (2).
1) 다음 절에서, 우리는 어떻게 상쇄가능성의 개념이 게티어 문제를 해결하기 위한 목적으로 활용될 수 있는지 논의할 것이다. 이 절에서는 우리는 하나의 다른 해법, 막다른 길로 밝혀진 그러한 해법을 고찰할 것이다.<20> 이 해법의 요지는 다음 조건으로 표현된다:

C1    S의 p에 대한 정당성은 어떠한 거짓에도 의존하지 않는다.

이 조건은 우리에게 스미스와 존스의 Ford차에 대한 그의 숙고에 대한 게티어의 원래 사례에 대해 옳은 결론을 준다. 스미스가 (2)를 연역할 때, 그는 전제로서 (1)을 사용한다. 그러나 (1)은 거짓이고, 그래서 (2)를 믿기 위한 그의 정당성은 거짓에 의존한다. 그래서 S는 조건 C1을 충족시키지 않고, 따라서 (2)를 알지 못한다.

2) Nevertheless, supplementing the JTB account with C1 does not solve the Gettier problem. There are other Gettier-type cases in which a justified true belief meets C1, yet does not qualify as an instance of knowledge. Let's consider a case of that type.
2) 그럼에도 불구하고, JTB 설명에 C1을 보충하는 것은 게티어 문제를 해결하지 못한다. C1을 충족시키는 정당화된 참인 믿음이지만 앎의 사례로서 자격을 얻지는 못하는 다른 게티어 유형의 사례들이 있다. 그러한 유형의 사례 하나를 고찰해 보도록 하자.

3) Suppose you look out your window and see your cat in your yard. Naturally, you believe, "My cat is in the yard." But what you take to be your cat isn't cat at all but a hologram so perfect that, by vision alone, it cannot be distinguished from an actual cat. Unbeknownst to you, your neighbor has acquired the equipment to project holograms, and he is playing a trick on you. Furthermore, suppose that your cat is in your yard, but is lying right underneath the window, where you cannot see her from your present point of view. Your belief that your cat is in your yard, then, happens to be true. Furthermore, you are completely justified in believing that your cat is in the yard because your cat can be found there quite frequently, the hologram bears a striking resemblance to your cat, and you have no grounds whatever to suspect your neighbor of projecting cat-holograms into your yard.
3) 당신이 당신의 창밖을 바라보고 당신의 마당에 있는 당신의 고양이를 본다고 가정해 보자. 자연스럽게, 당신은 "나의 고양이가 마당에 있다"라고 믿는다. 그러나 당신이 당신의 고양이라고 받아들인 것은 전혀 고양이가 아니고 단지 눈만으로는 그것이 실제 고양이와 구별될 수 없는 아주 완벽한 홀로그램이다. 당신이 모르는 사이에, 당신의 이웃이 홀로그램 영사기를 구했고, 그는 지금 당신을 속이고 있다. 더 나아가서, 당신의 고양이가 당신의 마당에 있지만, 창문 바로 밑, 당신이 당신이 바라보는 현재 위치에서는 그 고양이를 볼 수 없는 곳에 누워 있다고 가정하자. 당신의 고양이가 당신의 마당에 있다는 당신의 믿음은 그래서 우연히도 참이다. 더욱이, 당신은 당신의 고양이가 당신의 마당에 있다고 믿음에 있어서 완전히 정당화되는데 왜냐하면 당신의 고양이가 매우 자주 거기에서 발견될 수 있었고, 그 홀로그램은 당신의 고양이와 현저하게 유사한 것을 내놓으며, 당신은 당신의 이웃이 고양이-홀로그램을 당신의 마당에 영사하고 있으리라 의심할 그 어떤 근거도 가지고 있지 않기 때문이다.

4) In this case, the proposition you are justified in believing - my cat is in the yard - is not deduced from anything. Rather, you have a certain perceptual experience (that of seeing what seems to you to be your cat), and that experience triggers in you the belief that your cat is in the yard. Your justification for your belief is not, therefore, inferred from a further proposition that is false, and hence does not depend on any falsehood. Hence, if the JTB account is supplemented with C1, we get the wrong result for cases like this one.
4) 이 사례에서, 당신이 믿음 - 나의 고양이는 마당에 있다 - 에 있어서 정당화된다는 명제는 어떤 것으로부터도 연역되지 않는다. 오히려, 당신은 특정한 지각 경험 (당신에게 당신의 고양이로 보이는 어떤 것을 보고 있다는 경험) 을 가지고, 그 경험은 당신 안에서 당신의 고양이가 그 마당에 있다는 믿음을 촉발시킨다. 당신의 믿음에 대한 당신의 정당화는 그러므로 그 이상의 거짓인 명제로부터 추론되지 않으며, 따라서 어떠한 거짓에도 의존하지 않는다. 따라서, 만일 JTB 설명이 C1으로 보충된다면, 우리는 이와 같은 사례들에 대해 잘못된 결과를 얻게 된다.


A Defeasibility Solution to The Gettier Problem
게티어 문제에 대한 상쇄자 해법

1) Since Gettier examples involve the factual defeat of S's justification for believing the proposition in question, many epistemologists attempt to solve the Gettier problem by formulating conditions that require that S's justification remain undefeated.<21> There are, of course, rival proposals, among which this defeasibility approach is just one contender. Indeed, it would be fair to say that what is conspicuous about the literature on the Gettier problem is, first, the multitude of approaches and, second, the lack of a settled solution. Unfortunately, for reasons of space, we cannot go beyond a discussion of the defeasibility approach here.
1) 게티어 예시들은 S가 문제가 되는 명제를 믿기 위한 정당성에 대해 사실적 상쇄자를 포함하므로, 다수의 인식론자들은 게티어 문제를 S의 정당성이 상쇄되지 않은 채로 남을 것을 요구하는 조건들을 만들어냄으로써 해결하고자 시도한다.<21> 물론 경쟁하는 제안들이 있고, 이러한 상쇄가능성 접근법은 그 중에서 단지 하나의 경쟁이론일 따름이다. 덧붙여서, 게티어 문제에 대해서 논문에 대해서 눈에 띄는 것은, 우선, 접근법들의 다양함이고, 둘째로, 정착된 해법의 부재(결핍)이다. 불행하게도, 지면의 이유들로, 우리는 상쇄가능성 접근법에 대한 논의에서 더 넘어갈 수 없다.

2) The defeasibility approach is based on the idea that the full range of relevant facts must not defeat - in the factual sense of the word - S's justification for believing that p. The following condition captures the idea behind this approach:

C2    There is no proposition d such that d factually defeats S's evidence for believing that p.

C2 gives us the right result for the Gettier case we have considered. In Gettier's case about Smith and Jones's Ford, we assumed the following proposition is true:

(1) The Ford Jones is currently driving is a Hertz rental car.

(1) defeats the epistemizing potential of Smith's justification for believing that either Jones owns a Ford or Brown is in Barcelona. Hence, although Smith is justified in believing that either Jones owns a Ford or Brown is in Barcelona, he does not know it.
2) 상쇄가능성 접근법은 관련 사실들의 전 영역이 반드시 S가 p라는 것을 믿기 위한 정당성을 - 세계의 사실적 의미에서 - 상쇄시키지 않아야만 한다는 발상에 기초된다. 다음 조건은 이러한 접근법의 이면에서 그 발상을 포착한다:

C2    p를 믿기 위한 S의 증거를 사실적으로 상쇄시키는 그러한 명제 d는 아무것도 없다.

C2는 우리에게 우리가 고찰하였던 게티어 사례에 대한 옳은 결론을 준다. 스미스와 존스의 Ford차에 대한 게티어의 사례에서 우리는 다음 명제가 참이라고 가정하였다:

(1) 존스가 현재 몰고 있는 Ford차는 헤르츠가 빌려준 차이다.

(1)은 존스가 Ford차를 소유하고 있거나 브라운이 바르셀로나에 있다고 믿기 위한 스미스의 정당성에 있어서 그 인식화 가능성을 상쇄시킨다. 따라서, 비록 존스가 Ford차를 소유하고 있거나 브라운이 바르셀로나에 있다고 믿음에 있어서 스미스가 정당화된다 할지라도, 그는 그 사실을 알지 못한다.

3) In the case about your cat in the yard, this is true proposition:

(2) What you are looking at is a hologram that looks like a cat.

(2) is a factual defeater for the evidence you have for believing that your cat is in the yard. Hence you don't know that your cat is in the yard.
3) 마당의 당신 고양이에 대한 사례에서, 이것은 참인 명제이다:

(2) 당신이 바라보고 있는 것은 고양이처럼 보이는 홀로그램이다.

(2)는 당신의 고양이가 마당 안에 있다는 믿음에 대해 당신이 지니는 정당성에 대한 사실적 상쇄자이다. 따라서 당신은 당신의 고양이가 마당 안에 있다는 것을 알지 못한다.

4) It is important to note that when we apply C2 to particular cases, we must take into account the total body of S's evidence. The following case illustrates this. Suppose S's justified in believing

(3) This sheet of paper is blue

because the sheet of paper looks blue to him and he knows that

(4) This sheet of paper is a grade-appeal form and all such form are blue.

Suppose further that the following proposition is true:

(5) Blue light is shining on it.

(5) defeats S's perceptual evidence for believing (3). Nevertheless, C2 is met because (5) does not defeat S's evidence for (3) in its totality. Part of S's evidence is his knowledge of (4), which by itself is sufficient to justify S's belief. Hence, although there is a true proposition that defeats one element of S's justification for (3), S has nevertheless knowledge of (3).
4) 우리가 C2를 특수한 사례들에 적용할 때, 우리가 반드시 S의 증거에 속하는 총체를 고려해 넣어야만 한다는 것을 주의하는 것이 중요하다. 다음 사례는 이 점을 설명한다. S가 다음과 같이 믿음에 있어서 정당화된다고 생각해 보자.

(3) 이 한 장의 종이는 파랗다.

왜냐하면 그 한 장의 종이는 그에게 파랗게 보이고 또한 그가 다음과 같은 사실을 알기 때문이다.

(4) 이 한 장의 종이는 성적-이의 양식서이고 그러한 양식서는 모두 파랗다.

더 나아가 다음 명제가 참이라고 생각해 보자:

(5) 파란 빛이 그 양식서를 비추고 있다.
(5)는 (3)을 믿음에 대한 S의 지각적 증거를 상쇄시킨다. 그럼에도 불구하고, C2는 충족되는데 왜냐하면 (5)는 (3)에 대한 S의 증거를 그 전체에 있어서 상쇄시키지는 않기 때문이다. S의 증거 중 일부는 (4)에 대한 그의 앎, 그 자체로 S의 믿음을 정당화시키기에 충분한 그러한 앎이다. 따라서, 비록 S의 (3)에 대한 정당화의 한 요소를 상쇄시키는 참 명제가 있다 할지라도, S는 그럼에도 불구하고 (3)에 대한 앎을 가진다.


An Amended JTB Account of Knowledge
앎에 대한 수정된 JTB 설명

1) According to the traditional account, knowledge involves three ingredients: truth, belief, and justification. The Gettier problem demonstrates that a further ingredient is needed. Advocates of the defeasibility approach propose that this missing ingredient is the absence of factual defeat. Thus they would add a fourth condition to the traditional account:

S knows that p if and only if
(i) p is true;
(ii) S believes that p;
(iii) S has justifying evidence for believing that p;
(iv) there is no proposition d that factually defeats S's evidence for believing that p.

Note, however, that condition (iv) makes the truth condition redundant.<22> Whenever p is false, there will be a true proposition d that defeats whatever justification S may have for p: the negation of p, and any proposition that entails the falsehood of p. Hence condition (iv) fails to be met whenever p is false. To see why this is so, recall our definition of evidential defeat.

d defeats e as evidence for p if and only if e is evidence for believing that p, but e in conjunction with d is not evidence for believing that p.

Suppose p is false, and S has evidence e in support of p. Since p is false, the negation of p, not-p, is true. We must ask, therefore, whether e in conjunction with not-p is evidence for e. And the answer is that it is not, for nothing is evidence for p if it is conjoined with the negation of p.(?)
1) 전통적인 설명에 따르자면, 앎은 세 가지 요소들을 포함한다. 즉 참, 믿음, 그리고 정당성이 그것이다. 게티어 문제는 그 이상의 요소가 필요하다는 것을 증명한다. 상쇄가능성 접근법의 지지자들은 이 빠져 있는 요소가 사실적 상쇄의 부재라고 제안한다. 그래서 그들은 전통적 설명에 네 번째 조건을 추가할 것이다.

S는 p를 안다 iff
(i) p가 참이고,
(ii) S가 p라는 것을 믿으며,
(iii) S는 p라고 믿는 것을 정당화하는 증거를 가지고,
(iv) S가 p라고 믿는 증거를 사실적으로 상쇄시키는 명제 d는 아무것도 없다.

그렇지만 조건 (iv)가 참 조건을 잉여적인 것으로 만든다는 점을 주의하도록 하자.<22> p가 거짓인 언제라도, S가 p에 대해 가질 그 어떠한 정당화든 상쇄시키는 참 명제 d, p의 부정이 있을 것이고, p의 거짓을 수반하는 어떤 명제든지 있을 것이다. 따라서 조건 (iv)는 p가 거짓인 그 어느 때라도 충족되는 데에 실패한다. 이것이 왜 그러한지 보기 위해, 증거적 상쇄에 대한 우리의 정의를 상기해 보도록 하자.

d는 p에 대한 증거로서의 e를 e가 p라고 믿는 증거이지만 e와 d의 결합에 있어서는 p라고 믿는 증거가 아니라면 그리고 오로지 그러할 때에만 상쇄시킨다.

p가 거짓이고, S는 p를 뒷받침함에 있어서 증거 e를 가진다고 생각해 보자. p가 거짓이므로, p의 부정 not-p는 참이다. 그러므로 우리는 e가 not-p와 관련하여 e에 대한 증거인지 물어야만 한다. 그리고 그 대답은 그렇지 않다는 것인데, 왜냐하면 만일 p의 부정과 관련된다면 그 어떤 것도 p에 대한 증거가 아니기 때문이다.(?)

2) Let us consider a simple case of believing a false proposition. Suppose that, while thinking a walk in the countryside, you believe

(1) There is a sheep in the field

because you see an animal that looks like a sheep to you. In fact, it is a dog that, because of some bizarre features, looks from your point of view exactly like a sheep. Thus the following proposition is true:

(2) What you are looking at is actually a dog that looks like a sheep.

This proposition factually defeats your evidence for believing (1). If you were to acquire evidence for (2), you would no longer be justified in believing (1). So condition (iv) of the amended JTB account of knowledge is not met, and we get the result that you don't know there is a sheep in the field.
2) 거짓 명제를 믿는 단순한 사례 하나를 고찰해 보도록 하자. 생각하면서 시골길을 걷는 동안에, 당신이 다음과 같이 믿었다고 생각해 보자.

(1) 들판에 양 한 마리가 있다.

왜냐하면 당신은 당신에게 한 마리 양처럼 보이는 동물 하나를 보기 때문이다. 사실, 그것은 약간 독특한 외형들 때문에 당신이 보는 위치에서 양처럼 보이는 개이다. 따라서 다음 명제는 참이다:

(2) 당신이 바라보고 있는 것은 사실 양처럼 보이는 개이다.

이 명제는 사실적으로 당신의 (1)이라는 믿는 증거를 상쇄시킨다. 만일 당신이 (2)에 대한 증거를 획득하게 된다면 당신은 더 이상 (1)을 믿음에 있어서 정당화되지 않을 것이다. 그래서 앎에 대한 수정된 JTB 설명의 조건 (iv)는 충족되지 않고, 우리는 들판에 양이 있다는 것을 당신은 알지 못한다는 결과를 얻는다.

3) As you can see, then, adding condition (iv) to the traditional account makes condition (i) superfluous. The amended account, therefore, can be stated in three conditions only:

S knows that p if and only if
(i) S believes that p;
(ii) S has justifying evidence for believing that p;
(iii) there is no proposition d that factually defeats S's evidence for believing that p.

3) 당신이 볼 수 있는 것처럼, 그래서, 전통적 설명에 조건 (iv)를 추가하는 것은 조건 (i)을 불필요한 것으로 만든다. 수정된 설명은 그러므로 단지 세 가지 조건들만으로 진술될 수 있다:

S는 p라는 것을 안다 iff
(i) S가 p라는 것을 믿는다;
(ii) S는 p라고 믿는 것을 정당화하는 증거를 가진다;
(iii) S가 p라고 믿는 증거를 사실적으로 상쇄시키는 명제 d는 아무것도 없다.

 

 

-蟲-

Conclusive and Nonconclusive Evidence
결정적 증거와 비결정적 증거

1) Justification comes in degrees: some beliefs are more justified than others. Examples of beliefs that are justified to the highest degree are beliefs in the elemetary truths of arithmetic and axioms of logic, as well as simple beliefs about one's own conscious states. Beliefs that are justified to lesser degrees are those about the physical world, the past, the future, and things that are presently unobserved.
1) 정당화는 등급들이 있다: 일부 믿음들은 다른 믿음들보다 더욱 정당화된다. 높은 정도로 정당화되는 믿음들의 예들은 산술의 기본 진리들과 논리의 공리들에 대한 믿음들, 또한 한 개인의 의식적 상태들에 대한 단순한 믿음들이다. 덜 정당화되는 믿음들은 물리적 세계, 과거, 미래, 현재 관찰되지 않는 것들에 대한 믿음들이다.

2) The degree to which a belief is justified depends, among other things, on whether that belief is supported by conclusive or nonconclusive evidence. Conclusive evidence guarantees the truth of the belief it supports, and thus affords complete certainty to the possessor of such evidence. Nonconclusive evidence, on the other hand, does not guarantee the truth of the belief it supports. Thus, if one has nonconclusive evidence for a proposition p, one's justification for p is of a lower degree than complete certainty.
2) 믿음이 정당화되는 정도는 특히 그 믿음이 결정적인 증거에 의해서 지지되는지 혹은 비결정적인 것에 의해 지지되는지에 의존한다. 결정적 증거는 그 증거가 뒷받침하는 믿음의 참을 보장하고, 그래서 그러한 증거의 소유자에게 완전한 확실성을 제공한다. 다른 한편 비결정적 증거는 그 증거가 뒷받침하는 믿음의 참을 보장하지 않는다. 그래서, 만일 p 명제에 대해 비결정적 증거를 가진다면, p에 대한 정당성은 완전한 확실성보다 더 낮은 정도의 것에 속한다.

3) Consider the belief "I exist." As Descartes argued in the first of his Meditations, if I didn't exist, then I couldn't be thinking about my own existence. Consequently, it is impossible to mistakenly believe oneself to exist. My evidence for believing that I exist - introspective awareness of the fact that I am thinking about the question whether I exist - is conclusive: it guarantees that my belief that I exist is true, and thus provides me with complete certainty.
3) "내가 존재한다"라는 믿음을 고찰해 보자. 그의 『방법서설』에서 주장하였듯, 만일 내가 현존하지 않는다면, 나는 나의 고유한 현존에 대해 생각하고 있을 수 없을 것이다. 결론적으로, 실수로 자신을 현존한다 믿는 것은 불가능하다. 내가 현존한다고 믿기 위한 나의 증거 - 내가 현존하는지 하는 물음에 대해 내가 생각하고 있다는 사실에 대한 내성적 자각 - 은 결정적이다. 그 증거는 내가 현존한다는 나의 믿음이 참임을 보장하고, 그래서 나에게 완전한 확실성을 제공한다.

4) In contrast, my evidence for believing that there is a desk in my office is nonconclusive. I presently have the experience of seeing that desk in front of me. However, it is logically possible that I who am having that experience am a brain in a vat whose sensory experiences are the result of direct stimulation through a computer.<16> If I were such a brain in a vat, then exhypothesi I would not be in Brown Hall 109 but in a mad scientist's laboratory. Consequently, having the perceptual evidence I am currently having does not provide me with a logical guarantee that the belief that evidence supports is true.
4) 반대로, 내 사무실에 책상이 있다고 믿기 위한 나의 증거는 비결정적이다. 나는 현재 내 앞에 책상을 보는 경험을 하고 있다. 그렇지만, 그러한 경험을 하고 있는 내가, 통 속의 뇌, 그 뇌의 감각 경험들이 컴퓨터를 통한 직접자극의 결과인 그러한 통 속의 뇌라는 것도 논리적으로는 가능하다.<16> 만일 내가 그러한 통 속의 뇌라면, 가설에 따라 나는 브라운 관 109호가 아니라 한 미친 과학자의 연구실에 있을 것이다. 결론적으로, 내가 현재 가지고 있는 지각적 경험을 갖는다는 것은 나에게 그 증거가 뒷받침하는 믿음이 참이라는 논리적 보증을 제공하지 않는다.

5) The distinction between conclusive and nonconclusive evidence is an epistemological one. Do not confuse it with the distinction between conclusive and inconclusive evidence that is part of our ordinary language. When I'm in my office looking at my desk, then my evidence for believing that there is a desk there is certainly not inconclusive in the sense in which that term is ordinarily used. That is to say, outside of the context of epistemology, we would take my evidence to be conclusive - to leave no doubt whatsoever as to the presence of my desk. However, in the sense in which we shall use the term here, my evidence is nonconclusive since it does not guarantee my belief's truth.
5) 결정적 증거와 비결정적(nonconclusive) 증거 사이의 구분은 인식론적 구분이다. 그것을 우리의 일상언어의 일부인 결정적 증거와 미결정적(inconclusive) 증거 사이의 구분과 혼동하여서는 안 된다. 내가 내 사무실에서 나의 책상을 바라보고 있을 때, 책상이 있다고 믿기 위한 나의 증거는 일상적으로 사용되는 용어의 뜻에서 확실성이 있고 미결정적이지 않다. 그것은 인식론의 문맥 바깥에서, 우리가 나의 증거를 결정적인 - 나의 책상의 현전에 관한 것은 그 무엇이든지 의심의 여지를 남겨두지 않는 - 것으로 받아들인다는 말이다. 그렇지만 우리가 여기에서 사용하는 의미로 나의 증거는 비결정적인데 왜냐하면 그것은 나의 믿음의 참을 보장하지는 않기 때문이다.

6) One way of expanding one's stock of justified beliefs is to use deduction. Consider the following argument.

(1) I exist.
Therefore:
(2) Someone exists.

Premise (1) is certain, and it is certain that (2) follows deductively from (1). Thus the deductive link between (1) and (2) transmits certainty from (1) to (2). In general, because deduction is truth-preserving, it allows us to start out with premises for which we have conclusive evidence and to end up with conclusions for which we have conclusive evidence.<17> Conclusive evidence for a conclusion cannot be achieved with nondeductive types of derivation. For example, if p does not entail q but only makes q probable, then I cannot have conclusive evidence for q even if my evidence for p is conclusive.
6) 우리의 정당화된 믿음들의 양을 확장시키는 한 가지 방법은 연역을 사용하는 것이다. 다음 논증을 고찰해 보자.

(1) 나는 존재한다.
그러므로:
(2) 누군가는 존재한다.

전제 (1)은 확실하고, (2)가 연역적으로 (1)로부터 귀결한다는 것은 확실하다. 그래서 (1)과 (2) 사이의 연역적 연결은 (1)로부터 (2)에게로 확실성을 전달한다. 일반적으로, 연역은 진리-보존적이기 때문에, 우리로 하여금 우리가 결정적 증거를 가지고 있는 전제들로 시작하는 것과 우리가 결정적 증거를 가지는 결론들로 끝내는 것을 가능하게 한다. <17> 결론에 대한 결정적 증거는 비연역적 유형의 도출을 가지고 성취될 수 없다. 예를 들어, 만일 p가 q를 수반하지 않지만 q를 개연적이도록 만들기만 한다면, 나는 설령 p에 대한 나의 증거가 결정적이라 할지라도 q에 대한 결정적 증거를 가질 수 없다.

7) It should be noted that a deductively derived conclusion cannot enjoy a higher degree of justification than the premise (or premises) from which it is derived. To illustrate this, we will consider another argument.

(1) My wife is presently at home
Therefore:
(2) Presently, someone is at home

Suppose I am justified in believing (1) because I know that at the present time my wife is normally at home. Clearly, though, this kind of evidence does not provide me with certainty for (1). After all, she might be out on an errand. Hence, although the deductive step from (1) to (2) is certain, the argument's conclusion is not because the premise from which it is derived is not.
7) 연역적으로 도출된 결론은 그 결론이 도출된 전제 (혹은 전제들) 보다 더 높은 정도의 정당성을 향유할 수 없다는 점이 주의되어야 할 것이다. 이를 설명하기 위해, 우리는 또 다른 논증을 고찰할 것이다.

(1) 나의 아내는 지금 집에 있다
그러므로
(2) 지금, 누군가는 집에 있다

내가 지금 시간에 나의 아내가 보통 집에 있다는 것을 알기 때문에 (1)을 믿음에 있어서 정당화된다고 가정하자. 분명히, 그럴지라도, 이러한 종류의 증거는 나에게 (1)에 대한 확실성을 제공하지 않는다. 무엇보다도, 그녀는 심부름을 하러 갔을지도 모른다. 따라서, 비록 (1)로부터 (2)로의 연역적 단계가 확실하다 할지라도, 그 논증의 결론은 확실하지 않은데 왜냐하면 그 결론이 도출된 전제가 확실하지 않기 때문이다.


Justification and Defeasibility
정당성과 상쇄가능성

1) One's justification for believing a proposition p is sensitive to changes in the total body of one's evidence. As one's evidence changes, one can gain, lose, and regain justification for believing something. Suppose S sees a sheet of paper that looks to him. S believes

(1) This sheet of paper is blue
on the perceptual evidence that
(2) The sheet of paper looks blue to S.
Suppose further that S is informed that
(3) Blue light is shining on that sheet of paper.
If what S has to go on is no more than (2) and (3), S is not justified in believing (1) because (3) defeats (2) as S's justification for believing (1). We can define the concept of evidential defeat as follows:

Evidential Defeat
D defeats e as evidence for p if and only if e is evidence for believing that p, but e in conjunction with d is not evidence for believing that p.
S's justification for believing a proposition p is defeated if S has evidence for another proposition that defeats S's evidence for p. This can happen in two ways: (i) S can acquire evidence that justifies S in believing that p is false; or (ii) S can acquire evidence that destroys S's justification for believing that p is true, without, however, justifying S in believing that p is false. Let us call defeaters of the former type contradicting defeaters and those of the latter type undermining defeaters.<18> (3) is an example of an undermining defeater. It destroys S's justification for believing that (1) is true, but it does not justify S in believing that (1) is false.
1) 명제 p를 믿기 위한 정당성은 그 증거의 총체에 있어서의 변화들에 민감하다. 증거가 변화할 때, 어떤 것을 믿기 위한 정당성은 습득할 수 있고, 잃을 수 있으며, 재습득할 수 있다. S가 그에게 보이는 한 장의 종이를 본다고 가정해 보자. S는 다음과 같이 믿는다

(1) 이 한 장의 종이는 파랗다
다음과 같은 지각적 증거에 의거해서 그러하다
(2) 그 한 장의 종이가 S에게 파랗게 보인다.
더 나아가 S가 다음과 같이 알게 된다고 가정해 보자
(3) 푸른 빛이 그 한 장의 종이를 비추고 있다.
만일 S가 근거로 삼아야 할 것이 (2)와 (3) 말고는 없다면, S는 (1)을 믿음에 있어서 정당화되지 않는데 왜냐하면 (3)이 S가 (1)을 믿기 위한 정당화로서의 (2)를 상쇄시키기 때문이다. 우리는 증거적 상쇄를 다음과 같이 정의할 수 있다.

증거적 상쇄
D는 p에 대한 증거 e를 e가 p라는 것을 믿기 위한 증거이지만 d와의 결합에 있어서 e가 p를 믿기 위한 증거가 아니라면 또 오로지 그러한 경우에만 상쇄시킨다.
S의 명제 p를 믿기 위한 정당화는 만일 S가 p에 대한 S의 증거를 상쇄시키는 또 다른 명제에 대한 증거를 가지고 있다면 상쇄된다. 이러한 일은 두 가지 방식으로 일어날 수 있다: (i) S는 p가 거짓이라고 믿음에 있어서 S를 정당화시키는 증거를 획득할 수 있다. 또는 (ii) S는 p가 거짓이라고 믿음에 있어서 S를 정당화함이 없이 p가 참이라고 믿기 위한 S의 정당화를 파괴하는 증거를 획득할 수 있다. 우리는 전자의 유형에 속하는 상쇄자들을 반박 상쇄자들이라 부르고 후자 유형에 속하는 것들을 기반약화 상쇄자들이라 부르도록 하자. <18> (3)은 기반약화 상쇄자의 일례이다. (3)은 (1)이 참이라고 믿기 위한 S의 정당성을 파괴하지만, (1)이 거짓이라고 믿음에 있어서 S를 정당화하지는 않는다.

2) The following is an example of a contradicting defeater. Suppose again that S believes (1) on the basis of (2). Suppose further that S is informed of the truth of the following two propositions:

(4) This sheet of paper is a grade-change form.
(5) Crade-change forms are always pink.

In this case, S has an explanation of why the sheet of paper look blue - namely (3) - and has further evidence indicating in believing that (1) is false. The conjunction of (4) and (5), therefore, functions as a contradicting defeater.
2) 다음의 것들은 반박 상쇄자들의 예시이다. 다시 S가 (2)에 기초하여 (1)을 믿는다고 가정하자. 나아가 S가 다음 두 가지 명제들에 대해 참이라고 알게 된다고 가정하자.

(4) 이 한 장의 종이는 성적정정 양식서이다.
(5) 성적정정 양식서들은 언제나 분홍색이다.

이 경우, S는 어째서 그 한 장의 종이가 파랗게 보이는지 - 즉 (3) - 에 대한 설명을 가지고 나아가 (1)이 거짓이라고 믿음에 대해 지시하는 증거를 가진다. (4)와 (5)의 결합은 그러므로 반박 상쇄자로서 기능한다.

3) A defeater for p can, in turn, be defeated. If this happens, then the justification S originally had for p is restored. Suppose, once again, that S believes (1) on the basis of (2). S is informed that (3) is true, and thus his justification for believing (1) is defeated. However, subsequently S is informed of the following two propositions:

(6) This sheet of paper is a grade-appeal form.
(7) Grade-appeal froms are always blue.

This information provides S with further relevant evidence indicating that (1) is true after all. Since this evidence is not perceptual in nature, it is not affected by the fact that blue light is shining on the sheet of paper. After all, a sheet of paper that is blue to begin with will still look blue when illuminated by blue light. Hence, as soon as S learns of the facts expressed by (6) and (7), he is again justified in believing that the sheet of paper he is looking at is blue.
3) p에 대한 상쇄자가 이번에는 상쇄될 수 있다. 만일 이러한 일이 일어난다면, S가 원래 p에 대해 가졌던 정당성은 복구된다. 다시 한 번, S가 (2)에 기초하여 (1)을 믿는다고 가정해 보자. S는 (3)이 참이라고 알게 되고, 그래서 (1)을 믿기 위한 그의 정당성은 상쇄된다. 그렇지만, 이어서 S는 다음 두 가지 명제들에 대해 알게 된다.

(6) 이 한 장의 종이는 성적이의 양식이다.
(7) 성적이의 양식들은 언제나 파랗다.

이 정보는 S에게 (1)이 무엇보다도 참이라고 지시하는 더욱 유관한 증거를 제공한다. 이 증거는 본성에 있어서 지각적이지 않으므로, 푸른 빛이 그 한 장의 종이를 비추고 있다는 사실에 의해 영향을 받지 않는다. 무엇보다도, 처음에 파란 그 한 장의 종이는 파란 빛에 의해 비추어질 때에도 여전히 파랗게 보일 것이다. 따라서, S가 (6)과 (7)에 의해 표현되는 사실들에 대해 알게 되자마자, 그는 다시 그가 바라보고 있는 종이가 파랗다고 믿는 데에 있어서 정당화된다.


Defeasibility and The Gettier Problem
상쇄가능성과 게티어 문제

1) In the last section, we saw that justification can be defeated because some elements of S's evidence can defeat other elements of S's evidence. In this section, we shall see that - as is illustrated by Gettier cases - justification can also be defeated by certain facts: facts that bear evidentially on, but are not part of, S's evidence and defeat S's justification for believing that p in the sense that his justification for p remains intact but fails to give him knowledge of p. Let us keep these two types of defeat separate by distinguishing between evidential and factual defeat of justification. The following two definitions specify how these concepts differ:

Justificational Defeat
d evidentially defeats S's justification for believing that p if and only if (i) S has evidence e for believing that p; (ii) S has also evidence e' for a proposition d that defeats e as evidence for p.

Factual Defeat
d factually defeats S's justification for believing that p if and only if (i) S has evidence e for believing that p; (ii) there is a proposition d such that d is true, S does not have evidence for d, and d defeats e as evidence for p.
1) 전 절에서, 우리는 정당화가 S의 증거들 중 일부 요소들이 S의 증거들 중 다른 요소들을 상쇄시킬 수 있기 때문에 상쇄될 수 있음을 보았다. 이번 절에서, 우리는 - 게티어 사례들에 의해 설명되는 것과 같이 - 정당화는 또한 특정한 사실에 의해 상쇄될 수 있음을 볼 것이다. 그 사실들은 S의 증거와 증거적으로 관련은 있지만 S의 증거의 일부는 아니며 S가 p를 믿기 위한 정당성을 상쇄시키는데 그의 p에 대한 정당성이 온전히 남지만 그에게 p에 대한 앎을 제공하는 데에는 실패한다는 의미에서 그러하다. 이러한 두 종류의 상쇄를 정당화에 대한 증거적 상쇄와 사실적 상쇄 사이의 구별을 통해 구분하도록 하자. 다음 두 가지 정의들은 이러한 개념들이 어떻게 다른지 명시한다.

정당화적 상쇄
d는 증거적으로 S의 p라는 믿음을 위한 정당성을 (i) S가 p를 믿기 위한 증거를 가지고 (ii) S가 또한 p에 대한 증거로서의 e를 상쇄시키는 명제 d를 위한 증거 e'를 가진다면 또 그러할 때에만  상쇄시킨다.

사실적 상쇄
d는 사실적으로 S의 p라는 믿음을 위한 정당성을 (i) S가 p를 믿기 위한 증거 e를 가지고 (ii) d가 참이고, S는 d에 대한 증거를 가지고 있지 않으며, d는 p에 대한 증거로서의 e를 상쇄시킨다는 그러한 명제 d가 있다면 또 그러할 때에만 상쇄시킨다.

2) Suppose some elements of my evidence support p in such a way that, if I had no further relevant evidence, I would be justified in believing that p. Suppose further that other elements of my evidence defeat the evidence I have for p. In that case, I am not justified in believing that p. However, if there is a fact that is hidden from me - hidden in the sense that I don't have evidence for it - that defeats my evidence for p, then although I remain justified in believing that p, I am not in a position to know that p.<19>
2) 만일 내가 그 이상의 어떠한 적절한 증거도 가지고 있지 않다면, 내가 p를 믿음에 있어서 정당화될 것이라는 그런 방식으로 나의 증거의 일부 요소들이 p를 지지한다고 가정해 보자. 나아가 나의 증거의 다른 요소들이 내가 p에 대해 가지는 그 증거를 상쇄시킨다고 가정해 보자. 그 경우, 나는 p를 믿음에 있어서 정당화되지 않는다. 그렇지만, 만일 나로부터 감추어진 - 내가 그에 대한 증거를 가지지 않는다는 의미로 감추어진 -  p에 대한 나의 증거를 상쇄시키는 사실이 있다면, 내가 p라고 믿음에 있어서 정당화된 채로 남는다 할지라도, 나는 p라는 것을 아는 입장에 있지 않다.<19>

3) To keep these two types of defeat properly distinct, we shall distinguish between epistemizing and nonepistemizing justification. Epistemizing justification is justification that is both evidentially and factually undefeated; it turns true beliefs into knowledge. Nonepistemizing justification is justification that is factually defeated without being evidentially defeated. Nonepistemizing justification's shortcoming is taht, for reasons beyond S's ken, it is undermined by certain facts about S's situation, and thus fails to epistemize S's belief that p - that is, to turn it into knowledge.
3) 이러한 두 가지 유형의 상쇄를 적절하게 구별함을 유지하기 위해서, 우리는 인식화하는 정당화와 비인식화 정당화 사이에 구별을 해야 할 것이다. 인식화하는 정당화는 증거적으로도 사실적으로 상쇄되지 않는 정당화이다. 그러한 정당화는 참인 믿음들을 앎으로 바꾼다. 비인식화 정당화는 증거적으로 상쇄됨이 없이 사실적으로 상쇄되는 정당화이다. 비인식화 정당화의 결점은 그의 이해를 넘어서는 이유들로 인하여, S의 상황에 대한 특정 사실들에 의해 약화되고, 그래서 S의 p라는 믿음을 인식화하는 데에 - 즉, 그 믿음을 앎으로 바꾸는 데에 실패한다는 것이다.

4) In light of these considerations, it is clear that the phrase "S's justification for believing that p is defeated" is an ambiguous one. It might refer either to evidential or to factual defeat. Let us avoid ambiguity by adopting the following convention: When S's justification for p is evidentially defeated, we shall simply say that S's justification for p is defeated. If, on the other hand, S's justification for p is factually but not evidentially defeated, we shall say that the epistemizing potential of S's justification for p is defeated.
4) 이러한 고찰들의 조명 속에서, "S의 p를 믿기 위한 정당성이 상쇄된다"라는 구절은 모호한 것이라는 점이 분명해진다. 그 구절은 아마도 증거적 상쇄 또는 사실적 상쇄를 지시할 것이다. 다음 규정을 적용함으로써 모호성을 피하도록 하자: S의 p에 대한 정당화가 증거적으로 상쇄될 때, 우리는 단순히 S의 p에 대한 정당화가 상쇄된다고 말해야 할 것이다. 다른 한편 만일 S의 p에 대한 정당화가 사실적으로 상쇄되지만 증거적으로는 상쇄되지 않는다면, 우리는 S의 p에 대한 정당화의 인식화할 잠재적 가능성이 상쇄된다고 말해야 할 것이다.

5) Let us see how this convention works in a concrete case. Consider again the Gettier case we discussed earlier. Smith is justified in believing

(1) Jones owns a Ford
and deduces
(2) Either Jones owns a Ford or Brown is in Barcelona.

Since Brown happens to be in Barcelona, (2) is true. But Smith doesn't know (2) because there is (or so let's suppose) the following fact about his situation that is hidden from him:

(3) The Ford Jones drives is a Hertz rental car.

(3) is a contradicting defeater for (1): if Smith had evidence for believing (3), then he would be justified in believing that (1) is false. However, (3) is hidden from Smith; he has no evidence at all for (3). Consequently, (3) does not defeat Smith's justification for believing (2); it merely defeats the epistemizing potential of his justification for (2), and thus prevents Smith from knowing (2). Hence, in spite of (3), Smith is fully justified in believing (2). Indeed, it is an essential element of Gettier cases in general that the subject's justification remains intact. Gettier cases, after all, are cases of justified true belief.
5) 이러한 규정이 구체적인 사례에서 어떻게 작용하는지 보도록 하자. 다시 우리가 먼저 논의하였던 게티어 사례를 고찰해 보자. 스미스는 다음과 같이 믿음에 있어서 정당화된다.

(1) 존스는 Ford차를 소유하고 있다.
그리고 다음과 같이 연역한다.
(2) 존스가 Ford차를 소유하고 있거나 브라운이 바르셀로나에 있다.

브라운이 우연히 바르셀로나에 있으므로, (2)는 참이다. 그러나 스미스는 (2)를 알지 못하는데 왜냐하면 그로부터 감추어진 그의 상황에 대한 다음과 같은 사실이 있기 때문이다 (또는 우리가 그렇게 가정하도록 하자).

(3) 존스가 모는 Ford차는 헤르츠가 빌려준 차이다.

(3)은 (1)에 대한 반박적 상쇄자이다. 즉 만일 스미스가 (3)을 믿기 위한 증거를 가지고 있다면, 그는 (1)이 거짓이라고 믿음에 있어서 정당화될 것이다. 그렇지만, (3)은 스미스로부터 감추어져 있다. 그래서 그는 (3)에 대해 전혀 아무런 증거도 가지고 있지 않다. 결론적으로, (3)은 스미스의 (2)를 믿기 위한 정당성을 상쇄시키지 않는다. 그리고 단지 (2)에 대한 그의 정당성의 인식화의 잠재적 가능성을 상쇄시킬 뿐이며, 그래서 스미스로 하여금 (2)를 알지 못하도록 한다. 따라서, (3)에도 불구하고, 스미스는 (2)를 믿음에 있어서 완전히 정당화된다. 덧붙여서, 주체의 정당성이 온전히 남는다는 것은 일반적으로 게티어 사례들의 본질적 요소이다. 게티어 사례들은 무엇보다도 정당화된 참인 믿음에 대한 사례들이다.


<16> 이러한 가능성을 부정하는 일부 철학자들이 있다 할지라도, 그들조차 아마도 내가 너무나 생생하여 사실은 내가 미친 과학자가 환각을 유발시키고 있는 연구실에 있을 때 나의 사무실에서 내 책상을 바라보고 있다고 믿을 만큼 그렇게 생생한 환각을 겪고 있을지도 모른다는 것을 인정할 것이다.

<17> 물론, 전제들이 확실한 연역적 논증은 오직 유도의 각 단계가 또한 확실할 때에만 확실한 결론을 낳는다.


-蟲-
Truth

1) There are several competing philosophical theories about the nature of truth: the correspondence theory, verificationism, and pragmatism.<8> Here is what these theories assert:

The Correspondence Theory
The belief that p is true if and only if it corresponds with the fact that p.

Verificationism
The belief that p is true if and only if it is an instance of (idealized) rational acceptability.<9>

Pragmatism
The belief that p is true if and only if it is useful.<10>

1) 참이라는 것의 본성에 대한 여러 경쟁적인 철학적 이론들이 있다. 그것은 대응론, 증거론, 그리고 실용주의이다. <8>이러한 이론들이 주장하는 것은 이와 같다.

대응론
p라는 믿음은 그 믿음이 p라는 사실과 대응한다면 그리고 그러할 때에만 참이다.

증거론
p라는 믿음은 그 믿음이 (이상화된) 합리적 수용가능성의 한 사례라면 그리고 그러할 때에만 참이다.<9>

실용주의
p라는 믿음은 그 믿음이 유용하다면 그리고 그러할 때에만 참이다.<10>

2) Each of these theories has its shortcomings. The problem with the correspondence theory is that unless we know (i) what a fact is and (ii) what it is for a belief to correspond to a fact, the definition doesn't tell us much. Moreover, if we try to explain what a fact is, we will see that it is difficult to do so without using the concept of truth. Hence it is doubtful that the correspondence theory can provide us with a noncircular account of truth.
2) 이러한 이론들 각각은 그 단점들을 지니고 있다. 대응론의 문제는 우리가 (i) 무엇이 사실인지 그리고 (ii) 믿음이 사실에 일치한다는 것이 무엇인지 모르는 한, 그 정의가 우리에게 많은 것을 말해주지 않는다는 것이다. 더구나, 만일 우리가 사실이 무엇인지 설명하고자 노력한다면, 우리는 참에 대한 개념을 사용하지 않고 그렇게 하는 것이 어렵다는 사실을 알게 될 것이다. 따라서 대응론이 우리에게 참에 대한 비순환적 설명을 제공할 수 있다는 것은 의심스럽다.

3) Next, consider verificationism. Since "rational acceptability" is just another term for "justification," what verificationism tells us is that a true belief in a certain area is one that enjoys ideal justification - the best kind of justification we can come by in that area. The problem with this view is that it's not so easy to explain why it shouldn't be possible for a belief to enjoy such justification and nevertheless be false. Take the belief that there is an external world of physical bodies. This belief is a good candidate for an ideally justified belief. Many philosophers believe, however, that it is logically possible for this belief to be false. Descartes, for example, took it to be a logical possibility that an evil demon was deceiving him into believing that there is a world of physical objects, and many contemporary philosophers think that it is logically possible to be a brain in a vat that is somehow kept alive and stimulated so that the illusion of a normal life is generated. But if it is possible to be deceived by an evil demon or to be a brain in a vat, then it is possible for ideally justified beliefs about physical objects to be false. Since verificationists can't allow for evil-demon deception and unsuspecting brains in vats, they face the uphill battle of having to show what precludes such things from being possible.<11>
3) 다음으로, 증거론을 고찰해 보자. "합리적 수용가능성"은 단지 "정당성"의 또다른 말이므로, 증거론이 우리에게 말하는 것은 특정한 영역에서 참인 믿음이 이상적 정당화 - 우리가 그 영역에서 얻을 수 있는 최선의 정당화 - 를 향유하는 그러한 한 가지의 것이라는 것이다. 이러한 관점과 관련된 문제는 그 관점이 믿음이 거짓이라고 할지라도 그럼에도 불구하고 그러한 정당화를 누리는 것, 그것이 어째서 가능하지 않아야 하는지 설명하는 일이 쉽지 않다는 점이다. 물리적 실체들의 외부세계가 있다는 믿음을 취해보자. 이 믿음은 이상적으로 정당화된 믿음의 훌륭한 후보이다. 그렇지만 다수의 철학자들이 이 믿음이 거짓이 되는 일이 논리적으로 가능하다고 믿는다. 예를 들어 데카르트는 한 사악한 악마가 그로 하여금 물리적 대상들의 세계가 있다고 믿도록 그를 기만하고 있었다는 것을 논리적인 가능성의 하나로 취했었고, 여러 현대 철학자들은 어떤 식으로든 계속 살아있으며 자극을 받아 일상적 삶이라는 환각이 산출되도록 된 통 속의 뇌가 있을 논리적 가능성을 생각한다. 그러나 만일 사악한 악마에 의해 기만받는 것이나 통 속의 뇌가 있는 것이 가능하다면, 물리적 대상들에 대해 이상적으로 정당화된 믿음들이 거짓일 것도 가능하다. 증거론자들은 사악한 악마의 기만과 의심하지 않는 통 속의 뇌를 허용할 수 없으므로, 그들은 그러한 것들을 존재할 가능성을 막는 어떤 것을 보여주어야만 하는 난관에 직면한다.
(진리치와 정당성은 독립적인가, 인식주체와 인식대상은 분리되어 있는가, 존재론과 인식론의 관계, 등등의 문제)

4) Finally, let's consider pragmatism, which proposes that a true belief is a useful belief. It is doubtful that truth and usefulness are as closely linked as this theory supposes. It is easy to imagine true beliefs that are not useful at all. Suppose you are about to take an oral examination on a difficult subject. You believe - unfortunately - that you are badly prepared. In all likelihood, this belief will not be useful to you; it might even make you more nervous than you would be anyhow, and thus lower your chances of passing the exam. Now suppose you believe - falsely - you are well prepared. This belief might be useful to you because it will make you more confident, and thus able to perform as well as your present level of ability allows. It would appear, therefore, that true beliefs can fail to be useful and useful beliefs can fail to be true.
4) 끝으로, 참인 믿음은 유용한 믿음이라고 제안하는 실용주의를 고려해 보도록 하자. 참과 유용성이 이 이론이 생각하는 만큼 밀접하게 연계된다는 것은 의심스럽다. 전혀 쓸모 없는 참인 믿음들을 상상하는 일은 쉽다. 당신이 한 어려운 주제에 대한 구두시험을 막 치려는 참이라고 가정해 보자. 당신은 - 불행하게도 - 당신이 준비가 영 안 되어 있다고 믿는다. 십중팔구 이 믿음은 당신에게 유용하지 않을 것이다. 그 믿음은 당신을 여느때보다 훨씬 불안하게 만들기조차 할 것이고, 그래서 당신이 시험에 통과할 가능성을 낮출 것이다. 이제 당신이 - 거짓으로 - 아주 잘 준비되었다고 믿는다고 가정해 보자. 이 믿음은 아마도 당신에게 유용할 것인데 왜냐하면 그 믿음은 당신이 더욱 자신감 있도록 해줄 것이고, 그래서 당신의 현재 수준에서 허용하는 만큼의 능력발휘를 가능하게 할 것이다. 그러므로 참인 믿음들이 유용하지 않을 수 있고 유용한 믿음들이 참이 아닐 수 있는 것으로 보일 것 같다.
(인식론적 유용성과 윤리적-가치적 유용성의 차이.)

5) In spite of their respective shortcomings, each of these views has its advocates. Since we cannot delve any further into these matters here, we shall take a neutral stance concerning the nature of truth. However, we shall presuppose that in doing epistemology, we should be guided by the following two points about the relation between truth and justification:

(i) It is possible that p is true although there is no one who has a justified belief that p.
(ii) It is possible for S to be completely justified in believing that p although p is false.

5) 그 이론들의 개별적 단점들에도 불구하고, 이러한 관점들은 각기 그 지지자들을 가진다. 우리는 여기에서 이러한 문제들에 대해 더 멀리까지 뒤적거릴 수 없으므로, 참의 본성에 대해 중립적 입장을 취해야 할 것이다. 그렇지만, 우리는 인식론을 수행함에 있어서 우리가 참과 정당성 사이의 관계에 대한 다음 두 가지 점들에 의해 인도되어야 한다고 추정해야 할 것이다.

(i) p라는 정당화된 믿음을 가진 사람이 한 사람도 없다 할지라도 p가 참인 것은 가능하다.
(ii) p가 거짓이라 할지라도 S가 p를 믿음에 있어서 완전하게 정당화되는 것은 가능하다.

6) We will use the following example to illustrate the first point: Let p be a truth about what 999 is equal to. I don't have a justified belief that p for two reasons: first, I lack the requisite belief; and second, I didn't make any attempt to compute what that figure is equal to, so I have no justification for forming a belief about the matter.
6) 우리는 첫번째 사항을 설명하기 위해 다음의 예시를 사용할 것이다. p가 999에 등가인 어떤 것에 대해 참이라고 하자. 나는 두 가지 이유들로 p라는 정당화된 믿음을 가지지 못한다. 첫째, 나는 필요한 믿음을 결여하고 있다. 둘째, 나는 그 수치가 무엇인지 계산하기 위한 어떠한 시도도 하지 않았고, 그래서 나는 그 문제에 대해 하나의 믿음을 형성하기 위한 어떠한 정당성도 가지고 있지 않다.

7) To illustrate the second point, let us suppose that, unbeknownst to the public, Susan Rook, one of the anchors on CNN, has an identical twin sister, Sibyl Rook. Sometimes Sibyl, pretending to be Susan, reads the news, and except for a few people on the CNN staff, no one knows this is going on. Suppose you turn on CNN and believe yourself to be watching and listening to Susan Rook, when in fact Sibyl Rook is acting as the anchor on that occasion. Since you have no reason to suspect such a deception, your belief is justified. Nevertheless, it is false. Hence it is possible to have a justified false belief.
7) 두번째 사항을 설명하기 위해, 대중적으로는 알려지지 않은 CNN 앵커 수잔 룩이 똑같은 쌍둥이 자매 시뷜 룩을 하나를 가지고 있다고 생각해 보자. 가끔 쉬빌은 수잔인 척하고 뉴스를 진행하고, 소수의 CNN 스텝들을 제외하고는 아무도 이런 일이 벌어지고 있다는 것을 모른다. 당신이 CNN을 켜고, 사실 시뷜 룩이 마침 앵커를 하고 있을 때, 당신 스스로 수잔 룩을 보고 또 듣고 있다고 믿는다고 생각해 보자. 당신은 그러한 속임수를 의심할 아무런 이유도 없으므로, 당신의 믿음은 정당화된다. 그럼에도 불구하고, 그 믿음은 거짓이다. 따라서 정당화된 거짓 믿음을 가지는 것은 가능하다.

8) You may think that these points are inconsistent with verificationism. Note, however, that verificationism doesn't identify truth simply with justification, but with ideal justification. Hence a verificationist could accommodate the first point by arguing that a true belief about what 999 is equal to is one that is capable of ideal justification, whether or not I actually hold that belief and whether or not I actually have justification for it. For example, she could say that the equation in question is a truth because the community of the best-trained mathematicians endorses it. Concerning the second point, she could argue that when you mistake Sibyl for Susan Rook, you are completely justified in that belief although it is not an instance of a belief that enjoys ideal justification.<12>
8) 당신은 이러한 사항들이 증거론에는 맞지 않는다고 생각할지 모른다. 그렇지만 증거론은 참을 단순한 정당화와 동일시하지는 않고, 이상적 정당화와 동일시한다는 점을 주의해야 한다. 따라서 증거론자는 첫번째 사항에 대해 여지를 남겨둔다. 그러니까 999와 등가인 어떤 것에 대한 참인 믿음이 이상적으로 정당화될 수 있는 한 가지의 것이라고, 내가 실제로 그 믿음을 고수하든 하지 않든 그리고 내가 실제로 그 믿음에 대한 정당성을 가지든 갖지 않든 말이다. 예를 들어, 그녀(?)는 그 질문에서 방정식은 참인데 왜냐하면 최고로 숙련된 수학자들의 집단이 그것을 보증하기 때문이라고 말할 수 있을 것이다. 두 번째 사항과 관련해서, 그녀는 당신이 시뷜을 수잔 룩으로 착각했을 때, 당신은 그 믿음에 있어서 비록 그 믿음이 이상적 정당성을 향유하는 믿음의 일례가 아닐지라도 완전히 정당화된다고 주장할 수 있다.

Belief
믿음

1) The standard view is that a belief is an attitude one can have toward a proposition.<13> Epistemologists distinguish among the following three propositional attitudes. One can:

(i) believe that p (take p to be true);
(ii) disbelieve that p (take p to be false);
(iii) suspend judgment as to p (neither believe nor disbelieve p).
1) 표준적 관점은 믿음이 한 사람이 명제에 대해 취할 수 있는 하나의 태도라는 것이다.<13>

(i) p라는 것을 믿는다 (p를 참인 것으로 취한다);
(ii) p라는 것을 믿지 않는다 (p를 거짓인 것으로 취한다);
(iii) p에 관해 판단을 유보한다 (p를 믿지도 불신하지도 않는다).

2) Of course, disbelief is just a special case of belief, for if you disbelieve that p, then you believe that the negation of p is true.
2) 물론, 불신은 믿음의 굉장히 특수한 경우인데, 만일 당신이 p라는 것을 불신한다면, 당신은 p의 부정이 참이라는 것을 믿기 때문이다.

3) The concepts of theism, atheism, and agnosticism can be used to illustrate these three propositional attitudes. Let the proposition in question be that God exists. The theist believes that proposition, the atheist disbelieves it, and the agnostic neither believes nor disbelieves it, but suspend judgment on the matter of God's existence.
3) 유신론, 무신론, 불가지론의 개념들이 이러한 세 가지 명제적 태도들을 설명하기 위해 사용될 수 있다. 신이 현존한다는 것이 문제가 되는 명제라고 하자. 유신론자는 그 명제를 믿고, 무신론자는 그 명제를 불신하며, 불가지론자는 그 명제를 믿지도 불신하지도 않지만, 신의 현존에 대한 문제에 대해 판단을 유보한다.

4) Note that, with regard to any proposition one consider, one cannot escape taking one of these three attitudes. If you wonder whether God exists - that is, if you consider the proposition "God exists" - then you must either beileve that God exists, disbelieve it, or suspend judgment on the question.
4) 한 사람이 고찰하는 어떤 명제에 대해서든, 그는 이 세 가지 태도들 중 하나를 취하는 일을 피할 수 없다는 점을 주의하라. 만일 당신이 신이 현존하는지 궁금해 한다면 - 즉, 만일 당신이 "신이 현존한다"라는 명제를 고찰한다면 - 당신은 신이 현존한다고 믿거나, 그것을 불신하거나, 또는 그 물음에 대해 판단을 유보해야만 한다.

5) Beliefs, of course, are held with various degree of intensity. You might be slightly inclined to believe that God exists, or you might be generally convinced that God exists although at times you have doubts, or you might feel so certain that he exists that there is no room in your mind for any doubt.
5) 물론 믿음들은 여러 상이한 강도로 주장된다. 당신은 신이 현존한다는 것을 약간 믿고 싶어 할지도 모르고, 또는 당신이 때로 의심들을 품을지라도 대체로 신이 현존한다고 생각할지도 모르며, 혹은 신이, 당신의 마음에 어떠한 의심의 여지도 없이 현존한다고 강한 확신을 가질지도 모른다.

6) Before moving on to justification, let us consider one more distinction epistemologists make concerning the concept of belief: that between occurrent and standing beliefs. An occurent belief is one that is presently before your mind. For example, having just read about God's existence, your belief that God exists (or does not exist) might be one you are presently focusing on. If so, it is an occurent belief. Occurent beliefs can be either standing beliefs or beliefs that are newly formed. For example, that two and two equal four is a standing belief of yours that is not normally occurent. In contrast, a belief such as "The phone is ringing" is typically an occurent belief without being a standing belief.
6) 정당화로 넘어가기에 앞서, 믿음의 개념에 관련해 인식론자들이 만든 또 하나의 구별을 고찰해 보도록 하자. 그 구별은 일시적 믿음과 지속적 믿음 사이의 차이이다. 일시적 믿음은 지금 당신 마음 속에 떠오른 믿음이다. 예를 들어, 방금 막 신의 현존에 대해 읽었기 때문에, 신이 현존한다는(또는 현존하지 않는다는) 당신의 믿음은 당신이 지금 주의를 기울이고 있는 것일 터이다. 만일 그렇다면, 그것은 일시적인 믿음이다. 일시적 믿음들은 지속적 믿음들이 되거나 새롭게 형성되는 믿음들이 될 수 있다. 예를 들어, 2에 2를 더하면 4와 같다는 것은 보통 일시적이지 않은 당신의 지속적 믿음이다. 반대로, "전화가 울리고 있다"와 같은 그런 믿음은 지속적인 믿음이 아닌 전형적으로 일시적인 믿음이다.


Justification
정당성

1) According to the JTB account of knowledge, for a true belief to amount to knowledge, it must be justified. Now, if the JTB account were true, we should be able to characterize justification as that which "epistemize" true belief - as that which turns true belief into knowledge. We have seen, however, that the three conditions of the JTB account are not sufficient for knowledge. What epistemizes true beilef is justification plus something else, or "de-Gettiered" justification. But if justification is not what "epistemizes" true belief, how can we charaterize it?
1) 앎에 대한 JTB 설명에 따르면, 참된 믿음이 앎이 되기 위해서 그 믿음은 반드시 정당화되어야만 한다. 이제, 만일 JTB 설명이 참이라면, 우리는 정당화를 참인 믿음을 "인식화하는" 어떤 것으로서 규정할 수 있어야 할 것이다. - 즉 참인 믿음을 앎으로 만드는 그러한 어떤 것으로 말이다. 그렇지만 우리는 JTB 설명의 세가지 조건들이 앎에 불충분하다는 것을 보았다. 참인 믿음을 인식화하는 것은 정당화에 다른 어떤 것을 더하는 것이거나, "게티어문제를 해결한" 정당화이다. 그러나 만일 정당화가 참인 믿음을 "인식화하는" 어떤 것이 아니라면, 우리는 그것을 어떻게 규정할 수 있을까?

2) Epistemologists frequently answer this question by distinguishing between two kinds of true beliefs: those that are lucky guesses and those that are not. Recall the example of a lucky guess we considered avobe: A professor believes, without taking a head count,

(1) And here we have full attendence once again

and he happens to be right. We assumed that his belief lacked justification because, instead of basing it on evidence acquired through counting, he simply succumbs to wishful thinking. The reason the professor's belief must be viewed a "lucky guess" is that he doesn't have evidence in support of (1). In fact, his knowledge that in the second week of semester class attendance usually drops makes it likely that (1) is false. Thus he has no reason to think that (1) is true, and if subsequently he were to assess the situation, he'd have to say that it was mere luck that he got things right. Let us, then, define a lucky guess as follows:

Lucky guess
S's belief that p is a lucky guess if and only if (i) p is true; (ii) S believes that p; (iii) S has no evidence for believing that p is true.

Note that we are not defining the concept of a lucky guess by simply saying: A lucky guess is a belief the truth of which is just a matter of luck. To do so would be misguided because, as Gettier cases illustrate, even a completely justified belief can be true as a result of sheer luck. Consider the Gettier case we stated earlier. Smith believes:

(2) Either Jones owns a Ford or Brown is in Barcelona.

Smith's reasoning in support of (2) is impeccable. He has good evidence for believing that Jones owns a Ford, and he knows that (2) is entailed by the proposition that Jones owns a Ford. So as far as Smith's evidence and reasoning go, his belief is by no means a lucky guess. Yet, given that the first disjunct of (2) is false, there is a sense in which (2) is true by sheer luck. Brown just happens to be in Barcelona on the day Smith is ruminating about Jones's Ford. Had Brown changed his travel plans, he might have been somewhere else.
2) 인식론자들은 흔히 이 물음에 대해 참인 믿음들을 두 종류로, 즉 요행한 추측인 참믿음들과 그렇지 않은 것들로 구별함으로써 대답한다. 위에서 우리가 고찰하였던 요행한 추측의 예를 기억해 보자. 한 교수가 인원수를 세지도 않고 다음과 같이 믿는다.

(1) 여기 다시 한 번 전원이 출석하였다.

그리고 그는 우연히 맞았다. 우리는 그의 믿음이 정당성을 결여했다고 추정했는데, 왜냐하면, 세어 봄을 통해 획득된 증거에 기초하는 대신에, 그가 단순히 희망적인 생각에 굴복하였기 때문이다. 그 교수의 믿음이 틀림없이 "요행한 추측"으로 보이는 까닭은 그가 (1)을 뒷받침하는 증거를 가지고 있지 못하다는 사실이다. 사실, 학기 둘째 주 강의실 출석률이 일반적으로 떨어진다는 그의 앎은 (1)이 거짓일 것처럼 만든다. 그래서 그는 (1)이 참이라고 생각할 아무런 이유도 없고, 만일 나중에 그가 그 상황을 평가한다면, 그는 그가 맞추었던 것이 순전히 행운이었다고 말해야 할 것이다. 다음으로 우리는 요행한 추측을 다음과 같이 정의하도록 하자.

요행한 추측
p라는 S의 믿음은 (i) p가 참이고 (ii) S가 p를 믿으며 (iii) S가 p를 참이라고 믿을 아무런 증거도 가지고 있지 않다면 그리고 그러한 경우에만 요행한 추측이다.

우리가 요행한 추측을 단순히 그 참이 단지 요행인 그러한 믿음이라고만 말함으로써 요행한 추측의 개념을 정의하고 있지는 않다는 것을 주의하자. 그렇게 하는 것[요행한 추측을 요행한 참으로만 정의하는 것]은 오해될 것인데 왜냐하면 게티어의 사례들이 보여주듯 완전히 정당화된 믿음조차 행운의 결과로서 참이될 수 있기 때문이다. 우리가 앞서 진술하였던 게티어 사례를 고려해 보자. 스미스는 다음 명제를 믿는다.

(2) 존스가 Ford차를 소유하고 있거나 브라운이 바르셀로나에 있다.

(2)를 뒷받침하는 스미스의 추론은 흠잡을 것이 없다. 그는 존스가 Ford차를 소유하고 있다고 믿을 훌륭한 증거를 가지고 있고, (2)라는 그의 앎은 존스가 Ford차를 소유하고 있다는 명제에 의해 수반된다. 스미스의 증거와 추론이 그러한 한에서, 그의 믿음은 요행한 추측일 수 없다. 하지만, (2)의 첫 번째 선언지가 거짓임을 고려해 보면, (2)가 순전히 운으로 참이라는 의미이다. 브라운은 단지 우연히 스미스가 존스의 Ford차에 대해 생각하고 있는 그 날 바르셀로나에 있는 것이다. 브라운이 그의 여행 계획을 변경했다면, 그는 다른 어떤 곳에 있었을지 모른다.

3) We must distinguish, then, between two kinds of lucky beliefs. First, a belief can be lucky because, in relation to certain relevant facts, its truth was not a likely outcome. Second, a belief can be lucky because, in relation to the subject's evidence, its truth was not a likely outcome. Let us call the former a lucky truth (for lack of a better name), and the latter a lucky guess. Justification is what prevents a true belief from being a lucky guess, but not from being a lucky truth.
3) 그래서 우리는 두 종류의 요행한 믿음들을 구별해야만 한다. 첫째로, 한 믿음은 특정한 관련 사실들과의 관계에서 그것의 참이 도출되지 않았을 성싶기 때문에 요행한 것일 수 있다. 둘째로, 한 믿음이 그 주체의 증거와의 관계에서 그것의 참이 도출되지 않을 것 같기 때문에 요행한 것일 수 있다. 우리는 전자를 요행한 참 (더 나은 이름이 없으니) 이라 부르고, 후자를 요행한 추측이라 부르도록 하자. 정당화는 참인 믿음이 요행한 추측이 되지 않도록 막는 것이지만, 요행한 참이 되지 않도록 막는 것은 아니다.

4) We are now in a position to reevaluate the significance of the Gettier problem. What the Gettier problem shows us is that in order for a true belief to qualify as knowledge, it must satisfy two conditions: it must not be a lucky guess (that is, it must be justified), and it must not be a lucky truth. A true belief that isn't a lucky guess - like Smith's belief that (2) is true - may still be a lucky truth, and thus fall short of being knowledge. Hence in order to solve the Gettier problem, epistemologists have to figure out what kind of condition can prevent a true belief from being a lucky truth. Later in this chapter, we shall consider two proposals about what such a condition might look like. Before we can do that, however, we must examine a few more points about the nature of justification.
4) 이제 우리는 게티어 문제의 의미를 재평가하는 위치에 있다. 게티어 문제가 우리에게 보여주는 것은 참인 믿음이 앎의 자격을 얻기 위해서 그 믿음이 반드시 두 가지 조건들을 만족시켜야만 한다는 것이다. 그것은 반드시 요행한 추측이 아니어야만 하고 (즉, 반드시 정당화되어야 하고), 반드시 요행한 참이 아니어야만 한다. 요행한 추측이 아닌 참인 믿음 - (2)가 참이라는 스미스의 믿음과 같이 - 은 여전히 요행한 참일 것이고, 그래서 앎으로는 부족하다. 따라서 게티어의 문제를 해결하기 위해서, 인식론자들은 어떤 종류의 조건이 참인 믿음을 요행한 참이 되지 않도록 막을 수 있는지 알아내야 한다. 이 장 뒤에서, 우리는 그러한 한 가지 조건일 것 같기도 한 어떤 것에 대한 두 가지 제안들을 고찰할 것이다. 그렇지만, 그 일을 하기에 앞서 우리는 정당화의 본성에 대한 조금 더 많은 사항들을 검토해야만 한다.


The Property and The Activity of Justification
정당성의 속성과 활동

1) It is important to distinguish between the activity of jsutifying a belief and a belief's property of being justified. When you engage in the activity of justifying a belief, you are explaining what evidence, or what reasons, you have in support of that belief. Your aim of to convince others - or perhaps yourself - that the belief in question is justified. About the logical relation between justification as a property and justification as an activity, two points must be made.
1) 믿음을 정당화하는 활동과 정당화된 것으로서 믿음의 속성을 구분하는 것은 중요하다. 당신이 믿음을 정당화하는 활동을 시작할 때, 당신은 무엇이 당신이 그러한 믿음을 뒷받침하기 위해 가지고 있는 증거인지, 또는 근거들인지 설명하고 있다. 당신의 목표는 그 믿음이 정당화되느냐는 문제에 있어서 다른 사람들을 - 혹은 어쩌면 당신 자신을 - 납득시키는 것이다. 속성으로서의 정당성과 활동으로서의 정당화 사이의 논리적 관계에 대해서, 두 가지 사항들이 구성되어야만 한다.

2) First, a belief of yours can have the property of being completely justified even if you have not engaged in the activity of justifying it. Consider your present belief that your are reading book. Obviously, unless very odd circumstances obtain, this is a justified belief, even though (in all likelihood) you have not bothered to explain to anyone your evidence in support of that belief.
2) 첫째로, 당신의 믿음은 완전히 정당화된 것으로서의 속성을 지닐 수 있는데 설령 당신이 그 믿음을 정당화하는 활동을 하지 않았더라도 그러하다. 당신이 책을 읽고 있다는 당신의 현재 믿음을 고려해 보자. 분명히, 아주 기이한 상황들이 존재하지 않는 한, 이 믿음은 정당화된 믿음이고, 설령 (아마도) 당신이 누군가에게 그 믿음을 뒷받침하는 당신의 증거를 설명하고 싶어하지 않는다 할지라도 그러하다.

3) Second, it is possible for you to have no idea how to justify a certain belief although that belief is, as a matter of fact, justified. Suppose that on your first day in an epistemology class, the instructor asks you what your justification is for believing that you exist. Suppose further that the question completely dumbfounds you, so that you can't think how to respond to it. Does that mean you are not justified in believing you exist? Clearly not. Hence it is one thing for a belief to be justified, and quite another for the believer to be able to show that the belief is justified.
3) 둘째, 당신이 특정한 믿음을 설령 사실은 정당화된다 할지라도 어떻게 정당화하는지 아무런 생각도 없을 수 있다. 당신의 인식론 강의 첫날, 강의자가 당신에게 당신이 현존한다는 것을 믿기 위한 당신의 정당성이 뭐냐고 묻는다고 가정해 보자. 더 나아가서 그 물음이 완전히 당신의 말문을 막히게 만들어서, 당신이 그 물음에 어찌 답할지 생각할 수도 없다고 하자. 그것이 당신으로 하여금 당신이 현존한다는 것을 믿는 데에 있어서 정당화되지 않는다는 뜻인가? 분명히 아니다. 따라서 믿음이 정당화된다는 한 가지의 것과 그 믿는 사람이 그 믿음이 정당화된다는 것을 보여줄 수 있다는 또 다른 것은 전혀 다른 것이다.


Justificaiton and Evidence
정당성과 증거

1) There is a tight connection between one's justification for believing that p and one's evidence for p. According to some philosophers, we can even say that S is justified in believing that p if and only if believing p fits S's evidence.<14> Others would further tighten the connection between justification and evidence; they would say that one is justified in believing that p if and only if one has adequate evidence for p and believes that p because of that evidence.<15>
1) p를 믿기 위한 정당성과 p에 대한 증거 사이에는 긴밀한 관련이 있다. 일부 철학자들에 따르자면, 우리는 S가 p라는 것을 믿음에 있어서 p를 믿는 것이 S의 증거에 들어맞는다면 그리고 오직 그러한 때에만 정당화된다고 말할 수도 있다.<14> 다른 철학자들은 정당성과 증거 사이의 연결을 더욱 팽팽하게 조일 것이다. 그들은 한 사람이 p를 믿음에 있어서 그가 p에 대한 적절한 증거를 가지고 그 증거 때문에 p라는 것을 믿는다면 또 오직 그러한 때에만 정당화된다고 말할 것이다.

2) Traditionally, philosophers have recognized four sources of evidence: perception, introspection, reason, and memory. Perceptual evidence comes from our five senses: we have visual, auditory, olfactory, tactile, and gustatory experiences that provide us with evidential clues about the properties of physical objects. Introspection allows us to "look" inside ourselves, to know what mental states we are in: whether we are hungry, tired, excited, thinking about unicorns, worried about the future, and so on. Reason is the faculty that permits us to recognize such propositions as the following: p follows from q, p and q cannot both be true, p could not possibly be false, and p and q make r probable. Finally, memory permits us to retain knowledge that is derived from one of the others sources.
2) 전통적으로, 철학자들은 증거의 네 가지 원천들을 인정해 왔다. 그것들은 지각, 내성(성찰), 추론, 그리고 기억이다. 지각 증거는 오감으로부터 온다. 우리는 우리에게 물리적 대상들의 속성들에 대한 증거가 되는 조각들을 제공하는 시각, 청각, 후각, 촉각, 미각 경험들을 가진다. 내성은 우리로 하여금, 우리가 어떤 정신적 상태들에 처해 있는지, 그러니까 배가 고픈지, 피곤한지, 흥분되어 있는지, 유니콘들에 대해 생각하고 있는지, 미래에 대해 염려하고 있는지 따위를 알 수 있도록, 우리 자신의 내면을 "보는" 것을 가능케 한다. 추론은 우리로 하여금 다음과 같은 그런 명제들을 이해할 수 있도록 한다. p는 q로부터 결과한다, p와 q가 모두 참일 수 없다, p가 거짓임은 가능할 수 없다, p와 q는 r을 개연적으로 만든다. 끝으로, 기억은 우리로 하여금 다른 원천들 중의 하나로부터 도출된 앎을 유지시킨다.

3) A reliable authority is one more commonly recognized source of evidence. Evidence based on a reliable authority can be acquired from an expert in a certain subject, an encyclopedia, an article in a trustworthy magazine, or the like. It should be noted, however, that evidence based on a reliable authority can be fully explained in terms of the other categories. Consider how you would acquire knowledge in Biology 101. The learning process in the classroom involves your senses: you hear the professor's voice, and you see what she writes on the board. Moreover, the professor's lectures and the textbook for this course convey a body of knowledge that was gained through cognitive processes and research methods combining perceptual observation, reason, and memory.
3) 또 하나 일반적으로 인정되는 증거의 원천은 신뢰할 만한 권위이다. 신뢰할 만한 권위에 기초된 증거는 특정 주제의 전문가, 백과사전, 믿을 만한 지면, 또는 그와 비슷한 것으로부터 획득될 수 있다. 그렇지만 신뢰할 만한 권위에 기초된 증거가 그 외의 범주들로 완전히 설명될 수 있다는 점이 주의되어야 할 것이다. 당신이 어떻게 생물학 101에서 앎을 얻을 것인지 고려해 보자. 그 강의실에서의 학습과정은 당신의 감각들을 포함한다. 당신은 교수의 목소리를 듣고, 그녀가 칠판에 적는 것을 본다. 더욱이, 그 교수의 강의와 이 과정을 위한 교재는 지각적 관찰, 추론, 그리고 기억으로 조합된 인지 과정과 조사 방법들을 통하여 얻어진 앎의 집합체를 전달한다.

<11> 그래서 증거론( 또는 종종 이 관점이 그렇게 불리는 것으로서 반실재주의)를 지지하는 퍼트남은 정교한 논증을 내놓았다. 그에 따르면 통 속의 뇌는 가능할 수 없다.(?)

<12> 분명히, 이상적 조건들 아래에서, 당신은 당신이 실수로 수잔 룩이라고 믿는 그 여성의 참된 정체성을 밝히는 증거를 가질 수 있을 것이다.

-蟲-
Chapter One
1장

Knowledge and Justification
앎과 정당화

Epistemology
인식론

Epistemology is the study of knowledge and justified belief. To engage in this study is to seek answers to the following questions:

Q1. What is knowledge?
Q2. What do we know?
Q3. What is it for a belief to be justified?
Q4. Which of our beliefs are justified?
인식론은 앎과 정당화된 믿음에 대한 연구이다. 이 연구에 참여한다는 것은 다음 물음들에 대한 답변들을 모색한다는 것이다.
질문 1. 앎이란 무엇인가?
질문 2. 우리는 무엇을 아는가?
질문 3. 믿음이 정당화된다는 것은 무엇인가?
질문 4. 우리의 믿음들 중 어떤 것이 정당화되는가?

1) To answer question 1, we must define the concept of knowledge, while to answer question 2, we must determine the extent of knowledge. Question 3 calls for an analysis of the concept of justification, which involves stating criteria that tell us when beliefs are justified and when they are not. And if we wish to answer question 4, we must figure out the extent of what we are justified in believing. According to skeptics, the extent of what we know and what we are justified in believing is smaller than we ordinarily think. They reason that we have no way of ruling out that an evil demon is deceiving us without our being aware of the deception, or that a mad scientist has reduced us to unsuspecting brains in vats. According to nonskeptics, the claim that we have no way of ruling out such skeptical hypotheses is false. Nonskeptics believe, therefore, that the actual extent of knowledge and justified belief is, by and large, what we ordinarily think it is.
1) 질문 1.에 답하기 위해서는, 우리가 반드시 앎의 개념을 정의해야만 하는 반면, 질문 2.에 대답하기 위해서 우리는 반드시 앎의 범위를 확정하여야만 한다. 질문 3.은 우리에게 언제 믿음들이 정당화되고 또 언제 정당화되지 않는지를 알려줄 기준들을 진술하는 것을 수반하는 정당화의 개념에 대한 분석을 요구한다. 그리고 만일 우리가 질문 4.에 답하기를 바란다면, 우리는 반드시 우리가 믿음에 있어서 정당화되는 범위를 이해해야만 한다. 회의주의에 따르면, 우리가 아는 것과 우리가 믿음에 있어서 정당화되는 것의 범위는 우리가 일상적으로 생각하는 것보다 더욱 작다. 그들은 우리에게 사악한 악마가 우리가 알아차리지 못하도록 우리를 속이고 있다거나, 한 미친 과학자가 우리를 이상한 낌새를 채지 못하는 통 속의 뇌로 뒤바꿨다는 것을 배제할 아무런 방법도 없다고 추론한다. 비회의론자들에 따르면 우리가 그러한 회의주의적 가정들을 배제시킬 어떠한 방법도 가지고 있지 않다는 주장은 거짓이다. 비회의론자들은 그러므로 앎과 정당화된 믿음의 실제 범위는 대체로 우리가 일상적으로 생각하는 범위 정도라고 믿는다.

2) The four questions listed above are theoretical in nature. Epistemologists, however, are also interested in a practical question: What should we believe?<1> In order to answer this question, epistemologists must develop appopriate methodologies - methodologies that, if followed, help their practitioners to acquire justified beliefs, or even knowledge, and to avoid unjustified beliefs and error. Our focus in this book will be mostly theoretical. We shall discuss issues of practical epistemology only insofar as they bear on theoretical questions.
2) 위에 열거된 네 가지 물음들은 실상 이론적이다. 그렇지만 인식론자들은 또한 실천적인 문제에도 흥미를 가진다. 즉 우리가 믿어야 하는 것은 무엇인지<1>에 대해서도 관심을 가진다. 이 물음에 대답하기 위해, 인식론자들은 반드시 적절한 방법론들을, 만일 그 방법론들에 따른다면 그 방법론들의 실천자들이 정당화된 믿음들, 또는 앎까지도 획득하도록 돕고 정당화되지 않은 믿음들과 오류를 피하도록 돕는 그러한 방법론들을 개발해야만 한다. 이 책에서 우리는 주로 이론적인 데에 촛점을 맞출 것이다. 우리는 실천적 인식론의 문제들을 오로지 그 문제들이 이론적 물음들과 관련되는 한에서만 논의할 것이다.

3) Many philosophers think there is a close conceptual connection between knowledge and justification. They hold that a belief cannot amount to knowledge unless it is justified. On that view, we cannot answer the two questions about knowledge wihtout answering the two questions about justification. However, since the nature and extent of justified belief is an independently interesting issue, questions 3 and 4 would still remain central questions of epistemology even if these philosophers were wrong.
3) 많은 철학자들이 앎과 정당화 사이에 밀접한 개념적 연관이 있다고 생각한다. 그들은 믿음이 정당화되지 않는 한 그것은 앎이 될 수 없다고 주장한다. 그러한 관점에서, 우리는 정당화에 대한 두 가지 물음들에 대답하지 않고 앎에 대한 두 가지 물음들에 답할 수가 없다. 그렇지만, 정당화된 믿음의 본성과 범위는 그 자체로 흥미로운 문제이기 때문에, 물음 3.과 4.는 설령 [앎과 정당화 사이에 연관이 있다고 하는]그 철학자들이 틀렸다손 치더라도 여전히 인식론의 중심문제들로 남을 것이다.

4) In this chapter, we weill focus on the definition of knowledge and the role justification plays in that definition. In Chapter 2, we will consider precisely what we wish to accomplish when we attempt to analyze concepts such as knowledge and justification. In Chapter 3 through 8, we will consider various theories about the nature of justification. In Chapter 9, we will be concerned with issues of methodology, with how epistemology ought to be done. Finally, in Chapter 10, we will examine the extent of justified belief and knowledge and discuss skeptical and nonskeptical responses to questions 2 and 4.
4) 이 장에서, 우리는 앎에 대한 정의와 그 정의에 있어서 정당화가 수행하는 역할에 촛점을 맞출 것이다. 2장에서는 우리가 앎과 정당화 같은 개념들을 분석하고자 시도할 때 우리가 성취하길 바라는 바로 그것을 고찰할 것이다. 3장에서 8장까지에서는 정당화의 본성에 대한 다양한 이론들을 고찰할 것이다. 9장에서는 방법론에 대한 문제들, 인식론이 어떻게 수행되어야 하는지에 관계할 것이다. 끝으로 10장에서는 정당화된 믿음과 앎의 범위를 검토하고 질문 2.와 질문 4.에 대한 회의주의와 비회의주의의 응답들을 논의할 것이다.


Propositional Knowledge
명제적 앎

1) When epistemologists concern themselves with knowledge, what they are interested in is the propositional meaning of the word "know," which must be distinguished from two other meanings. First, the word "know" can be used in the sense of "knowing how" to do a certain thing. For example, we may say: "He knows how to cook an omelet," or "She knows how to play the violin," or "They know how to pick good stocks." Second, the word "know" can be used in the sense of "being acquainted with." In this sense, knowing something, such as a place, a person, or a movie, involves becoming acquainted with it through a personal experience. I know Paris if I have been there and explored the city a bit; I know Arnold Schwarzenegger if I have met him; and I know the movie Vertigo if I have seen it.
1) 인식론자들이 앎에 관계할 때, 그들이 흥미를 가지는 것은 "안다"라는 말의 명제적 의미, 그 단어의 두 가지 다른 뜻들과 반드시 구별되어야만 하는 그러한 명제적 의미이다. 첫째로, "안다"라는 말은 특정한 어떤 것을 "할 줄 안다"는 의미로 사용될 수 있다. 예를 들어 우리는 "그는 오믈렛을 만들 줄 안다"라거나 "그녀는 바이올린을 켤 줄 안다," 또는 "그들은 좋은 가축을 고를 줄 안다"고 말할 것이다. 둘째로, "안다"라는 말은 "접한 적이 있다"라는 의미로 사용될 수 있다. 이러한 의미에서 장소, 사람, 영화 따위의 어떤 것을 안다는 것은 개인적 경험을 통해서 그것과 접하게 되었다는 것을 뜻한다. 내가 만일 파리에 가 본 적이 있고 그 도시를 잠시 여행한 적이 있다면 나는 파리를 안다; 만일 내가 아놀드 슈왈츠제네거를 만난 적이 있다면 나는 그를 안다; 그리고 Vertigo라는 영화를 본 적이 있다면 나는 그 영화를 안다.

2) Knowledge in the propositional sense, however, is knowledge of facts; it is knowing that so-and-so. This kind of knowledge is called "propositional" because the phrase "so-and-so" is to be replaced with a sentence expressing a proposition. For example, replacing "so-and-so" in "I know that so-and-so" with the sentence "The opossum is a nocturnal animal" results in "I know that the opossum is a nocturnal animal."
2) 그렇지만 명제적인 의미에서 앎이란 사실들에 대한 앎이다. 그러한 앎은 이러저러하다는 것을 안다는 것이다. 이러한 종류의 앎이 "명제적"이라고 불리는데 왜냐하면 "so-and-so"라는 관용구가 명제를 표현하는 하나의 문장으로 대체되는 것이기 때문이다. 예를 들어 "나는 이러저러하다는 것을 안다"라는 데에서 "이러저러하다"를 "주머니쥐는 야행성 동물이다"라는 문장으로 대체시키면 "나는 주머니쥐가 야행성 동물이라는 것을 안다"라는 것이 된다.

3) In order to ask the question, "What is knowledge?" in complete generality, let us use the letter S to stand for the subject to whom knowledge is attributed. Furthermore, instead of using the phrase "so-and-so" to represent the proposition known, let us use the letter p. This allows us to reformulate our question thus: What is it for S to know that p?<2> In contemporary epistemology, the dominant approach to answering this question is to state a set of conditions that are both necessary and sufficient for a person S to know that p.
3) 최대한 일반적으로  "앎이란 무엇인가?"라는 물음을 던지기 위해서, 그의 앎이 그에게 귀결되는 주체를 대신해서 S라는 문자를 사용하도록 하자. 여기에 더해서, 알려진 명제를 표현하기 위해 "이러저러하다"라는 관용구를 사용하는 대신 문자 p를 사용하도록 하자. 이것은 우리로 하여금 우리의 물음을 이와 같이 재정식화하도록 해준다. S가 p를 안다는 것은 무엇인가?<2> 현대 인식론에서, 이 물음에 대한 지배적인 접근은 S라는 사람이 P라는 것을 아는 데에 필요하고도 충분하기도 한 일련의 조건들을 진술하는 것이다.


Knowledge as Justified True Belief
정당화된 참인 믿음으로서의 앎

There is an analysis of knowledge, often referred to as the traditional or standard account, that holds that knowledge is justified true belief. We may state the conditions of the standard account of knowledge as follows:

The JTB Account
S knows that p if and only if(iff) (i) p is true; (ii) S believes that p; and (iii) S is justified in believing that p.
종종 전통적 또는 표준적 설명으로 언급되는 앎에 대한 분석이 있는데, 그것은 앎이란 정당화된 참인 믿음이라 주장한다. 우리는 앎에 대한 표준적 설명에 속하는 조건들을 다음과 같이 진술할 것이다.

정당화된 참인 믿음 설명
만일 (i) p가 참이고, (ii) S가 p라는 것을 믿으며, (iii) S가 p를 믿는 것이 정당화된다면, 그리고 오직 그러한 경우에만 S는 p를 안다.

1) The first condition, the truth condition, requires that p be true. What is known must be true, or to put it the other way around, it is impossible to know a proposition that is false. Here are two examples. First, since it is false that there is a fax machine on my desk, I cannot know that there is a fax machine on my desk. Second, many people believe that there are space aliens who periodically visit earth and abduct humans to perform experiments upon them. Some people claim that they themselves have been the victims of such abductions. According to the truth condition, if it is false that these people were abducted by space aliens, then they do not know they were abducted, however strongly they believe it.
1) '참 조건'인 첫 번째 조건은 p가 참일 것을 요구한다. 알려지는 것은 반드시 참이어야 하거나, 혹은 그걸 반대로 두고, 한 명제에 대해 그것이 거짓임을 아는 것이 불가능하다. 여기에 두 가지 예시들이 있다. 첫째로, 내 책상 위에 팩스기기가 있다는 것이 거짓이기 때문에, 나는 내 책상 위에 팩스기기가 있다는 것을 알 수 없다. 둘째로, 많은 사람들은 주기적으로 지구를 방문하고 인간들에게 실험을 하기 위해 그들을 납치하는 외계인들이 있다고 믿는다. 일부 사람들은 그들 자신들이 그러한 유괴들의 희생자들이었노라고 주장한다. 참 조건에 따르면, 이러한 사람들이 외계인들에 의해 유괴되었다는 것이 거짓이라면, 그들은 그들이 유괴되었다는 것을 알지 못하지만, 그들은 강력하게 그걸 믿는다.

2) The second condition, requiring that S believe that p, is called the belief condition, there is, for example, the truth expressed by the 978th entry in the Chicago telephone book (assuming the possessor of the number listed in that entry hasn't disconnected service). Now, if I have got no idea what that number is and whom it belongs to, I don't have a belief about that matter, and thus don't know the truth contained in that entry.
2) S가 p를 믿을 것을 요청하는 두 번째 조건은 믿음 조건이라 불리는데, 예를 들어 시카고 전화번호부에 978번째 항목이 명시된 사실이 있다(그 항목에 기재된 번호의 소유자가 서비스를 끊지 않았다고 가정하면). 이제, 만일 내가 그 번호가 뭔지 그리고 그 번호가 누구 번호인지 아무 생각도 없다면, 그 문제에 대해서 나는 하나의 믿음을 가지지 않고, 그래서 그 항목에 포함된 진실을 알지 못한다.

3) According to the third condition of the JTB account, the justification condition, a true belief amounts to knowledge only if it is justified. This condition was first suggested by Plato. In his dialogue Theatetus, Socrates asks: What shall we say that knowledge is? Theatetus answers that knowledge is true opinion. Socrates is not pleased with this answer, and formulates an objection to it. In reply, Theatetus says:

There is a distinction, Socrates, which I have heard made by someone else, but I had forgotten it. He said that true opinion, combined with reason, was knowledge, but that the opinion which had no reason was out of the sphere of knowledge: and that things of which there is no rational account are not knowledge.<3>
3) JTB 설명의 세 번째 조건인 정당화 조건에 따르면, 참인 믿음은 오직 그것이 정당화될 때라야만 앎이 된다. 이 조건은 플라톤에 의해 처음으로 제안되었다. 그의 대화편 『테아이테토스』에서 소크라테스는 우리가 앎이라고 말해야 할 것은 무엇인지 묻는다. 테아이테토스는 앎이란 참된 의견이라고 대답한다. 소크라테스는 이 대답에 달가워하지 않고, 그에 대해 하나의 반박을 내놓는다. 테아이테토스는 다음과 같이 대답한다:

소크라테스, 제가 다른 누군가에 의해 만들어진 것을 들었던 것은 다른 점이 있습니다만, 저는 잊어 버렸습니다. 그는 근거와 결합된 참된 의견은 앎이지만, 아무런 근거도 없는 의견은 앎의 테두리 밖에 있다고, 그리고 아무런 합리적인 설명도 없는 그러한 것들은 앎이 아니라고 말했습니다.<3>

4) The distinction Theatetus alludes to is that between beliefs supported by reason and beliefs not supported by reason, and his proposal is that true beliefs qualify as knowledge only if they are supported by reason. In contemporary parlance, the distinction is that between justified and unjustified beliefs, and the condition Theatetus is proposing is that for a true belief to amount to knowledge, it must be justified.
4) 테아이테토스가 언급하는 차이점은 근거에 의해 뒷받침되는 믿음들과 근거에 의해 뒷받침되지 않는 믿음들 사이의 차이이고, 그의 제안은 참된 믿음들이 오로지 근거에 의해 뒷받침되어야만 앎의 자격을 얻는다는 것이다. 현대적 용어로, 그 차이는 정당화된 믿음들과 정당화되지 않은 믿음들 사이의 차이이고, 테아이테토스가 내놓고 있는 조건은 참된 믿음이 앎이 되기 위해서는, 그것이 반드시 정당화되어야만 한다는 것이다.

5) What is the rationale for claiming that justification is necessary for knowledge? Many epistemologists would say that an unjustified true belief is no more than a lucky guess, and that a lucky guess falls short of being knowledge.
5) 정당화가 앎의 필요조건이라고 주장하기 위한 근거는 무엇인가? 많은 인식론자들은 정당화되지 않은 믿음은 요행스런 추측에 지나지 않을 것이라고, 그리고 요행한 추측은 앎이기에 부족하다고 말할 것이다.

6) Here is an example of what is meant by calling a belief a "lucky guess." Consider a professor who enjoys full attendance in his logic class during the first week of the semester. On Monday of the second week, there does not appear to be an obvious drop in attendance, and the professor forms the belief: "And here we have full attendance once again." As a matter of fact, the professor's belief is true. However, he has no evidence to support his belief, for there are too many students taking his course for him to recognize how many are in attendance just by glancing around the classroom. Moreover, the fact that there was full attendance throughout the entire first week does not mean that there will be full attendance throughout the second week. Actually, after the first week of a course, a drop in attendance is to be expected. Hence, in order to be justified in believing that there is full attendance, the professor would have to count how many students are actually in his classroom. But he doesn't do that. Instead, he succumbs to wishful thinking. He is hoping for full attendance, and when he looks out on a nearly full classroom, he just can't help believing that all of his student are present. The professor's belief, then, is unsupported by evidence and triggered by wishful thinking. Though ture, it is an unjustified belief - a lucky guess - and thus is not an instance of knowledge.
6) 하나의 믿음을 '요행한 추측"이라고 이름으로써 뜻하는 것이 무엇인지에 대한 하나의 예시가 여기 있다. 학기 첫 주 동안 그의 논리학 수업에 전원이 출석해서 즐거워하는 한 교수를 생각해 보자. 둘째 주 월요일에, 출석률이 뚜렷하게 감소한 것으로는 보이지 않았고, 그 교수는 다음과 같은 믿음을 형성한다. "여기에서 우리는 다시 한 번 전원 출석을 하였다." 사실, 그 교수의 믿음은 참이다. 그렇지만, 그는 그의 믿음을 뒷받침할 아무런 증거도 가지고 있지 않은데, 왜냐하면 그의 수업을 듣는 학생들이 너무나 많아서 그가 얼마나 많은 사람들이 출석하였는지 단지 강의실을 한 번 죽 훑어 보는 것만으로는 알아볼 수 없기 때문이다. 더욱이, 첫째 주 전체를 통해 전원이 출석했다는 그 사실은 둘째 주를 통해서도 전원이 참석할 것임을 의미하진 않는다. 실제로, 강의 첫째주 이후로는 출석률 감소가 예상되게 마련이다. 따라서, 전원이 출석했다는 믿음에 있어서 정당화되기 위해서, 그 교수는 그의 강의실에 실제로 얼마나 많은 학생들이 있는지 헤아려 보아야 할 것이다. 그러나 그는 그렇게 하지 않는다. 대신에, 그는 낙관적 생각에 굴복한다. 그는 전원 출석을 희망하고 있으며, 그가 거의 가득 찬 강의실을 바라보았을 때, 그는 그저 그의 학생 전부가 출석해 있다고 믿을 수밖에 없다. 그 교수의 믿음은 그래서 증거에 의해 뒷받침되지 않고 낙관적 생각에 의해 촉발된다. 참이라 할지라도, 그것은 정당화되지 않은 믿음 - 요행한 추측 - 이며 그래서 앎의 사례가 아니다.


The Gettier Problem
게티어 문제

1-1) According to the standard account, knowledge is justified true belief. In his famous paper "Is Justified True Belief Knowledge?" Edmund Gettier demonstrated, however, that the three conditions of the standard account are not sufficient.<4> Let us consider one of the two counterexamples he presented in that paper.
Suppose Smith is justified in believing

(1) Jones owns a Ford.
1-1) 표준적 설명에 따르자면 앎이란 정당화된 참인 믿음이다. 그렇지만 에드문트 게티어는 그의 유명한 논문 "정당화된 참인 믿음은 앎인가?"에서 표준 설명의 세 가지 조건들이 불충분하다는 것을 증명하였다.<4> 그가 그 논문에서 제시한 두 가지 반례들 중 하나를 살펴보도록 하자.
스미스가 다음과 같이 믿는 것이 정당화되었다고 가정하자.

(1) 존스는 Ford차를 소유하고 있다.

1-2) because, as far back as Smith can remember, Jones has always owned a Ford, and he just got a ride in a Ford Jones claimed was his. As a matter of fact, Jones sold his old Ford and is currently driving a Ford he rented from Hertz. Hence (1) is false. Suppose further that Smith applies the laws of deductive logic to (1) and deduces the following three propositions about the whereabouts of his friend Brown:

(2) Either Jones owns a Ford or Brown is in Boston.
(3) Either Jones owns a Ford or Brown is in Barcelona.
(4) Either Jones owns a Ford or Brown is in Brest-Litovsk.
1-2) 스미스가 기억할 수 있는 한에서 존스는 언제나 Ford차를 가지고 있었고, 존스가 존스 자신의 차라고 주장했던 Ford차를 방금 타고 갔기 때문이다. 사실은, 존스는 그의 오래된 Ford를 팔았고 지금은 그가 헤르츠로부터 빌렸던 Ford차를 운정하고 있다. 따라서 (1)은 거짓이다. 나아가 스미스가 연역 논리의 법칙을 (1)에 적용하여 그의 친구 브라운의 소재에 대해 다음과 같은 세 가지 명제들을 도출했다고 가정해 보자.

(2) 존스가 Ford차를 가지고 있거나 브라운이 보스턴에 있다.
(3) 존스가 Ford차를 가지고 있거나 브라운이 바르셀로나에 있다.
(4) 존스가 Ford차를 가지고 있거나 브라운이 브레스트-리토프스크에 있다.

1-3) By sheer coincidence, Brown happens to be in Barcelona. Hence (3) happens to be true. Smith, however, has no evidence at all for believing that Brown is in Barcelona. Nevertheless, according to Gettier, Smith is justified in believing (3). Gettier defends this claim by appealing to the following principle:

If S is justified in believing p, and p entails q,<5> and S believes q because he deduces q from p, then S is justified in believing q.<6>
1-3) 순전히 우연의 일치로, 브라운은 마침 바르셀로나에 있다. 따라서 (3)은 우연히도 참이다. 그렇지만 스미스는 브라운이 바르셀로나에 있다는 것을 믿을 아무런 증거도 전혀 가지고 있지 않다. 그럼에도 불구하고, 게티어에 따르면, 스미스는 (3)을 믿음에 있어서 정당화된다. 게티어는 이러한 주장을 다음 원칙에 호소함으로써 옹호한다.

만일 S가 p를 믿음에 있어서 정당화된다면, 그리고 p가 q를 수반한다면,<5> 또 S가 p로부터 q를 도출하였기 때문에 q를 믿는다면, S가 q를 믿음에 있어서 정당화된다.<6>

1) Although some philosophers have challenged this principle, it must be admitted that it is quite plausible. Its plausibility derives from the fact that there is no better way of expanding the stock of one's justified beliefs than through deduction.<7> If we apply this principle to our case, we can reason thus: Smith is justified in believing (1), he recognizes that (1) entails (3), and thus deduces (3) from (1); consequently, he is justified in believing (3). However, does he know that (3) is true? Since (1) is false, and Smith has no evidence at all for believing that Brown is actually in Barcelona, we'd have to say that he does not. Hence (3) is an example of a justified true belief that falls short of being knowledge.
1) 일부 철학자들이 이 원칙에 저항하였다 할지라도, 그 원칙이 타당하다는 것은 인정될 수밖에 없다. 그 원칙의 정당성은 한 사람이 가지고 있는 정당화된 믿음을 확장시키는 방법으로 연역을 통하는 것보다 더 나은 방법은 아무것도 없다는 사실로부터 나온다.<7> 만일 우리가 이 원칙을 우리의 경우에 적용한다면, 우리는 따라서 스미스가 (1)을 믿음에 있어서 정당화된다고, 그리고 그가 (1)이 (3)을 수반한다는 것을 알고 있으며, 따라서 (1)로부터 (3)을 도출한다고, 즉 결론적으로, 그는 (3)을 믿음에 있어서 정당화된다고 추론할 수 있다. 그렇지만, 그가 (3)이 참임을 아는가? (1)이 거짓이므로, 그리고 스미스는 브라운이 실제로 바르셀로나에 있다는 것을 믿을 아무런 증거도 전혀 없으므로, 우리는 그가 알지 못한다고 말해야 할 것이다. 따라서 (3)은 앎이기에는 불충분한 정당화된 참인 믿음의 일례이다.

2) In order to solve the Gettier problem, the definition of knowledge must be modified either by adding a fourth condition or by building an appropriate clause into the justification condition. Later in this chapter, we will return to this issue. In the meantime, let's briefly consider each of the three concepts that figure in the traditional account of knowledge: truth, belief, and justification.
2) 게티어 문제를 해결하기 위해, 앎에 대한 정의는 네 번째 조건을 추가함으로써든 아니면 정당화 조건에 적합한 구절을 붙박음으로써든 수정되어야만 한다. 이 장의 뒷부분에서, 우리는 이 문제로 되돌아올 것이다. 그 동안에, 앎에 대한 전통적인 설명에 등장하는 세 가지 개념들, 참, 믿음, 그리고 정당화의 각각을 간략하게 고찰하도록 하자.

<1> According to Roderick Chisholm (1989), the practical purpose of epistemology can be put thus: "We want to do our best to improve our set of beliefs - to replace those that are unjustified by others that are justified and to replace those that have a lesser degree of justification with others that have a greater degree of justification" (p.1). For an elaboration of the theoretical and practical dimensions of epistemology, see Sosa (1991), essay 14.
<1> 로데릭 치솔름(1989)에 따르면, 인식론의 실천적 의도는 따라서 "우리는 우리의 일련의 믿음들을 개선시키기 위해 최선을 다하기를 바란다 - 정당화되지 않은 믿음들을 정당화된 다른 믿음들로 정당화의 정도가 부족한 믿음들을 정당화의 정도가 더욱 큰 다른 믿음들로 대체하기를 바란다"(p.1)라고 제시될 수 있다. 인식론의 이론적 차원과 실천적 차원에 대한 노작에 대해서는 소사(1991), 소론 14를 보라.

<2> An expression of the form "S knows that p" is called a "schema." A schema is a formula that is neither true nor false, but can be transformed into a sentence that is true or false by making appropriate replacements. Thus, if we replace "S" with "Socrates" and "p" with "Courage is a virtue," the result is: "Socrates knows that courage is a virtue." Since strict adherence to the "that" clause can result in awkward formulations, we shall also use the phrase "S knows p," taking it to be understood that substituting a proposition for "p" will call for adding a "that."
<2> "S가 p라는 점을 안다"라는 형태의 표현은 "도식"이라 불린다. 도식은 참도 거짓도 아니지만 적절한 대체물들을 만듦으로써 참 또는 거짓인 문장으로 변형될 수 있는 식이다. 그래서, 만일 우리가 "S"를 "소크라테스"로 그리고 "p"를 "용기는 하나의 탁월함이다"로 대체한다면, 그 결과는 "소크라테스는 용기가 하나의 탁월함이라는 것을 안다"가 된다. "that(~라는 점, 사실, 것)" 절에 대한 엄격한 고수는 어색한 어구들을 귀결할 수 있으므로, 우리는 또한 "S가 p를 안다"라는 구를, "that"을 추가할 것을 요구할 "p"라는 명제를 대신하는 것으로 이해되는 것으로 취하면서, 사용할 것이다.

<3> Theatetus 200d - 201d. Quoted from The Dialogues of Plato, 4th ed., trans. Benjamin Jowett (Oxford: The Clarendon Press, 1953).
 
<4> See Gettier (1963).

<5> By saying that p entails q, we mean that it's not possible for p to be true and q to be false.
<5> p가 q를 수반한다고 말함으로써, 우리는 p가 참이면서 q가 거짓이기는 불가능하다는 것을 의미한다.

<6> This principle asserts that justification is transmitted from one proposition to another through recognized entailments. It is also referred to as the "closure" principle because it tells us that moving from one proposition to another on the basis of a recognized entailment does not get us outside of the closed area of justified beliefs. See Dancy (1985), pp. 10f.
<6> 이 원칙은 정당화가 의식된 계사 한정들을 통해서 한 명제에서 다른 명제로 전해진다고 주장한다. 그것은 또한 "폐쇄 원칙"으로 언급되기도 하는데 왜냐하면 그것이 우리에게 의식된 계사 한정에 기초한 한 명제로부터 다른 명제로의 이동이 정당화된 믿음들의 폐쇄된 영역 밖으로 나가지 않는다는 것을 말해주기 때문이다.

<7> The reason for that is that deduction is truth-preserving. If p is true and q can be deduced from p, then it is impossible that q is false. See Klein (1992), p.460.
<7> [연역보다 더 나은 정당화된 믿음들의 확장방법이 없는]그 이유는 연역이 진리-보존적이라는 것이다. 만일 p가 참이고 q가 p로부터 연역된다면, q가 거짓이기는 불가능하다.

-蟲-
Preface

1) This book presents a systematic, up-to-date account of the landscape of contemporary epistemology. To ensure that it is suitable for introductory epistemology courses, I have explained each issue from scratch, presupposed very little philosophical knowledge, and included an account of the logical and conceptual tools that are used for philosophical discussion and analysis. Numerous examples illustrate the problems, theories, objections, and rebuttals that make up the meat of each topic, and study questions and excercises at the end of each chapter review and reinforce the material covered in that chapter.
1) 이 책은 현대 인식론의 조망에 대한 체계적인 최신의 설명을 제시한다. 이 책이 입문 수준의 인식론 과정들을 위해 적합하다는 것을 보장하기 위해, 나는 각각의 문제를 아무런 예비 없이 설명하였고, 아주 적은 철학적 지식만을 상정하였으며, 철학적 논의와 분석을 위해 사용되는 논리적이고 개념적인 수단들에 대한 설명을 포함시켰다. 많은 예시들은 각 주제의 골자를 이루는 문제들, 이론들, 반박들, 그리고 반증들을 설명하며, 각 장 말미의 학습과제들과 연습문제들은 해당 장에서 다루어진 제재[題材]를 복습하고 보강한다.

2) The writer of a textbook of limited length must make some kind of trade-off between breadth and depth, between including every important topic and doing full justice to every topic included. This book, I believe, offers a reasonable compromise. It contains chapters on all the central topics of epistemology - the analysis of knowledge, epistemic justification, foundationalism, coherentism, reliabilism, traditional versus naturalistic epistemology, and skepticism - and discusses each of these topics in depth. Thus it is of interest to both undergraduates and beginning graduate students who wish to familiarize themselves with the epistemological territory.
2) 제한된 난이도의 교재를 저술하는 사람은 반드시 넓이와 깊이 사이에서, 중요한 모든 주제를 포함하는 일과 포함된 각각의 모든 주제를 완전히 공평하게 다루는 일 사이에서, 일종의 균형을 이루어야만 한다. 내가 믿기로 이 책은 합리적인 절충을 제공한다. 이 책은 인식론에 대한 모든 중심 주제들에 대한 장들을 - 앎에 대한 분석, 인식론적 정당화, 토대론, 정합론, 신빙론, 전통적 인식론 대(對) 자연주의적 인식론, 회의주의 - 포함하며 이러한 주제들 하나하나를 깊이있게 논의한다. 그래서 인식론 분야에 정통하고자 하는 학부생들과 석사 초년생들 모두에게 이 책은 흥미있는 것이다.

3) There are sections in this book in which, some will think, I unnecessarily digress from the main theme or pursue a particular issue for too long. Since diverging judgments about such matters can't be avoided, I propose that instructors simply choose from the table of contents to make up a menu of their own liking.
3) 이 책에는 일부 사람들에게 내가 불필요하게 핵심 주제에서 벗어나거나 지나치게 장황하게 특정 주제를 파고든다고 생각하게 될 부분들이 있다. 그러한 문제들에 대해 의견이 갈리는 것은 피할 수 없기 때문에, 나는 강의자들이 그들의 고유한 취향에 맞는 계획표를 구성하도록 목차에서 단순히 선택할 것을 제안한다.

4) One epistemological topic conspicuously missing from this book is perception. I decided to omit this topic because including it would have meant either making the book too long for an introductory test or omitting some other topic I considered too important to leave out. I suggest that instructor who want to cover perception use supplementary course materials. Another feature of this book that needs explaining is the location of the chapter on skepticism. Since skeptical arguments are an excellent device for getting students excited about the study of epistemology, a discussion of them might reasonably be expected at the book's beginning. A satisfactory treatment of skepticism, however, requires a good grasp of what epistemology is about, plus a certain amount of skill at philosophical reasoning. For these reasons, I have placed the discussion of skepticism at the very end of the book.
4) 이 책에서 명백하게 결여된 한 가지 인식론적 주제는 지각이다. 나는 이 주제를 생략하기로 결정하였는데 왜냐하면 그 주제를 포함한다는 것이 이 책을 입문 수준의 시도로서는 지나치게 길게 만들거나 내가 배제시킬 수 없을 만큼 중요하게 고려하는 다른 어떤 주제를 생략한다는 것을 의미했을 터이기 때문이다. 나는 지각을 포괄하길 원하는 강의자가 추가적인 강의교재들을 사용할 것을 제안한다. 설명을 요하는 이 책의 또 다른 특징은 회의주의를 다루는 장의 위치이다. 회의적 논증들은 학생들을 인식론에 열광케 하기 위한 훌륭한 장치이기 때문에, 그 논증들에 대한 논의는 마땅히 이 책의 서두에 올 것이 예상될 것이다. 그렇지만 회의주의에 대한 만족스러운 논의는 인식론이 무엇에 대한 것인를 훌륭하게 이해할 것을, 거기에 더하여 철학적 추론에 있어서 일정 정도의 기술을 요한다. 이러한 이유들로, 나는 회의주의에 대한 논의를 이 책의 말미에 두었다.

5) I have tried, as far as possible, to link particular theories to the philosophers who advocate them because I think the best way to penetrate a philosophical issue is to look at the writings of philosophers who are widely regarded as representing opposing camps on that issue. Hence much of the book is devoted to discussing William Alston as a critic of the deontological concept of epistemic justification; Laurence Bonjour and Keith Lehrer as advocates of coherentism; Alvin Goldman as a proponent of reliabilism; W. V. Quine, Alvin Goldman, and Hilary Kornblith as advocates of naturalized epistemology. Other contemporary philosophers whose writings either figure in, or have influenced this book are Robert Audi, Roderick Chisholm, Alvin Plantinga, and Ernest Sosa.
5) 나는 가능한 한 특수한 이론들이 그 이론들을 지지한 철학자들과 연계되도록 노력하였는데, 왜냐하면 나는 철학적 문제를 통관하는 가장 좋은 방법이 그 문제에 대해 반대 진영을 대표하는 것으로 널리 알려진 철학자들의 저술들을 살펴보는 것이라고 생각하기 때문이다. 따라서 이 책의 상당부분은 인식론적 정당화의 의무론적 이해에 대한 비판자로서 윌리엄 알스톤을, 정합론에 대한 대변자들로서 로렌스 봉주르와 케이트 레러를, 신빙론의 제안자로서 앨빈 골드만을, 자연주의화된 인식론에 대한 지지자들로서 W. V. 콰인, 앨빈 골드만, 그리고 힐러리 콘빌스를 논의하는 데에 할애된다. 이 책에 저술들이 등장하거나 또는 영향을 미친 다른 동시대 철학자들은 로버트 아우디, 로드릭 치셜럼, 앨빈 플랜팅가, 그리고 어니스트 소사이다.

6) I don't believe that a textbook writer should attempt to adopt a neutral point of view. Concealing one's own stand on the issues under consideration usually results in a stale and lackluster product. Thus on occasion I have added my own two cents to the issues I discuss. Of course I have done my best to give each theory and point of view a fair hearing.
6) 나는 교재 저술가가 중립적 관점을 취해야 한다고는 믿지 않는다. 그 주제들에 대한 한 개인의 고유한 입장을 고려 하에 감추는 것은 보통 진부하고 생기 잃은 산물을 낳는다. 그래서 가끔 나는 나 자신의 2센트 짜리(시덥잖은) 생각들을 내가 논의하는 문제들에 덧붙였다. 물론 나는 각각의 이론과 관점을 공평하게 듣기 위해 최선을 다했다.

7) I am indebted to Ted Bolen for his encouragement and support, to my colleague Casey Swank for reading the entire manuscript and discussing it with me, to Noah Lemos for his helpful comments on several chapters, and to my reviewers, Mark Bernstein, Douglas C. Long, and Paul E. Tibbetts, for their comments and criticisms. I am especially grateful to a reviewer who would like to remain anonymous for generously providing me with a vast amount of ciritical comments.
7) 나는 테드 볼렌에게 그의 응원과 지지에 대한 감사를 전하며, 나의 동료 케이시 스웽크에게는 전체 원고를 읽어주고 나와 함께 논의해준 점에 감사를 표하고, 노아 레모스에게는 여러 장들에 대한 그의 유익한 논평들을, 그리고 나의 논평자들인 마크 번슈타인과 더글라스 C. 롱, 그리고 폴 E. 티벳츠에게는 그들의 논평들과 비평들을 감사하는 바이다. 나는 특히나 나에게 엄청난 양의 비평적 논평들을 아낌없이 내놓은, 익명으로 남기를 바란 한 사람의 논평자에게 감사를 표한다.

-蟲-
Apology: Defence of Socrates

Plato

1) I don't know how you, fellow Athenians, have been affected by my accusers, but for my part I felt myself almost transported by them, so persuasively did they speak. And yet hardly a word they have said is true. Among their many falsehoods, one especially astonished me: their warning that you must be cereful not to be taken in by me, because I am a clever speaker. It seemed to me the height of impudence on their part not to be embarrassed at being refuted straight away by the facts, once it became apparent that I was not a clever speaker at all - unless indeed they call a "clever" speaker one who speaks the truth. If that is what they mean, then I would admit to being an orator, although not on a par with them.
1) 아네테 동포 여러분, 저는 여러분들께서 저의 고발자들에게 어떻게 영향을 받으셨는지 모릅니다만, 저로서는 제 자신이 그들에 의해 거의 잊혀진 듯이 여겨졌습니다. 그만큼 그들이 그럴싸하게 말했으니 말입니다. 그렇긴 하지만 그들이 했던 말은 거의 진실이 아닙니다. 그들의 여러 거짓말들 중에서, 특히나 한 가지 것이 저를 경악하게 하였습니다. 그것은 여러분들께서 저에게 사로잡히지 않도록 주의해야만 하며, 그 이유는 제가 영리한 연설가이기 때문이란 그들의 경고였습니다. 제가 전혀 언변이 능란한 사람이 아니었다는 것이 밝혀지기만 하면, 사실들로 인해 즉시 반박되리란 점에 그들로서 부끄러워하지 않는다는 건 제게는 더할 나위 없이 뻔뻔스러운 노릇으로 보입니다 - 덧붙여 그들이 진실을 말하는 사람을 두고 "능란한" 언변을 가진 자라 부르지 않는 한 말입니다. 만일 그들이 그런 뜻으로 하는 말이라면, 저는 그들과는 급이 다른 연설가라고 시인할 것입니다.

2) As I said, then, my accusers have said little or nothing true; whereas from me you shall hear the whole truth, though not, I assure you, fellow Athenians, in language adorned with fine words and phrases or dressed up, as theirs was: you shall hear my points made spontaneously in whatever words occur to me - persuaded as I am that my case is just. None of you should expect anything to be put differently, because it would not, of course, be at all fitting at my age, gentlemen, to come before you with artificial speeches, such as might be composed by a young lad.
2) 그러니까, 제가 말씀드렸듯, 저를 고발한 자들은 거의 사실을 말하지 않았거나 아무런 사실도 말하지 않았습니다. 하지만 저에게서는 여러분들께서 모든 진실을 듣게 되실 겁니다. 비록, 제가 여러분들께 장담하건데, 아테네 동포 여러분, 저들의 말들이 그러했던 것 같이 훌륭한 단어들과 구절들로 장식되거나 치장된 말로는 아닐지라도 말씀입니다. 그러니까 여러분들께서는 즉흥적으로 제게 떠오르는 - 저로서는 제 경우가 옳다고 납득되는 - 그런 아무 말로나 이루어진 제 입장들을 듣게 되실 겁니다. 여러분들 중 그 누구도 달리 기대하셔서는 안 될 것인데, 왜냐하면 여러분, 여러분 앞에 어린 녀석들이나 하듯이 꾸며낸 말들을 가져 오는 것은 제 나이에 전혀 어울리지 않을 것이기 때문입니다.

3) One thing, moreover, I would earnestly beg of you, fellow Athenians. If you hear me defending myself with the same arguments I normally use at the banker's tables in the market-place (where many of you have heard me) and elsewhere, please do not be suprised or protest on that account. You see, here is the reason: this is the first time I have ever appeared before a court of law, although I am over 70; so I am literally a stranger to the diction of this place. And if I really were a foreigner, you would naturally excuse me, were I to speak in the dialect and style in which I had been brought up; so in the present case as well I ask you, in all fairness as I think, to disregard my manner of speaking - it may not be as good, or it may be better - but to consider and attend simply to the question whether or not my case is just; because that is the duty of a judge, as it is an orator's duty to speak the truth.
3) 더욱이 한 가지를 제가 여러분들께 진심으로 간청드려야 하겠습니다, 아테네인 여러분. 만일 여러분들께서 제가 평소에 (여러분들 중 많은 분들께서 제게 이야기를 들으셨던) 아고라의 환전소에서 그리고 다른 어디에서든 하던 똑같은 말들을 가지고 제 자신을 변호하는 것을 들으신다면, 부디 놀라시거나 그런 설명에 항의하지 말아 주시길 바랍니다. 보시다시피, 여기 그 이유가 있으니 말입니다. 제가 일흔이 넘었습니다만서도, 이번이 저로서는 처음으로 법정앞에 나서 본 것입니다. 그래서 저는 이런 자리에서의 말투에 말 그대로 문외한입니다. 그리고 만일 제가 정말로 이방인이었더라면, 여러분들께서는 자연스럽게 저를 양해해 주셨을 것입니다, 제가 사투리로 제가 훈육받아왔던 그러한 식으로 말할 것을 말입니다. 그래서 지금 이 경우에도 여러분께 제가, 저의 말하는 방식을 - 그게 썩 좋지 못할 수도 있고, 아니면 더 나을 수도 있겠습니다만 - 개의치 않고 단순히 제 주장이 올바른지 아닌지 하는 물음만을 고려하고 또 주의를 기울이는 것을, 제 생각에는 온당하게 부탁드립니다. 왜냐하면 그것이 판정단들의 책임이기 때문입니다, 진실을 말하는 것이 연설가의 몫이듯이 말입니다.

4) To begin with, fellow Athenians, it is fair that I should defend myself against the first set of charges then turn to the later charges and the more recent ones. You see, I have been accused before you by many people for a long time now, for many years in fact, by people who spoke not a word of truth. It is those people I fear more than Anytus and his crowd, though they too are dangerous. But those others are more so, gentlemen: they have taken hold of most of you since childhood, and made persuasive accusations against me, yet without an ounce more truth in them. They say that there is one Socrates, a "wise man," who ponders what is above the earth and investigates everything beneath it, and turns the weaker argument into the stronger.
4) 아테네 시민여러분, 우선 제가 첫 번째 고소자들의 무리에 대항해 제 자신을 변호하고 그 다음에 나중 고발자들과 더 최근의 고발자들에게 맞서는 것이 마땅할 것입니다. 아시다시피, 저는 지금으로부터 오래 전부터, 실로 여러 해에 걸쳐서, 많은 사람들에 의해 여러분들앞에 고소되어 왔습니다. 그 사람들은 진실이라고는 한 마디도 하지 않은 사람들입니다. 그러한 사람들을 저는 아니토스와 그의 무리들보다 더욱 두려워합니다. 그들도 마찬가지로 위험하긴 하지만 말입니다. 그러나 저 다른 사람들이 더욱 위험합니다, 여러분. 그들은 여러분들 거의 모두를 어릴 적부터 사로잡아 왔고, 저에 대해 그럴 듯한 혐의들을 지어왔습니다, 그것들 중에 조금도 더한 진실은 없지만 말입니다. 그들은 소크라테스라는 한 사람이, "현명한 자"가, 땅밑에 뭐가 있을지 골몰하고 그 아래의 모든 것들을 연구하며, 더 약한 논변을 더욱 강한 것으로 바꾸는 자가 있다고 말합니다.

5) Those accusers who have spread such rumour about me, fellow Athenians, are the dangerous ones, because their audience believes that people who inquire into those matters also fail to acknowledge the gods. Moreover, those accusers are numerous, and have been denouncing me for a long time now, and they also spoke to you at an age at which you would be most likely to believe them, when some of you were children or young lads; and their accusations simply went by default for lack of any defence. But the most absurd thing of all is that one cannot even get to know their names or say who they were - except perhaps one who happens to be a comic playwright. The ones who persuaded you by malicious slander, and also some who persuade others bacause they have been persuaded themselves, are all very hard to deal with: one cannot put any of them on the stand here in court, or cross-examine anybody, but one must literally engage in a sort of shadow-boxing to defend oneself, and cross-examine without anyone to answer. You too, then, should allow, as I just said, that I have two sets of accusers: one set who have accused me recently, and the other of long standing to whom I was just referring. And please grant that I need to defend myself against the latter first, since you too heard them accusing me earlier, and you heard far more from them than from these recent critics here.
5) 저에 대한 그런 소문을 퍼뜨린 그러한 고발자들은, 아테네 시민여러분, 위험한 자들입니다. 왜냐하면 그들의 청중들은 그러한 문제들을 조사하는 사람들이 또한 신들을 인정하지 않기도 한다고 믿기 때문입니다. 더욱이, 그러한 고발자들은 수가 많고, 지금으로부터 오래 전부터 저를 비난해 오고 있으며, 그들이 또한 여러분들께서 거의 틀림없이 그들을 믿으셨을 그러한 나이대에, 여러분들 중 몇분은 아이들이셨거나 나이 어린 사내들이셨을 때에, 여러분들께 말을 했었기 때문입니다. 그리고 그들의 고발들은 어떤 변론도 없었기 때문에 단순한 결석재판이었습니다. 그러나 무엇보다도 가장 말도 안 되는 것은 그들의 이름들을 알거나 그들이 누구인지 말할 수조차 없다는 점입니다. - 아마도 희극 작가일 한 사람은 제외하고 말입니다. 악질적인 중상모략으로 여러분들을 설득했던 그런 자들과 또한 그들 스스로 설득되었던 탓에 다른 이들을 설득시키는 몇몇 사람들은 모두 대하기가 무척이나 어렵습니다. 그래서 여러분들께서도 제가 말했던 것처럼 제가 두 부류의 고발자들을 가진다는 것을 인정해 주실 것입니다. 한 부류는 최근에 저를 고발한 사람들이고, 아주 오래 된 다른 부류는 제가 방금 말하던 그 사람들입니다. 그리고 제가 우선 뒤엣 사람들에 대항해 제 자신을 변호해야 한다는 것을 인정해 주시길 부탁드립니다. 왜냐하면 여러분들도 마찬가지로 일찍부터 저들에게서 저를 고발하는 것을 들으셨고, 여기에 있는 최근의 비판자들에게서보다도 그들로부터 훨씬 더 많이 들으셨기 때문입니다.


6) Very well, then. I must defend myself, fellow Athenians, and in so short a time must try to dispel the slander which you have had so long to absorb. That is the outcome I would wish for, should it be of any benefit to you and to me, and I should like to succeed in my defence - though I believe the task to be a difficult one, and am well aware of its nature. But let that turn out as God wills(as gods will): I have to obey the law and present my defence.
6) 그럼, 좋습니다. 저는 저 자신을 변호해야만 합니다, 아테네 시민여러분, 그리고 아주 짧은 시간 안에 여러분들이 아주 오랫동안 받아들여 오신 그 중상을 떨쳐 드려야만 합니다. 그것이 제가 바라는 결과이고, 여러분들께도 제게도 득이 되는 것일 터이며, 저는 저의 변호를 성공할 것 같습니다 - 비록 제가 그 일이 어려운 일에 속한다고 믿고, 그 본성을 잘 알고 있지만 말입니다. 그러나 그것은 신들의 뜻대로 되도록 놓아 둡시다. 저는 법에 따라 제 자신의 변호를 내놓아야 하니까요.

7) Let us examine, from the beginning, the charge that has given rise to the slander against me - which was just what Meletus relied upon when he drew up this indictment. Very well then, what were my slanderes actually saying when they slandered me? Let me read out their deposition, as if they were my legal accusers.
7) 우리는 저에 대한 중상을 생기게 하였던 그 죄목을 처음부터 검토해 보도록 합시다 - 그것은 멜레토스가 이 고발장을 작성할 적에 근거하였던 바로 그 죄목입니다. 좋습니다, 저를 중상하는 자들이 저를 중상하고 있던 때에 실제로 말하고 있던 것은 무엇입니까? 저는 그들의 증언문을 낭독하고자 합니다, 마치 그들이 저의 법적인 고발자들인 것처럼 말입니다.

8) "Socrates is guilty of being a busybody, in that he inquires into what is beneath the earth and in the sky, turns the weaker argument into the stronger, and teaches others to do the same."
8) "소크라테스는 참견쟁이 죄인입니다. 그가 지하의 것과 하늘의 것을 탐구하고, 더 약한 논변을 더욱 강한 것으로 바꾸며, 다른 이들이 똑같은 짓을 하도록 가르친다는 점에서 그렇습니다."

9) The charges would run something like that. Indeed, you can see them for yourselves, enacted in Aristophanes' comedy: in that play, a character called "Socrates" swings around, claims to be walking on air, and talks a lot of other nonsense on subjects of which I have no understanding, great or small.
9) 그 죄목들은 그와 같은 어떤 것들이 될 것입니다. 덧붙여서, 여러분들께서는 스스로, 아리스토파네스의 희극에서 상연되는 그것들을 보실 수 있습니다. 그 연극에서, "소크라테스"라고 불리는 한 인물은 빙글빙글 돌고, 허공 중을 걷고 있다고주장하며, 제가 많든 적든 이해하지 못하는 주제들에 대해 무의미한 다른 많은 것들을 말합니다.

10) Not that I mean to belittle knowledge of that sort, if anyone really is learned in such matters - no matter how many of Meletus' lawsuits I might have to defend myself against - but the fact is, fellow Athenians, those subjects are not my concern at all. I call most of you to witness yourselves, and I ask you to make that quite clear to one another, if you have ever heard me in discussion (as many of you have). Tell one another, then, whether any of you has ever heard me discussing such subjects, either briefly or at length; and as a result you will realize that the other things said about me by the public are equally baseless.
10) 제가 그런 종류의 지식들을 하찮다고 말하려는 것은 아닙니다, 만일 누구든 정말로 그런 문제들에 대해 학식이 있다면 말씀입니다 - 제가 그에 대항해 제 자신을 변호해야 할 멜레토스의 소송내용들이 얼마나 많든지 상관 없이 - 그러나 사실은, 아테네인 여러분, 그러한 주제들은 전혀 제 관심사가 아닙니다. 저는 여러분들 대부분을 여러분들 자신들의 증인으로 청하며, 여러분들께 저는, 만일 여러분들께서 한 번이라도 저에게서 논의하는 것을 들으신 적이 있다면(여러분들 중 많은 분들께서 그러하셨듯이), 서로가 서로에게 그것을 분명히 밝혀 주시기를 부탁드립니다. 그러니 서로에게 말씀해 주십시오, 여러분들 중 누구시든 그러한 주제들로 제가 논의하는 것을 들으신 일이 있으신지, 짧든 길든 말씀입니다. 그리고 결론적으로 여러분들께서는 대중들이 저에 대해 말하는 그 이외의 이야기들도 똑같이 근거없다는 것을 알아차리실 것입니다.

11) In any event, there is no truth in those charges. Moreover, if you have heard from anyone that I undertake to educate people and charge fees, there is no truth in that either - though for that matter I do think it also a fine thing if anyone is able to educate people, as Gorgias of Leontini, Prodicus of Ceos, and Hipias of Elis profess to. Each of them can visit any city, gentlemen, and persuade its young people, who may associate free of charge with any of their own citizens they wish, to leave those associations, and to join with them instead, paying fees and being grateful into the bargain.
11) 어쨌든, 그러한 죄목들 중에는 아무런 진실도 없습니다. 더욱이, 만일 여러분들께서 누구에게서든 제가 사람들을 교육하고 값을 받으려 한다고 들으셨다면, 거기에도 어느 것 하나 진실은 없습니다 - 그 문제에 대해서 제가 누구든 사람들을, 레온티노이의 고르기아스, 케오스의 프로디코스, 그리고 엘리스의 히피아스가 공언하듯이, 그렇게 가르칠 수 있다면 그것 또한 훌륭한 일이라고 생각하긴 합니다만 말씀입니다. 그들 각자는 어느 도시든 방문할 수 있습니다, 시민 여러분, 그리고 그 도시의 젊은 사람들, 그들이 원하는 그들의 시민 동료들 중 누구에게는 공짜로 어울릴 그런 사람들을 그러한 연대들을 내버려 두고, 대신에 그들과 어울리라고, 값을 치르고 또한 감사하면서 그러하라고 설득할 수 있습니다.

12) On that topic, there is at present another expert here, a gentleman from Paros; I heard of his visit, because I happened to run into a man who has spent more money on sophists than everyone else put together - Callias, the son of Hipponicus. So I questioned him, since he has two sons himself.
12) 그 주제에 관해서, 당장 여기에 또 다른 전문가가 있습니다, 파로스에서 온 시민 말입니다; 저는 그가 방문하였다는 것을 들었는데, 왜냐하면 제가 마침 다른 모두를 합한 것보다도 더 많은 돈을 소피스트들에게 써온 한 사람을 만났기 때문입니다 - 그는 히포니코스의 아들 칼리아스입니다. 그래서 저는 그에게 물었습니다, 그가 자신의 두 아들을 두었기 때문입니다.

13) "Callias," I said, "if your two sons had been born as colts or calves, we could find and engage a tutor who could make them both excel superbly in the required qualities - and he'd be some sort of expert in horse-rearing or agriculture. But seeing that they are actually human, whom do you intend to engage as their tutor? Who has knowledge of the required human and civic qualities? I ask, because I assume you've given thought to the matter, having sons yourself. Is there such a person," I asked, "or not?"
"Certainly," he replied.
"Who is he?" I said; "Where does he come from, and what does he charge for tuition?"
"His name is Evenus, Socrates," he replied; "He comes from Paros, and he charges 5 minas."
13) "칼리아스," 제가 말했습니다, "만일 당신의 두 아들이 망아지들이나 송아지들로 태어났었다면야, 우리는 그 친구들을 모두를 필요한 소질들에 있어서 월등히 빼어나게 만들 수 있는 교사를 찾고 또 고용할 수도 있을 거에요. - 또 그는 말사육이나 농사에 있어서 어떤 종류의 전문가겠지요. 하지만 그 친구들이 실제로는 사람인지라, 당신은 누구를 그들의 교사로 고용할 생각인가요? 누가 꼭 필요한 인간적이고 시민적인 소질들에 대해 지식을 가지고 있나요? 저는, 당신이 당신 자신의 아이들을 두고 있기 때문에 그 문제를 곰곰히 생각했을 거라고 짐작하기에 이렇게 묻는 것입니다. 그런 사람이 있나요," 제가 물었습니다, "혹은 없나요?"
"확실히 있지요." 그가 대답했습니다.
"그가 누군가요?" 제가 말했습니다. "그는 어디에서 왔고, 수업료는 얼마인가요?"
"그의 이름은 에우에노스에요, 소크라테스." 그가 대답했습니다. "그는 파로스에서 왔고, 5므나를 받습니다."

14) I thought Evenus was to be congratulated, if he really did possess that skill and imparted it for such a modest charge. I, at my rate, would certainly be giving myself fine airs and graces if I possessed that knowledge. But the fact is, fellow Athenians, I do not.
14) 저는 에우에노스가 축하받을 사람이라 생각했습니다. 만일 그가 정말로 그런 기술을 지니고 있었고 그런 대단찮은 값으로 그걸 전해주었다면 말입니다. 저라면 제가 그런 지식을 가졌더라면 잘난 체를 하고 자랑하였을 것이 확실합니다. 그러나 사실은 아테네인 여러분, 저는 그런 지식을 가지고 있지 못합니다.

15) Now perhaps one of you will interject: "Well then, Socrates, what is the difficulty in your case? What is the source of these slanders against you? If you are not engaged in something out of the ordinary, why ever has so much rumour and talk arisen about you? It would surely never have arisen, unless you were up to something different from most people. Tell us what is it, then, so that we don't jump to conclusions about you."
15) 이제 아마도 여러분들 중 한 분께서는 말참견을 하실지도 모릅니다. "좋아요, 소크라테스, 당신의 소송에서 어려운 점이 뭡니까? 당신에 대한 이런 모략들의 원천은 뭡니까? 만일 당신이 일상적인 것들을 벗어나는 어떤 것에 참여하지 않는다면, 어째서 당신에 대해 그렇게나 많은 소문들과 말들이 있던 겁니까? 당신이 대부분의 사람들과 다른 뭔가로 바쁘지 않았다면야, 그런 일은 당연히 절대로 일어나지 않았을 겁니다. 그러니까 우리가 당신에 대해 성급하게 결론내리지 않도록 그것이 무엇인지 우리에게 말씀해 주십시오."

16) That speaker makes a fair point, I think; and so I will try to show you just what it is that has earned me my reputation and notoriety. Please hear me out. Some of you will perhaps think I am joking, but I assure you that I shall be telling you the whole truth.
16) 그렇게 말씀하시는 분께서는 제 생각에 맞는 말을 하고 계십니다. 또 그래서 저는 여러분께 바로 그것이 제게 저의 평판과 악명을 가져다 준 것임을 보여드리고자 노력할 것입니다. 제 말을 끝까지 들어주시기 바랍니다. 여러분들 중 몇몇분께서는 아마도 제가 농담을 하고 있다고 생각하실 겁니다만, 저는 여러분들께 제가 모든 진실을 말씀드리고 있을 것임을 보장합니다.

17) You see, fellow Athenians, I have gained this reputation on account of nothing but a certain sort of wisdom. And what sort of wisdom is that? It is a human kind of wisdom, perhaps, since it might just be true that I have wisdom of that sort. Maybe the people I just mentioned possess wisdom of a superhuman kind; otherwise I cannot explain it. For my part, I certainly do not possess that knowledge; and whoever says I do is lying and speaking with a view to slandering me -
17) 아시다시피, 아테네인여러분, 저는 이러한 평판을 다른 무엇도 아닌 어떤 종류의 지혜 때문에 얻게 되었습니다. 그러면 그게 어떤 종류의 지혜이겠습니까? 그것은 아마도 인간적인 종류의 지혜일 터인데, 왜냐하면 제가 그러한 종류의 지혜를 가지고 있다는 것은 정말로 사실일 테니 말씀입니다. 어쩌면 제가 방금 말씀드린 사람들은 인간을 넘어서는 종류의 지혜를 지니고 있을지도 모릅니다. 그 외에는 제가 그걸 설명할 수가 없습니다. 저로서는 확실히 그런 지식은 가지고 있지 않습니다. 그리고 제가 그런 지식을 가지고 있다고 말하는 그 누구라도 거짓말을 하고 있고 저를 중상하려는 시선으로 말하고 있습니다 -

18) Now please do not protest, fellow Athenians, even if I should sound to you rather boastful. I am not myself the source of the story I am about to tell you, but I shall refer you to a trustworthy authority. As evidence of my wisdom, if such it actually be, and of its nature, I shall call to witness before you the god at Delphi.
18) 아테네인 여러분, 이제 제발 반발하지 말아 주시기 바랍니다. 설령 제가 여러분께 되레 뽐내는 것처럼 들린다 할지라도 말씀입니다. 제가 여러분께 말씀드리려는 이야기는 제 자신이 출처가 아닙니다. 하지만 저는 여러분께 믿을 만한 증언을 제시해 드릴 것입니다. 만일 그러한 것이 정말로 있다면 제 지혜에 대한 증거로서, 그리고 그 지혜가 어떤 성격의 것인지에 대한 증거로서, 저는 여러분들 앞에 델피의 신을 증인으로 청할 것입니다.

19) You remember Chaerephon, of course. He was a friend of mine from youth, and also a comrade in your party, who shared your recent exile and restoration. You recall too what sort of man Chaerephon was, how impetuous he was in any undertaking. Well, on one occasion he actually to put the following question to it - as I said, please do not make a disturbance, gentlemen - he went and asked if there was anyone wiser than myself; to which the Pythia responded that there was no one. His brother here will testify to the court about that story, since Chaerephon himself is deceased.
19) 여러분들께서는 물론 카이레폰을 기억하실 겁니다. 그는 어려서부터 제 친구들 중 한 사람이었고, 또한 여러분들의 부대에서 여러분들의 추방과 귀국을 함께 하였던 한 사람의 동료이기도 하였습니다. 여러분들께서도 카이레폰이 어떤 종류의 사람이었는지 기억하실 겁니다. 어떤 일을 맡든지 그는 어찌나 성급하였던지요. 자, 어떤 기회에 그는 실제로 그 지혜에 대해 다음과 같음 물음을 던졌습니다 - 제가 말씀드렸던 것처럼, 시민여러분, 소란을 일으키지 말아주시길 제발 부탁드립니다 - 그는 가서 물었습니다, 혹시 저보다 지혜로운 누군가가 있는지를 말씀입니다. 그 물음에 대해 퓌티아는 아무도 없다고 대답했습니다. 그의 형제가 여기에서 그 이야기에 대해 법정에서 증언할 것입니다. 카이레폰 자신은 고인이 되었기 때문입니다.

20) Now keep in mind why I have been telling you this: it is because I am going to explain to you the origin of the slander against me. When I heard the story, I thought to myself: "What ever is the god saying? What can his riddle mean? Since I am all too conscious of not being wise in any matter, great or small, what ever can he mean by pronouncing me to be the wisest? Surely he cannot be lying: for him that would be out of the question."
20) 이제 어째서 제가 이 이야기를 여러분들께 말씀드려 왔는지 명심하십시오. 그것은 제가 여러분들께 저에 대한 그 중상의 근원을 계속해서 설명드리고 있기 때문입니다. 제가 그 이야기를 들었을 때, 저는 속으로 생각했습니다. "대체 그 신께서 말씀하신 것은 무슨 이야기지? 그 분의 수수께끼는 뭘 뜻하는 걸까? 나는 대단한 것이든 사소한 것이든 어떤 문제에 있어서도 현명하지 않다는 걸 너무나도 잘 알고 있는데, 그분께서 나를 가장 현명하다고 선언하심으로써 도대체 무슨 뜻을 전하실 수 있으실까? 확실히 그 분께서 거짓말을 하실 수는 없으니까. 그분에 대해 그건 물을 여지가 없지."

21) So for a long time I was perplexed about what he could possibly mean. But then, with great reluctance, I proceeded to investigate the matter somewhat as follows. I went to one of the people who had a reputation for wisdom, thinking there, if any where, to disprove the oracle's utterance and declare to it: "Here is someone wiser than I am, and yet you said that I was the wisest."
21) 그래서 오랜 동안 저는 그 분께서 뜻하셨을 수 있을 것에 대해 당혹스러워 하게 되었습니다. 그러나 그리고 나서, 정말로 마지못해서, 저는 다음과 같은 다소간의 문제를 살펴보기를 계속하였습니다. 저는 지혜로 이름 높은 사람들 중 하나에게 찾아갔습니다. 어디에서든, 그곳에서 그 신탁의 말씀이 틀렸음을 증명하고 다음과 같은 것, "여기 저보다 더욱 지혜로운 사람이 있는데, 당신께서는 제가 가장 지혜롭다고 말씀하셨습니다." 라는 것을 분명히 할 생각으로 말입니다.

22) So I interviewed this person - I need not mention his name, but he was someone in public life; and when I examined him, my experience went something like this, fellow Athenians; in conversing with him, I formed the opinion that, although the man was thought to be wise by many other people, and especially by himself, yet in reality he was not. So I then tried to show him that he thought himself wise without being so. I thereby earned his dislike, and that of many people present; but still, as I went away, I thought to myself: "I am wiser than that fellow, anyhow. Because neither of us, I dare say, knows anything of great value; but he thinks he knows a thing when he doesn't; whereas I neither know it in fact, nor think that I do. At any rate, it appears that I am wiser than he in just this one small respect: if I do not know something, I do not think that I do."
22) 그래서 저는 이러한 사람을 만나서 이야기해 보았습니다 - 제가 그의 이름을 말할 필요는 없겠습니다만, 그는 공인 중의 한 사람이었습니다; 그리고 제가 그를 검토했을 때, 저는 이와 같은 어떤 것을 경험하게 되었습니다, 아테네인 여러분. 그와 대화하면서, 저는 그 사람이 여러 다른 사람들에 의해 현명한 사람으로 생각되었고 또 특히 그 자신이 그렇게 생각했다 할지라도, 실상 그는 현명하지 않았다는 생각을 가지게 되었습니다. 그래서 저는 다음으로 그에게 그가 현명하지 않으면서도 그 자신을 현명하다고 생각한다는 점을 보여주려고 애썼습니다. 그렇게 함으로써 저는 그에게 미움을 샀고, 여기 많은 분들의 미움도 샀습니다. 하지만 여전히, 제가 떠나는 동안에도, 저는 속으로 생각했습니다. "내가 어떻든 저 사람보다는 현명하군. 왜냐하면 우리들 중 누구도 솔직히 말해서 대단히 가치있는 것에 대해서는 어떤 것도 알지 못하니까. 하지만 그는 그가 알지 못하면서도 어떤 것을 안다고 생각해. 반면에 나는 정말로 그것을 알지 못하고, 내가 안다고 생각하지도 않아. 어쨌든, 바로 이 작은 한 가지 면에서는 내가 그보다 현명하다는 게 드러나는군. 만일 내가 어떤 것을 모른다면, 나는 내가 안다고 생각하지 않는다는 것 말이야."

23) Next, I went to someone else, among people thought to be even wiser than the previous man, and I came to the same conclusion again; and so I was disliked by that man too, as well as by many others.
23) 다음으로, 저는 또 다른 누군가에게 갔습니다. 사람들 사이에서 먼젓번 사람보다 훨씬 더 현명하다고 여겨진 사람입니다. 그리고 저는 다시 같은 결론에 이르렀습니다. 그래서 저는 다른 많은 사람들뿐만 아니라 저 사내에게도 미움을 받게 되었습니다.

24) Well, after that I went on to visit one person after another. I realized, with dismay and alarm, that I was making enemies; but even so, I thought it my duty to attach the highest importance to the god's business; and therefore, in seeking the oracle's meaning, I had to go on to examine all those with any reputation for knowledge. And upon my word, fellow Athenians - because I am obliged to speak the truth before the court - I truly did experience something like this: as I pursued the god's inquiry, I found those held in the highest esteem were practically the most defective, whereas men who were supposed to be their inferiors were better off in respect of understanding.
24) 자, 그 이후에 저는 또 다른 다음 사람을 계속해서 찾아갔습니다. 저는 충격과 공포를 느끼면서 제가 적들을 만들고 있다는 사실을 깨달았습니다. 그러나 그렇긴 하지만, 저는 신의 일에 가장 높은 가치를 두는 것이 저의 의무라고 생각했습니다. 그래서, 신탁의 의미를 찾으면서, 저는 앎에 있어서 어떤 명성이라도 가진 모든 사람들을 계속해서 검토해야 했습니다. 그리고 맹세컨데, 아테네 시민여러분 - 왜냐하면 제가 법정앞에서 진실을 이야기할 책임이 있으니 말입니다 - 저는 진실로 이와 같은 어떤 일을 겪었습니다. 제가 신의 탐구를 추구하는 동안, 저는 가장 존경받는 그러한 사람들이 특히나 가장 부족한 사람들이었던 반면, 그들보다 못한 사람들로 여겨졌던 사내들이 이해심의 측면에서는 더욱 나았다는 것을 알게 되었습니다.

25) Let me, then, outline my wanderings for you, the various "labours" I kept undertaking, only to find that the oracle proved competely irrefutable. After I had done with the politicians, I turned to the poets - including tragedians, dithyrambic poets, and the rest - thinking that in their company I would be shown up the poems over which I thought they had taken the most trouble, and questioned them about their meaning, so that I might also learn something from them in the process.
25) 다음으로 저는 여러분들께 저의 방랑을 개략적으로 보여드리고자 합니다. 제가 떠맡았던 여러가지 "일들"에 대해서 말씀입니다. 그것은 오직 그 신탁이 완전히 반박할 수 없는 것으로 드러남을 발견하기 위한 것이었습니다. 제가 정치인들과 이야기를 나누고 나서, 저는 시인들 - 비극 시인들, 주신 찬가 작가들, 그리고 나머지 시인들을 포함한 사람들 - 에게로 넘어갔습니다. 그들의 무리 속에서 저는 제 생각에 그들이 가장 공을 들였다고 여겨지는 시들을 내놓았고, 그들에게 그 시들의 의미를 물었습니다. 그 과정에서 저 또한 그들로부터 어떤 것을 배울까 해서 말씀입니다.

26) Now I'm embarrassed to tell you the truth, gentlemen, but it has to be said. Practically everyone else present could speak better than the poets themselves about their very own compositions. And so, once more, I soon realized this truth about them too: it was not from wisdom that they composed their works, but from a certain natural aptitude and inspiration, like that of seers and soothsayers - because those people too utter many fine words, yet know nothing of the matters on which they pronounce. It was obvious to me that the poets were in much the same situation; yet at the same time I realized that because of their compositions they thought themselves the wisest people in other matters as well, when they were not. So I left, believing that I was ahead of them in the same way as I was ahead of the politicians.
26) 지금 제가 여러분들께 진실을 말씀드리기가 당혹스럽습니다만, 시민여러분, 그 진실을 말해야 하겠습니다. 실제로 여기 계신 다른 모든 분들께서 그 시인들 자신들보다도 바로 그들 자신의 작품들에 대해 더욱 잘 말씀하실 수 있을 것입니다. 그래서, 다시 한 번, 저는 곧 그들에 대해서도 이러한 사실을 깨닫게 되었습니다. 그들이 그들의 작품들을 구성하였던 것은 지혜로부터가 아니었고, 어떤 타고난 소질과 영감, 예지자들과 예언자들의 것과 같은 그런 것들로부터라는 사실입니다. - 왜냐하면 그런 사람들도 마찬가지로 많은 훌륭한 말들을 하지만, 그들이 그에 대해 말하는 그러한 문제들에 대해서는 아무것도 모르기 때문입니다. 저에게는 시인들도 완전히 똑같은 상황에 있었다는 것이 명백했습니다. 하지만 동시에 저는 그들이 그들의 작품들 때문에 그들 자신을 다른 문제들에서도 또한 가장 지혜로운 사람들로 생각한다는 것을 깨닫게 되었습니다. 그들이 지혜롭지 않을 때에도 말입니다. 그래서 저는 제가 정치인들보다 나은 것과 똑같은 방식으로 그들보다도 낫다고 믿으면서 떠났습니다.

27) Then, finally, I went to the craftsmen, because I was conscious of knowing almost nothing myself, but felt sure that amongst them, at least, I would find much valuable knowledge. And in that expectation I was not disappointed: they did have knowledge in fields where I had none, and in that respect they were wiser than I. And yet, fellow Athenians, those able craftsmen seemed to me to suffer from the same failing as the poets: because of their excellence at their own trade, each claimed to be a great expert also on matters of the utmost importance; and this arrogance of theirs seemed to eclipse their wisdom. So I began to ask myself, on the oracle's behalf, whether I should prefer to be as I am, neither wise as they are wise, nor ignorant as they are ignorant, or to possess both their attributes; and in reply, I told myself and the oracle that I was better off as I was.
27) 그리고 끝으로, 저는 장인들에게 갔습니다. 왜냐하면 저는 제 자신이 거의 아무것도 모른다는 점을 알고 있었습니다만, 분명 그들 가운데에서는 최소한 제가 훨씬 가치있는 앎을 발견하리라 확신하였기 때문입니다. 그리고 그러한 기대에서 제가 실망하지는 않았습니다. 그들은 제가 알지 못했던 분야들에 지식을 갖고 있었습니다. 그리고 그 점에서 그들은 저보다 더욱 지혜로웠습니다. 하지만, 아테네인 여러분, 그러한 능력있는 장인들이 제게는 시인들과 똑같은 잘못을 범하고 있는 것으로 보였습니다. 그들의 고유한 사업에 있어서 그들의 탁월함 때문에, 가장 중요한 문제들에 대해서도 각자가 훌륭한 전문가임을 주장하였습니다.그리고 이러한 그들의 오만은 그들의 지혜를 바래게 하는 듯이 보였습니다. 그래서 저는 자문하기 시작하였습니다. 신탁을 위하여, 제가 그들이 현명한 그러한 모습으로 현명하지도, 그들이 무지한 것과 같이 그렇게 무지하지도 않고 지금 제 모습처럼 있는 것을 선호해야 할지, 아니면 그들의 속성들 모두를 지니려고 해야 할지; 그리고 저는 대답하였습니다. 저는 제 자신과 그 신탁에 대고 제가 저였던 그대로 있는 것이 더 낫다고 말했습니다.

28) The effect of this questioning, fellow Athenians, was to earn me much hostility of a very vexing and trying sort, which has given rise to numerous slanders, including this reputation I have for being "wise" - because those present on each occasion imagine me to be wise regarding the matters on which I examine others. But in fact, gentlemen, it would appear that it is only the god who is truly wise; and that he is saying to us, through this oracle, that human wisdom is worth little or nothing. It seems that when he says "Socrates," he makes use of my name, merely taking me as an example - as if to say, "The wisest amongst you, human beings, is anyone like Socrates who has recognized that with respect to wisdom he is truly worthless."
28) 이러한 물음의 결과로, 아테네인 여러분, 저는 아주 성가시고 괴로운 종류의 많은 적대심을 얻었습니다. 그것은 엄청난 비방을 불러 일으켰습니다. 제가 "지혜로운" 자라는 이 평판을 포함해서 말씀입니다. - 왜냐하면 제가 다른 이들을 검토하는 문제들을 고려하면서 그 때마다 참석자들이 저를 지혜로운 자라고 상상했기 때문입니다. 하지만 사실, 여러분, 참으로 지혜로운 것은 오직 신들뿐이라는 것이 드러날 것입니다. 그리고 이 신탁을 통해 그 신께서 우리에게 말씀하시고 계신 것은 인간적 지혜는 거의 가치가 없거나 아무런 가치도 없다는 점이라는 것도 드러날 것입니다. 그분께서 "소크라테스"라고 말씀하실 적에, 그분께서는 제 이름을 가지고 단지 예시로 삼으시는 것으로 보입니다. 마치 이렇게 말씀하시는 것처럼 말입니다. "너희 인간들 중 가장 지혜로운 자는, 누구든 소크라테스처럼 지혜에 있어서 그가 참으로 아무것도 아님을 깨달은 자이다."

29) That is why, even to this day, I still go about seeking out and searching into anyone I believe to be wise, citizen or foreigner, in obedience to the god. Then, as soon as I find that someone is not wise, I assist the god by proving that he is not. Because of this occupation, I have had no time at all for any activity to speak of, either in public affairs or in my family life; indeed, because of my service to the god, I live in extreme poverty.
29) 그것이 오늘까지도 제가 여전히 계속해서 제가, 시민이든 외래인이들, 지혜롭다 생각하는 누구든 그 신께 복종하는 뜻에서 찾아내고 또 살펴보는 이유입니다. 그리고, 금새 저는 누군가가 현명하지 않다는 것을 알아 차리고, 그가 현명하지 않다는 것을 밝혀냄으로써 그 신을 돕습니다. 이와 같은 일로 인해, 저는 사회적 문제들에 대해서도 제 가정사에 대해서도 이렇다 할 활동을 할 시간이 전혀 없었습니다. 더욱이, 신에 대한 저의 봉사 때문에, 저는 극도로 빈한하게 삽니다.

30) In addition, the young people who follow me around of their own accord, the ones who have plenty of leisure because their parents are wealthiest, enjoy listening to people being cross-examined. Often, too, they copy my example themselves, and so attempt to cross-examine others. And I imagine that they find a great abundance of people who suppose themselves to possess some knowledge, but really know little or nothing. Consequently, the people they question are angry with me, though not with themselves, and say that there is a nasty pestilence abroad called "Socrates," who is corrupting the young.
30) 덧붙여서, 자발적으로 저를 따라 다니는 젊은 사람들, 그들의 부모님들께서 매우 부유하신 덕분에 넘치는 여가를 지닌 친구들은 반문하는 사람들의 이야기를 듣기를 즐깁니다. 종종, 마찬가지로 그들도 스스로 저의 일을 따라하고, 그렇게 다른 이들을 반문하고자 시도합니다. 그리고 저는 그 친구들이, 자신들은 어떤 지식을 지니고 있다고 생각하지만 실제로는 거의 아는 게 없거나 아무것도 모르는 넘칠 정도로 많은 사람들을 발견한다 생각합니다. 결론적으로, 그 친구들이 심문하는 그 사람들은 그 친구들 자신들이 아님에도 불구하고 저에게 화를 냅니다. 그리고 "소크라테스"라는 끔찍한 역병이, 젊은이들을 타락시키고 있는 그런 질병이 있다고 말합니다.

31) Then, when asked just what he is doing or teaching, they have nothing to say, because they have no idea what he does; yet, rather than seem at a loss, they resort to the stock charges against all who pursue intellectual inquiry, trotting out "things in the sky and beneath the earth," "failing to acknowledge the gods," and "turning the weaker argument into the stronger." They would, I imagine, be loath to admit the truth, which is that their pretensions to knowledge have been exposed, and they are totally ignorant. So because these people have reputations to protect, I suppose, and are also both passionate and numerous, and have been speaking about me in a vigorous and persuasive style, they have long been filling your ears with vicious slander. It is on the strength of all this that Meletus, along with Anytos and Lycon, has proceeded against me: Meletus is aggrieved for the poets, Anytus for the craftsmen and politicians, and Lycon for the orators. And so, as I began by saying, I should be surprised if I could rid your minds of this slander in so short a time, when so much of it has accumulated.
31) 그래서, 그 사람이 하고 있는 일이나 가르치고 있는 바로 그것이 무엇인지 질문을 받을 적에는, 그들이 할 말은 아무것도 없는데, 왜냐하면 그들은 그가 하는 일에 대해 아무것도 모르기 때문입니다. 그러나 오히려 당황해 하는 듯하며, 그들은 "하늘에 있는 것들과 땅 밑에 있는 것들," "신들을 인정하지 않음," 그리고 "더 약한 논변을 더욱 강한 것으로 뒤바꿈"을 둘러대면서, 지성적 탐구를 추구하는 모든 이들에 대한 상투적인 죄목들에 의존합니다. 제 생각에 그들은 진실을 받아들이길 꺼려할 것 같습니다. 그 진실이란 그들의 지식에 대한 허세들이 폭로되었다는 것이고, 그들이 전부 다 무지하다는 것입니다. 그래서 제 생각에는 이러한 사람들이 평판들을 지켜내려고, 그리고 또한 그들이 열정적이기도 하고 또 엄청 많기도 하기 때문에, 격렬하고 설득력 있어 보이는 방식으로 저에 대해 말해 오고 있었고, 그들은 여러분들의 귓가에 사나운 모략들을 채워넣어 왔던 것입니다. 이러한 모든 것들에 힘입어 멜레토스가, 아뉘토스와 뤼콘과 마찬가지로, 계속해서 저에게 반대해온 것입니다. 멜레토스는 시인들을, 아뉘토스는 장인들과 청지가들을, 그리고 뤼콘은 예언가들을 위해서 분개하고 있습니다. 그래서 제가 처음 말했던 것처럼, 만일 제가 여러분들의 마음에서 이러한 모략을 너무도 짧은 시간 안에, 너무나 많이 쌓여 버린 시간에, 떨쳐낼 수 있다면, 저는 놀랄 것입니다.

32) There is the truth for you, fellow Athenians. I have spoken it without concealing anything from you, major or minor, and without glossing over anything. And yet I am virtually certain that it is my very candour that makes enemies for me - which goes to show that I am right: the slander against me is to that effect, and such is its explanation. And whether you look for one now or later, that is what you will find.
32) 여러분들을 위한 진실입니다, 아테네인 여러분. 저는 그 진실을 여러분들께 주된 것이든 사소한 것이든 어느 것 하나 숨김 없이, 그리고 어떤 것에 대해서든 꾸밈 없이 말씀드렸습니다. 그렇지만 저는 거의 확신합니다. 저에 대한 적들을 만든 것은 제 자신의 지나친 솔직함이라는 것을 말씀입니다. - 그 솔직함은 제가 옳다는 것을 보여주고 있습니다. 저에 대한 중상은 그런 취지의 것이고, 그러한 것이 그 중상에 대한 설명입니다. 그리고 여러분들께서 그것을 지금 기대하시든 나중에 바라시든, 여러분들께서 발견하실 것이 그것입니다.


-蟲-


P.S 영역은 어감이 진짜 다 죽어 버리는구나. 이런 거 보면 박종현 선생님 번역이 얼마나 멋진지도 알겠고 기종석 선생님 말씀대로 영어보다도 한국말이 고대 그리스어에 더 가까운 면이 있다는 말도 좀 와닿는다. 일단 영어공부가 먼저이니 어쩔 수 없지만. 대학원 가면 『변론』 강독할껴+_+!!

"πείσομαι δὲ μᾶλλον τῷ θεῷ ἢ ὑμῖν, καὶ ἕωσπερ ἂν ἐμπνέω καὶ οἷός τε ὦ, οὐ μὴ παύσωμαι φιλοσοφῶν καὶ ὑμῖν παρακελευόμενός τε καὶ ἐνδεικνύμενος ὅτῳ ἂν ἀεὶ ἐντυγχάνω ὑμῶν"

The Absurd
부조리

Thomas Nagel

1) Most people feel on occasion that life is absurd, and some feel it vividly and continually. Yet the reasons usually offered in defense of this conviction are patently inadequate: they not really explain why life is absurd. Why then do they provide a natural expression for the sense that it is?
1) 대부분의 사람들은 이따금 삶이 부조리하다고 느끼고, 몇몇은 그것을 선명하고도 지속적으로 느낀다. 하지만 일반적으로 이러한 확신에 대한 반박으로 제공되는 근거들은 명백히 불충분하다. 왜냐하면 그것들은 어째서 삶이 부조리한지를 실제로 설명하지는 않기 때문이다. 그래서 왜 그들은 삶이 그러하다는 감각에 대해 자연스러운 표현을 내놓는가?


1) Consider some examples. It is often remarked that nothing we do now will matter in a million years. But if that is true, then by the same token, nothing that will be the case in a million years matters now. In particular, it does not matter now that in a million years nothing we do now will matter. Moreover, even if what we did now were going to matter in a million years, how could that keep our present concerns from being absurd? If their mattering now is not enough to accomplish that, how would it help if they mattered a million years from now?
1) 몇 가지 예들을 생각해 보자. 종종 우리가 지금 문제시하는 그 어떤 일도 백만년 동안 문제시되지는 않을 것이라고 이야기되곤 한다. 그러나 만일 그것이 참이라면, 동전의 양면으로서, 지금 백만년 동안 문제시되는 경우일 것은 아무것도 없을 것이다. 특히, 우리가 지금 문제로 삼는 어떤 것도 백만년 동안 문제가 되지는 않으리라는 것은 지금 문제가 되지 않는다. 더 나아가서 설령 우리가 지금 문제시 삼는 것이 백만년 동안 문제시될 것이라 하더라도, 어떻게 그것이 부조리한 존재에 대한 우리의 작금의 우려를 지속시킬 수 있는가? 만일 그들이 지금 문제삼고 있음이 그것을 성취하기에 충분치 못하다면, 그것들이 지금으로부터 백 만년 동안 문제시될지라도 어떻게 그것이 도울 것인가?

2) Whether what we do now will matter in a million years could make the crucial difference only if its mattering in a million years depended on its mattering, period. But then to deny that whatever happens now will matter in a million years is to beg the question against its mattering, period; for in that sense one cannot know that it will not matter in a million years whether (for example) someone now is happy or miserable, without knowing that it does not matter, period.
2) 우리가 지금 문제로 삼는 일이 백만년 동안 문제시될지 아닐지는 오로지 그 문제삼음이 백만년 동안 그 문제삼음의 기간에 의존하는 경우에만 결정적인 차이를 만들 수 있을 것이다. 그러나 그래서 지금 일어나는 그 무엇이든지 백만년 동안 문제시되리라는 것을 부정하는 일은 그 문제삼음, 기간에 반대해 물음을 구하는 것이다; 그런 의미에서, 기간은 문제시되지 않는다는 것을 알지 못하고서는 누구도, (예를 들어) 누군가 지금 행복하지 아니면 비참한지 그것이 백만년 동안 문제시되지 않으리라고 알 수 없기 때문이다.

3) What we say to convey the absurdity of our lives often has to do with space or time: we are tiny specks in the infinite vastness of the universe; our lives are mere instants even on a geological time scale, let alone a cosmic one; we will all be dead any minute. But of course none of these evident facts can be what makes life absurd, if it is absurd. For suppose we lived forever; would not a life that is absurd if it lasts seventy years be infinitely absurd if it lasted through eternity? And if our lives are absurd given our present size, why would they be any less absurd if we filled the universe (either because we were larger or because the universe was smaller)? Reflection on our minuteness and brevity appears to be intimately connected with the sense that life is meaningless; but it is not clear what the connection is.
3) 우리가 우리의 삶들에 대해 부조리함으로 전하고자 말하는 것은 종종 공간이나 시간을 가지고 해야한다. 우리는 우주의 무한한 광막함 속에서 미세한 얼룩들이다; 우리의 삶들은 우주적인 것이기는 고사하고 지질학적 시간 규모에조차 단지 순간적일 따름이다; 우리는 모두 어느 순간엔가 죽게 될 것이다. 그러나 물론 이러한 명백한 사실들 중 어느 것 하나 삶을 부조리하게 만드는 것일 수 없다. 그것이 만일 부조리하다면 말이다. 우리가 영원토록 살았다고 가정해 보자; 부조리한 삶은 만일 그것이 7년 동안 계속된다면 설령 그 삶이 영원을 통해 계속될지라도 한없이 부조리하지 않겠는가? 그리고 만일 우리에게 현재 주어진 크기만큼의 우리의 삶들이 부조리하다면, 어째서 그 삶들은 우리가 우주를 채워넣는다 할지라도 조금도 덜 부조리하지 않은가 (우리가 더 크든 혹은 우주가 더 작든 간에)? 우리의 순간적인 덧없음에 대한 반성은 삶이 무의미하다는 감각과 내밀하게 결속되어 있는 것으로 드러난다. 그러나 그 연결이 무엇인지는 분명하지 않다.

4) Another inadequate argument is that because we are going to die, all chains of justification must leave off in mid-air: one studies and works to earn money to pay for clothing, housing, entertainment, food, to sustain oneself from year to year, perhaps to support a family and pursue a career - but to what final end? All of it is an elaborate journey leading nowhere. (One will also have some effect on other people's lives, but that simply reproduces the problem, for they will die too.)
4) 또 다른 불충분한 논증은 우리가 죽을 것이기 때문에, 타당화의 모든 연쇄는 공중에서 중단해야만 한다는 것이다. 누군가는 옷을 사고 집을 사고 여가를 즐기고 음식을 사기 위한 돈을 벌기 위해, 해마다 스스로 먹고 살기 위해, 어쩌면 가족을 부양하고 경력을 쌓기 위해 공부하고 일한다 - 하지만 어떤 궁극적인 목적을 위한 것인가? 고작해야 그것은 아무데로도 이끌지 않는 정교한 여행이다. (또한 누군가는 다른 사람들의 삶들에 영향을 끼칠 것이지만, 그것은 단순히 문제를 재생산하는 것일 따름인데, 왜냐하면 그들도 마찬가지로 죽을 것이기 때문이다.)

5) There are several replies to this argument. First, life does not consist of a sequence of activities each of which has as its purpose some later member of the sequence. Chains of justification come repeatedly to an end within life, and whether the process as a whole can be justified has no bearing on the finality of these end-points. No further justification is needed to make it reasonable to take aspirin for a headache, attend an exhibition of the work of a painter one admires, or stop a child from putting his hand on a hot stove. No larger context or further purpose is needed to prevent these acts from being pointless.
5) 이 논증에 대한 다양한 응답들이 있다. 첫째로, 삶은 행위들의 연속된 사건, 그 연속된 사건의 어떤 다음 구성요소를 목표로 지니는 각각의 행위들의 연속된 사건으로 구성되지 않는다는 것이다. 정당화의 연쇄들은 반복적으로 삶 속에서 결말에 이르고, 그 전체로서의 경과가 정당화될 수 있는지는 이러한 종점들의 최종성과 아무런 관련이 없다. 두통 때문에 아스피린을 먹는 것, 누군가가 존경하는 화가의 작품 전시회에 참여하는 것, 또는 아이가 뜨거운 난로 위에 손을 대는 일을 멈추도록 하는 것을 합리화하기 위해 아무런 더 나아간 정당화도 필요치 않다. 이러한 행동들이 맹목적이 되는 것을 막기 위해 아무런 더 큰 문맥이나 더 나아간 의도도 필요치 않다.

6) Even if someone wished to supply a further justification for pursuing all the things in life that are commonly regarded as self-justifying, that justification would have to end somewhere too. If nothing can justify unless it is justified in terms of something outside itself, which is also justified, then an infinite regress results, and no chain of justification can be complete. Moreover, if a finite chain of reasons can not justify anything, what could be accomplished by an infinite chain, each link of which must be justified by something outside itself?
6) 설령 누군가가 삶 속에서 보통 자기 정당화로 간주되는 삶의 모든 것들을 추구함에 대한 더 나아간 정당화를 내놓기를 바란다 할지라도, 그 정당화도 마찬가지로 어딘가에서 끝나야 할 것이다. 만일 그 어떤 것도 그것 자체로부터 외부의, 또한 정당화된 그러한 어떤 것으로 정당화되지 않는 한 정당화할 수 없다면, 무한후퇴가 귀결될 것이고, 그 어떠한 정당화의 연쇄도 완결될 수 없을 것이다. 더욱이, 만일 하나의 유한한 이유들의 연쇄가 어떤 것도 정당화할 수 없다면, 무한한 연쇄, 그 각각의 연결이 그 자체 바깥의 어떤 것에 의해 정당화되어야만 하는 그 연쇄로써 무엇이 성취될 수 있을 것인가?

7) Since justifications must come to an end somewhere, nothing is gained by denying that they end where they appear to, within life - or by trying to subsume the multiple, often trivial ordinary justifications of action under a single, controlling life scheme. We can be satisfied more easily than that. In fact, through its misrepresentation of the process of justification, the argument makes a vacuous demand. It insists that the reasons available within life are incomplete, but suggests thereby that all reasons that come to an end are incomplete. This makes it impossible to supply any reasons at all.
7) 정당화들은 반드시 어디에선가 끝나야만 하기 때문에, 그 정당화들이 삶 속에서 나타난 곳에서 끝난다는 것에 대한 부정을 통해서 - 혹은, 다양하고 종종 사소하고 일상적인 행위의 정당화들을 단일하고 제어하는 삶의 도식 아래로 포괄하려는 시도를 통해서는 아무것도 획득되지 않는다. 우리는 그보다 더 쉽사리 만족할 수 있다. 사실상, 정당화의 과정에 대한 와전을 통하여, 그 논증은 얼빠진 요구를 한다. 그 논증은 삶 속에서 가용한 근거들이 불완전하다고 주장하지만, 그로써 끝으로 치닫는 이유들 모두가 불완전하다고 추측한다. 이것은 그 어떤 이유들이라도 제공하는 것이 전혀 불가능하도록 만든다.

8) The standard arguments for absurdity appear therfore to fail as arguments. Yet I believe they attempt to express something that is difficult to state, but fundamentally correct.
8) 부조리성에 대한 표준적인 논증들은 그러므로 논증들로서는 실패한 것으로 드러난다. 하지만 나는 그 논증들이 진술하기 어려운, 하지만 근본적으로 정확한 어떤 것을 표현하고자 시도한다고 믿는다.



1) In ordinary life a situation is absurd when it includes a conspicuous discrepancy between pretension or aspiration and reality: someone gives a complicated speech in support of a motion that has already been passed; a notorious criminal is made president of a major philanthropic foundation; you declare your love over the telephone to a recorded announcement; as you are being knighted, your pants fall down.
1) 일상생활에서 하나의 상황은 그 상황이 요구나 염원과 실재 사이에서 뚜렷한 불일치를 포함할 때 부조리하다. 누군가 이미 지나가 버린 행위를 지지하느라 장광설을 늘어놓는다; 악명 높은 범죄자가 주요한 박애주의적 재단의 의장이 된다; 당신은 당신의 사랑을 수화기 너머 녹음된 소식으로 선언한다; 당신이 기사 작위를 받고 있을 때, 당신의 바지가 흘러 내린다.

2) When a person finds himself in an absurd situation, he will usually attempt to change it, by modifying his aspirations, or by trying to bring reality into better accord with them, or by removing himself from the situation entirely. We are not always willing or able to extricate ourselves from a position whose absurdity has become clear to us. Nevertheless, it is usually possible to imagine some change that would remove the absurdity - whether or not we can or will implement it. The sense that life as a whole is absurd arises when we perceive, perhaps dimly, an inflated pretension or aspiration which is inseparable from the continuation of human life and which makes its absurdity inescapable, short of escape from life itself.
2) 한 개인이 부조리한 상황에 처한 자신을 알아차릴 때, 그는 보통, 그의 바람들을 수정하거나, 또는 현실을 그 바람들에 더 잘 어우러지도록 이끌거나, 혹은 스스로 완전히 그 상황으로부터 벗어남으로써 그 상황을 바꾸고자 시도할 것이다. 우리가 항상 우리에게 그 부조리가 선명하게 된 입장에서 해방되려고 하거나 해방될 수 있거나 한 것은 아니다. 그럼에도 불구하고, 그 부조리를 제거할 어떤 변화를 상상하는 것은 일반적으로 가능하다 - 우리가 그 변화를 시행할 수 있거나 시행하려고 하든 아니든 간에 말이다. 전체로서의 삶이 부조리하다는 감각은 우리가 아마도 어렴풋하게, 인간 삶의 지속과 불가분하고 그 삶의 부조리를 삶 자체로부터 달아나지 않는 한 피할 수 없도록 만드는 과장된 요구나 바람을 인지할 때 일어난다.

3) Many people's lives are absurd, temporarily or permanently, for conventional reasons having to do with their particular ambitions, circumstances, and personal relations. If there is a philosophical sense of absurdity, however, it must arise from the perception of something universal - some respect in which pretension and reality inevitably clash for us all. This condition is supplied, I shall argue, by the collision between the seriousness with which we take our lives and the perpetual possibility of regarding everything about which we are serious as arbitrary, or open to doubt.
3) 많은 사람들의 삶들은 주기적으로든 영속적으로든 부조리한데, 특정한 야망들, 사정들, 인간관계들과 관련한 관습적인 이유들 때문이다. 그렇지만 만일 부조리에 대한 철학적 감각이 있다면, 그것은 틀림없이 보편적인 어떤 것 - 요구와 실재가 우리 모두에게 불가피하게 충돌하는 어떤 측면에서 - 으로부터 일어난다. 나는 다음과 같이 주장할 것이다. 이러한 상태는 우리가 우리들의 삶들을 대하는 진지함과 우리가 심각하게 대하는 모든 것들을 임의적인 것으로, 또는 의심의 여지가 있는 것으로 간주할 지속적인 가능성 사이의 충돌에 의해 충족된다.

4) We cannot live human lives without energy and attention, nor without making choices which show that we take some things more seriously than others. Yet we have always available a point of view outside the particular form of our lives, from which the seriousness appears gratuitous. These two inescapable viewpoints collide in us, and that is what makes life absurd. It is absurd because we ignore the doubts that we know cannot be settled, continuing to live with nearly undiminished seriousness in spite of them.
4) 우리는 활력과 흥미 없이도, 우리가 어떤 것들을 다른 것들보다 더욱 진지하게 고려한다는 것을 보여주는 그러한 선택들 없이도 인간적인 삶들을 살아낼 수 없다. 하지만 우리가 언제나 우리의 삶들의 특수한 형식, 그로부터 그 심각함이 까닭없는 것으로 드러나는 그러한 형식 외부의 가용한 관점을 지니는 것은 아니다. 이러한 두 가지 피할 수 없는 관점들은 우리 안에서 충돌하고, 그것이 바로 삶을 부조리하게 만드는 것이다. 그것은 부조리한데 왜냐하면 우리는 우리가 알고 있는 의심들이 정착될 수 없다는 것을, 그 의심들 대신 거의 줄어들지 않는 진지함을 가지고 삶을 계속 살아 나가면서, 무시하기 때문이다.

5) This analysis requires defense in two respects: first as regards the unavoidability of seriousness; second as regards the inescapability of doubt.
5) 이러한 분석은 첫 번째는 진지함의 불가피성을 고려하는 것으로서, 두 번째는 의심의 피할 수 없음을 고려하는 것으로서, 두 가지 측면에서 옹호를 요한다.

6) We take ourselves seriously whether we lead serious lives or not and whether we are concerned primarily with fame, pleasure, virtues, luxury, triumph, beauty, justice, knowledge, salvation, or mere survival. If we take other people seriously and devote ourselves to them, that only multiplies the problem. Human life is full of effort, plans, calculation, success and failure: we pursue our lives, with varying degrees of sloth and energy.
6) 우리는 우리가 진지한 삶들을 이끌어 나가는지 아닌지 그리고 우리가 우선적으로 명성, 기쁨, 덕성, 화려함, 영광, 아름다움, 정의로움, 앎, 희생, 또는 순전히 생존에 관심을 두는지 우리 자신을 진지하게 고려한다. 만일 우리가 다른 사람들을 진지하게 대하고 그들에게 우리 자신들을 헌신한다면, 그것은 오로지 문제를 증대시킬 뿐이다. 인간 삶은 노력, 계획들, 추정, 성광과 실패로 가득 차 있다. 우리는 다양한 정도의 나태와 활력으로 우리들의 삶들을 추구한다.

7) It would be different if we could not step back and reflect on the process, but were merely led from impulse to impulse without self-consciousness. But human beings do not act solely on impulse. They are prudent, they reflect, they weigh consequences, they ask whether what they are doing is worthwhile. Not only are their lives full of particular choices that hang together in larger activities with temporal structure: they also decide in the broadest terms what to pursue and what to avoid, what the priorities among their various aims should be, and what kind of people they want to be or become. Some men are faced with such choices by the large decisions they make from time to time; some merely by reflection on the course their lives are taking as the product of countless small decisions. They decide whom to marry, what profession to follow, whether to join the Country Club, or the Resistance; or they may just wonder why they go on being salesmen or academics or taxi drivers, and then stop thinking about it after a certain period of inconclusive reflection.
7) 만일 우리가 한 걸음 물러나 그 과정을 반성할 수 없다면, 허나 순전히 충동에서 충동으로 자의식 없이 이끌리는 것이었더라면 그것은 달랐을 터이다. 그러나 인간 존재들은 오로지 충동으로만 행위하진 않는다. 그들은 신중하고, 반성하며, 결과들의 경중을 따지고, 그들이 하고 있는 일이 가치있는 일인지 따져 묻는다. 그들의 삶들은 더 큰 활동들 속에서 세속적 구조와 함께 묶인 특수한 선택들로 가득 차 있을 뿐만이 아니다. 그 삶들은 또한 가장 넓은 표현으로 추구해야 할 것과 피해야 할 것, 그들의 다양한 목적들 중에서 우선시되어야 할 것들, 그들이 그러하길 바라거나 그렇게 되기를 바라는 종류의 사람들을 결정한다. 몇몇 사람들은 그들이 때때로 행하는 결정들에 의해 그러한 선택들에 직면하게 된다; 일부는 단순히 무수한 작은 결정들의 산물로서 그들의 삶들이 취하고 있는 경로에 대한 반성으로써 그렇게 된다. 그들은 누구와 결혼할지, 어떤 직업을 택할지, 컨트리 클럽, 또는 저항 운동에 참여할지 말지를 결정한다; 또는 그들은 어째서 그들이 세일즈맨이나 학자나 택시 운전수인 채로인지 단순히 궁금할지도 모르고, 그래서 결론이 나지 않는 약간의 반성의 시간 뒤에 그에 대해 생각하기를 멈춘다.

8) Although they may be motivated from act to act by those immediate needs with which life presents them, they allow the process to continue by adhering to the general system of habits and the form of life in which such motives have their place - or perhaps only by clinging to life itself. They spend enormous quantities of energy, risk, and calculation on the details. Think of how an ordinary individual sweats over his appearance, his health, his sex life, his emotional honesty, his social utility, his self-knowledge, the quality of his ties with family, colleagues, and friends, how well he does his job, whether he understands the world and what is going on in it. Leading a human life is a full-time occupation, to which everyone devotes decades of intense concern.
8) 비록 그들이 삶이 그들에게 제시하는 그들의 즉각적인 필요들에 의해 행위에서 행위로 동기부여를 받을지는 몰라도, 그들은 일반적인 습관체계와 그러한 동기들이 자리하는 삶의 형식에 들러붙음으로써 - 또는 아마도 오로지 삶 자체에 매닮으로써 - 그 과정이 지속되는 것을 허용한다. 그들은 막대한 양의 활력, 위험, 세부사항들에 대한 추산을 감수한다. 일상적인 개인이 그의 외모, 건강, 성생활, 감정적 진솔함, 사회적 효용, 자각, 가족, 동료들, 친구들과 엮인 그의 능력, 그의 일을 얼마나 잘 하는지, 그가 세계와 그 안에서 굴러가는 사정을 이해하는지 아닌지에 대해 얼마나 땀 흘리며 애쓰는지 생각해 보라. 인간적 삶을 이끄는 것은 직업이고, 직업에 대해 누구나 수십년의 강한 관심을 쏟아 붓는다.

9) This fact is so obvious that it is hard to find it extraordinary and important. Each of us lives his own life - lives with himself twenty-four hours a day. What else is he supposed to do - live someone else's life? Yet humans have the special capacity to step back and survey themselves, and the lives to which they are committed, with that detached amazement which comes from watching an ant struggle up a heap of sand. Without developing the illusion that they are able to escape from their highly specific and idiosyncratic position, they can view it sub specie aeternitatis - and the view is at once sobering and comical.
9) 그것이 비범하고 중요하다고 알아차리기가 어렵다는 바로 그 사실은 매우 분명하다. 우리들 각자는 그 고유한 삶을 살아간다 - 그의 하루 24시간을 보내면서. 그가 해야 한다고 생각하는 그 밖에 것은 무엇인가 - 다른 누군가의 삶을 사는 것인가? 하지만 인간들은 물러서서 그들 자신을, 그들이 전념하는 삶들을, 한 마리 개미가 모래 더미를 헤치고 나아가려 분투하는 것을 보면서 일어나는 그 사심없는 경탄과 함께 탐구하는 특별한 능력을 가지고 있다. 그들이 그들의 상당히 구체적이고 유별난 위치로부터 달아날 수 있다는 환상을 전개시키지 않고서, 그들은 영원한 상 아래에서 그것을 볼 수 있다 - 그리고 그 관점은 퍼뜩 정신이 들도록 하고 재미있다.

10) The crucial backward step is not taken by asking for still another justification in the chain, and failing to get it. The objections to that line of attack have already been stated; justifications come to an end. But this is precisely what provides universal doubt with its object. We step back to find that the whole system of justification and criticism, which controls our choices and supports our claims to rationality, rests on responses and habits that we never question, that we should not know how to defend without circularity, and to which we shall continue to adhere even after they are called into question.
10) 결정적인 물러섬은 연쇄에 있어서 여전히 또 다른 정당화를 요구함으로써, 그리고 그 정당화를 획득하는 데에 실패함으로써는 얻어지지 않는다. 그러한 공격노선에 대한 반박들은 이미 진술되어왔다; 정당화들은 끝나고야 만다. 그러나 이것이 바로 그 반대와 더불어 보편적 의심을 제공하는 것이다. 우리는 우리의 선택을 통제하고 우리의 합리성에 대한 주장들을 뒷받침하는 정당화와 비평의 전체 체계를 알아내기 위해 물러선다. 그 체계는 우리가 단 한 번도 의문을 품지 않은, 우리가 순환 없이 어떻게 옹호할지 알지 못해야 하는 응답들과 습관들, 그리고 그것들이 의문시된 이후에조차 그것들을 신봉하기를 지속해야 할 그러한 응답들과 습관들에 달려 있다.

11) The things we do or want without reasons, and without requiring reasons - the things that define what is a reason for us and what is not - are the starting points of our skepticism. We see ourselves from outside, and all the contingency and specificity of our aims and pursuits become clear. Yet when we take this view and recognize what we do as arbitrary, it does not disengage us from life, and there lies our absurdity: not in the fact that such an external view can be taken of us, but in the fact that we ourselves can take it, without ceasing to be the persons whose ultimate concerns are so coolly regarded.
11) 우리가 이유없이, 이유를 따지지 않고 행하거나 바라는 것들 - 우리에게 무엇이 이유이고 무엇이 이유가 아닌지 규정하는 것들 - 그것들은 우리의 회의에 있어서 출발점들이다. 우리는 우리 자신을 외부로부터 바라보고, 우리의 목적들의 모든 우발성과 특수성 그리고 추구들은 명백하게 된다. 하지만 우리가 이러한 관점을 취하고 우리가 하는 일이 임의적임을 인지할 때, 그것은 우리를 삶으로부터 풀어주지 않고, 거기, 그러한 하나의 항구적 관점이 우리에게 취해질 수 있다는 사실에가 아니라, 그의 궁극적 관심들이 매우 차분하게 고려되는 그러한 개인들로 있기를 중단함이 없이 우리가 우리 자신을 떠맡을 수 있다는 사실에 우리의 부조리가 자리한다.



1) one may try to escape the position by seeking broader ultimate concerns, from which it is impossible to step back - the idea being that absurdity results because what we take seriously is something small and insignificant and individual. Those seeking to supply their lives with meaning usually envision a role or function in something larger than themselves. They therefore seek fulfillment in service to society, the state, the revolution, the progress of history, the advance of science, or religion and the glory of God.
1) 누군가는 더욱 광범위한 궁극적 관심들, 그로부터 소급이 불가능한 관심들을 추구함으로써 그 위치에서 도피하고자 애쓸지도 모른다 - 우리가 진지하게 고려하는 것이 작고 사소하며 사적인 어떤 것이기 때문에 부조리가 생긴다는 발상 때문에. 그들의 삶들을 의미로 채우기를 추구하는 그러한 자들은 보통 그들 자신들보다 더욱 거대한 어떤 것에서 하나의 역할이나 기능을 마음 속에 그린다. 그러므로 그들은 사회, 국가, 혁명, 역사의 진보, 학문의 진전, 또는 종교와 신의 영광에서 성취를 추구한다.

2) But a role in some larger enterprise cannot confer significance unless that enterprise is itself significant. And its significance must come back to what we can understand, or it will not even appear to give us what we are seeking. If we learned that we were being raised to provide food for other creatures fond of human flesh, who planned to turn us into cutlets before we got too stringy - even if we learned that the human race had been developed by animal breeders precisely for this purpose - that would still not give our lives meaning, for two reasons. First, we would still be in the dark as to the significance of the lives of those other beings; second, although we might acknowledge that this culinary role would make our lives meaningful to them, it is not clear how it would make them meaningful to us.
2) 하지만 어떤 더 거대한 사업에서의 역할은 그 사업 자체가 의미있지 않는 한 의미부여를 할 수 없다. 그리고 그 사업의 의미는 우리가 이해할 수 있는 것으로 되돌아와야만 하거나, 우리에게 우리가 추구하고 있는 어떤 것을 주는 것으로 보이지조차 않을 것이다. 만일 우리가 인육을 좋아하는 다른 생명체들을 위해, 우리가 (힘줄로) 너무 질겨지기 전에 얇게 져민 고기로 만들 계획을 하고 있는 그들을 위해 음식으로 제공되도록 생겨나게 되었다는 것을 안다 할지라도 - 설령 우리가 인간종은 바로 이러한 의도로 동물 사육자들에 의해 진화되어 왔다는 것을 안다고 할지라도 - 그것은 여전히 우리 삶들에 의미를 부여하진 않을 것인데, 두 가지 이유들에서 그러하다. 첫째로 우리는 여전히 그러한 다른 존재자들의 삶들의 의미에 대해서는 암흑 속에 자리할 것이다. 둘째로 비록 우리가 이러한 식용으로의 역할이 우리의 삶들을 그들에게 의미있도록 만들 것임을 인정한다 칠지라도, 어떻게 그것이 그들을 우리에게 의미있도록 만들지는 분명하지 않다.

3) Admittedly, the usual form of service to a higher being is different from this. One is supposed to behold and partake of the glory of God, for example, in a way in which chickens do not share in the glory of coq au vin. The same is true of service to a state, a movement, or a revolution. People can come to feel, when they are part of something bigger, that it is part of them too. They worry less about what is peculiar to themselves, but identify enough with the larger enterprise to find their role in it fulfilling.
3) 확실히, 더 고차원의 존재에 대한 봉사의 통상적인 형태는 이와 다르긴 하다. 예를 들어 누군가는 신의 영광에 대해 닭들이 코코뱅(적포도주 닭국, 탕?)의 영광에 몫을 가지지는 않는(그와는 다른) 그런 방식으로 바라보고 참여하게 되어 있다. 국가, 운동, 또는 혁명에 봉사함에 있어서도 같다. 사람들은, 그들이 어떤 더욱 커다란 것의 일부일 때, 그 어떤 것도 마찬가지로 그들의 일부라는 것을 느끼게 될 수 있다. 그들은 그들 자신들에게 특유한 것에 대해서는 덜 염려하지만, 그 안에서 성취감을 주는 그들의 역할을 발견하도록 하는 더욱 거대한 사업은 충분히 알아본다.

4) However, any such larger purpose can be put in doubt in the same way that the aims of an individual life can be, and for the same reasons. It is as legitimate to find ultimate justification there as to find it earlier, among the details of individual life. But this does not alter the fact that justifications come to an end when we are content to have them end - when we do not find it necessary to look any further. If we can step back from the purposes of individual life and doubt their point, we can step back also from the progress of human history, or of science, or the success of a society, or the kingdom, power, and glory of God, and put all these things into question in the same way. What seems to us to confer meaning, justification, significance, does so in virtue of the fact that we need no more reasons after a certain point.
4) 그렇지만, 그런 어떤 더욱 거대한 의도든 개별적 삶의 목적들이 의심에 놓일 수 있는 동일한 방식으로, 그리고 똑같은 이유들로 인하여 의심을 받을 수 있다. 그곳에서 궁극적인 정당화를 발견하는 것이 정당한 만큼 더 앞서서, 개별적인 삶의 구체적인 것들 사이에서 그 의심을 찾는 것은 정당하다. 그러나 이것은 우리가 기꺼이 그것들을 끝으로 가질 때 - 우리가 그것을 더 이상 살펴볼 필요를 찾지 못할 때 - 정당화들이 끝난다는 사실을 대체하지 않는다. 만일 우리가 개별적 삶의 의도들로부터 물러나 그러한 것들의 관점을 의심할 수 있다면, 우리는 또한 인간사 또는 학문의 진보, 사회나 왕국의 성공, 신의 권능과 영광으로부터도 물러날 수 있고, 이러한 모든 것들을 같은 방식으로 의문시할 수 있다. 우리에게 의미, 정당성, 중요성을 부여하는 것으로 보이는 것은 우리가 특정한 지점 이후로는 그 이상의 아무런 이유들도 필요로 하지 않는다는 사실 덕분에 그렇게 의미부여하는 것이다.

5) What makes doubt inescapable with regard to the limited aims of individual life also makes it inescapable with regard to any larger purpose that encourage the sense that life is meaningful. Once the fundamental doubt has begun, it cannot be laid to rest.
5) 개인적 삶의 제한된 목적들과 관련하여 의심을 불가피하게 만드는 것은 또한 그 어떤 더욱 거대한 의도, 삶이 의미롭다고 느끼도록 추동하는 그러한 의도와 관련해서도 의심을 피할 수 없도록 만든다. 근본적인 회의가 시작되어 버리고 나면, 그것은 파묻힐 수가 없다.

6) Camus maintains in The Myth of Sisyphus that the absurd arises because the world fails to meet our demands for meaning. This suggests that the world might satisfy those demands if it were different. But now we can see that this is not the case. There does not appear to be any conceivable world (containing us) about which unsettlable doubts could not arise. Consequently the absurdity of our situation derives not from a collision between our expectations and the world, but from a collision within ourselves.
6) 『시쉬포스 신화』에서 카뮈는 세계가 우리의 의미에 대한 요구들과 만나는 데에 실패하기 때문에 부조리가 일어난다고 주장한다. 이것은 세계가 달랐더라면 그러한 요구들을 충족시켰을지도 모른다는 것을 시사한다. 하지만 이제 우리는 이것이 사실이 아님을 볼 수 있다. 그 세계에 대해 진정되지 않는 의심들이 일어날 수 없을 (우리를 포함하는)어떠한 상상 가능한 세계도 있을 성싶지 않다. 결론적으로 우리의 처지에 대한 부조리는 우리의 기대들과 세계 사이의 충돌로부터가 아니라, 우리들 자신들 안에서의 충돌로부터 유래한다.


1) It may be objected that the standpoint from which these doubts are supposed to be felt does not exist - that if we take the recommended backward step we will land on thin air, without any basis for judgment about the natural responses we are supposed to be surveying. If we retain our usual standards of what is important, then questions about the significance of what we are doing with our lives will be answerable in the usual way. But if we do not, then those questions can mean nothing to us, since there is no longer any content to the idea of what matters, and hence no content to the idea that nothing does.
1) 이러한 의심들이 현존하지 않는 것으로 느껴지게끔 되어 있는 그러한 관점은 반박될지도 모른다 - 만일 우리가 권장된 물러섬을 취한다면 우리는 허공중에 내려앉을 것이라는 관점은, 우리가 조사하고 있어야 할 자연스러운 반응들에 대한 판단을 위한 어떤 배경도 없이. 만일 우리가 무엇이 중요한지에 대한 우리의 일상적 표준들을 유지한다면, 우리의 삶들과 더불어 우리가 하고 있는 일의 중요성에 대한 물음들은 일상적인 방식으로 대답 가능한 것이 될 것이다. 하지만 만일 우리가 그렇게 하지 않는다면, 그러한 물음들은 우리에게 아무것도 의미할 수 없는데, 왜냐하면 더 이상 문제가 되는 것에 대한 생각에 어떠한 내용도 없고, 따라서 아무것도 문제삼지 않는 생각에는 아무런 내용도 없기 때문이다.

2) But this objection misconceives the nature of the backward step. It is not supposed to give us an understanding of what is really important, so that we see by contrast that our lives are insignificant. We never, in the course of these reflections, abandon the ordinary standards that guide our lives. We merely observe them in operation, and recognize that if they are called into question we can justify them only by reference to themselves, uselessly. We adhere to them because of the way we are put together; what seems to us important or serious or valuable would not seem so if we were differently constituted.
2) 그러나 이러한 반박은 물러섬의 본성에 대한 오해이다. 그것은 우리에게 무엇이 진정으로 중요한 것인지에 대한 이해를 제공하도록 되어 있지 않고, 그래서 우리가 그와 반대로 우리의 삶들이 사소하다는 것을 보도록 한다. 우리는 절대로 이러한 반성들의 경로 안에서 우리의 삶들을 인도하는 일상적 표준들을 폐기하지 않는다. 우리는 단지 사용 중에 그것들을 관찰하고, 만일 그것들이 의문시된다면 우리가 오직 그것들 자체를 지시하는 언급하는 것만으로 그것들을 정당화할 수 있다는 것을 헛되이 인정한다. 우리는 그 표준들을 고집하는데 왜냐하면 우리가 결합되는 방식 때문이다; 우리에게 중요하거나 심각하거나 가치있어 보이는 것은 우리가 다르게 된다면 그렇게 보이지 않을 것이다.

3) In ordinary life, to be sure, we do not judge a situation absurd unless we have in mind some standards of seriousness, significance, or harmony with which the absurd can be contrasted. This contrast is not implied by the philosophical judgment of absurdity, and that might be thought to make the concept unsuitable for the expression of such judgments. This is not so, however, for the philosophical judgment depends on another contrast which makes it a natural extension from more ordinary cases. It departs from them only in contrasting the pretensions of life with a larger context in which no standards can be discovered, rather than with a context from which alternative, overriding standards may be applied.
3) 일상적인 삶에서, 틀림없이, 우리는 우리가 심각성, 중요성, 부조리가 대조될 수 있는 그러한 조화에 대한 어떤 표준들을 마음에 품지 않는 한 한 상황을 부조리하고 판단하지 않는다. 이 대조는 부조리에 대한 철학적 판단에 의해 시사되지 않되, 아마도 그러한 판단들의 표현으로 적합한 개념을 만드는 것으로 생각될 것이다. 그렇지만 이것은 그렇지도 않은데, 철학적 판단이 그 판단을 더욱 일상적인 경우들로부터 자연스러운 확장을 하도록 만드는 또 다른 대조에 의존하기 때문이다. 철학적 판단은, 대안적이고 우선적인 규준들이 그로부터 수반될지 모르는 문맥을 가지고 보다는, 오로지 삶의 요구들을 아무런 규준들도 발견될 수 없는 더 큰 문맥을 가지고 대조함에 있어서만, 일상적 경우들로부터 벗어난다.


1) In this respect, as in others, philosophical perception of the absurd resembles epistemological skepticism. In both cases the final, philosophical doubt is not contrasted with any unchallenged certainties, though it is arrived at by extrapolation from examples of doubt within the system of evidence or justification, where a contrast with other certainties is implied. In both cases our limitedness joins with a capacity to transcend those limitations in thought(thus seeing them as limitations, and as inescapable).
1) 다른 측면들에서 처럼, 이러한 측면에서 부조리에 대한 철학적 지각은 인식론적 회의주의와 유사하다. 두 경우 모두 궁극적인 철학적 의심은 어떠한 불가침한 확실성들과도 대조되지 않는다, 비록 그것이 외삽법에 의해 증거들 또는 정당화의 체계 안에서의 의심의 예시들로부터, 다른 확실성들과의 대조가 시사되는 곳에 도달하게 된다 할지라도 말이다. 두 경우들 모두 우리의 제한성이 사유 속에서(따라서 그 제한들을 제한들로서, 그리고 불기피한 것으로서 보면서) 그러한 제한들을 초월할 수 있는 능력과 결부된다.

2) Skepticism begins when we claim knowledge. We notice that certain types of evidence convince us, that we are content to allow justifications of belief to come to an end at certain points, that we feel we know many things even without knowing or having grounds for believing the denial of others which, if true, would make what we claim to know false.
2) 회의주의는 우리가 앎을 주장할 때에 시작된다. 우리는 특정한 유형의 증거들이 우리를 납득시킨다는 것, 우리가 믿음에 대한 정당화들이 특정한 지점들에서 끝나도록 허용하게 되어 있다는 것, 우리는 만일 참이라면 우리가 안다고 주장하는 것을 틀린 것으로 만들 그러한 다른 것들에 대한 부정을 믿기 위한 근거들을 알거나 가지지 않고서조차도 우리가 많은 것들을 안다고 느낀다는 것을 알아차린다.

3) For example, I know that I am looking at a piece of paper, although I have no adequate grounds for claiming I know that I am not dreaming; and if I am dreaming then I am not looking at a piece of paper. Here an ordinary conception of how appearance may diverge from reality is employed to show that we take our world largely for granted; the certainty that we are not dreaming cannot be justified except circularly, in terms of those very appearances which are being put in doubt. It is somewhat farfetched to suggest I may be dreaming; but the possibility is only illustrative. It reveals that our claims to knowledge depend on our not feeling it necessary to exclude certain incompatible alternatives, and the dreaming possibility or the total-hallucination possibility are just representatives for limitless possibilities most of which we cannot even conceive.
3) 예를 들어 나는 내가 종이 한 장을 보고 있다는 것을 아는데, 비록 내가 꿈을 꾸고 있지 않다는 것을 안다고 주장하기 위한 타당한 근거들을 아무것도 가지고 있지 않다 하더라도 그러하다; 그리고 만일 내가 꿈을 꾸고 있다면 나는 한 장의 종이를 보고 있지 않다. 여기에서 겉보임이 어떻게 현실과 나뉠는지에 대한 일상적 이해는 우리가 우리의 세계를 굉장히 당연시한다는 것을 보여주기 위해 사용된다; 우리가 꿈을 꾸고 있지 않다는 확실성은 순환적으로, 의심받는 바로 그러한 겉보임으로 정당화되는 것 이외에는 정당화될 수 없다. 그것은 어느 정도 에둘러서 내가 꿈을 꾸고 있는지도 모른다는 것을 시사하는 것이다; 그러나 그 가능성은 오로지 예증적인 것일 뿐이다. 그것은 앎에 대한 우리의 주장들이 양립 불가능한 특정한 대안들을 배제할 필요가 있다고 느끼지 않는다는 것에 의존한다는 것을 밝히고, 꿈을 꾸고 있을 가능성이나 총체적-환각의 가능성이, 그 대부분을 우리가 지각조차 할 수 없는 그러한 무제한한 가능성들에 대해 대표적인 것들이다.

4) Once we have taken the backward step to an abstract view of our whole system of beliefs, evidence, and justification, and seen that it works only, despite its pretensions, by taking the world largely for granted, we are not in a position to contrast all these appearances with an alternative reality. We cannot shed our ordinary responses, and if we could it would leave us with no means of conceiving a reality of any kind.
4) 우리가 우리의 믿음들, 증거, 그리고 정당화의 전 체계에 대한 추상적 관점으로 물러섬을 취하고 나면, 그리고 그것이 그것의 요구들에도 불구하고 세계를 대체로 당연시함으로써만 유효하다는 것을 보고 나면, 우리는 이러한 모든 겉모습들을 대안적 실재성과 대조시키는 입장에 서지 않는다. 우리는 우리의 일상적인 반응들을 없앨 수 없고, 만일 우리가 없앨 수 있다면 그것은 어떤 종류의 실재성이든 지각할 방법도 없이 우리를 남겨둘 것이다.

5) It is the same in the practical domain. We do not step outside our lives to a new vantage point from which we see what is really, objectively significant. We continue to take life largely for granted while seeing that all our decisions and certainties are possible only because there is a great deal we do not bother to rule out.
5) 실천적 영역에서도 그것은 마찬가지이다. 우리는 우리의 삶들 바깥 우리가 무엇이 실재인지, 객관적으로 중요한지 그로부터 보는 그러한 새로운 유리한 위치로 나아가지 않는다. 우리는 우리의 모든 결정들과 확실성들이 오로지 우리가 제외시킬 것을 신경쓰지 않는 많은 것 때문에 가능한데 반해 삶을 대체로 당연시하기를 계속한다.

6) Both epistemological skepticism and a sense of the absurd can be reached via initial doubts posed within systems of evidence and justification that we accept, and can be stated without violence to our ordinary concepts. We can ask not only why we should believe there is a floor under us, but also why we should believe the evidence of our senses at all - and at some point the framable questions will have outlasted the answers. Similarly, we can ask not only why we should take aspirin, but why we should take trouble over our own comfort at all. The fact that we shall take the aspirin without waiting for an answer to this last question does not show that it is an unreal question. We shall also continue to believe there is a floor under us without waiting for an answer to the other question. In both cases it is this unsupported natural confidence that generates skeptical doubts; so it cannot be used to settle them.
6) 인식론적 회의주의와 부조리에 대한 감각 모두 우리가 허용하는 증거와 정당화의 체계들에서 제기된 초기의 의심들을 통해 도달될 수 있고, 우리의 일상적 이해들과 충돌함이 없이 진술될 수 있다. 우리는 어째서 우리가 우리 아래에 바닥이 있다는 것을 믿어야 하는지 물을 수 있을 뿐만 아니라, 어째서 우리가 좌우간 우리의 감각들의 증거를 믿어야 하는지도 물을 수 있다 - 그리고 어떤 점에서는 고안해 낼 수 있는 물음들이 답변들보다 오래 갈 것이다. 유사하게, 우리는 우리가 왜 아스피린을 먹어야 하는지만 물을 수 있는 게 아니라, 도대체 왜 우리가 우리 자신의 안정을 위해 애써야 하는지도 물을 수 있다. 우리가 이 후자의 질문에 대한 대답으로 지체없이 아스피린을 먹어야 한다는 사실은 그것이 비현실적인 물음이라는 것을 보여주지는 않는다. 우리는 또한 나머지 질문에 대한 대답으로 지체없이 우리 아래에 바닥이 있다고 계속해서 믿어야 한다. 두 경우들 모두에서 이 뒷받침되지 않은 자연스러운 믿음은 회의주의적 의심들을 산출하는 것이다; 그래서 그것은 그 물음들을 가라앉히기 우해 사용될 수 없다.

7) Philosophical skepticism does not cause us to abandon our ordinary beliefs, but it lends them a peculiar flavor. After acknowledging that their truth is incompatible with possibilities that we have no grounds for believing do not obtain - apart from grounds in those very beliefs which we have called into question - we return to our familiar convictions with a certain irony and resignation. Unable to abandon the natural responses on which they depend, we take them back, like a spouse who has run off with someone else and then decided to return; but we regard them differently (not that the new attitude is necessarily inferior to the old, in either case).
7) 철학적 회의주의는 우리에게 우리의 일상적 믿음들을 버리는 일을 불러 일으키지는 않지만, 그 믿음들을 특유한 풍미를 더한다. 그 믿음들의 참이 우리가 - 우리가 의문시한 바로 그러한 믿음들을 근거로 삼는 것 이외에은 - 믿기 위한 아무런 근거도 가지지 않는 가능성들과 양립불가능하다고 인정한 이후 우리는 어떤 반어와 체념을 가지고 우리의 친숙한 이해들로 되돌아온다. 그 믿음들이 의존하는 자연스러운 반응들을 폐기할 수 없고, 우리는 그 믿음들을 되가져온다. 다른 누군가와 떠났다가 되돌아오기로 결심하는 배우자처럼. 하지만 우리는 그 믿음들을 다르게 본다 (어느 경우든 그 새로운 태도가 필연적으로 옛 사람들보다 열등하다는 것은 아니다.)

8) The same situation obtains after we have put in question the seriousness with which we take our lives and human life in general and have looked at ourselves without presuppositions. We then return to our lives, as we must, but our seriousness is laced with irony. Not that irony enables us to escape the absurd. It is useless to mutter: "Life is meaningless; life is meaningless…" as an accompaniment to everything we do. In continuing to live and work and strive, we take ourselves seriously in action no matter what we say.
8) 우리가 우리의 삶들과 일반적인 인간적 삶을 대하는 그러한 진지함을 의문시 하고 추정들 없이 우리 자신을 본 뒤에 똑같은 상황이 된다. 그리고 우리는 우리가 마땅히 그러해야 하듯 우리의 삶들로 돌아오지만 우리의 심각함은 역설과 엮인다. 역설이 우리가 부조리를 피하도록 해줄 수 있다는 것은 아니다. "삶은 무의미하다; 삶은 무의미해…"라고 투덜거리는 것이 우리가 하는 모든 일에 조미료로서 무용하지는 않다. 계속해서 살아가고 일하고 분투하면서, 우리는 우리가 뭐라 하든 관계 없이 실천에 있어서 우리 자신을 진지하게 대한다.

9) What sustains us, in belief as in action, is not reason for justification, but something more basic than these - for we go on in the same way even after we are convinced that the reasons have given out. If we tried to rely entirely on reason, and pressed it hard, our lives and beliefs would collapse - a form of madness that may actually occur if the inertial force of taking the world and life for granted is somehow lost. If we lose our grip on that, reason will not give it back to us.
9) 실천에서처럼 믿음에 있어서 우리를 살아가게 하는 것은 정당화가 그 까닭이 아니라, 이러한 것들보다 더 기본적인 어떤 것이다 - 우리가 그 이유들이 끝나 버렸다는 것을 알게 된 이후에조차 같은 방식으로 계속 살기 때문이다. 만일 우리가 전적으로 근거에만 매달리려 했다면, 그리고 그것을 강력하게 밀어 붙였다면, 우리의 삶들과 믿음들은 충돌하였을 것이다 - 만일 세계와 삶을 당연시함의 관성이 어떻게든 상실된다면 실제로 일어날지 모르는 광기의 형태. 우리가 그러한 능력을 잃어 버린다면, 이유가 우리에게 그걸 돌려주지는 않을 것이다.


1) In viewing ourselves from a perspective broader than we can occupy in the flesh, we become spectators of our own lives. We cannot do very much as pure spectators of our own lives, so we continue to lead them, and devote ourselves to what we are able at the same time to view as no more than a curiosity, like the ritual of an alien religion.
1) 우리가 몸으로 접할 수 있는 것보다 더 넓은 관점에서 우리 자신을 바라봄에 있어서, 우리는 우리 자신의 삶들에 대한 관중들이 된다. 우리는 우리 자신의 삶들에 대한 순수한 관중들로서 아주 그렇게 굴 수는 없고, 그래서 우리는 그 삶들을 끌어 나아가기를 계속하고, 동시에 우리 자신을 우리가 호기심 이상의 아무것도 아닌 것으로 바라볼 수 있는 것에 헌신한다. 낯선 종교의 의식처럼.

2) This explains why the sense of absurdity finds its natural expression in those bad arguments with which the discussion began. References to our small size and short lifespan and to the fact that all of mankind will eventually vanish without a trace are metaphors for the backward step which permits us to regard ourselves from without and to find the particular form of our lives curious and slightly surprising. By feigning a nebula's-eye view, we illustrate the capacity to see ourselves without presuppositions, as arbitrary, idiosyncratic, highly specific occupants of the world, one of countless possible forms of life.
2) 이것은 왜 부조리에 대한 감각이 논의를 시작했던 그러한 좋지 못한 논증들에서 그 자연스러운 표현을 발견하는지를 설명한다. 우리의 작고도 짧은 생애와 모든 인류가 결국 흔적도 없이 사라질 것이라는 사실에 대한 언급들은 우리로 하여금 우리 자신을 외부에서 고려하고 우리의 삶들의 특유한 형태를 신기하고 약간은 놀라운 것으로 여기도록 해주는 물러섬에 대한 은유들이다. 우주적 관점을 가장함으로써, 우리는 우리 자신을 추정들 없이, 세계의 제멋대로이고 기이하며 고도로 특수한 입주자들로, 무수한 가능한 삶의 형태들 중의 하나로 보는 능력을 보여준다.

3) Before turning to the question whether the absurdity of our lives is something to be regretted and if possible escaped, let me consider what would have to be given up in order to avoid it.
3) 우리들 삶들의 부조리가 거부되어야 할, 그리고 만일 가능하다면 회피되어야 할 어떤 것인지 하는 물음으로 돌아가기에 앞서, 나는 그 부조리를 피하기 위해 포기되었어야 할 것이 무엇인지를 고려해 보고자 한다.

4) Why is the life of a mouse not absurd? The orbit of the moon is not absurd either, but that involves no strivings or aims at all. A mouse, however, has to work to stay alive. Yet he is not absurd, because he lacks the capacities for self-consciousness and self-trascendence that would enable him to see that he is only a mouse. If that did happen, his life would become absurd, since self-awareness would not make him cease to be a mouse and would not enable him to rise above his mousely strivings. Bringing his new-found self-consciousness with him, he would have to return to his meager yet frantic life, full of doubts that he was unable to answer, but also full of purposes that he was unable to abandon.
4) 어째서 생쥐의 삶은 부조리하지 않은가? 달의 궤도도 부조리하진 않지만, 그것은 전혀 아무런 노력들도 목적들도 수반하지 않는다. 그렇지만 쥐는 살아남기 위해 애써야 한다. 하지만 그 쥐는 부조리하지 않은데, 그 쥐가 그저 한 마리 쥐에 불과하다는 것을 볼 수 있도록 해줄 자기의식과 자기초월을 위한 능력들을 결여하고 있기 때문이다. 만일 그러한 일이 벌어진다면, 그 쥐의 생은 부조리해질 것인데, 왜냐하면 자각은 그를 한 마리 쥐인 채로 머무르도록 하지 않을 것이고 그의 쥐스러운(?) 노력들에 초연할 수 있도록 해주지 않을 것이기 때문이다. 새롭게 발견된 그의 자기의식을 가지고, 그 쥐는 메마르고 아직도 정신없이 바쁜 그의 생, 그가 답할 수 있었던 의심들로 가득 찬, 뿐만 아니라 그가 버릴 수 있었던 의도들로 가득 찬 삶으로 돌아가야 할 것이다.

5) Given that the transcendental step is natural to us human, can we avoid absurdity by refusing to take that step and remaining entirely within our sublunar lives? Well, we cannot refuse consciously, for to do that we would have to be aware of the viewpoint we were refusing to adopt. The only way to avoid the relevant self-consciousness would be either never to attain it or to forget it - neither of which can be achieved by the will.
5) 초월적 걸음이 우리 인간에게 자연스러운 것임을 고려하면, 우리는 그 걸음을 떼기를 거부하고 전적으로 우리의 지상의 삶들 속에 머무름으로써 부조리를 피할 수 있을까? 자, 우리는 의식적으로 거부할 수 없는데, 왜냐하면 우리가 택하기를 거부하고 있던 관점을 알아차려야 했을 것이라는 점 때문이다. 그 상대적인 자기의식을 피할 유일한 방법은 그 자기의식을 절대로 얻지 않거나 그것을 잊어 버리는 것일 터이다 - 둘 중 어느 것도 의지로 성취될 수는 없다.

6) On the ohter hand, it is possible to expend effort on an attempt to destroy the other component of the absurd - abandoning one's earthly, individual, human life in order to identify as completely as possible with that universal viewpoint from which human life seems arbitrary and trivial. (This appears to be the ideal of certain Oriental religions.) If one succeds, then one will not have to drag the superior awareness through a strenuous mundane life, and absurdity will be diminished.
6) 다른 한편으로, 부조리의 다른 요소들을 파괴하고자 시도하는 데에 노력을 들일 수는 있다 - 세속적이고, 개인적이며, 인간적인 삶을 가능한한 완전히, 인간적 삶이 그로부터 임의적이고 제멋대로인 것으로 보이는 보편적 관점과 동일시하기 위해서 폐기하기 위해서. (이것은 특정한 동양 종교들의 이상으로 드러난다.) 만일 누군가 성공한다면, 그는 고되고 지루한 삶을 통해 우월한 깨달음을 이끌어내지 않아도 될 것이고, 부조리는 감소될 것이다.

7) However, insofar as this self-etiolation is the result of effort, will-power, aceticism(a's'ceticism?), and so forth, it requires that one take oneself seriously as an individual - that one be willing to take considerable trouble to avoid being creaturely and absurd. Thus one may undermine the aim of unworldliness by pursuing it too vigorously. Still, if someone simply allowed his individual, animal nature to drift and respond to impulse, without making the pursuit of its needs a central conscious aim, then he might, at considerable dissociative cost, achieve a life that was less absurd than most. It would not be a meaningful life either, of course; but it would not involve the engagement of a transcendent awareness in the assiduous pursuit of mundane goals. And that is the main condition of absurdity - the dragooning of an unconvinced transcendent consciousness into the service of an immanent, limited enterprise like a human life.
7) 그렇지만, 이러한 자기-황화가 노력, 의지력, 고행 같은 것들의 결과인 한, 그것은 자신을 한 개인으로서 진지하게 대하는 것을 - 만들어진 부조리한 존재를 벗어나는 일을 중요한 문제로 기꺼이 받아들일 것을 요구한다. 그래서 누군가는 비세속적 목적을 그것이 지나치게 격하게 추구함으로써 약화시킬지도 모른다. 그럼에도, 만일 누군가 단순히 그의 개인적이고 동물적인 본성으로 하여금 충동에 흘러가고 반응하는 것을 허용한다면, 가장 중요한 의식적 목표에 대한 그 충동의 요구들에 대한 추구를 이룸이 없이, 그는 아마도, 중대한 해리적 값을 지불하고, 대다수의 사람들보다는 덜 부조리했던 삶을 이룩할지 모른다. 물론 그것은 의미있는 삶도 아닐 것이다; 그러나 일상적인 목적들에 대한 근면한 추구에 있어서 초월적 깨달음에 대한 약속을 수반하지도 않을 것이다. 그리고 그것이 부조리의 주된 상태이다 -  인간적 삶과 같이 인지되지 않은 초월적 의식에 대한 내재적이고 엄격한 사업에 봉사하도록 강요하는 것.

8) The final escape is suicide; but before adopting any hasty solutions, it would be wise to consider carefully whether the absurdity of our existence truly presents us with a problem, to which some solution must be found - a way of dealing with prima facie disaster. That is certainly the attitude with which Camus approaches the issue, and it gains support from the fact that we are all eager to escape from absurd situations on a smaller scale.
8) 마지막 도피는 자살이다; 그러나 어떤 성급한 해결책들을 택하기에 앞서, 우리의 현존에 대한 부조리가 실로 우리에게 - 언뜻 재난인 듯한 것을 다루는 방법 - 어떤 해결책이 반드시 발견되어야만 하는 그러한 문제를 제기하는지 주의깊게 고려해 보는 것은 현명한 일일 것이다. 그것은 분명히 카뮈가 그 문제를 대하는 태도이고, 그 태도는 우리 모두가 부조리한 상황들로부터 더 작은 규모로 달아나기를 열망한다는 사실로부터 지지를 얻는다.

9) Camus - not on uniformly good grounds - rejects suicide and the other solutions he regards as escapist. What he recommends is defiance or scorn. We can salvage our dignity, he appears to believe, by shaking a fist at the world which is deaf to our pleas, and continuing to live in spite of it. This will not make our lives un-absurd, but it will lend them a certain nobility.
9) 카뮈는 - 일관되게 좋은 근거들로는 아니지만 - 자살을 거부하고 그가 현실도피로 간주한 다른 해결책들도 거부한다. 그가 요구하는 것은 저항이나 경멸이다. 그는 우리가 우리의 존엄을 구제할 수 있다고 믿는 것으로 보이는데, 우리의 간청에 귀를 막아 버린 세계에 주먹질을 해댐으로써, 그리고 그럼에도 불구하고 계속해서 살아냄으로써 그렇게 할 수 있다는 것이다. 이것은 우리 삶을 부조리하지 않도록 만들지는 못할 것이지만, 그 삶들에 어떤 고결함을 부여할 것이다.

10) This seems to me romantic and slightly self-pitying. Our absurdity warrants neither that much distress nor that much defiance. At the risk of falling into romanticism by a different route, I would argue that absurdity is one of the most human things about us: a manifestation of our most advanced and interesting characteristics. Like skepticism in epistemology, it is possible only because we possess a certain kind of insight - the capacity to transcend ourselves in thought.
10) 이것이 나에게 낭만적이고 약간은 자기동정적으로 보인다. 우리의 부조리는 많은 고통도 많은 저항도 타당하게 하지 않는다. 다른 경로를 통해 낭만주의에 빠질 위험을 무릅쓰고, 나는 부조리가 우리에게 있어서 가장 인간적인 것들 중 하나라고 주장할 것이다: 우리의 가장 진보되고 흥미로운 특징들의 징후로서 말이다. 인식론에서 회의주의와 같이, 그것은 오로지 우리가 어떤 종류의 통찰을 지니는 것 때문으로만 가능하다 - 사유 속에서 우리 자신을 초월하는 능력 말이다.

11) If a sense of the absurd is a way of perceiving our true situation (even though the situation is not absurd until the perception arises), then what reason can we have to resent or escape it? Like the capacity for epistemological skepticism, it results from the ability of understand our human limitations. It need not be a matter for agony unless we make it so. Nor need it evoke a defiant contempt of fate that allows us to feel brave or proud. Such dramatics, even if carried on in private, betray a failure to appreciate the cosmic unimportance of the situation. If sub specie aeternitatis there is no reason to believe that anything matters, then that does not matter either, and we can approach our absurd lives with irony instead of heroism or despair.
11) 만일 부조리에 대한 감각이 우리의 참된 상황을 이해하는 방법이라면 (설령 그 상황이 그 지각이 이루어지기 전까지는 부조리하지 않더라 할지라도), 그래서 우리가 그것을 분개하거나 피해야 하는 무슨 이유를 가질 수 있는가? 인식론적 회의주의를 위한 능력과 마찬가지로, 그것은 우리의 인간적 제한들을 이해하는 능력으로부터 귀결한다. 그것은 우리가 그것을 그렇게 만들기 전까지는 그렇게 극심하게 고통스러울 일일 필요가 없다. 그것이 우리로 하여금 용기나 자긍심을 느끼도록 허락하는 숙명에 대한 저항적 경멸을 불러일으킬 필요가 있는 것도 아니다. 그러한 극적인 것은, 사적으로 다루어진다 하더라도, 그 상황의 우주적 하찮음을 평가하는 데에 실패하였음을 드러낸다. 만일 영원한 상 아래에서 어떤 것이든 문제시된다고 믿을 아무런 이유도 없다면, 그것도 문제될 게 없을 것이고, 우리는 우리의 부조리한 삶을 영웅주의나 체념 대신에 반어로 접근할 수 있을 것이다.

-蟲-
The Province of Philosophy

J. J. C. Smart

1) No one answer can be given to the question "What is philosophy?" since the words "philosophy" and "philosopher" have been used in many ways. Some people, for example, think of philosophy as offering the consolation of a religion, and of the philosopher as a man who receives with equanimity the bufferings of life. This has very little to do with the way in which academic people, including myself, use the word "philosophy." I do not feel particularly unqualified to be an academic philosopher because I am not "philosophical" when I am bowled out first ball at cricket. As I propose to use the word "philosophy" it will stand primarily for an attempt to think clearly and comprehensively about: (a) the nature of the universe, and (b) the principles of conduct. In short, philosophy is primarily concerned with what there is in the world and with what we ought to do about it. Notice that I have said both "to think clearly" and "to think comprehensively." The former expression ties up with the prevailing conception of philosophy as linguistic or conceptual analysis, and the latter ties up with another common conception of philosophy as the rational reconstruction of language so as to provide a medium for the expression of total science.
1) "철학이란 무엇인가?"라는 물음에 그 누구도 대답을 할 수 없는데 왜냐하면 "철학"과 "철학자"라는 말이 여러 방식으로 사용되어 왔기 때문이다. 예를 들어 몇몇 사람들은 철학이 종교적 위안을 제공하는 것이라고 생각하고, 철학자는 삶의 격동과 파란을 평정을 가지고 맞아들이는 사람으로 생각한다. 이것은 나를 포함한 학계 사람들이 "철학"이라는 말을 사용하는 방식과 아주 약간 관계가 있다. 나는 딱히 강단 철학자가 되기에 완전무결하다고  느끼지는 않는데 왜냐하면 내가 크리켓 경기에서 첫타를 실패했을 때 나는 "철학적"이지는 않기 때문이다. 내가 "철학"이라는 말을 사용하려고 할 때 그 말은 주로 다음과 같은 것들에 대해 분명하고 포괄적으로 생각하려는 시도를 나타낼 것이다. (a) 우주의 본성, 그리고 (b) 실천의 원칙들. 간단히 말해서 철학은 주로 세계 내에 있는 것과 우리가 그에 대해 해야 할 일에 관련된다. 내가 "분명하게 생각하는 것"과 "포괄적으로 생각하는 것" 모두를 이야기 했다는 점을 주의하라. 앞서의 표현은 언어적이거나 개념적인 분석으로 유행하는 철학의 개념과 얽혀 있고, 후자의 표현은 언어의 합리적 재구성으로서 그래서 학문 총체에 대한 표현을 위한 도구를 제공하는 것으로서 철학의 또 다른 일반적인 이해와 묶여 있다.

2) Thus, a man might analyze biology in a certain way. He might argue, as I shall do, that living organisms, including human beings, are simply very complicated physico-chemical mechanisms. This man might also analyse physics as the ordering and predicting of sense experiences. For the sake of argument let us concede that such a man might be thinking clearly, he would not be thinking comprehensively. As biologist he would be thinking of man as a mechanism, as very much a part of nature, a macroscopic object interacting with its environment. As physicist, however, he would be thinking of this great world of nature as just a matter of the actual and possible experiences of sentient beings, and so, in a sense, he would be putting nature inside man. To think comprehensively he would have to discover a way of thought which enabled him to think both as biologist and as a physicist. Presumably a comprehensive way of thought would be one which brought all intellectual disciplines into a harmonious relationship with one another. It may turn out that there are some realms of discourse, such as theology, which cannot be brought into a harmonious relationship with the various sciences. Any attempt to do so may result in violence to logic or to scientific facts, or may involve arbitrariness and implausibility. (Consider, for example, the implausibility of a theory which asserts that the mechanistic account of evolution by natural selection and mutation is broadly true but the there is a special discontinuity in the case of man, to whom was super-added an immortal soul.) If this is so, such anomalous branches of discourse will have to be rejected and will not form part of the reconstruction of our total conceptual scheme.
2) 그래서, 한 사람은 아마도 특정한 방식으로 생명 활동을 분석할 것이다. 그는 아마도 내가 하려는 것처럼 인간을 포함한 생명 또는 유기체들이 단순히 매우 복잡한 물리-화학적 기계장치들이라고 주장할 것이다. 이 사람은 또한 아마도 물리학을 감각 경험들에 대한 정리와 예측으로 해석할 것이다. 논증을 위해 그러한 사람이 아마도 분명하게 생각하고 있으리라고, 그가 포괄적으로 생각하고 있지는 않을 것이라고 치자. 생물학자로서 그는 인간을 기계장치로, 상당수 자연의 일부로, 주위환경에 관심을 기울이는 거시적 대상으로 생각하고 있을 것이다. 그렇지만 물리학자로서 그는 자연의 이 광대한 세계를 바로 지각을 지닌 존재자들의 실제적이고 가능한 경험들의 문제로 생각하고 있을 것이고, 그래서 어떤 의미로는 그가 사람 내부에 자연을 놓고 있을 것이다. 포괄적으로 생각하기 위해 그는 그가 생물학자로서도 또 물리학자로서도 생각할 수 있도록 하는 사유의 한 방법을 발견해야 할 것이다. 짐작컨데 사유의 포괄적 방법은 모든 지적 분야들을 상호 조화로운 관계로 가져다 놓을 한 가지의 것일 터이다. 그 방법은 신학과 같은 어떤 담론의 영역들, 다양한 학문들과 조화로운 관계로 이끌릴 수 없는 몇몇 영역들이 있다는 데에로 나아갈지도 모른다. 그렇게 하려는 어떠한 시도든 논리 혹은 과학적 사실들에 폭력을 낳거나, 또는 천단(擅斷)을 수반하게 될지 모른다. (예를 들어 자연선택과 돌연변이에 의한 진화에 대한 기계적 설명은 폭넓게 진실이지만 인간의 경우에 있어서는 불멸하는 영혼이 부가되었다는 특별한 단절이 있다고 주장하는 믿기 어려운 이론을 생각해 보라.) 만일 이것이 그렇다면, 담론의 그러한 변칙적인 곁가지들은 거부되어야 할 것이고 우리의 총체적 개념적 기획(도식)의 부분을 구성하지 않을 것이다.

3) So much, for the moment, about the "nature of the universe" or "world view" part of philosophy. Let us now briefly consider the second part of philosophy, which is concerned with the principles of conduct … as has been generally recognised in modern philosophy, it is not possible to deduce propositions about what ought to be done purely from propositions about what is the case. It follows that the principles of conduct are by no means unambiguously determined by our general philosophy. Nevertheless, in their laudable objection to those who would deduce ethics from the nature of the world (and in particular to some of those biologists who would base ethics on the theory of evolution and the like) philosophers have tended to obscure the fact that our general philosophical and scientific beliefs may stongly influence our ethical principles. For example, if one of our principles of conduct were that we should do what is commanded by a personal God and if our world view were one which left no place for such a God, then this principle of conduct would have to be given up, or at least we should have to find some other reason for adhering to it. In this [selection], which will be naturalistic in temper, I do not wish to concern myself with the general question of the legitimacy or illegitimacy of theology. The example of theology was brought up simply to show in a vivid way that metaphysics can be relevant to ethics. We must certainly not jump from the impossibility of deducing "ought" solely from "is" to the untenable position that our general philosophical and scientific views have no bearing on our ethical ones.
3) 우선은 "우주의 본성"이나 "세계관" 철학의 부분은 그 정도로 하도록 하자. 이제 간략하게 철학의 둘째 부분을 살펴보도록 하자. 그 부분은 행위의 원칙들과 관련된다. 일반적으로 근대 철학에서 인정되어 왔듯이, 순수하게 그 상황이 무엇인지에 대한 명제들로부터 행해져야 할 것이 무엇인지에 대한 명제들이 연역되는 것은 불가능하다. 결과적으로 우리의 일반적인 철학으로는 행위의 원칙들이 모호하지 않게 결정될 방법이 없다. 그럼에도 불구하고, 세계의 본성으로부터 윤리학을 연역하려는 그러한 사람들에 대한 (그리고 특히 윤리학을 진화론과 그 유사한 것에 기초지으려는 그러한 생물학자들의 일부에 대한) 그 원칙들의 칭찬할 만한 반박에 있어서 철학자들은 우리의 일반적인 철학적이고 과학적인 믿음들이 우리의 윤리적 원칙들에 강력하게 영향을 미칠지 모른다는 사실을 모호하게 만드는 경향이 있었다. 예를 들어, 만일 우리의 행위원칙들 중 하나가 인격신에 의해 명령된 일을 해야 한다는 것이라면, 그리고 만일 우리의 세계관이 그러한 신을 위한 아무런 자리도 남겨두지 않는 그러한 한 가지의 것이라면, 이 행위원칙은 포기되어야 할 것이거나, 혹은 최소한 우리는 그 원칙에 덧붙일 다른 어떤 이유를 찾아야 할 것이다. 이러한 자연주의적 성향이 될 선택지에 있어서 나는 신학의 적법성과 위법성에 대한 일반적인 물음에 전념하고 싶지는 않다. 신학의 예는 단순히 형이상학이 윤리학에 관계될 수 있다는 것을 생생하게 보여주기 위해 제기되었다. 확실히 우리는 단지 "is"만으로는 그로부터 "ought"를 연역함이 불가능하다는 것으로부터 우리의 일반적인 철학적이고 과학적인 관점들이 우리의 윤리적 관점과 아무런 관계도 없다는 옹호될 수 없는 입장으로 비약하지 말아야만 한다.


철학과 무의미의 배제

1) I have been suggesting a conception of philosophy as the attempt to acquire a synoptic view of the world. On this account of philosophy it shares the tentaive character of the sciences. We must never think that we have acquired, even in outline, the final truth, for science inevitably provides wurprises for us, and we may have to make important revisions of even our most general notions. We may hope, however, that our synoptic account will be nearer to the final truth than is that of common sense. Now in recent years it has been argued in some quarters that in philosophy we are not concerned as scientists are, with the distinction between truth and falsity, but with that between sense and nonsense. As philosophers, according to this conception, it is not our business to say what the owrld is in fact like: we must leave this to scientists and historians. What we can do, and what we are by our training peculiarly fitted to do, is to help to ensure that we, together with scientists and historians, at least utter falshoods: that we and they do not fall into nonsense which has not even achieved the distinction of an intelligible falshood. Let me illustrate the notion of nonsense by means of example based on Alice in Wonderland.Suppose that a man came and said that he had seen a miaowing and blinking cat's head which was unattached to a body. I should be disposed in this case to disbelieve the man, and to say that what he told me was false. I should feel that I understood him: that I knew what it would be like for such an event as he reported to occur, but that I did not believe that any such event ever had or would occur. His report would contradict various secure beliefs that I possessed, particularly in the field of animal physiology. Now let us suppose that the man had reported not that he had seen a cat's head by itself but that he had seen simply a grin by itself. Not even a grinning mouth unattached to a head, but simply a grin all on its own. In this case I should not know what was meant at all. I should not be disposed to say that I understood what the man said, even though I disbelieved in the truth of his report. I should say, rather that what he said was nonsense, neither true nor false, and so I could not even disbelieve him.
1) 나는 세계에 대한 개괄적 관점을 획득하기 위한 시도로서 철학의 이해를 제안하고 있었다. 철학에 대한 이러한 설명에 대해서 그것은 학문들의 잠정적 성격을 공유한다. 우리는 결코 우리가 개략적으로라도 궁극적인 진리를 획득했다고 생각해서는 안 되는데, 과학은 불가피하게 우리에게 뜻밖의 일들을 제공하기 때문이고, 우리는 우리의 가장 일반적인 관념들에 대해서조차 중대한 수정을 할 필요가 있을지 모르기 때문이다. 그렇지만 우리는 아마도 우리의 개괄적 설명이 일반 상식의 설명이 있는 것보다는 궁극적 진리에 더 가까이 있을 것이라 희망할지 모른다. 이제 최근 몇 년 동안 일부 사람들에게서 철학에 있어서 우리는 과학자들이 그러하듯 진리와 오류 사이에 대조와 관련하지 않고 의미와 무의미 사이의 대조에 관계한다는 주장이 있어 왔다. 철학자들로서, 이러한 이해에 따르면, 세계가 실제로 무엇과 같은 것인지 말하는 것은 우리의 일이 아니다. 우리는 이 일을 과학자들과 역사학자들에게 넘겨야만 한다. 우리가 할 수 있는 일, 우리가 우리의 훈련을 통해 특유하게 할 수 있도록 맞추어진 일은 과학자들과 역사학자들과 더불어 우리가 최소한 우리와 그들이 지성적인 거짓과 명확한 차이조차 이루지 않은 무의미에 빠져든다는 거짓말들을 한다는 것을 보장하도록 돕는 것이다. 『이상한 나라의 앨리스』에 기초한 한 가지 예시로써 그 무의미의 관념을 묘사해 보고자 한다. 한 사람이 와서 그가 몸뚱이에 붙어있지 않은 야옹거리고 깜빡거리는 고양이의 머리를 봤다고 말했다고 치자. 나는 이 경우 그 사람을 불신할 것 같고, 그가 나에게 말했던 것이 거짓이었다고 말할 듯하다. 나는 내가 그를 이해했다고 느낄 것이다. 즉 나는 그가 일어났다고 전하는 그러한 한 사건과 같을 그러한 어떤 것을 알고 있었다고 느낄 것이다. 그러나 나는 그러한 어떤 사건이라도 결코 일어났거나 일어나리라고 믿지 않았다고 느낄 것이다. 그의 말은 내가 지니고 있던 여러 안정적인 믿음들에 모순될 것이고, 특히 동물 생리학의 영역에서 그러할 것이다. 이제 그가 그 자체로 고양이 머리를 봤다고 하지 않고 단순히 웃음 그 자체를 봤다고 말한다고 가정해 보자. 머리에 붙어있는 웃는 입이 아니라, 단순히 웃음이 그 자체로 전부인 것을. 이 경우 나는 도대체 무슨 의미였는지 전혀 알지 못할 것이다. 나는 내가 그가 말했던 것을 이해했다고 말할 것 같지 않다. 설령 내가 그의 말을 참이라고 믿지 않는다고 하더라도 말이다. 나는 그가 말했던 것이 무의미하다고 말하지 참이거나 거짓이라 말하지는 않을 것이고, 그래서 나는 그를 믿지 않을 수조차 없을 것이다.

2) Now it is indubitable that there are sentences which have appeared to be meaningful and which nevertheless have turned out to be nonsense. I shall mention one such sentence in a moment. And so even though the remarks of traditional philosophers (say, about the famous trio of topics, God, Freedom, and Immortality) may not be obvious nonsense, like the report of the catless grin, they may be nonsense all the same.
2) 이제 의미있는 것으로 보이는 문장들과 그럼에도 불구하고 의미 없는 것으로 밝혀진 문장들이 있다는 것은 의심의 여지가 없다. 나는 곧 그러한 문장 하나를 언급할 것이다. 그리고 설령 전통적인 철학자들의 언급들이 (말하자면, 유명한 세 가지 주제들, 신, 자유, 불멸에 대하여) 고양이 없는 웃음에 대한 말처럼 명백하게 무의미하지 않을지 몰라도 그것들은 그래도 마찬가지로 무의미할 것이다.

3) Here is a sentence, couched entirely in the respectable terminology of pure mathematics, which at first sight may appear to some readers (assuming that they have not encountered it before) to be perfectly meaningful, though perhaps rather dry and abstract. It was first concocted by Bertrand Russell. (Russell's paradox.) The sentence is: "The class of all classes not members of themselves is a member of itself." There appear to be plenty of classes of objects which are not members of themselves. The class of criminals is not a criminal (the police do not have to seek the class of criminals after they have arrested all criminals), and the class of football teams in the league is not a further football team. Most classes therefore appear not to be members of themselves. But some classes do appear to be members of themselves: certainly the class of classes does. For is not the class of classes a class? It would therefore seem to be perfectly intelligible to pose the question of whether the class of all classes not members of themselves is or is not one of those classes which are members of themselves. Unfortunately, this question admits neither the answer "yes" nor the answer "no." For if the class of all classes not members of themselves is a member of itself, then it follows that it is one of those classes which are not members of themselves. And if it is not a member of itself, then it is a member of itself. Either way we get a contradiction. It follows that we can neither say that the sentence "the class of all classes not members of themselves is a member of itself" express a truth, nor can we say that it expresses a falsehood. We are forced to conclude that it is meaningless.
3) 여기 한 문장이 있다. 그 문장은 순수 수학의 훌륭한 용어들에 전적으로 자리매김되어 있고, 척 보기에 아마 몇몇 독자들에게 (그들이 일전에 그것을 마주했던 적이 없다고 가정하면) 완벽하게 의미있는 것으로 보일 그러한 문장이다. 아마 건조하고 추상적일 터라도 말이다. 그 문장은 버트란드 러셀에 의해 처음으로 고안되었다. (러셀의 역설이다.) 그 문장은 "그 자신들의 원소들이 아닌 집합들 모두의 집합은 그 자신의 원소이다."라는 문장이다. 그들 자신들의 원소들이 아닌 대상들의 집합들은 충분할 만큼 많아 보인다. 범죄자들의 집합은 한 사람의 범죄자가 아니고 (경찰들은 그들이 모든 범죄자들을 체포한 뒤에 범죄자들의 집합을 좇을 필요가 없다), 리그에서 풋볼 팀들의 집합은 도 하나의 풋볼 팀이 아니다. 대부분의 지합들은 그러므로 그것들 자신들의 원소들로 보이지 않는다. 그러나 일부 집합들은 그것들 자신들의 원소들로 보인다. 특히 집합들의 집합이 그러하다. 집합들의 집합은 하나의 집합이 아니기 때문인가? 그러므로 그것들 자신들의 원소들이 아닌 모든 집합들의 집합이 그것들 자신들의 원소들인 그러한 집합들 중의 하나인지 아닌지에 대한 물음을 제기하는 것은 완벽하게 이해될 수 있을 것으로 보인다. 불행하게도, 이 물음은 '네'라는 대답도 '아니오'라는 대답도 허용하지 않는다. 왜냐하면 만일 그 자신들의 원소들이 아닌 모든 집합들의 집합이 그 자신의 원소라면, 따라서 그 집합은 그것들 자신들의 원소들이 아닌 그러한 집합들 중의 하나라는 것이 두따를 터이기 때문이다. 그리고 만일 그 집합이 그 자신의 원소가 아니라면, 그 집합은 그 자신의 원소일 것이다. 어떤 식으로도 우리는 모순을 취하게 된다. 우리가 "그것들 자신들의 원소들이 아닌 모든 집합들의 집합은 그 자신의 원소이다"라는 문장이 참을 나타낸다고 말할 수도 없고, 우리가 그 문장이 거짓을 나타낸다고 말할 수도 없다는 것이 뒤따른다. 우리는 그것이 무의미하다고 결론내릴 수밖에 없다.

4) The above paradox is particularly important and instructive, because it shows how unsuspected possibilities of nonsense can break out even in the rigorous and austere terminology of mathematics. For those readers who may not find abstractions about classes to their taste I shall mention a similar, though less important paradox, which may be even more succinctly stated. Consider the sentence "This sentence is false." The sentence is about itself. Is it true or false? It can be neither, because if it is true it is false and if it is false it is true. It is important to note that the above paradoxical sentences are not mere contradictions. You can assert the negation of a contradiction. That is, a contradiction is just plain false. "2+2=5" is a contradiction, and so "2+2≠5" is a truth. Contradictions have their uses, for they occur in proofs by reductio ad absurdum. If you can deduce "2+2=5" you can normally deduce that the negation of one or other of the premisses is true. I have said "normally" here, because it is important to use reductio ad absurdum methods only when you are reasonably sure of the meaningfulness of the sentence you are trying to prove. If the sentence you are trying to prove is meaningless it may be like one of the paradoxical sentences above and you may be able to deduce a contradiction both from it and from its negation. In which case the deduction of a contradiction from its negation does not ensure its truth. This consideration may be of interest to some readers, in that it may throw light on the fact that certain mathematicians, the so-called "intuitionists," Brouwer and his school, reject proof by reductio ad absurdum in circumstances in which classical mathematicians do not. It is, of course, the case that there are sentences which classical mathematicians regard as meaningful and which the intuitionists hold to be meaningless.
4) 위의 역설은 특히 중요하고도 유익한데, 왜냐하면 그것은 어떻게 무의미의 예상치 못한 가능성들이 엄격하고 근엄한 수학용어들에서조차 튀어나올 수 있는지 보여주기 때문이다. 그들의 경험에 대해서 집합들에 대한 추상들을 발견하지 못할 그러한 독자들을 위해 나는 좀 덜 중요한 역설일지라도 아마 더욱 간결하게 진술되었을 유사한 역설을 하나 언급할 것이다. "이 문장은 거짓이다"라는 문장을 고려해 보자. 그 문장은 그 문장 자체에 대한 것이다. 그것은 참인가 거짓인가? 그것은 참일 수도 거짓일 수도 없는데, 왜냐하면 만일 그것이 참이라면 그것은 거짓일 터이고 그것이 거짓이라면 그것은 참일 터이기 때문이다. 위의 역설적 문장들이 단순한 모순들이 아님을 주목하는 것이 중요하다. 당신은 모순에 대한 부정을 주장할 수 있다. 즉, 하나의 모순은 단지 분명한 거짓이다. "2+2=5"는 모순이고, 그래서 "2+2≠5"는 참이다. 모순들은 그것들의 용도들을 지니는데, 왜냐하면 그것들은 귀류법(모순으로 되돌림)에 의한 증명들에서 발생하기 때문이다. 만일 당신이 "2+2=5"를 연역할 수 있다면 당신은 일반적으로 그 전제들 중의 하나 혹은 그 이상의 전제들의 부정이 참임을 연역할 수 있다. 나는 여기에서 "일반적으로"라고 말했는데, 왜냐하면 귀류법의 방식들을 오로지 당신이 합리적으로 당신이 증명하고자 하는 문장의 의미있음에 대해 확신할 때에만 사용하는 것이 중요하기 때문이다. 만일 당신이 증명하려는 문장이 의미없다면 그것은 위의 역설적 문장들 중의 하나와 같은 것일 터이고 당신은 그 문장으로부터도 그 문장의 부정으로부터도 모순을 연역할 수 있을 것이다. 어떤 경우에는 그것의 부정으로부터 모순에 대한 연역이 그것의 참을 보장하지 않는다. 이러한 고려사항은 아마도 일부 독자들에게 흥미로울 것인데, 그것이 특정한 수학자들, 소위 "직관론자들", 브로우베르와 그의 학파가 고전적 수학자들이 증명하지 않은 상황들에서의 귀류법에 의한 증명을 거부한다는 사실에 대해 실마리를 던져준다는 점에서 그러하다. 물론 그것은 고전적 수학자들이 의미있는 것으로 간주하고 직관론자들은 무의미하다고 주장하는 문장들이 있는 경우이다.

5) The sort of possibility of nonsense to which I have been drawing attention in the last few paragraphs is a subtle and insidious one. Nonsense of a sort has always been recognised: consider "I married a prime number" and "Virtue is triangular." It is an insight of the last fifty years (though foreshadowed by the philosophyically subtle humour of Lewis Carroll) that there can be important and nonobvious possibilities of nonsesne. This insight was generalised by Wittgenstein and by those much influenced by him into a complete philosophy of philosophy.
5) 내가 마지막 적은 문단들에서 주의를 이끌었던 무의미의 그런 종류의 가능성은 교묘하고 은밀한 것이다. 일종의 무의미는 언제나 고려되어 왔다: "나는 소수와 결혼했다"와 "덕은 삼각이다"를 고려해 보라. 무의미의 중요하고 불확신한 가능성들이 있을 수 있다는 것은 지난 50년 동안의 통찰이다 (설령 루이스 캐롤의 철학적이고 미묘한 농담에 의해 전조되었다 할지라도 말이다). 이러한 통찰은 비트겐슈타인과 그에게 영향을 받은 많은 이들에 의해 철학에 대한 완결된 철학으로 일반회되었다.

6) It is clear that technique for recognising non-obvious nonsense is highly desirable, and I should agree that the development and application of such a technique is at least part of the task of philosophy. How does this connect up with my conception of philosophy as the development of a synoptic outlook? Can the elimination of nonsense change our world view? At first sight the answer to this is in the negative. If the nonsense really is nonsense it cannot form part of a world view, even a false one. So it looks as though elimination of nonsense removes dead wood but does not affect the living branches of our knowledge. This answer is, however, too hasty. It may well be that by using nonsensical premisses, in addition to a set A of meaningful ones, we may be able to deduce a set B of meaningful sentences which are not deducible from A alone. I shall show how to deduce the false but meaningful sentence. "The moon is made of green cheese" from the nonsensical sentence "This sentence is false." Let us represent the sentence "This sentence is false" by the symbol "S" for short.
6) 불확실한 무의미를 알아보기 위한 기술이 상당히 가치있다는 것은 분명하고, 나는 그러한 기술의 발전과 적용이 최소한 철학적 과업의 일부임을 동의할 것이다. 어떻게 이것이 대략적인 개괄의 발전으로서 철학에 대한 나의 관점과 연결되는가? 무의미의 제거는 우리의 세계관에 변화를 가져올 수 있는가? 얼핏 봐서 이에 대한 대답은 부정적이다. 만일 무의미가 정말로 무의미하다면 그것은 세계관의 일부를 구성할 수 없고, 틀린(거짓) 세계관의 일부조차 구성할 수 없다. 그래서 비록 무의미의 제거가 죽은 나무를 제거하는 것으로 보일지라도 그것은 우리들의 지식에 있어서 살아있는 가지들에 영향을 미치지 않는다. 그렇지만 이러한 대답은 성급하다. 의미있는 것들의 집합 A에 추가함에 있어서 무의미한 전제들을 사용함으로써, 우리는 A만으로부터는 연역될 수 없는 의미있는 문장들의 집합 B를 연역할 수 있게 될지 모른다는 것은 아마도 좋은 것일 터이다. 나는 어떻게 거짓이지만 의미있는 문장을 연역할 수 있는지 보이고자 한다. 무의미한 문장 "이 문장은 거짓이다"로부터 "달은 녹색 치즈로 만들어진다"를 연역한다. "이 문장은 거짓이다"를 기호 "S"로 간략하게 표현하도록 하자.

7) From "This sentence is false" we can deduce "This sentence is not false." That is, from S we can deduce not-S. However, from S we can deduce "S or the moon is made of green cheese." But not-S (which we have already deduced) together with "S or the moon is made of green cheese" enables us to deduce "The moon is made of green cheese."
7) "이 문장은 거짓이다"로부터 우리는 "이 문장은 거짓이 아니다"라는 문장을 연역할 수 있다. 즉, S로부터 우리는 not-S를 연역할 수 있다. 그렇지만, S로부터 우리는 "S이거나 달은 푸른 치즈로 만들어진다"를 연역할 수 있다. 하지만 not-S(우리가 이미 연역하였던)는 "S이거나 달은 푸른 치즈로 만들어진다"와 더불어 우리가 "달은 푸른 치즈로 만들어진다"를 연역할 수 있도록 한다.

8) Thus given the nonsense "This sentence is false" we can deduce that the moon is made of green cheese. We have been able to do this because the nonsense in question issues in a contradiction, and from a contradiction we can, by the method of the last paragraph, deduce any sentence whatever. It is not obvious, however, that all nonsensical sentences issue in a contradiction. Some seem so far off the rails of meaningful discourse that it is not even possible to use them to demonstrate their own senselessness. Thus, it is not obvious that "I married a prime number" or "A bodiless grin appeared in the room" issue explicitly in contradiction. Nevertheless, my derivation of the proposition that the moon is made of green cheese should make it plausible that a philosopher should be able to deduce false conclusions from true premisses if he makes his deduction through unrecognised nonsense. The deduction would, of course, be an incorrect one, but it would be incorrect in a very unobvious and subtle way. The detection of its incorrectness would depend on the detection of hidden nonsense. A good example of this sort of thing, in the history of philosophy, suggested to me by D. M. Armstrong, is perhaps Aristotle's deduction of the false, though meaningful, proposition that the heavenly bodies are of a different is by way of the nonsense that the heavenly bodies obey laws of the same nature as the laws of logic, i.e., laws of a sort of logical hardness.
8) 그래서 "이 문장은 거짓이다"라는 무의미가 주어지면 우리는 달이 푸른 치즈로 만들어진다는 것을 연역할 수 있다. 우리는 이것을 할 수 있었는데 모순에 있어서의 현안문제들에 있어서 그 무의미가, 그리고 하나의 모순으로부터 우리가 마지막 문단의 방법을 통해 어떤 무엇이든 문장을 연역할 수 있기 때문이다. 그렇지만 모든 무의미한 문장들이 모순에 있어서 문제를 제기한다는 것은 확실하지 않다. 일부는 그것들을 그것들의 고유한 무의미성을 증명하기 위해 사용하는 것조차 가능하지 않다는 의미있는 논의의 경로에서 꽤나 멀리 떨어져 있어 보인다. 그래서, "나는 소수와 결혼했다"나 "몸뚱이 없는 미소가 방안에 나타났다"가 분명하게 모순에 있어서 문제를 제기한다는 것은 분명하지 않다. 그럼에도 불구하고, 달은 푸른 치즈로 만들어진다라는 명제에 대한 나의 유도는 철학자가 거짓 결론들을 참된 전제들로부터 만일 그가 그의 연역을 자각되지 않은 무의미를 통해 이룬다면 연역할 수 있어야 한다는 것을 타당하게 만들어야 할 것이다. 그 연역은 물론 부정확한 한 가지의 것일 터이지만, 그것은 매우 불확실하고 미묘한 방식으로 부정확한 것일 터이다. 그것의 부정확성에 대한 연역은 숨은 무의미의 연역에 의존할 것이다. 철학사에 있어서 이러한 종류의 것에 대한 좋은 예시는 D. M. 암스트롱에 의해 나에게 제시된 것으로, 아마도 설령 유의미할지라도 거짓인 '천체들은 지구의 실체와는 다른 실체에 속한다'라는 명제에 대한 아리스토텔레스의 연역일 것이다. 그의 연역은 천체들이 논리법칙들로서, 즉 일종의 논리적 견고성의 법칙들로서 동일한 자연법칙들을 따른다는 무의미에 속하는 방식에 의한 것이다.

9) It may be thought that my example of a deduction that the moon is made of green cheese proves too much. For if it proves anything it proves that from "This sentence is false" (or from a simple nonparadoxical contradiction such as "2+2=5") we could deduce anything whatever. But philosophers, however metaphysical they may be, are not satisfied to assert any proposition whatever. There are some propositions which they wish to assert and there are other propositions which they wish to deny. A system of thought which harboured a contradiction would, on the contrary, degenerate into the happy assertion of anything whatever. To this objection we must reply that in practice a system will degenerate in this way only if the contradiction is detected. If the contradiction is not detected it cannot in practice provide a route for the deduction of any proposition whatever. It is like a way out of prison which is quite unkown to the prisoners: as far as they are concerned it might as well not exist, and the bolts and bars do not lose any of their effectiveness.
9) 달은 푸른 치즈로 만들어진다는 연역에 대한 나의 예시가 감당할 수 없다고 생각될지도 모른다. 왜냐하면 만일 그것이 어떤 것이든 증명한다면 그것은 "이 문장은 거짓이다"로부터 (혹은 "2+2=5"와 같은 단순한 비역설적 모순으로부터) 우리가 어떤 무엇이든 연역할 수 있으리란 것을 증명할 터이기 때문이다. 그러나 철학자들, 하지만 형이상학적일 그들은 어떤 무슨 명제든 주장하는 것으로 만족하지 않는다. 그들이 주장하길 바라는 일부 명제들이 있고 그들이 거부하고자 하는 다른 명제들이 있다. 하나의 모순을 품었던 사유의 체계는 반대로 어떤 아무것에 대한 행복한 단언으로 퇴락할 것이다. 이러한 반론에 대해 우리는 실질적으로 하나의 체계는 만일 모순이 발견된다면 오로지 그러한 경우에만 이러한 방식으로 퇴락할 것이라고 응답해야만 한다. 그것은 감옥의 수감자들에게 전혀 알려지지 않은 그러한 탈출구와 같은 것이다. 그들이 그것을 전혀 존재하지 않는 것이리라 우려하는 한, 그리고 빗장들과 쇠창살들이 그 힘의 어느 것 하나 잃지 않는 한.

10) I conclude therefore that it must not be supposed that the view that philosophy consists only in the elimination of nonsense implies the proposition that philosophy has no effect on our world view. It may cause us to shed some of our beliefs about the world because it may enable us to see that we have accepted these beliefs only on the strength of a fallacious deduction through a nonsensical part of language. This conclusion is far stronger than another one, which is conceded by most philosophers, that the elimination of nonsense leads to clarity of thought and so helps the progress of the sciences.
10) 나는 그러므로 철학이 오로지 무의미의 제거로만 이루어진다는 관점이 철학은 우리의 세계관에 아무런 영향도 미치지 않는다는 명제를 함축한다고 가정되어서는 결코 안 된다고 결론내린다. 그것은 아마도 우리에게서 세계에 대한 우리의 믿음들 중 일부를 없앨 터인데 왜냐하면 그것이 아마도 우리로 하여금 우리가 이러한 믿음들을 오로지 잘못된 연역의 힘에서 언어의 무의미한 부분을 통해서만 받아들였음을 볼 수 있도록 할 것이기 때문이다. 이러한 결론은 대부분의 철학자들에 의해 인정되는 다른 결론, 무의미의 제거가 사유의 분명함으로 이끌고 그래서 학문들의 진보를 돕는다는 결론보다 너무 강하다.


Philosophy as More Than the Elimination of Nonsense
무의미의 제거 그 이상의 것으로서 철학

1) That philosophy is at least the elimination of nonsense and the clarification of thought is something of which I have not the least doubt. However, I should also wish to argue that philosophy is more than this, and that it is the business of the philosopher to decide between various synoptic hypotheses on grounds of plausibility. Of course, scientists have to decide between hypotheses, and with a slight over-simplification we may say that they do so by means of observation and experiment. It may be, however, that no available method of experiment and observation will decide between two hypotheses. The philosopher may legitimately, I think, feel it within his province to speculate on the relative palusibilities of the two hypotheses if they are of such generality and importance as to affect our overall world view. For example, in the sequel I shall be concerned to argue for the palusibility of the view that the human brain is no more than a physical mechanism, that no vitalistic or purely psychical entities or laws are needed to account for its operations. This type of philosophical thinking links up closely with the purely clarificatory sort of philosophy, since part of my strategy will be to try to expose confusions in a priori philosophical arguments for the opposite hypothesis. Of course, those who produce such a priori arguments will probably deny that what they are arguing for is a "hypothesis": they will hold that their view is true as a matter of logic, just as a mathematical proposition perhaps is. I shall, however, indicate why I think that such philosophers are too sanguine in regarding philosophy as pure logic.
1) 철학이 최소한 무의미의 제거이며 사고의 명증화라는 것은 내가 작은 의심도 품지 않는 어떤 것이다. 그렇지만, 나는 또한 철학이 이 이상의 것이라고 주장하고자 하며, 타당성에 근거하는 다양한 개략적인 가정들 사이에서 결정을 내리는 일이 철학의 과업이라고 주장하고자 한다. 물론 과학자들은 가설들 사이에서 결정을 내려야 하고, 우리는 약간 과도한-단순화로써 그들이 관찰과 실험의 방법으로 그러한 일을 한다고 말할는지 모른다. 그렇지만 실험과 관찰의 그 어떠한 가용한 방법도 두 가설들 사이에서 결정을 내리지 못할 것이다. 철학자는 만일 그 가설들이 우리의 전반적인 세계관에 영향을 미치는 것으로서 그러한 일반성과 중요성을 가진다면 내가 생각하기에는 정당하게 두 가설들의 상대적인 타당성들에 대해 추정하는 그의 분야에 있어서 그것(실험과 관찰이 결정내리지 못한다는 것)을 느낄 것이다. 예를 들어, 이어지는 곳에서 나는 인간 뇌가 물리적 기계장치 이상의 아무것도 아니라고 주장하기를 바랄 것이고, 어떠한 활력론적이거나 순수하게 심리학적인 실체들 또는 법칙들이 그 두뇌의 활동들을 설명하기 위해 필요하지 않다고 주장하기를 바랄 것이다. 이러한 유형의 철학적 생각은 순수하게 명확화하는 종류의 철학과 밀접하게 관련을 맺는데, 왜냐하면 나의 전략 중 일부가 반대가설들을 위한 선험적인 철학적 논증들에서의 혼동들을 노출시키고자 시도하는 것이 될 것이기 때문이다. 물론 그러한 선험적 논증들을 산출해내는 사람들은 아마도 그들이 그를 위해 논증하고 있는 어떤 것이 "가설"이라는 점을 부정할 것이다. 그들은 그들의 관점이 논리적 문제로서 참이라고 주장할 것인데, 바로 수학적 명제들이 아마도 그러한 것과 마찬가지이다. 그렇지만 나는 어째서 내가 그러한 철학자들은 철학을 순수 논리학으로 간주함에 있어서 지나치게 낙관적이라고 생각하는지 밝힐 것이다.

2) A philosopher might have to decide betweeen two hypotheses for which there not only is no available empirical test but for which there could be no possible empirical test. I shall illustrate this point by reference to the hypothesis that the universe began to exist ten minutes before I began writing this sentence, but with everything just as it was ten minutes ago. (Fossils in the rocks, photographs in the pocket, memory traces in the brain, light rays in interstellar space, and so on.) Of course this is not a hypothesis which any philosopher is likely to hold, though the English naturalist and biblical theologian Philip Gosse produced a very similar theory in order to reconcile geology and the book of Genesis. Gosse held that the world was created only a few thousand years ago, exactly as stated in the book of Genesis, but that God had also created the various eroded canyons, fossils as if of prehistoric animals and plants, and so on. In short, he held that the world was created a few thousand years ago just as in fact it was (on the usual geological and evolutionary account) at that time. Clearly Gosse's theory was immune to empirical refutation, and he was extremely pained when both the scientific world and the theological world spurned his ingenious reconciliation. Nevertheless, though it is not a live philosophical theory, the hypothesis that the world began ten minutes ago, just as it was ten minutes ago, will serve to illustrate my methodological point. It is clear that no experiment or observation could upset the hypothesis that the world began ten minutes ago just as it was ten minutes ago. If I mention our memories of last week's football match the reply will be that these are not true memories: the football match never happened, but we came into existence ten minutes ago complete with pseudo-memories of the non-existent game. If I point to newspaper photographs of the football match the reply will be that the newspaper, complete with photographs, itself began to exist ten minutes ago. And so on.
2) 어떤 철학자는 그를 위한 아무런 가용한 경험적 시험도 없을 뿐만 아니라 그를 위한 아무런 가능한 경험적 시험도 있을 수 없는 그러한 두 가설들 사이에서 결정을 내려야 할지 모른다. 나는 우주가 이 문장을 쓰기 시작하기 10분 전에, 하지만 그 우주가 10분 전에 그러하였을 것이었던 것과 똑같이 모든 것들을 가지고 존재하기 시작했다는 가설을 언급함으로써 이 점을 묘사하고자 한다. (암석들 속의 화석들, 주머니 속 사진들, 뇌 속 기억의 흔적들, 행성간 공간에 광선들 등.) 물론 이것은 어떤 철학자도 주장할 것 같지 않은 가설이다. 설령 잉글랜드 박물학자이자 성서신학자인 필립 고쓰가 매우 유사한 이론을 지질학과 창세기를 조화시키기 위해 내놓았었다고 할지라도 말이다. 고쓰는 세계가 단지 몇 천 년 전에, 정확하게 창세기에 쓰여졌듯이 창조되었을 뿐이며, 그러나 신은 또한 풍화침식된 다양한 협곡들, 선사시대 동식물들의 것과 같은 화석들 따위의 것들도 창조했다고 주장했다. 간단히 말해서, 그는 세계가 몇 천 년 전 (통상의 지질학적이고 진화론적인 설명에 기초해) 실제로 그 당시에 그러하였을 것과 똑같이 창조되었다고 주장했다. 명백하게 고쓰의 이론은 경험적 반박으로부터 면제되었고, 그는 과학적 세계와 신학적 세계 모두가 그의 기발한 조화를 일축시켰을 때 극심하게 고통받았다. 그럼에도 불구하고, 비록 그것이 살아있는 철학적 이론이 아니라고 할지라도, 세계가 10분 전에 그러하였을 바로 그 세계와 똑같이 10분 전에 시작했다는 그 가설은 나의 방법론적 핵심을 묘사하는 데에 도움이 될 것이다. 그 어떠한 실험이나 관찰도 세계가 10분 전에 그것이 그러하였을 바로 그 모습으로 10분 전에 시작하였다는 그 가설을 뒤엎을 수 없을 것이라는 점은 분명하다. 만일 내가 우리의 지난 주 풋볼 경기에 대한 기억들을 언급한다면 그 응답은 이러한 기억들이 참된 기억들이 아니라는 것일 터이다. 그 풋볼 경기는 결코 일어났던 적이 없고, 그러나 우리는 비-실재인 경기에 대한 가상-기억들을 완비한 채 10분 전에 현존하기 시작했다는 것이다. 만일 내가 그 풋볼 경기에 대한 신문 사진들을 지적한다면 그 대답은 그 신문이 사진들을 갖춘 채 그 자체로 10분 전에 존재하기 시작했다는 것일 터이다. 뭐 그 따위 응답들이 돌아올 것이다.

3) Some philosophers would say that since there could be no experimental or observational way of deciding the question whether or not the world came into existence ten minutes ago just as it was ten minutes ago, the assertion or denial that the world began ten minutes ago is without sense. This seems to me to be unplausible. There seems to be nothing contradictory in the notion of a world suddenly springing into existence in this way. Moreover, suppose that I am suffering from an intense toothache. I should not take kindly to the view that in a year's time there would be no meaningful difference between the hypothesis that the world exists now, complete with my toothache, and the hypothesis that the world will spring into existence next year, just as it will be next year, with pseudo-traces, such as memories and empty gums, as if of my present toothache.
3) 일부 철학자들은 세계가 10분 전에 그러할 바로 그와 같이 10분 전에 존재하게 되었는지 아닌지에 대한 물음을 결론내리기 위한 어떠한 실험이나 관찰의 방법도 있을 수 없기 때문에, 세계가 10분 전에 있게 되었다는 것에 대한 긍정이나 부정은 의미가 없다고 말할 것이다. 내가 보기에 이것은 부당하다. 세계가 이런 식으로 갑작스럽게 뿅하고 존재하게 되었다는 관념에 있어서 아무런 모순도 없는 듯이 보인다. 더 나아가서, 내가 극심한 치통으로 고통받고 있다고 가정하자. 한 해 동안 나는 세계가 나의 치통을 갖춘 채로 지금 현존한다는 가설과 세계가 그것이 다음 해에 그러한 바로 그 모습으로 기억들과 빈 잇몸들, 마치 지금의 내 치통처럼 가상-흔적들을 가지고 다음 해에 갑자기 튀어나와 존재할 것이라는 가설 사이에 아무런 유의미한 차이도 없으리라는 관점에 호의적이지 않을 것이다.

4) It is hard, however, without losing all sense of reality, to deny that the hypothesis that the world began ten minutes ago just as it was ten minutes ago is a meaningful one. (Though an unbelievable one.) Indeed, though there are no possible observations or experiments which could distinguish between this hypothesis and the more usual one, there are considerations, hard though they may be to formulate, of simplicity and palusibility, which should determine us to reject the "ten minutes ago" hypothesis. For this hypothesis presents us with a cosmology depending on a highly complex and arbitrary set of initial conditions. If the "ten minutes ago" hypothesis is accepted, then we have to take as a brute fact, for which no explanation could possibly be given, that ten minutes ago there were certain footprints on the beach at Glenelg, South Australia, that there were certain light waves in the depths of intergalactic space, that there were certain definite "photographs" in my breast pocket, that there were certain types of pseudo-prehistoric bones in the rock strata. All these facts would have to be taken as just "flat" and in principle inexplicable. Now it is true that on any hypothesis there is an element of arbitrariness in nature. Why have we five fingers rather than four or six? Nevertheless this arbitrariness can be understood as due to the element of sheer accident involved in the large-scale non-accident of evolution by natural selection. This arbitrariness, and other sorts of arbitrariness, such as the occurrence of hard rocks here and soft rocks there, of blue stars here and red stars there, is, on the normal hypothesis that the world has existed for a very long time, much what we should expect. It would be surprising rather if everything were neat and orderly. But this sort of arbitrariness is not like the extraordinary and universal arbitrariness of the initial conditions which we find in the "ten minutes ago" hypothesis.
4) 그렇지만 모든 현실감을 잃지 않고 세계가 10분 전에 그것이 그러했을 바로 그 모습으로 10분 전에 시작되었다는 가설이 의미있는 것임을 부정하기란 어렵다. (설령 믿을 수 없는 것이라 할지라도 말이다.) 더욱이, 이 가설과 좀 더 일반적인 것 사이를 구별할 수 있을 가능한 관찰들이나 실험들이 아무것도 없다고 할지라도, 고려들이 있는데, 비록 그 고려들이 정식화되기 어려울지 모른다 하더라도, 우리로 하여금 "10분 전" 가설을 거부하도록 결정지을 단순성과 타당성에 대한 그러한 고려들이 있다. 이러한 가설은 우리에게 고도로 복잡하고 임의적인 일련의 초기조건들에 의존하는 우주론을 보여준다. 만일 "10분 전" 가설이 허용된다면, 우리는 그에 대한 아무런 설명도 가능하게 제시될 수 없는 맹목적 진실로서 10분 전에 남부 오스트레일리아 글레넬그 해변에 특정한 발자국들이 있었다는 것을, 은하간 공간의 한 가운데 특정한 광선들이 있었다는 것을, 내 가슴팍 주머니에 어떤 확실한 "사진들"이 있었다는 것을, 특정한 종류의 가상-선사 뼈들이 암석 지층들 속에 있었다는 것을 받아들여야 한다. 이러한 모든 사실들은 마치 '단호한' 것처럼 그리고 원칙적으로 설명될 수 없는 것처럼 취해져야 할 것이다. 이제 어떤 가설에 대해서든 본성적으로 임의적인 요소가 있음은 사실이다. 왜 우리는 넷이나 여섯이 아니라 다섯 손가락을 가지는가? 글머에도 불구하고 이러한 임의성은 자연선택에 의한 진화의 대규모 비-우연에 있어서 수반되는 순전한 우연의 요소 때문인 것으로 이해될 수 있다. 이러한 임의성, 그리고 여기에는 단단한 암석들이 있고 저기에는 부드러운 암석들이 있는 것과 같이, 여기에는 푸른 별들이 있고 저기에는 붉은 별들이 있다는 것처럼 다른 종류의 임의성은 세계가 매우 오랜 시간 동안 존재해 왔다는 평범한 가설에 대해서, 우리가 예상할 어떤 것 그 이상의 것들이다. 만일 모든 것이 질서정연하다면 그것이 오히려 놀라울 것이다. 그러나 이러한 종류의 임의성은 우리가 "10분 전" 가설에서 발견하는 초기조건들의 비범하고 보편적인 임의성과 같은 그런 것은 아니다.

5) The example of the hypothesis that the universe began to exist ten minutes ago seems to show that it is possible to choose, on grounds of plausibility, between two hypotheses between which there can be no empirical test. I shall myself consider one important case of this sort [in Philosophy and scientific Realism] when I shall argue for the view that our conscious experiences are not identical with brain processes but that they are correlated with brain processes. Here once more we have, as we shall see, two hypotheses between which no empirical test could decide. I shall argue on plausible grounds for the former (materialistic) hypothesis against the later (dualistic) hypothesis. Before I can do this I shall, of course, have to argue that certain a priori philosophical arguments against materialism are not as cogent as they seem at first sight to be. The plausible arguments I shall use are of various sorts, but one of these is worthy of specific mention. This is Occam's razor. It depends on the precept "Do not multiply entities beyond necessity." This is a familiar maxim not only of philosophical method but also of scientific method. For example, if biochemical reactions will explain a certain phenomenon of cell growth, then there is no need to postulate, in addition to the biochemical reactions which we know to occur anyway, a life force or some irreducibly biological law of nature. (Occam himself is popularly supposed to have applied his razor to the metaphysical problem of universals, though I gather that there is a good deal of doubt the historical accuracy of this.) It might turn out that in cases where we need to talk of universals, such as justice and whiteness, we could manage equally well by talking of the words "just" and "white." If we can think of words as marks on paper and the like (the trouble, of course, is that words, unlike particular inscriptions, themselves turn out to be universals), then we can effect an economy. For we know that ink marks on paper and the like occur anyway, and if they will do all the explanatory tasks that are needed we need not bring in the airy fairy and altogether dubious entities justice and whiteness.
5) 우주가 10분 전에 시작되었다는 가설의 예는 타당성에 기초하여 어떠한 경험적 시험도 가능하지 않은 두 가설들 사이에서 선택할 수 있다는 것을 보여주는 듯하다. 나는 스스로 이러한 종류의 한 가지 중요한 경우를 내가 우리의 의식 경험들이 뇌의 진행과 동일시되지 않지만 그 경험들은 뇌의 진행과정들과 관련된다고 주장하고자 할 때 고려할 것이다 [『철학과 과학적 사실주의』에서]. 우리는 여기에서 우리가 보게 될 것과 같이 한 번 더 어떠한 경험적 시험도 그 사이에서 결정할 수 없는 그러한 두 가지 가설들을 가진다. 나는 후자 (이원론적) 가설에 대해 전자 (유물론적) 가설을 위해 타당한 근거들에 대해서 주장할 것이다. 내가 이러한 것을 할 수 있기에 앞서서 나는 물론 특정한 선험적 철학논증들이 유물론에 반대해 그것들이 첫눈에 그렇게 보이는 만큼 그렇게 설득력있지는 않다는 것을 주장해야 할 것이다. 내가 사용할 그 타당한 논증들은 다양한 종류의 것들이지만, 이것들 중 하나는 특별히 언급할 가치가 있다. 이것은 오캄의 면도날이다. 그 면도날은 "필연성을 넘어서서 실체들을 증대시키지 말라"라는 수칙에 의존한다. 이것은 철학적 방법에 대해서만 친숙한 격언이 아니라 과학적 방법에 대해서도 그러한 것이다. 예를 들어, 만일 생물화학적 반응들이 세포 성장의 특정한 현상을 설명할 것이라면, 우리가 어쨌든 발생할 것을 알고 있는 생물화학적 반응들에 더하여 생명력이나 어떤 환원불가능한 생물학적 자연법칙을 상정할 필요는 전혀 없다. (오캄 자신은 일반적으로 그의 면도날을 보편자들에 대한 형이상학적 문제에 적용시켰던 것으로 생각된다. 비록 내가 이에 대한 역사적 정확성을 의심할 그럴싸한 이유가 있음을 알아냈다고 하더라도 말이다.) 우리가 올바름과 흼과 같은 보편자들에 대해 이야기할 필요가 있는 경우들에 우리가 "올바름"과 "흼"이라는 단어들을 취함으로써 똑같이 잘 다룰 수 있으리라는 것은 아마도 드러날 것이다. 만일 우리가 종이에 적힌 그리고 그와 같은 것으로서 단어들에 대해 생각할 수 있다면(물론 특별히 적힌 글자들과 달리 단어들 그 자체는 보편적인 것으로 드러난다는 것은 문제이다), 우리는 경제적인 효과를 낼 수 있다. 왜냐하면 우리는 종이 위의 잉크 표시들과 어떻게든 일어나는 그와 같은 것들을 알고, 만일 그것들이 필요한 모든 설명작업들을 해낼 것이라면 우리는 애매하고(비현실적이고) 죄다 미심쩍은 실체들에서 올바름과 흼을 가져올 필요가 없다.

6) I suspect that considerations of plausibility, turning on the notions of simplicity and arbitrariness, of Occam's razor and the like, have an important and indeed indispensable place in philosophical argument. This is partly because philosophy is carried out in a natural language, not in some artificial language, with rigid formation and transformation rules explicitly laid down as in a formal logical or mathematical system. Hence, though it is often possible to persuade another philosopher that he has landed himself in inconsistency or in nonsense, and that he must therefore give up certain of his tenets, it is never possible to prove this to him. He can always patch up the inconsistencies and nonsenses in his language by means of supplementary rules and hypotheses. We shall have to present an alternative in chapter 6 [of Philosophy and Scientific Realism] to the so-called libertarian theory of free-will. At first sight this theory is easy to refute, for the libertarian seems to hold that acting freely is something intermediate between being determined and acting by pure chance. Logic would seem to leave no room for such an intermediate possibility. The libertarian will reply that if I define "pure chance" as "not being determined," then his "acting freely" is a sub-species of what I call "pure chance." This sub-species is not properly pure chance, but consists in acting from reasons, not from causes. I then reply to the libertarian with the stock philosophical arguments showing that reasons are not a sort of para-cause and that acting from reasons is not incompatible with acting from causes. The obdurate libertarian is sure to prepare yet another line of defence and get round this objection in some way. (As I well know from inconclusive phiosophical discussions on this topic.)
6) 나는 타당성에 대한 고려들이 오캄의 면도날과 혹은 그와 같은 것들의 단순성과 임의성의 관념들을 중심으로 철학적 논증에 있어서 중요하고 더욱이 불가결한 자리를 차지하리라 짐작한다. 이는 부분적으로 철학이 형식논리 또는 수학적 체계에서와 같이 분명하게 놓인 엄격한 구조와 전환규칙들을 가지고 어떤 인공언어에서 수행되지 않고 자연언어에서 수행되기 때문이다. 그러므로, 비록 다른 철학자에게 그가 모순이나 무의미에 빠졌다고, 그리고 그는 그러므로 그의 근본원리들 중 특정한 것을 포기해야만 한다고 설득하는 일이 종종 가능하다고 하더라도, 이것을 그에게 증명하는 것은 절대로 불가능하다. 그는 언제나 그의 언어에 있어서 모순들과 무의미들을 보조규칙들과 가설들을 수단으로 삼아 대충 기워낼 수 있다. 우리는 [『철학과 과학적 사실주의』의] 6장에서 소위 자유의지에 대한 자유의지론이라는 것에 대한 하나의 대안을 표명해야 할 것이다. 딱 봐서 이것은 반박하기 쉬운 이론인데, 왜냐하면 자유의지론자는 자유롭게 행위하는 것이 결정된 것과 순전히 우연한 행동 사이의 어떤 중간인 것이라 주장하는 것처럼 보이기 때문이다. 논리학은 그러한 중간적 가능성을 위한 아무런 여지도 남겨두지 않는 것으로 보일 것이다. 자유의지론자는 만일 내가 "순전한 우연"을 "결정되지 않은 것"으로 정의한다면 그의 "자유롭게 행위함"은 내가 "순전한 우연"이라 부르는 것의 아종(변종)일 것이라고 응답할 것이다. 이 아종은 온전히 순전한 우연은 아니지만, 이유들로부터의 행위로 이루어지고 원인들로부터의 행위로 이루어지진 않는다. 그러면 나는 그 자유의지론자에게 이유들이 일종의 준-원인들이 아니며 그 이유들로부터의 행위는 원인들로부터의 행위와 양립불가능한 것이 아님을 보여주는 상투적인 철학적 논증들을 가지고 답한다. 완고한 자유의지론자는 아직 방어의 다른 노선이 예비되어 있고 이 반대를 어떤 식으로 해결할 수 있음에 틀림없다고 확신한다. (내가 이 문제에 대한 결정적이지 못한 철학적 논의들로부터 잘 알고 있듯이 말이다.)

7) This characteristic inconclusiveness of philosophical argument is a fact familiar to all philosophers. If they were to take it seriously more of them would be favourably disposed to my conception of philosophy as in part depending on merely plausible considerations. If a philosopher keeps on patching up his theory we may try to persuade him that his way of talking is becoming more and more baroque and is ill-fitting to our scientific knowledge. The libertarian philosopher of free-will may, if he is ingenious enough, render himself immune to our logical arguments, but only at the cost of great artificiality in his theory, and at the price of bringing in a great discontinuity in the story of animal evolution. Just where in the line of evolution, the primates, or sub-men, or early men, does this "soul," or power of free choice in the libertarian sense, become superadded to man as he appears in the usual biological story? It would, moreover, have to be very special creation: it is impossible that the evolution of such a metaphysical entity could be explained in the usual mechanistic terms, natural selection acting on gene pools (a gene being a complex nucleic acid). Of course, if the philosopher is happy with the broque quality of his theory and with its artificiality of fit with total science, then there is no more to be done. In many cases, however, plausible considerations of the sort I have suggested may have a persuasive force that purely abstract considerations of consistency and the like may not have. With ingenuity these last can be got round, but if the methods of getting round them have to be supplemented every century, or every decade, in order to take account of advances in science, then it will be a very romantically minded philosopher who will not begin to feel uneasy.
7) 철학적 논증에 대한 이러한 특징적인 결정적이지 못함은 모든 철학자들에게 친숙한 하나의 사실이다. 만일 그들이 그것을 보다 더 진지하게 취할 것이라면 그들은 단지 타당한 고려들에 의존하는 부분으로서 철학에 대한 나의 이해에 호의적으로 이끌리게 될 것이다. 만일 한 철학자가 그의 이론을 기워내기를 고수한다면 우리는 그에게 그의 화법이 점점 더 복잡해지고 있으며 우리의 과학적 지식에 안 맞는다는 것을 설득하고자 노력할 것이다. 그 자유의지에 대한 자유의지론 철학자는 아마도, 만일 그가 충분히 기발하다면, 우리의 논리적 논증들로부터 면제되도록 만들 것인데, 그의 이론에 있어서 그 대단한 작위성의 대가뿐만 아니라 동물 진화론에 있어서의 엄청난 단절의 값까지 치르면서 그러할 것이다. 진화의 선상에서 대체 어디에서 영장류들, 혹은 하등-인간, 또는 초기 인간이 이러한 "영혼" 또는 자유의지론적 의미에서 자유로운 선택의 능력을 통상적 생물학적 논의에 있어서 그가 그렇게 드러나듯 인간에 덧붙이기 시작했는가? 더욱이 그것은 매우 특별한 창조여야 한다. 그러한 형이상학적 실체의 진화가 통상의 기계론적 용어들로 설명될 수 있다는 것은 불가능하기 때문이다. 유전자 풀들(복잡한 핵산인 하나의 유전자 존재)에 대한 자연 선택의 작용 같은 용어 말이다. 물론 만일 그 철학자가 그의 이론의 복잡성과 총체적 과학과의 적합도에 대한 작위성으로 만족한다면, 더 이상 할 일은 없다. 많은 경우들에 있어서 그렇지만 내가 제안했던 그러한 종류의 타당한 고려들은 아마도 일관성에 대한 순수하게 추상적인 고려들을 설득하는 힘을 가질 것이고 그 유사한 것들은 가지지 않을 것이다. 기발함을 가지고 이러한 문제들이 끝내 해결될 수 있지만, 만일 그 문제들을 해결하는 방법들이 매 세기, 혹은 매 세대 과학에서의 진보에 대한 설명을 취하기 위해 보충되어야만 한다면, 그것은 매우 낭만적인 정신을 가진, 불안을 느끼기 시작하지 않을 철학자일 것이다.

Synoptic Philosophy and Man's Place in Nature
종관 철학과 자연에서 인간의 자리

1) If philosophy is concerned, in the manner suggested above, with the rational reconstruction of our conceptual scheme, then it quite obviously covers a very wide field. There will therefore be some important issues which, for the purposes of this [selection], I shall be content to leave to one side. For example, I shall not be concerned with the much-vexed question of Platonism versus nominalism, that is, whether in addition to the concrete objects or events which exist in space and time we must postulate abstract objects as well. For example, do mathematicians assert the reality of numbers and classes? The two parts of the previous question can indeed be amalgamated if we accept Frege's and Russell's analysis of numbers as classes of classes of objects. In any case, in higher mathematics it is essential to introduce infinite classes, i.e., classes of numbers. That a class is an abstract object can most easily be seen if we consider the null class, which can be described e.g., as the class of twentieth-century terrestrial unicorns. The null class is a perfectly good class, and because it has no mambers there is no temptation to confuse it with the "heap" of its members. A class, unlike a heap, has a number. Consider the class of students who are in this room at a certain moment. It has, say, ten members. Contrast the spatially scattered "heap" of human protoplasm in this room. This has no number. It is made up of ten persons and 1015 livings cells and goodness knows how many molecules or atoms. Now science, since it includes mathematics, apparently has to mention classes. Does this mean that we must accept classes as real things postulated by science, on a par, perhaps, with electrons or the far side of the moon? The reason why I shall not discuss this issue of Platonism versus nominalism, or of the reality of abstract entities, is that it has little relevance to the question of man's place in the universe. The connecting theme of this book will be the attack on anthropocentric or near-anthropocentric strains of thought in philosophy. I shall attack phenomenalist and subjectivist theories of mind and matter, space and time. The question of whether the universe contains Platonic entities is neutral to these issues.
1) 만일 철학이 위에서 제안된 방식으로 우리의 개념적 도식에 대한 이성적 재구축에 관계한다면, 철학은 상당히 분명하게 매우 넓은 영역을 포괄할 것이다. 그러므로 이[선택]를 위하여 내가 기꺼이 한 측면을 남겨둘 다소 중요한 화두가 있을 것이다. 예를 들어, 나는 플라톤주의대(對) 유명론의 매우 골치아픈 물음, 즉 시간과 공간에 현존하는 구체적인 대상들이나 사건들에 덧붙여서 우리가 추상적인 대상들 또한 상정해야만 하는지의 물음에는 관여하지 않을 것이다. 예를 들어, 수학자들은 수들과 집합들의 실재성을 확언하는가? 앞선 물음의 두 부분들은 만일 우리가 대상들의 집합들의 집합들로서 수들에 대한 프레게와 러셀의 분석을 받아들인다면 실로 병합될 수 있다. 어떤 경우든, 고등 수학에서 유한 집합들, 즉 수들의 집합들을 도입하는 것은 본질적이다. 하나의 집합이 하나의 추상적 대상이라는 것은 만일 우리가 공집합을 고려한다면 가장 쉽게 보여질 수 있다. 공집합은 예를 들어 20세기 지상의 일각수들의 집합과 같은 것으로 묘사될 수 있다. 그 공집합은 완벽하게 훌륭한 집합이고, 그것이 아무런 원소들도 가지지 않기 때문에 그것과 그것의 원소들의 "더미"를 혼동하도록 하는 아무런 유혹거리도 없다. 더미와 달리 하나의 집합은 하나의 수를 가진다. 특정한 순간 이 방에 있는 학생들의 집합을 고려해 보라. 그 집합은 말하자면 10개의 원소들을 지닌다. 반면에 공간에 산재된 이 방 안의 인간 원형질 "더미"를 고려해 보라. 이 더미는 아무런 수도 가지고 있지 않다. 그것은 열 명의 사람들과 10의 15제곱에 달하는 살아있는 세포들과 가히 많은 분자들이나 원자들인 영양분으로 이루어진다. 이제 과학은 수학을 포함하기 때문에 외관상 집합들을 언급해야 할 듯하다. 이것은 우리가 집합들을 과학에 의해 상정된 실재적인 것들로 받아들여야만 한다는 것을 의미하는가, 아마도 전자들과 동등하게 혹은 달의 이면과 같이? 내가 플라톤주의와 유명론 사이의, 또는 추상적 실체들의 실재성에 대한 이 문제를 논의하지 않으려는 이유는 그 논의가 우주에서 인간의 자리에 대한 물음에 거의 관련이 없기 때문이다. 이 책의 연속적인 주제는 철학에 있어서 사유의 인간중심주의적인 또는 그에 가까운 압력들에 대한 공격이 될 것이다. 나는 정신과 물질, 공간과 시간에 대한 현상론적이고 주관주의적인 이론들을 공격할 것이다. 우주가 플라톤적 실체들을 포함하는지에 대한 물음은 이러한 문제들에 대해 중립적이다.

2) Of course, the days when man was thought to be physically at the centre of the universe are long over, but as I shall try to show, a disguised anthropocentricity still prevails in many fields of philosophy. For example, traditional phenomenalism in a sense puts this great world of nature "inside sentience": indeed, as we shall see, phenomenalism has appealed to no less a man than F. P. Ramsey on account of its apparent power to tame the vast astronomical spaces that threatened to overawe him. Moreover, I shall be concerned to refute the more recent phenomenalism, not of tables and chairs but of electrons and protons, which has attempted to deny the full-blooded reality of the sub-microscopic world, i.e., of objects of an order of magnitude very much smaller than those of macroscopic or roughly man-sized objects. In later chapters I shall attack anthropocentricity in prevailing theories of the secondary qualities and of consciousness, and I shall be concerned to put man in his place by defending the view that he is nothing more than a complicated physical mechanism. The ground for this part of the book is prepared by a chapter on the relations between physics and biology. The chapter on space and time might be thought to be outside the scope of the book, but the reader will, if he preseveres, discover that even in such notions as of past, present, and future there is a concealed anthropocentricity. In the final chapter there is a discussion of the relevance and lack of relevance of a materialistic metaphysics for ethics.
2) 물론 인간이 물리적으로 우주의 중심에 있는 것으로 생각된 시기는 오래되었지만, 내가 보여주고자 하는 것과 같이 가장된 인간중심성은 여전히 철학의 여러 영역들에 만연해 있다. 예를 들어 전통적인 현상론은 어떤 의미에서 자연의 이 광대한 세계를 "지각의 내부에" 놓는다. 더구나 우리가 볼 것과 같이 현상주의는 프랑크 P. 램지 보다 조금더 덜하지 않은 사람의 관심을 끌었는데 그를 위압하도록 위협하는 막대한 천문학적 공간들을 다스리는 현상주의의 분명한 능력 때문이다. 더 나아가서 나는 더욱 최근의, 책걸상들에 대해서가 아니라 전자들과 양자들에 대해서, 그것들이 준-미시세계, 즉 거시적이거나 또는 거칠게 말해서 인간적-규모의 대상들의 규모보다 훨씬 더 작은 자릿수의 대상들의 완전한 실재성을 부정하고자 시도한 현상주의를 반박하는 데에 관심을 가질 것이다. 이후의 장에서 나는 부차적 성질들에 대한 이론들과 의식에 대한 이론들의 팽배에 있어서 인간중심주의를 공격할 것이고, 인간을 그가 복잡화된 물리적 기계장치 이외의 다른 아무것도 아니라는 관점을 옹호함으로써 그의 자리에 정위하는 데에 관심을 가질 것이다. 그 책의 이러한 부분의 근거는 물리학과 생물학의 관계들에 대한 장으로써 준비된다. 공간과 시간에 대한 장은 아마도 그 책의 범위 바깥에 있는 것으로 생각될 테지만, 독자는 만일 그가 잊지 않는다면 과거, 현재, 미래와 같은 그러한 것들에 대한 그런 관념들에서조차 감추어진 인간중심성이 있음을 발견할 것이다. 마지막 장에는 윤리학을 위한 유물론적 형이상학의 타당성과 타당성의 결핍에 대한 논의가 있다.

-蟲-
Part 1. Philosophy and the Meaning of Life

The Value of Philosophy

Bertrand Russell


1) Having now come to the end of our brief and very incomplete review of the problems of philosophy, it will be well to consider, in conclusion, what is the value of philosophy and why it ought to be studied. It is the more necessary to consider this question, in view of the fact that many men, under the influence of science or of practical affairs, are inclined to doubt whether philosophy is anything better than innocent but useless trifling, hair-splitting distinctions, and controversies on matters concerning which knowledge is impossible.
   지금 철학의 문제들에 대한 우리의 간단하고 매우 불완전한 검토의 막바지에 이르러서, 다음과 같은 것을 고려하는 것이 좋을 것이다. 결론적으로, 철학의 가치는 무엇이며 어째서 철학이 연구되어야 하는지 말이다. 많은 사람들이 과학이나 실천적 문제들의 영향 아래에서, 지식일 수 없는 어떤 것과 관련하는 문제들에 대한 악의는 없지만 쓸데없이 자질구레하고, 골치아픈 구별들, 그리고 논란들 이외에 하등 나을 것 없는 것은 아닌지 하는 의심을 하게 되는 경향이 있다는 사실을 고려하면, 이 물음을 고려하는 것은 더욱이 필연적이다.

2) This view of philosophy appears to result, partly from a wrong conception of the ends of life, partly from a wrong conception of the kind of goods which philosophy strives to achieve. Physical science, through the medium of inventions, is useful to innumerable people who are wholly ignorant of it; thus the study of physical science is to be recommended, not only, or primarily, because of the effect on the student, but rather because of the effect on mankind in general. This utility does not belong to philosophy. If the sutdy of philosophy has any value at all for others than students of phiosophy, it must be only indirectly, through its effects upon the lives of those who study it. It is in these effects, therefore, if anywhere, that the value of philosophy must be primarily sought.
   철학에 대한 이러한 관점은 부분적으로는 삶의 목적들에 대한 잘못된 이해로부터, 일부는 철학이 성취하고자 분투하는 종류의 가치들에 대한 오해로부터 귀결된다. 물리과학은, 발명들의 매개를 통하여, 그 과학에 전적으로 무지한 헤아릴 수 없이 많은 사람들에게 유용하다. 그래서 물리 과학에 대한 연구가 권장되어야 하는 것인데 오로지 그 학생들에 대한 영향 때문만이 아니라, 혹은 그것이 주된 것이 아니라 오히려 일반적으로 인류에 대한 영향 때문이다. 이러한 유용성은 철학에 속하지 않는다. 만일 철학에 대한 연구가 철학도들보다도 다른 이들에게 도대체 어떤 가치라도 가진다면, 그것은 분명 오로지 간접적일 것임에 틀림없는데, 그 철학을 연구하는 이들의 삶들에 대한 그 연구의 영향들을 통해서이다. 그러므로 기껏해야 철학의 가치가 우선적으로 추구되어야만 한다는 것은 이러한 영향들에 있어서이다.

3) But further, if we are not to fail in our endeavour to determine the value of philosophy, we must first free our minds from the prejudices of what are wrongly called "practical" men. The "practical" man, as this word is often used, is one who recognizes only material needs, who realizes that men must have food for the body, but is oblivious of the necessity of providing food for the mind. If all men were well off, if poverty and disease had been reduced to their lowest possible point, there would still remain much to be done to produce a valuable society; and even in the existing world the goods of the mind are at least as important as the goods of the body. It is exclusively among the goods of the mind that the value of philosophy is to be found; and only those who are not indifferent to these goods can be persuaded that the study of philosophy is not a waste of time.
   그러나 더 나아가서, 만일 우리가 철학의 가치를 결정하려는 우리의 시도에 있어서 실패하지 않는다면, 우리는 분명 처음으로 우리의 정신들을 "현실적" 사람들이라고 잘못 불리는 것에 대한 편견들로부터 자유롭게 할 것이다. 그 "현실적인" 사람이란, 이 말이 종종 그렇게 쓰이듯이, 오로지 물질적 필요만을 인식하는 사람, 사람들이 반드시 몸을 위한 음식을 취해야만 한다는 것을 아는 사람, 그러나 마음의 양식을 제공해야 할 필요성은 의식하지 못하는 사람 중의 하나이다. 만일 모든 사람들이 잘 산다면, 그리고 가난과 질병이 그 최저치로 경감된다면, 가치있는 사회를 산출하기 위해 행해져야 할 많은 남은 일들이 여전히 있을 것이다. 그리고 현존하는 세계에서조차 정신에 좋은 것들은 최소한 몸에 좋은 것들 만큼은 중요하다. 철학의 가치가 발견되어야 하는 것임은 정신에 좋은 것들 중에서 독보적이다. 그리고 이러한 좋음들에 무관심하지 않은 그러한 이들만이 철학에 대한 연구가 시간낭비가 아니라는 것을 납득할 수 있다.

4) Philosophy, like all other studies, aims primarily at knowledge. The knowledge it aims at is the kind of knowledge which gives unity and system to the body of the sciences, and the kind which results from a critical examination of the grounds of our convictions, prejudices, and beliefs. But it cannot be maintained that philosophy has had any very great measure of success in its attempts to provide definite answers to its questions. If you ask a mathematician, a mineralogist, a historian, or any other man of learning, what definite body of truths has been ascertained by his science, his answer will last as long as you are willing to listen. But if you put the same question to a philosopher, he will, if he is candid, have to confess that his study has not achieved positive results such as have been achieved by other sciences. It is true that this is partly accounted for by the fact that, as soon as definite knowledge concerning any subject becomes possible, this subject ceases to be called philosophy, and becomes a separate science. The whole study of the heavens, which now belongs to astronomy, was once included in philosophy; Newton's great work was called "the mathematical principles of natural philosophy." Similarly, the study of the human mind, which was a part of philosophy, has now been separated from philosophy and has become the science of psychology. Thus, to a great extent, the uncertainty questions which are already capable of definite answers are placed in the sciences, while those only to which, at present, no definite answer can be given, remain to form the residue which is called philosophy.
   철학은 다른 모든 연구들과 같이 무엇보다도 앎에 목표를 둔다. 철학이 목표로 하는 그 앎이란 학문들의 몸체에 통일성과 체계를 부여하는 종류의 앎이고, 우리의 의견들, 선입견들, 믿음들의 근거들에 대한 비판적 검토로부터 귀결하는 종류의 앎이다. 그러나 철학이 철학의 물음들에 확고한 답변들을 내놓고자 하는 시도들에 있어서 어떤 매우 훌륭한 성공의 척도를 가지고 있다고 주장될 수는 없다. 만일 당신이 수학자나 광물학자 또는 역사학자나 어떤 다른 학자에게든 그의 학문으로써 확인된 진리들의 어떤 명확한 핵심을 묻는다면, 그의 대답은 당신이 듣고자 하는 그만큼 길게 늘어질 것이다. 그러나 만일 당신이 동일한 질문을 철학자에게 한다면, 그는 만일 그가 솔직하다면 그의 연구가 다른 학문들에 의해 성취된 그런 긍정적인 결과들을 성취하지 못했노라고 고백하려 할 것이다. 이것이 부분적으로 다음과 같은 사실에 의해, 그러니까 어떤 주제에 관련한 확실한 앎이 가능해지면 가능해질 수록, 이 주제가 철학이라 불리우길 그치고, 분리된 하나의 학문이 된다는 사실로 설명된다는 것은 맞는 말이다. 이제는 천문학에 속하는 천체에 대한 전체 연구는 일찍이 철학에 포함됐었다. 뉴턴의 위대한 저작은 "자연 철학의 수학적 원리들"이라고 불렸다. 비슷하게, 철학의 일부였던 인간 정신에 대한 연구는 이제 철학으로부터 분리되었고 심리과학이 되었다. 그래서 크게 보자면 이미 확실한 답변들이 가능한 불확실한 물음들은 학문들(과학들) 안에 놓이고, 반면에 현재 어떠한 확실한 답변도 주어질 수 없는 그러한 물음들만이 철학이라 불리우는 잔여 유산의 꼴로 남는다.

5) This is, however, only a part of the truth concerning the uncertainty of philosophy. There are many questions - and among them those that are of the profoundest interest to our spiritual life - which, so far as we can see, must remain insoluble to the human intellect unless its powers become of quite a different order from what they are now. Has the universe any unity of plan or purpose, or is it a fortuitous concourse of atoms? Is consiousness a permanent part of the universe, giving hope of indefinite growth in wisdom, or is it a transitory accident on a small planet on which life must ultimately become impossible? Are good and evil of importance to the universe or only to man? Such questions are asked by philosophy, and variously answered by various philosophers. But it would seem that, whether answers be otherwise discoverable or not, the answers suggested by philosophy are none of them demonstrably true. Yet, however slight may be the hope of discovering an answer, it is part of the business of philosophy to continue the consideration of such questions, to make us aware of their importance, to examine all the approaches to them, and to keep alive that speculative interest in the universe which is apt to be killed by confining ourselves to definitely ascertainable knowledge.
    그렇지만 이것은 철학의 불확실성에 관련한 단편적인 진실일 따름이다. 우리가 볼 수 있는 한, 인간 지성의 능력들이 지금 그러한 것으로부터 상당히 다른 질서가 되지 않는 한 그러한 지성에게는 풀 수 없는 것으로 남을 수밖에 없는 많은 물음들이 있다. -그리고 그것들 중에는 우리의 정신적 삶에 대해 가장 심오한 관심에 속하는 것들도 있다-. 우주는 어떤 단일한 계획이나 의도를 지니는가, 아니면 원자들의 우연한 마당인가? 의식은 지혜에 있어서 무한한 성장의 희망을 주는 우주의 영속적인 일부인가, 아니면 그것은 궁극적으로는 생명이 불가능하게 될 수밖에 없는 작은 행성 위에서의 무상한 하나의 사건인가? 선과 악은 우주에 대해 중요한가 인간에게만 중요한가? 그런 물음들은 철학에 의해 던져지고, 다양한 철학자들에 의해 여러가지로 답변된다. 그러나 답변들이 다른 방식으로 발견될 수 있는지 없는지, 철학에 의해 제시된 그 답변들은 그것들 중에 어떤 것도 증명 가능한 참이 아닌 것처럼 보일 듯하다. 하지만, 그렇지만 조금이나마 하나의 답변을 발견할 희망은 있을 것이다, 그러한 물음들에 대한 숙고를 계속하는 것, 우리가 그러한 물음들의 중요성을 깨닫도록 만드는 것, 그 물음들에 대한 모든 접근들을 검토하는 것, 그리고 우리들 자신을 명확하게 확인 가능한 지식에 제한함으로써 사멸해 버리는 경향이 있는 세계에 대한 그러한 사변적 흥미를 살아있도록 유지하는 것은 철학적 과업의 일부이다.

6) Many philosophers, it is true, have held that philosophy could establish the truth of certain answers to such fundamental questions. They have supposed that what is of most importance in religious beliefs could be proved by strict demonstration to be true. In order to judge of such attempts, it is necessary to take a survey of human knowledge, and to form an opinion as to its methods and its limitations. On such a subject it would be unwise to pronounce dogmatically; but if the investigations of our previous chapters have not led us astray, we shall be compelled to renounce the hope of finding philosophy any definite set of answers to such questions. Hence, once more, the value of philosophy must not depend upon any supposed body of definitely ascertainable knowledge to be acquired by those who study it.
    여러 철학자들은 정말로 철학이 그러한 근본적 물음들에 대한 특정한 답변들의 진리를 확립시킬 수 있으리라 주장해 왔다. 그들은 종교적 믿음들에 있어서 가장 중요한 어떤 것이 엄격한 증명에 의해 참인 것으로 증명될 수 있으리라 생각했다. 그러한 시도들을 판단하기 위해서, 인간적 지식에 대해 조사할 필요가 있고, 그 방법들과 제한들에 관해서 하나의 견해를 형성할 필요가 있다. 그러한 하나의 주제에 대해서 독단적으로 표명하는 것은 현명하지 못한 일일 것이다. 그러나 만일 우리의 앞선 장들에 속하는 연구들이 우리를 잘못된 길로 이끌지 않았다면, 우리는 철학을 그러한 물음들에 대한 어떤 한정된 일련의 답변들로 발견할 희망을 포기하도록 강제될 것이다. 그러므로, 다시 한 번, 철학의 가치는 분명 철학을 연구한 이들에 의해 획득될 확실하게 확인될 수 있는 지식의 어떠한 가정된 몸체에도 의존하지 않음에 틀림없다.

7) The value of philosophy is, in fact, to be sought largely in its very uncertainty. The man who has no tincture of philosophy goes through life imprisoned in the prejudices derived from common sense, from the habitual beliefs of his age or his nation, and from convictions which have grown up in his mind without the co-operation or consent of his deliberate reason. To such a man the world tends to become definite, finite, obvious; common objects rouse no questions, and unfamiliar possibilities are contemptuously rejected. As soon as we begin to philosophize, on the contrary, we find, as we saw in our opening chapters, that even the most everyday things lead to problems to which only very incomplete answers can be given. Philosophy, though unable to tell us with certainty what is the true answer to the doubts which it raises, is able to suggest many possibilities which enlarge our thoughts and free them from the tyranny of custom. Thus, while diminishing our feeling of certainty as to what things are, it greatly increases our knowledge as to what they may be; it removes the somewhat arrogant dogmatism of those who have never travelled into the region of liberating doubt, and it keeps alive our sense of wonder by showing familiar things in an unfamiliar aspect.
    사실상 철학의 가치란 것은 주로 그것의 바로 그 불확실성에서 추구되어야 할 것이다. 철학에 대해 맛조차 본 일이 없는 사람은 일반 상식, 그의 나이가 혹은 그의 국가에 속하는 습관적 믿음들, 그리고 그의 신중한 이성의 협력이나 인가 없이 그의 마음 안에서 자라난 사견들로부터 유래한 편견들에 갇힌 삶을 살아간다. 그러한 사람에게 세계는 분명하고, 유한하며, 명백하게 되는 경향이 있다. 공통의 대상들은 아무런 의문점들도 일으키지 않고, 낯선 가능성들은 경멸적으로 거부당한다. 반대로 우리가 철학화하기 시작할 수록 우리는 우리가 여는 장들에서 보았던 것처럼 가장 일상적인 것들조차 오직 극히 불완전한 답변들만 주어질 수 있는 문제들로 이끈다는 것을 알아차린다. 철학은 철학이 일으키는 그러한 의심들에 대해 무엇이 참된 답인지 우리에게 확신을 가지고 말하도록 할 수 없을지라도 우리의 생각들을 확장시키고 그러한 생각들을 관습의 독재로부터 자유롭게 하는 여러 가능성들을 제시할 수 있다. 그래서, 사물들이 무엇인지에 대한 확실성에 대한 우리의 느낌을 약화시키는 동안, 그것은 그것들이 무엇일지에 대한 우리의 지식을 엄청나게 증대시킨다. 그것은 자유로운 의심의 영역으로 탐험해 본 적이 없는 그러한 이들의 어떤 오만한 독단주의를 제거시키고, 친숙한 것들을 낯선 면모에서 보여줌으로써 우리의 호기심 살아있도록 한다.

8) Apart from its utility in showing unsuspected possibilities, philosophy has a value - perhaps its chief value - through the greatness of the objects which it contemplates, and the freedom from narrow and personal aims resulting from this contemplation. The life of the instinctive man is shut up within the circle of his private interests: family and friends may be included, but the outer world is not regarded except as it may help or hinder what comes within the circle of instinctive wishes. In such a life there is something feverish and confined, in comparison with which the philosophic life is calm and free. The private world of instinctive interests is a small one, set in the midst of a great and powerful world which must, sooner or later, lay our private world in ruins. Unless we can so enlarge our interests as to include the whole outer world, we remain like a garrison in a beleaguered fortress, knowing that the enemy prevents escape and that ultimate surrender is inevitable. In such a life there is no peace, but a constant strife between the insistence of desire and the powerlessness of will. In one way or another, if our life is to be great and free, we must escape this prison and this strife.
   예측하지 못한 가능성들을 보여줌에 있어서의 그 유용성과 별개로, 철학은 - 아마도 철학의 최고의 가치인 - 철학이 숙고하는 대상들의 중요성을 통해서, 그리고 이러한 심사숙고로부터 기인하는 협소하며 사적인 목적들로부터의 자유를 통해서, 하나의 가치를 지닌다. 본능적인 인간의 삶은 아마도 가족과 친구들이 포함될 그의 사적인 관심사들의 원환 안에 닫힌 채이다. 그러나 그 바깥의 세계는 본능적 소망들의 원환에 딸려 있는 것을 돕거나 방해할지 모르는 그러한 것을 제외하고는 고려되지 않는다. 그러한 삶에는, 철학적 삶이 안정되고 자유로운 것에 비하자면 과열되고 꽉 막힌 어떤 것이 있다. 본능적 관심사들의 사적인 세계는, 조만간 우리의 사적인 세계를 파괴시킬 것임에 틀림없는 위대하고 강력한 세계의 한가운데에 놓인 자그마한 하나의 세계이다. 우리가 그렇게 우리의 흥미들을 외부 세계 전체를 포함하는 것과 관련하여 확장시킬 수 없는 한, 우리는, 적들이 퇴로를 막고 있으며 궁극적인 투항이 불가피함을 알면서 포위된 요새의 군사들과 같이 남는다. 그러한 삶에는 아무런 평화도 없이 욕망의 주장과 의지의 무력함 사이의 끝없는 다툼만이 있다. 어떤 식으로든, 만일 우리의 삶이 훌륭하고 자유로운 것이려면, 우리는 반드시 이러한 감옥과 이 다툼으로부터 달아나야만 한다.

9) One way of escape is by philosophic contemplation. Philosophic contemplation does not, in its wides survey, divide the universe into two hostile camps - friends and foes, helpful and hostile, good and bad - it views the whole impartially. Philosophic contemplation, when it is unalloyed, does not aim at proving that the rest of the universe is akin to man. All acquisition of knowledge is an enlargement of the Self, but this enlargement is best attained when it is not directly sought. It is obtained when the desire for knowledge is alone operative, by a study which does not wish in advance that its objects should have this or that character, but adapts the Self to the characters which it finds in its objects. This enlargement of Self is not obtained when, taking the Self as it is, we try to show that the world is so similar to this Self that knowledge of it is possible without any admission of what seems alien. The desire to prove this is a form of self-assertion and, like all self-assertion, it is an obstacle to the growth of Self which it desires, and of which the Self knows that it is capable. Self-assertion, in philosophic speculation as elsewhere, views the world as a means to its own ends; thus it makes the world of less account than Self, and the Self bounds to the greatness of its goods. In contemplation, on the contrary, we start from the not-Self, and through its greatness the boundaries of Self are enlarged; through the infinity of the universe the mind which contemplates it achieves some share in infinity.
   도피의 한 가지 방법은 철학적 숙의에 의한 것이다. 철학적 숙의는 넓은 조망에서 우주를 두 가지 적대적 진영들로 - 친구들과 적들, 도움이 되는 쪽과 적대적인 쪽, 좋음과 나쁨 - 가르지 않는다. 그것은 전체를 치우침 없이 바라본다. 철학적 숙의는, 그것이 다른 것과 뒤섞이지 않을 때, 우주의 나머지가 인간에 가깝다는 것을 증명하는 데에 목적을 두지 않는다. 모든 지식의 습득은 자아의 확대이고, 그러나 이러한 확장은 직접적으로 추구되지 않을 때에 가장 잘 획득된다. 그것은, 미리 그 대상들이 이러저러한 성격을 가져야 하리라 바라지 않고, 자아를 그것이 그것의 대상들에게서 발견하는 성격들에 맞추는 그러한 연구에 의해서, 지식욕만이 홀로 작동할 때 얻어진다. 자아를 있는 그대로 취하면서 세계가 어떤 이질적인 그런 것의 어떤 승인도 없이 그 세계에 대한 지식이 가능한 이 자아와 매우 유사하다는 것을 우리가 보여주고자 노력할 때 이러한 자아의 확장은 획득되지 않는다. 이러한 것을 증명하려는 욕망은 자기주장의 한 형태이고, 모든 자기주장이 그러하듯, 그것은 자아가 욕망하는, 그리고 가능하다는 것을 알고 있는 성장에 걸림돌이다. 자기주장은 여느 곳에서나 마찬가지로 철학적 사변에 있어서도 세계를 그 자신의 고유한 목적들을 위한 수단으로 간주한다. 그래서 세계를 자아보다 덜 중요하게 여기며, 자아를 그 세계의 이익들의 훌륭함에 묶는다. 반대로 숙의에 있어서 우리는 비아(非我)로부터 출발하고, 그 비아의 거대함을 통하여 자아의 경계들이 확장된다; 우주의 무한성을 통해서 우주를 숙고하는 정신은 무한성의 어떤 참여를 성취한다.

10) For this reason greatness of soul is not fostered by those philosophies which assimilate the universe to Man. Knowledge is a form of union of Self and not-Self; like all union, it is impaired by dominion, and therefore by any attempt to force the universe into conformity with what we find in ourselves. There is a widespread philosophical tendency towards the view which tells us that Man is the measure of all things, that truth is man-made, that space and time and the world of universals are properties of the mind, and that, if there be anything not created by the mind, it is unkowable and of no account for us. This view, if our prvious discussions were correct, is untrue; but in addition to being untrue, it has the effect of robbing philosophic contemplation of all that gives it value, since it fetters contemplation to Self. What it calls knowledge is not a union with the not-Self, but a set of prejudices, habits, and desires, making an impenetrable veil between us and the world beyond. The man who finds pleasure in such a theory of knowledge is like the man who never leaves the domestic circle for fear his word might not be law.
    이러한 이유로 영혼의 위대함은 우주를 인간에 동화시키는 그러한 철학들을 통해서 발전되지 않는다. 지식은 자아와 비아의 통일의 한 형태이다; 모든 통일체와 마찬가지로, 지식은 권역에 의해 손상되며, 그러므로 우주를 우리가 우리 자신들에게서 발견한 것에 순응하도록 강제하고자 하는 그 어떠한 시도에 의해서든 손상된다. 인간이 만물의 척도이며, 진리는 인공의 것이고, 공간과 시간과 보편자들의 세계는 정신의 특징들이라고, 또 만일 그 어떤 것이든 정신에 의해 창조된 것이 없다면, 그것은 불가지한 것이며 우리에게 고려되지 않는 것이라고 우리에게 말하는 관점을 향하는 철학적 경향이 만연해 있다. 이러한 관점은 만일 우리의 아선 논의들이 정확했다면, 틀렸다. 그러나 틀린 것에 덧붙여서, 그 관점은 철학적 숙의에 가치를 부여하는 모든 것을 앗아가는 결과를 가지는데, 왜냐하면 그것이 숙고를 자아에 속박시키기 때문이다. 그 관점이 지식이라고 부르는 것은 비아와의 통일체가 아니라, 우리와 세계 사이 저편에 헤쳐낼 수 없는 장막을 드리우는 일련의 편견들, 습관들, 욕망들이다. 지식에 대한 그러한 이론에서 즐거움을 발견하는 그런 사람은 그의 말이 법칙이 아닐 것이 두려워 가족의 울타리를 결코 떠나본 적 없는 사람과 같다.

11) The true philosophic contemplation, on the contrary, finds its satisfaction in every enlargement of the not-Self, in everything that magnifies the objects contemplated, and thereby the subject contemplating. Everything, in contemplation, that is personal or private, everything that depends upon habit, self-interest, or desire, distorts the object, and hence impairs the union which the intellect seeks. By thus making a barrier between subject and object, such personal and private things become a prison to the intellect. The free intellect will see as God might see, without a here and now, without hopes and fears, without the trammels of customary beliefs and traditional prejudices, calmly, dispassionately, in the sole and exclusive desire of knowledge - knowledge as impersonal, as purely contemplative, as it is possible for man to attain. Hence also the free intellect will value more the abstract and universal knowledge into which the accidents of private history do not enter, than the knowledge brought by the senses, and dependent, as such knowledge must be, upon an exclusive and personal point of view and a body whose sense-organs distort as much as they reveal.
    반대로 참된 철학적 숙고는 비아의 모든 확장에서, 숙고된 대상들을 확대시키는 모든 것들에서, 그리고 그로써 주체가 숙고하는 것 전부에서 만족을 얻는다. 숙고에 있어서 개인적이고 사적인 모든 것, 습관, 사익, 또는 욕망에 의존하는 모든 것은 그 대상을 왜곡시키고, 그러므로 지성이 추구하는 통일체를 손상시킨다. 그리하여 주체와 객체 사이에 장벽을 만듦으로써, 그러한 개인적이고 사적인 것들은 지성의 감옥에게 감옥이 된다. 자유로운 지성은, '여기'와 '지금' 없이, 희망들과 두려움들 없이, 관습적 믿음들과 전통적 편견들의 구속들 없이, 평온하고, 평정하게, 오로지 전적으로 지식 - 비개인적인 것으로서, 그리고 순수하게 사색적인 것으로서, 인간이 획득할 수 있는 것으로서 - 에 대한 욕구 안에서 신이 볼 그러한 것을 볼 것이다. 그러므로 또한 자유로운 지성은 개인사의 사건들이 침범하지 않는 추상적이고 보편적인 지식을 감각에 의해 제공되고, 그러한 지식이 틀림없이 그러하듯 배타적이며 개인적인 관점과 밝혀내는 만큼 왜곡시키기도 하는 감각기관들을 지닌 육체에 의존하는 그러한 지식보다 더욱 가치있게 여길 것이다.

12) The mind which has become accustomed to the freedom and impartiality of philosophic contemplation will preserve something of the same freedom and impartiality in the world of action and emotion. It will view its purposes and desires as parts of the whole, with the absence of insistence that results from seeing them as infinitesimal fragments in a world of which all the rest is unaffected by any one man's deeds. The impartiality which, in contemplation, is the unalloyed desire for truth, is the very same quality of mind which, in action, is justice, and in emotion is that universal love which can be given to all, and not only to those who are judged useful or admirable. Thus contemplation enlarges not only the objects of our thoughts, but also the objects of our actions and our affections: it makes us citizens of the universe, not only of one walled city at war with all the rest. In this citizenship of the universe consists man's true freedom, and his liberation from the thraldom of narrow hopes and fears.
    철학적 숙의의 자유와 평등에 익숙해지기 시작한 정신은 행위와 감정의 세계에서 동일한 자유와 평등에 속하는 어떤 것을 지켜낼 것이다. 그 정신은 전체의 부분들로서 그 정신의 의도들과 욕망들을 그 나머지 전부가 그 어떤 한 사람의 행위들로도 영향받지 않는 세계의 미미한 파편들로서 그것들을 바라봄으로써 귀결되는 저항의 부재와 함께 보여줄 것이다. 숙고에 있어서 진리를 향한 뒤섞이지 않은 욕망인 그 공평성은 행위에 있어서 정의로운 정신, 그리고 감정에 있어서 모두에게 주어질 수 있는 보편적 사랑인 정신과 매우 동일한 질이고 유익하거나 존경스럽다고 판단된 이들에게만 그러한 것은 아니다. 그래서 숙고는 우리 사유의 대상들만이 아니라 또한 우리 행위들과 우리의 감성들의 대상들도 확장시킨다. 그것은 우리를 세계시민으로 만들며, 그저 다른 모든 세계와 맞서는 고립된 도시의 시민에 그치도록 하지 않는다. 이러한 세계시민성 안에는 인간의 참된 자유, 그리고 그의 협소한 희망들과 두려움들의 예속상태로부터의 해방이 자리한다.

13) Thus, to sum up our discussion of the value of philosophy; Philosophty is to be studied, not for the sake of any definite answers to its questions, since no definite answers can, as a rule, be known to be true, but rather for the sake of the questions themselves; because these questions enlarge our conception of what is possible, enrich our intellectual imagination and diminish the dogmatic assurance which closes the mind against speculation; but above all because, through the greatness of the universe which philosophy contemplates, the mind also is rendered great, and becomes capable of that union with the universe which constitutes its highest good.
     그래서 철학의 가치에 대한 우리의 논의를 요약하자면, 철학은 연구되어야 할 것이며, 하나의 법칙처럼 어떠한 한정된 대답들도 참인 것으로 알려질 수 없기 때문에 철학의 물음들에 대한 어떤 한정된 대답들을 위해서가 아니라, 오히려 물음들 그 자체들 때문에 연구되어야 하는 것이다. 왜냐하면 이러한 물음들은 무엇이 가능한지에 대한 우리의 이해를 확장시키고, 우리의 지적 상상력을 풍요롭게 하며 사색으로부터 정신을 폐쇄시키는 독단적 확언을 약화시키기 때문이다. 그러나 무엇보다도 철학이 숙의하는 우주의 광대함을 통하여 정신도 또한 위대해지며, 그 세계의 최상의 선으로 여겨지는 세계와의 통일체가 될 수 있게 되기 때문이다.


-蟲-
1) 플라톤 『에우튀프론』의 독자들은 전형적으로 에우튀프론의 경건(εὐσέβεια)에 대한 다섯 가지 정의 시도들에 대한 소크라테스의 논박적 검토에 주의를 집중한다 (1) 경건은 누구든지 부정의를 저지른 자에 맞서 소송을 거는 일이다(5d-6e), (2) 경건은 신들에 의해 사랑받는 것이다(6e-9d), (3) 경건은 모든 신들에 의해 사랑받는 것이다(9e-11b), (4) 경건은 신들이 그들의 가장 아름다운 산물을 산출하도록 봉사하는 올바름의 부분이다(11e-14b), (5) 경건은 기원과 제사의 기술이다(14b-15c). 그러나 이러한 논증적 사건들이 그 대화의 명백한 철학적 실체를 아주 잘 구성한다 할지라도 , 만일 우리가 그것들과 더불어 그 대화의 전반적인 의미까지 모두 충분히 이해하고자 한다면 그 대화의 참가자들을 몰아가는 그 복잡한 동기들을 또한 신중하게 철저히 검토 해 마땅하다. 플라톤 자신이 이런 종류의 검토를 유발시키길 바란다는 것은 분명해 보이는데, 왜냐하면 그는 이상하게 복잡하고 긴 극적 도입부를, 거칠게 말해서 그 대화편 길이의 3분의 1에 달하는 그런 도입부(그 주제들이 연이은 경건으로의 탐구를 끊임없이 알리는)를 내놓기 때문이다. 우리는 그래서 그 도입부의 가장 혼란스럽고 아직 논의가 덜 된 요소들 중 하나를 꽤 잘 살펴볼 수 있을 것이다; 즉, 에우튀프론의, 그가 그의 아버지를 그의 아버지가 저질렀다고 그가 상상하는 그런 종류의 살인들에 수반되는 μίασμα - 더러움 -  때문에, 그리고 공정한 정의가 그것을 요구하기 때문에 고소함에 있어서 정당화된다는, 그 단언이다(4b7-c3). 이 논문에서 나는 이러한 호소에 대한, 에우튀프론이 생각되어 왔던 것보다 더욱 도덕적으로 그리고 신학적으로 진보적일 것이라는 것을 보여주는, 그러나 또한 『에우튀프론』이 간접적으로 소크라테스에 대한 비-법률적 변호의 역할을 하는 방식을 새로이 조명하는 한 가지 새로운 설명을 주장하고자 한다.

2) 『에우튀프론』의 도입부는 두 부분으로 구성되는데, 첫번째 부분은 곧 있을 소크라테스의 공판을 소개하는 것이고 두번째 부분은 에우튀프론 자신의 송사를 상술하는 것이다. 여기에서 내 의도에 맞추어 나는 다음과 같이 가정하는 위험을 감수하고자 한다. 이 첫번째 부분와 뒤따르는 부분에서 플라톤이 소크라테스의 동기들을 에우튀프론과 관련하여 상대적으로 투명하고, 완전히 고결하며, 『변론』의 소크라테스에 의해 공언된 동기들(예를 들어 『변론』 22e-23b, 29c-31c)과 동일한 것으로 묘사한다는 것이다: 이처럼 소크라테스는 그것, '경건이 무엇인지'에 대한 앎의 탐구(15c-16a)가 결코 폐기되어서는 안 된다는 것을 지니는데, 왜냐하면 이 사업이 그 자체로 즐겁고 (주장컨데 '상상조차 할 수 없는 행복'; 『변론』 41c3-4) 경건할 뿐만 아니라, 경건이 '남은 삶을 더욱 나은 삶이도록' 허용하리라는 것을(15e7-16a4), 특히 그것의 지속이 불경의 습격에 의해 위협받을 것임을 이해하게 되는 것이기도 하기 때문이다(3b-d, 5a-c, 12e, 15e-16a).

3) 자연스럽게, 일부 주석가들은 철면피스러운 위선과 뒤엉킬 그러한 표현들을 발견했다. 존 베버스루이스에 따르자면 예를 들어 허세가 심하고 답답하며 패륜아에 '나치청년단에서도 꽤나 잘 나갔을' 광신교도로서 에우튀프론의 전형적인 성격은 그에 대한 소크라테스의 검토가 실제로 얼마나 모욕적이고 모자란 것인지에 대한 우리의 주목으로부터 주의를 떼어 놓았다. 우리가 이야기하게 되는 소크라테스는 처음에 적합한 정의에 대한 그의 기이한 원리를 가지고 에우튀프론을 압도하고(5c-d, 6d-e) 다음으로 이해가 아닌 단지 문자적 동의에 불과한 것을 바탕으로 궤변적으로 그를 두들긴다. 그러나 이러한 비평은 즐거울지는 모르지만, 소크라테스가 그러므로 에우튀프론의 영혼을 불경하고 부정의하며 위선적으로 외면한 죄가 있다는 것이 뒤따르진 않는다(베버스루이스, 184). 오히려, 베버스루이스께는 죄송하게도 (176, 184), 그것은 대화편 말미에서 법정 소환 전에  에우튀프론이 갑작스럽게 떠나게 되고, 그래서 그로써 그가 그의 잘못 세워진 소송을 몰수당한다고 우리가 생각하도록 플라톤이 일조하는 데에 가장 극적으로 효과적으로 보인다(후기 전통이 그러하듯이; D.L.2.29). 그래서 소크라테스의 전략은 에우튀프론이 잠재적으로 행위방식을 훼손시키는 쪽으로 나아가는 것을 만류함으로써 에우튀프론과 그의 아버지, 그리고 그의 친족들에게 유익하다. 이 모든 것이 그러함에도 불구하고 그렇지만 그것이 내가 지금 초점을 맞추려는 에우튀프론이다.

4) 플라톤의 이야기에 따르자면, 소크라테스와 에우튀프론의 우연한 만남보다 5년 앞서 에우튀프론의 소작인 중 하나(ὁ πελάτης)가 그의 가문 노예들 중 하나(ὁ οἰκέτης)를 술에 취해 홧김에 죽였다. 에우튀프론의 아버지는 그 살인자를 묶었고 구덩이에 내던졌다. 그리고 - 그 일꾼이 살인자였기 때문에 - 아테네 종교적 조언가들(ἐξηγηταί) 중 하나로부터 어떻게 처리할지에 대한 조언을 기다리는 동안 그 살인자를 신경쓰지 않았다. 결국, 그 일꾼은 배고픔과 추위로 죽었다. 이에 대하여 에우튀프론이 이제 아르콘 바실레우스 앞에 에우튀프론의 아버지에 대하여 살인죄로 소송을(δίκη φόνου) 가져오는데, 그의 아버지와 그 자신(그리고 짐작컨데 그의 친족들) 모두를 그가 이런 종류의 부정의한 죽임을 곁에 있으면서 본 μίασμα에 대해서 정화하기 위해서이다(4b-e). 소크라테스는 당연히 이 이야기에 놀란다. 그 자신의 아버지에게 소송을 걸음으로써 에우튀프론은 미친 사람 취급을 받는데(μαίνεσθαι, 4a1), 왜냐하면 남을 위해 가족을 고소한다는 것은(만일 실제로 불가능하지 않다면; 아래를라) 가장 드문 일일 것이기 때문이다. 더욱이,

-작성중-
꼴들의 내재

1) 초기 대화편들에서 꼴들이 그것들의 예시들과 동일하지 않으며 그것들의 예시들에 우선한다는 사실은 그것들이 그것들의 예시들 '안에' 있다는 주장에 반대하여 영향을 미친다. 로스 교수는 꼴들에 대한 이론의 발전에 있어서 '내재로부터 떠나 초월로 향하는 일반적 변동이 있다. 초기에 거의 모든 것들이 내재에 대해 이야기한다.'라고 주장해 왔다. 그는 후기 대화편들에서 특수자들은 결코 충분하게 그것들의 꼴들을 예시하거나 예증하지 못하는 것으로 여겨진다고 주장한다; 특수자들은 오로지 그 꼴들을 흉내낼 따름이다. 그러나 초기 대화편들에서 꼴은 '특수자 안에 내재하는 것으로' 다루어진다. 꼴은 그것들 안에 "현존한다"; 그것은 장인에 의해 "그것들 안에" 정위된다; 꼴은 그 예시들에게 '공통적'이다; 특수자의 경우에는 꼴을 "지니거나" 그 꼴에 "관여한다".

2) 이 주장의 힘은 '내재'의 의미에 달려 있다. 로스 교수가 인용하는 모든 표현들은 그리스어에서 추상명사들과 함께 정말로 일상적으로 사용된다; 예를 들어 한 행위에 올바름이 있다고 말하는 것은 한 행위가 올바르다고 말하는 것의 단순히 또 다른 방식, 그리고 이상적 방식일 따름이다. 꼴들은 추상 명사들의 의미들이기 때문에, 그런 표현들의 사용은 형이상학적으로 자연스럽다: 만일 꼴들이 그것들의 예시들 안에 내재적이라는 주장이 언어적 용법의 사실들을 요약할 따름이라면, 그 주장은 참이지만, 존재론적 지위의 문제가 관련되는 한에서는 무의미한 주장이다.

3) 다른 한편, 만일 '내재'라는 것이 언어의 특정 용례를 포괄하는 말일 따름이라기 보다는, 꼴들의 존재론적 지위에 속한 것으로 취해진다면, 초기 대화편들의 언어에 대한 로스 교수의 호소는 꼴들이 내재적이라는 그의 주장을 뒷받침하지 못할 것이다. 내재의 이 비일상적 의미를 그것의 추정적 반대로, 초월성이란 말로 한정하는 것도 가능하지 않다: 꼴들이 어떤 전문적 의미에서 사물들 '안에' 있다는 주장은 그것들이 '떨어져' 있다거나 '따로' 있다는 것에 대한 부정으로 대체될 수도 없다. 왜냐하면 로스 교수는 내재와 초월을 모순으로서 다루지 않고 상호보완적으로 다루기 때문이다. 이에 대한 이유는 플라톤이 '안에'라는 것을 장소로 사용하고 그것의 동종적인 것들을 중기와 후기 대화편들에서, 바로 그가 초기에 사용했던 것과 똑같이 사용하기 때문이다. 그리고 로스 교수는 이에 대해 다음과 같이 언급한다:

유일하게 가능한 결론은, 그[플라톤]가 그 중 어느 표현으로도 만족하지 못했던 반면, 그는 그 둘 모두를 사용하는 것보다 더 진리에 접근할 아무런 방법도 보지 못했다는 것일 듯하다. 그 하나는 보편개념과 그것의 특수자들 사이의 관계의 친밀성을 강조하고, 다른 하나는 모든 특수자들 각각이 어떤 보편자에 대해서도 완벽한 예증이 되는 데에 실패한다는 것을 강조하는 것이다. 그는 심지어 그 관계가 완전히 특유하고 정의하기 어렵다는 것조차 눈치챘었을 것이다. '분유'와 '모사' 모두 그에 대한 은유들이고, 그 두 가지 상호보완적인 은유들의 사용은 그 둘 중 하나만의 사용보다는 더 낫다.

그 결론은 따르기 어렵다. 만일 예증이 그 자체의 유에 속하는(특유한) 관계라면, 어떠한 분석도 불가능하고 어떠한 분석도 필요로 하지 않는다면, '모사'와 '분유'는 그에 대한 유용한 은유이기 어렵다; 은유는 어떤 종류의 유비적 관계성을 수반하고, 여기에는 유추를 위한 아무런 근거도 없다. '분유'는 '모사'가 그렇지 않은 것처럼 정확히 일상적으로 추상명사들과 함께 사용되기 때문에, 엄격한 의미에서 전혀 은유가 아니라는 것이 덧붙여질 것이다. 그러나 내재가 초월의 부정으로 정의될 수 없다는 것을 넘어서서, 그리고 로스 교수는 어디에서도 그 용어에 그 자체의 독립적 의미를 부여하는 데에 성공하지 못한다. 중기와 후기 대화편들에서 꼴들은 그것들을 모사하는 예시들에 의해 오로지 불충분하게만 예증된다는 것은 참이면서 중요하기도 하다; 그리고 어째서 그 문제들에 대한 그 진술이 만일 누군가 선택한다면 '초월'이라 불리지 않아야 하는지에 대한 어떠한 이유도 없다. 그러나 어떠한 독립적 의미도 '내재'에 할당되지 않았기 때문에, 만일 그 주장이 그 꼴들의 존재론적 지위에 대한 설명을 의미한다면 참도 거짓도 초기 대화편들에 있어서 꼴들은 그것들의 예시들 안에 내재적이라는 로스 교수의 주장에 덧붙지 않는다.

4) 그 문제의 참은 꼴들이 강한 의미에서 후기에서 만큼이나 초기 대화편들에 있어서도 그것들의 예시들로부터 '떨어져' 있다는 말이다. 그것들이 그것들의 예시들과 동일하지 않기 때문에, 그리고 존재론적으로 그것들의 예시들에 우선하기 때문이다. 즉, 그것들은 '따로' 존재한다. 초기 대화편들에서의 꼴들에 대한 이론과 후행했던 이론들 사이의 차이점은 분리에 대한 사실로 구성되지 않고, 분리가 생각되는 방식으로 구성된다. 중기 대화편들은 존재론적 지위에 대한 변경된 추정, 꼴들이 있는 방식에 대한 이론으로 전환된 추정을 제시한다.


『파이돈』에서의 분리

1) 분리의 주제는 『파이돈』에서 처음으로 감각과 반성 사이의 대조를 통해 소개된다. 소크라테스는 올바름과 같은 그런 어떤 것이 오로지 그 자체로 있고 또한 아름다움과 좋음도 그러하다는 시미아스의 논증을 유도해 냄으로써 시작한다. 그리고 나서 그는 그런 것들이 시각이나 여타의 감각들에 의해 파악될 수 없고 오로지 순수한 사유 그 자체만으로만 파악될 수 있다고 주장한다. 감각은 앎에 대한 탐구에 있어서 반성보다 하위에 놓인다(『파이돈』65d-e).

2) 자연스러운 확장에 의해 이 설명은 하나의 추론을 불러일으킨다: 감각의 대상들은 반성의 대상들보다 하위에 놓인다는 것이다. 이것은 『파이돈』에서 상기에 대한 두번째 논증의 핵심이다. 그 논증의 첫번째 가정은 같음의 꼴이 존재한다는 것이다(74a-b):

내 생각에 우리는 어떤 같은 것이 있다고 말하는군 - 난 막대기가 막대기와 같다거나 돌이 돌과 같다거나 그런 종류의 어떤 것이 아니라, 이러한 것들을 넘어서서 저편에 다른 어떤 것을 말하는 것이네만 - 같음 그 자체 말일세.

그리고 나서 그 같음과 감각적인 같음이 ἕτερα ὄντα, 상이한 종류의 것들임이 드러난다; 왜냐하면 감각적인 같음들은 하나에 대해서만 같음을 입증할 뿐 다른 것에 대해서는 그렇지 않고, 사실 정말로 같은 것들은 같지 않을 수도 없을 뿐더러 같음은 같지 않음일 수 없기 때문이다(74b-c). 그래서 같음은 그 같음이 같지 않음에 의해 같아질 수 없다는 점에서 같은 것들과는 다르고, 그러나 실상 감각적 같음들은 또한 같지 않을 것이다.

3) 같음이 그것의 감각적 사례들과 동일시되는 것이 아님을 보였을 때, 『파이돈』은 다음으로 계속해서 그것들의 관계를 특징지운다. 감각적 같음들이 같음이 같은 것처럼 같지는 않고, 그것이 그러한 것인 그런 종류의 것에는 미치지 못한다는 것이 동의된다; 그것들은 또 다른 종류의 것으로서 그러한 종류의 것인 것이기를 바라지만, 그것에 대해 하위에 자리한다; 그것들은 그 같음과 유사하지만, 그것에 대한 측면에서는 부족하다(74d-e). 그러나, 우리가 이 결핍을 재인할 수 있기 때문에, 우리가 그 자체 안에서 그 자체에 대해 같음에 대한 분명한 앎을 가졌다는 것이 뒤따를 수밖에 없다(75b):

우리가 보거나 듣거나 여타의 어떤 감각들을 사용하기 시작함에 앞서서, 우리는 반드시 같음 그 자체는 무엇인지에 대한 앎을 가지고 있었어야만 하네. 만일 우리가 이러한 감각적 같음들을 그 대상들에 귀속시킬 수 있다면 말이지. 그러한 모든 것들이 그와 같은 그러한 종류의 것이기를 애쓰고 있으나 그것에 대해 하위에 있다는 근거에서 말일세.

그래서 말하자면 우리는 같음에 대한 우리의 앎을 감각적 같음들로부터 추상할 수 없는데, 왜냐하면 그것들의 결핍 때문이다; 오히려, 우리는 반드시 감각적 같음들이 같음들임을 재인하기 위해 그러한 앎을 가져야만 한다. 동일한 설명이 75c-d에 대해 유효하다:

더 크거나 더 적은 그런 종류의 모든 것들은, 우리의 논증이 같음에 대해 아름다움 그 자체, 좋음 자체, 올바름, 경건에 대해서, 그리고 한 마디로 우리의 변증에 있어서 우리가 실재의 보증을 재가하는 그러한 모든 것들에 대해보다 조금도 덜 적용되지 않기 때문일세.

반성에 대한 감각의 하위성은 지성적 꼴들에 대한 감각적 예시들의 하위성으로 설명되는 것이다.

4) 여기에서 분리는 명백하게 비동일성, 독립성, 또는 우선성 보다 더 많은 어떤 것을 필요로한다. 그것은 꼴들에 대한 예시들이 꼴들에 대한 불완전한 모사들 또는 유사물들이라는 주장을 필요로 한다. 블라스토스 교수의 말을 빌리자면, 그것은 모사 이론을 필요로 한다. 그 이론에 자연스러운 귀결로 감각적인 것들과 꼴들은 그것들의 실재성의 정도에 있어서 다르다는, 꼴들은 그것들의 예시들보다 더욱 실재적이라는 이론이 나중에 결합되었다.


두 세계

1) 『파이돈』은 계속해서 감각적 대상들과 지적 대상들을 두 세계, 가시계와 비가시계에 대한 원칙으로 전개시켜 나아간다. 비슷한 것이 비슷한 것을 알고, 지성적 영혼은 그 영혼의 앎의 대상들, 영원한 것들에 가깝다. 가시계의 대상들, '그것이 실재인 뒤에 이름지어지는 그러한 것들'은 끊임없이 변화하는 필멸의 영역에 있고, 서로에 대한 면으로나 그 자체들에 대한 면으로나 결코 동일하지 않다. 반대로, 그것의 현존에 대한 실재성을 우리가 묻고 답하는 속에서 설명해 내는 꼴들은 언제나 동일한 것들에 대한 측면에서 동일한 방식으로 존재하고 본성에 있어서 단일하며 오로지 그것들 자체만으로 현존하고 변화를 허용하는 그 어떠한 환경에서 어떤 방식으로도 결코 속하지 않는다(78d-79b).

2) 『국가』에서 두 세계는 앎의 세계와 사견의 세계가 된다. 소크라테스는 거기에서 철학자, 지혜의 구애자를 정의하는 일을 떠맡고, 그를 그와 표면적으로만 유사한, 아름다운 볼 것들과 들을 것들의 구애자와 구별하는 일에 착수한다. 그 논의는 꼴들에 대한 이론을 전제함으로써 시작한다(Ⅴ, 475e-476a):

나는 자네가 나와 이 점에 있어서, 그러니까 아름다움은 추함에 반대이고, 그것들은 두 가지의 것들이라는 점에 동의하리라 믿는다네; 그리고 그러므로, 그것들의 각각은 하나라는 것도 말일세. 같은 것들은 올바름과 그릇됨, 좋음과 나쁨, 그리고 모든 특성들에 대해 참일세. 각각의 것은 그 자체로 하나의 것이지만, 행위들과 신체들과 각기 다른 것을 가지고 교류를 통하여, 각각은 많은 곳들에서 나타남으로써 많은 것들로 드러나지.

그것은 시각적인 것들과 청각적인 것들의 구애자들로부터 지혜를 사랑하는 자들을 구분하는 아름다움의 꼴에 대한 인식이다. 깨어 있든 잠들어 있든 꿈으 꾼다는 것은 어떤 것 같은 것을 단순히 그런 것 같은 것이 아니라 그것이 닮은 다름 아닌 바로 그것이라고 믿는 것이다. 철학자는, 그가 아름다움과 그것에 참여하는 사물들 사이의 구별을 파악할 수 있기 때문에, 깨어 있고 또 꿈꾸고 있지 않으며, 단지 사견이 아니라 앎을 가진다(476c-d).

3) 이 닮음은 실재성의 정도들을 시사한다. 완전히 알 수 있는 어떤 것은 완전히 실재적이다(476e-477a). 그래서 만일 있으면서도 있지 않은 그러한 어떤 것이 있다면, 그것은 완전한 실재와 전적인 비실재 사이에 놓일 것이다. 앎은 실재하는 것에 상응하기 때문에, 그리고 무지는 비실재에 상응하므로, 우리는 만일 실재와 비실재 사이의 중간인 어떤 것이 있다면 앎과 무지 사이의 중간인 어떤 것이 있다고 기대해야만 한다. 이것은 사견이거나 믿음이다; 왜냐하면 사견은 무지보다 더욱 선명하고 앎보다는 더 흐릿하기 때문이다 (477a-b). 앎과 사견은, 그것들이 산출하는 정신의 상태에 의해서도 그것들의 대상들에 의해서도 구별되는, 서로 다른 힘들이나 능력들이다 (477c-d). 앎에 의해 산출되는 정신의 상태는 틀릴 수 없는 것이고, 사견에 의해 산출되는 것은 틀릴 수 있는 것이다. 앎의 대상은 실재적인 것이고, 그것의 능력은 실재를 그것이 그것인 바로서 알 수 있는 능력이다; 사견의 대상은 비실재일 수 없는데, 왜냐하면 그 비실재는 아무것도 아니고, 그래서 실재와 비실재 사이의 어떤 중간적인 것이어야만 하기 때문이다 (478a-d).

4) 그 대상은 무엇인가? 이 물음에 답하기 위해, 우리는 아름다운 것들의 현존을 재인하는 이들에게, 그러나, 동일한 것들에 대한 측면에서 언제나 동일한 것으로 현존하는 아름다움의 꼴의 현존은 부정하는 그러한 이들에게로 고개를 돌려 물어야만 한다 (479a-b):

많은 아름다운 것들 중 어느 것이 또한 추함일 것으로 드러나지 않을 것인가? 혹은 올바른 것들은 어떤 것이 그릇되지 않은 것인가? 또는 경건한 것들은 어느 것이 불경건하지 않은 것인가? 여러 짝들이 짝들인 만큼이나 양분되는 것으로 드러난다. 큰 사물들은 작다; 가벼운 사물들은 무겁고, 더 이상 반대인 것보다 하나인 것이지 않다. 있다고 이야기되는 이러한 많은 것들 중 각각의 무엇이든, 그것들은 있다고 이야기되지 않는 바로 그만큼일 것이다. …그것들은 모호하고, 그것들에 대해 있는 것으로서든 없는 것으로서든 그 양쪽 모두로서든 둘 다 아닌 것으로서든 고정된 개념을 형성하는 것은 불가능하다.

사견의 대상들은 감각적이고, 그것들은 완전히 실재적이지도 그렇다고 아예 없지도 않다.

5) 두 세계에 대한 이러한 설명은 일련의 비율들에 의거한다: 꿈꾸고 있음이 깨어 있음에 맞서듯, 그렇게 사견은 앎에 맞서고, 더 흐릿한 것이 더 분명한 것에, 덜 실재적인 것이 실재적인 것에, 꼴들의 예시들이 꼴들에 맞선다. 『정체』Ⅹ,597a에서 목수는 침대의 꼴을 만드는 것이 아니라 그저 그 꼴의 유사물들인 어떤 것을 만드는 것으로 이야기된다; 그의 생산품을 완전히 실재적인 것으로 부르는 것은 오류인데, 왜냐하면 그것은 실재와 참에 대한 관점에서 흐릿하고, 또렷하지 않고, 미미하기 때문이다.

6) 두 세계는 후기 대화편 『티마이오스』에서 반복된다:

무엇이 언제나 실재이고 아무런 생성도 지니지 않는 그러한 것이고, 또 무엇이 언제나 생성 중이면서 절대로 실재이지 않은 그러한 것인가? 합리적 설명과 함께 사유로써 이해가능한 것은 언제나 변함없이 실재하는 것이다; 반면에 비지성적 감각과 더불어 믿음의 대상인 것은 생겨나고 사라지지만 결코 실재로 있음을 지니지는 못하는 그러한 것이다.

있음은 지적인 것이 감각적인 것에 맞서듯 생성에 맞선다.

7) 이것은 고도의 형이상학이다; 그러나 그것은 또한, 중요하게도, 우주에서 가장 높은 것과 인간 영혼에서 가장 깊은 것 사이의 유대를 강조하는 도덕적 종교적 원칙이다. 『향연』에서 구애자는 아름다움에 대한 그의 탐구에 있어서 또 다른 세계로 상승한다; 그리고 아름다움의 본성과 본질을 묘사함에 있어서 플라톤의 산문이 갑작스럽게 합창 시의 형식으로 주신 찬가들을 터뜨리는 곳에서 알아차렸다(211a-b):

첫째로, 그것은 언제나 있으며 생겨나거나 있게 되거나 소멸하지 않는다;
그것은 자라나거나 줄어들지도 않는다.
다시, 그것은 한 측면에서 아름답고 또 다른 측면에서는 아름답지 않은 그런 것이 아니다;
한 순간 아름답고 다른 순간 아름답지 않은 것도 아니다;
한 관계에서 아름답고 다른 관계에서는 추한 그런 것도 아니다;
한 사람에게는 아름다운 것으로서 그러나 다른 사람에게는 그렇지 않은 것으로서,
여기에서는 아름답고 저기에서는 추한 그러한 것도 아니다.

또 다시 그것은 얼굴과 두 손과 육신의 부분들의 방식으로
아름다운 것으로 드러날 것도 아니다;
아름다움은 앎으로서 아름다운 것도 아니고, 논증으로서도 그러하지 않다;
아름다움은 한 마리 짐승 안에서, 또는 지상에서, 혹은 천상에서, 다른 어떤 것 안에서든 그와 같이
어딘가에서 어떤 것 안에 있는 것으로서 아름다운 것도 아니다:

그러나 그것은 오로지 그 자체인 어떤 것이고,
본성에 있어서 영원히 하나이다.
모든 다른 아름다운 것들은,
그것들이 생겨나고 사라지는 그러한 방식으로 그 아름다움을 나누어 가진다.
그러나 그것은 더 늘지도 줄지도 않고,
그것은 영향을 받지 않는다.

이것은 찬가이면서도 형이상학적 서술이다. 플라톤의 문체는 특정한 생각들의 자극 아래에서 빨라진다; 그의 운율들에 대한 두드려대는 박자, 춤추는 박자를 번역상에서 보존하는 것은 불가능하다.

8) 이러한 시야의 높이에 도달한 구애자의 영혼은 완전히 뒤바뀐다 (『향연』212a):

산출하는 것은 오로지 그것을 통해서만 그 아름다움이 보일 수 있는 그 아름다움을 보는 중에만 그에게 닥쳐올 것이다. 그는 탁월함의 상이 아니라, 왜냐하면 그가 상을 접하지 않기 때문에, 진정한 탁월함을 산출할 것이다. 그리고 참된 탁월함을 산출함에 있어서, 그리고 그것을 양육함에 있어서, 그는 신께 사랑받게 된다; 그리고 그는, 그게 누구라 할지라도, 불멸한다.

출산의 은유는 자연스럽게 『정체』에서 사용된 성교와 결혼의 은유들과 연관을 맺는다:

앎에 대한 참된 구애자는 실재에 전력하도록 타고난 자이고, 사람들이 실재일 것이라 믿는 잡다한 것들 사이에서 머무를 수가 없는 자이다. 그러나 그는 인내를 가지고 그가 각각의 것의 본질적 본성을 실재와의 관련으로 인하여 실재를 파악할 수 있는 그의 영혼의 일부를 가지고 발견하기 전까지 희미해지거나 실패하지 않을 그의 길을 고수한다; 그리고 그가 그로써 실재하는 것에 도달하고 그것과의 통일에 들어갔을 때, 이 혼인의 자손은 지성과 진리이다; 그리하여 마침내, 앎과 참된 삶과 양식을 발견했을 때, 그는 그의 여정으로부터 휴식하게 된다.

『향연』과 『정체』에서 시야와 부활에 대한 플라톤의 강조는 엘레우시스 제전의 제례에 대한 어떤 유비이다. 그 제례는 풍요와 정화를 기원했고, 마지막 통과의례에서 끝났다. 그곳에서 제물들은 갑작스럽게 작열하는 빛 속에서 숭배자들의 호기심 어린 눈에 드러났고, 숭배자들은 신에 대한 그들의 유대와 그들의 구원에 대해 확신하게 되었다.

9) 두 세계의 원리는 또한 아이스퀼로스의 종교적 시야에 대한 유비를 지닌다:

9-1) 아이스퀼로스의 가장 마지막 작품들에 있어서 관찰가능한 단계에서, 합창의 부분은 여전히 극적이고, 대사와 동등한 중요성에 속한다. 그 두 요소들은 균등하게 균형잡힌다; 그러나 동시에 그것들은 상이한 세계들을 차지하기 시작했고, 그래서 우리는 하나로부터 다른 하나로의 이행을 느낄 수 있다. 그 결과는 이제 두 측면들을 지니는 극의 흥미로운 이중성이다. 그 하나는 보편과 영원이고, 다른 하나는 특수와 시간에 제약됨이다.

9-2) 이 현상의 본성은 우리가 희망하기로는 선명해질 것이다. 만일 우리가 『아가멤논』을 하나의 삽화로 취한다면 말이다. 이 극에서, 보이는 표현은 어떻게 트로이의 정복자가 귀국하였고 여왕에 의해 살해되었는지를 보여준다. 앞으로 계속되는 단계에서 그 사건들은 특수한 사건들, 설화적 시간과 현실적 공간에 정위된 사건들이다. 그 인물들은 특정 개인들, 신화적이거나 역사적인 인물들이다 - 아이스퀼로스에게는 이에 대해 아무런 차이도 없다 - 그들은 그 순간 살았던 그리고 지상의 그 지점을 디뎠던 이들이다. 그러나 합창곡들에서 그 행위는 시간과 공간 바깥 우주의 계획으로 들어 내어진다. 그래서, 그 무대가 선명할 때 가시적 표현은 시간에 대해 유보되고, 적잖은 고통받는 사로잡힌 필멸자들의 그 일시적 광경을 뛰어 넘어서서, 바로 그 곳에서 그 때, 죄악의 그물 속에서, 장엄한 거리와 무시무시한 윤곽 속에서, 산들보다도 더욱 불변하게, 제우스의 영원한 권고 속에서, 확립된 진실들이 닥쳐 온다. 순간적 격정의 그 맥박은 차츰 잦아든다; 바로 지금 우리를 숨막히는 집중 속에 붙잡아둔 그 인간적 의지들의 충돌과 갈등은 인형극의 규모로 침잠하고 사그라든다; 지속되는 운명의 노래가 피에 사로잡힌 오만, 숙명의 고통으로의 끈질긴 유혹에 이끌린 오만의 주제를 펼친다 하더라도. 마치 합창부에서 불확실한 더 높은 무대 위에서, 배우들의 또 다른 무리가 동시에 더욱 장엄하고 상징적인 극을 불러일으키는 것처럼. 이 비가시적 장면에서 네메시스와 아테의 격정과 감정(휘브리스와 페이쏘)이 걷는다 - 후기 풍자의 핏기없는 추상들이 아니라, 여전히 초자연적 실재성의 극찬하는 특징들에 덮인. 무시간적 순간에 인간적 삶의 광경에서 제우스의 전능한 두 눈에 알려진 측면에서 장막이 걷힌다; 그리고 그것이 다시 떨어질 때, 우리는 아가멤논과 클리템네스트라, 일깨워지고, 정화되고, 고양되며, 평안한 그들의 필멸의 비극으로 되돌아온다.

플라톤과 아이스퀼로스 모두 도덕적 질서, 공통된 세계의 운행들을 관장하는 그 질서의 현존을 단언하였다; 두 사람 모두 그 질서를 어떤 의미에서 '분리된' 것으로, 그 자체의 고유한 실재성을 가진 것으로 다룬다. 플라톤에게 있어서, 그 질서의 요소들은 주로 기하학과 도덕적 탁월함에서 발견되었다; 더 높은 단계로 나아가는 그 능력들은 동일성과 지혜, 좋음과 아름다움이고 만용과 복수가 아니다. 아이스퀼로스의 사유는 발상들의 동종의 그러나 더 오래된 방식의 계획, 그 뿌리들을 철학보다 오래된, 그리고 아마도 그리스보다도 더 오래된 신화들 속에 지니고 있던 계획 내에서 움직인다.


 플라톤 사유의 통일성

1) 초기 대화편들에서 꼴들은 그것들의 예시들로부터 분리되고, 그 점에서 그 꼴들은 그 예시들과 동일하지 않으며 존재론적으로 그 예시들에 우선한다. 이것은 후기에까지 참인 것으로 남는다. 그러나 중기 대화편들은 이 분리를, 결핍과 비실재의 심연으로 갈린 두 세계에 대한 원리를 수반하는 우주에 대한 새로운 관점으로 확장시킨다. 이 원리와 결부된 것은 초기 대화편들이 보여줄 수 있는 어떤 것과도 다른 종교적 태도이다; 초기 대화편들 어디에서도 꼴 - 어떤 꼴이든 - 이 경배의 적합한 대상이리라 시사되거나 함축되지 않기 때문이다. 그래서 초기와 중기의 대화편들 사이에 꼴들에 대한 이론에 있어서 변화 - 혹은 더 정확하게, 발전 - 가 있다.

2) 이러한 주장은 이 세기의 특히나 두드러지는 플라톤주의자들 중 일부가 주장해 왔던 것처럼, 플라톤의 사유는 통일체라고, 그리고 『에우튀프론』과 다른 초기대화편들에서의 꼴들에 대한 이론은 『파이돈』과 『정체』에서의 그것과 본질적으로 같은 것이라고 주장하는 사람들에 의해 거부될 것이다. 『에우튀프론』, 『라케스』, 『뤼시스』로부터 도출된 세 가지 주요 문헌들은 이 관점을 지지하기 위해 사용되어 왔다.

3) 『에우튀프론』은 꼴들이 기준들로서 사용되리라 전제하고, 기준으로서의 꼴에 대한 정의가 그 꼴의 반대항에 의해 그 꼴이 한정된다는 것을 시사할 수 없다고 전제한다. 이러한 것들은 『파이돈』의 모사이론에 있어서본질적 요소들이다; 그러나 그것들은 단일론 명제를 확립시켜주지 않는다. 꼴이 기준으로 사용되리라고 말하는 것이 한 가지 것이고, 다른 하나는 꼴에 대한 그 꼴의 예증들이 결핍된 사례들인 그러한 꼴이 모범이기 때문에 꼴이 그렇게 사용된다고 말하는 것이다. 『에우튀프론』에서 경건의 모든 예증이 각기 반드시 어떻게든 결핍된 경건이어야만 한다는 아무런 실마리도 없다; 그리고 한 가지 분명한 것은 꼴이 그 고유의 반대항에 의해 한정될 수 없다는 주장이고, 꼴의 모든 각각의 예증이 그것과 그것의 반대항 모두에 의해 한정되리라는 주장은 또 다른 것이다. 후자의 주장은 중기 대화편들에서 이루어지는 것이고, 유사물의 결핍에 대한 본질적인 것이다.

4) 『라케스』는 앎을, 과거, 현재, 미래에 독립적으로 적용된다는 의미에서 무시간적으로 만드는 앎에 대한 설명을 담고 있다(198d-199e). 앎의 무시간성은 그 앎의 대상들의 영원성을 수반한다는 것, 그리고 영원성은, 생겨나고 사라지는 사물들의 감소하는 실재성에 반대되는 것으로서 완전한 실재성을 수반한다는 것이 주장되어 왔다. 이러한 언급은, 만일 건전하다면, 단일론 명제를 뒷받침할 것이다. 그러나 분명히 앎이 과거, 현재, 미래에 적용되는 것으로서 무시간적이라고 주장하는 것은, 그것이 그러므로 반드시 영원한 대상들을 지녀야만 한다는 것과 또 다른 것이다; 플라톤이 이러한 주장들을 초기 대화편들 안에서 연계시켰었다는 것을 보여주는 그 어떤 증거도 없다.

5) 다시, 『뤼시스』가 있다. 거기에서 모든 가치있는 것들은 제일 가치있는 것의 상들인 것으로 이야기된다. 그리고 그런 식으로 결핍을 시사하는 것으로서 묘사된다. 『파이돈』이 꼴의 각각이 현존하고, 다른 것들은, 그것들이 그것들을 나눠 갖게 되기 때문에, 그것들 이후에 이름지어진다'고 주장하듯, 그렇게 『뤼시스』는 '사물들에 대해 가치 있는 어떤 것은 가치 있는 다른 어떤 것 때문이라고 이야기함에 있어서, 우리는 단순한 말 바꾸기를 내놓고 있는 듯이 보인다; 정말로 가치있는 것은 그 안에 소위 가치있다는 이러한 모든 것들이 당도하는 그러한 것 자체이다'(『뤼시스』 220a-b)라고 주장한다. 여기에서 문제가 될 듯이 보이는 것은 이름들 또는 어원의 제일 명칭과 파생 명칭이고, 이것은 두 세계의 원리에 속하는 중요한 언어적 귀결이다. 그러나 『뤼시스』에서 가치 있는 사물들의 제일 가치있는 것에 대한 관계는 꼴들의 예증들의 꼴들에 대한 관계가 아닌데, 왜냐하면 앞선 관계는 후자는 그렇지 않지만 목적들에 대한 수단들의 관계이기 때문이다; 경건한 것들은 경건함에 의해 경건하지만 그것들은 『뤼시스』에서 치료가 건강을 위해 가치있는 것처럼 경건함을 위해서 경건하지는 않다. 『뤼시스』에서 소크라테스는 윤리와 선택의 목표들에 관심을 두지 형이상학에 관심을 갖지는 않는다. 그래서 『뤼시스』는 나중에 『파이돈』에서 꼴들에 적용되었던 이름의 이론을 전제한다; 그러나 그 작품은 『파이돈』의 꼴들에 대한 이론을 전제하지는 않으며, 수단들의 목적들에 대한 관계에 있어서 수단들에 귀착되는 그 결핍은 특수자들의 결핍이 아니다. 『고르기아스』(467d)는 『뤼시스』의 요점을 또 다른 방식으로 놓는다: '만일 한 사람이 목적을 위해 어떤 것을 한다면, 그는 그가 하는 것을 바라는 게 아니라 그가 그것을 해서 이루려는 것을 바라는 것이다.'

6) 일반적으로 꼴들에 대한 이론의 단일론적 설명을 뒷받침하는 데에 인용되는 그 세 구절들은 그 설명이 참임을 수반하지 않는다. 그러나 고려되어야 할 또 다른 논증이 있다. 그것은 플라톤이 설령 초기 대화편들에서 꼴들에 대한 중기 이론을 진술하지 않았다 할지라도, 그가 그러므로 그것을 가지고 있지 않았다는 것은 침묵으로부터 언급으로의 논증이라는 것이다. 한 사람이 그가 쓸 때마다 매번 그가 아는 모든 것을 말할 필요는 없다.

7) 이 논증은 만일 초기 대화편들에 있어서 꼴들에 대한 이론이 꼴들의 예증들의 결핍과 감소되는 실재성을 시사한다는 것이 보일 수 있다면 더 많은 영향력을 지닐 것이다; 플라톤은 짐작컨데 최소한 우리만큼은 그런 함축들을 추적할 수 있었을 것이다. 그러나 그러한 함축은 존재하지 않는다. 반대로: 결핍과 감소되는 실재성은 중기 대화편들에서 앎의 문제들과 연관되고 초기 대화편들에서는 논의되지 않으며, 초기에 견지되는 주장들과 모순되는 그러한 문제들에 대한 해결책들과 연관된다.

8) 그래서 예를 들어 감소되는 실재성에 대한 주장은 『정체』에서 앎의 대상들을 의견의 대상들로부터 구별하는 수단으로 사용된다. 그러나 바로 그 앎과 의견의 구별은 『메논』(97a-98b)보다 더 앞에 놓이지 않는다; 그리고 앎과 의견이 상이한 대상들을 가진다는 주장은 『메논』에 부재할 뿐만 아니라, 의견은 거기에서 '근거에 대한 숙고'에 의해 앎으로 전환되기 때문에(98a; 85c 참조) 『메논』에 의해 암시적으로 반박된다.

9) 또 다시, 『파이돈』은 꼴들이 현존한다는 주장과 우리의 영혼들은 태어나기 전부터 존재했다는 주장이 동등하다(76d-e)고 견지하고, 이것의 배경이, 결국 꼴들에 대한 꼴들의 예시들의 결핍에 의존하는 상기의 원리이다. 그러나 초기 대화편들에서 소크라테스는 『파이돈』에서 이루어진 불멸성에 대한 주장들을 받아들이는 것과는 아주 동떨어진 - 만일 상기론과 결핍의 원리가 참이라면 거의 무시될 수 없는 주장들 - 불가지론자이다. 『변론』(20c-21d)에 있어서, 그의 무죄 선고에 표를 던졌던, 그리고 그의 개인적 관점들을 왜곡시킬 아무런 동기도 가지고 있지 않은 친구들과 동지들에게 말하면서, 소크라테스는 죽음에 대해 오로지 다음과 같은 것만을 주장하고자 하고 있다. 죽음을 좋은 것으로 생각할 이유가 있는데, 왜냐하면 그것은 아무것도 없음과 의식 없음, 깊고도 꿈조차 없는 잠과 같은 상태이거나 아니면 이 세계로부터 또 다른 세계로의 이행, 호머의 유행 이후 생각된 그러한 이행이기 때문이라는 것이다. 이러한 것들은 『파이돈』에서의 영혼이 불멸하며 죽음 이후에도 '오로지 그 자체로' 존재하리라는 뜨거운 신념으로 빛나는 사람의 말이 아니다.

10) 물론 플라톤의 사유에 대한 통일성이 있다; 그러나 그것은 금자탑의 단일성이 아니다. 그것은 성장과 발전의 단일성, 생명의 단일성이다.


변화를 위한 동기들

1) 만일 초기와 중기 대화편들이 꼴들의 지위들에 대한 그 대화편들의 평가에 있어서 다르다는 것이 사실이라면, 어떻게 그러한 차이가 설명될 것인지 물어볼 필요가 있다. 어떤 철학적 계기들이 그것을 촉진시켰는가? 초기와 중기 대화편들 양쪽에서 꼴들은 '분리'되어 있다. 그러나 '분리'는, 대화편들의 그러한 집단들 사이에서와 같이 상이한 의미들을 지닌다. 꼴들은 초기 대화편들에서 예증들로부터 구별되고 그보다 우선하는 것으로서 분리되어 있다. 중기 대화편들에서 그것들은 완전한 실재적 모범들, 그것들에 대해 그것들의 예증들이 결핍되어 있고 덜 실재적 예시들인 그러한 것으로서 분리되어 있다. 중기대화편들에서 플라톤이 답하고자 했던 존재론적 지위들에 대한 물음들이 초기에 물었던 것들과 다르다는 것이 뒤따라야 하고, 이것은 사실 참이다. 중기 대화편들에서 플라톤의 설명은 초기대화편들이 직면하지 않았던 인식론의 문제들에 의해 조건지어진다. 그러한 문제들은 회의주의와 선험적 지식을 두고 발생하였다. 그것들은 소크라테스적 변증에서가 아니라 그러한 변증에 대해서 일어났다; 특히, 그 문제들은 플라톤이 어떻게 소크라테스적 변증이 꼴들이나 본질들에 대한 탐구로서 가능한지에 대한 물음을 다루는 데에로 돌아섰을 때 일어났다.


회의주의와 존재론적 지위

1) 솔직한 사람은, 경건함'은 어떤 것이다' 또는 '실재하는 것이다'라고, 또는 그가 그의 행위들의 적절성을 결정하기 위한 기준으로 그것을 고려할 것이라고 이야기하게 됨에 있어서, 확실히 '좋아. 그게 어디 있는데?'라고 말할 것이라 기대된다. 그리고 그 자체로는 충분히 소박한 요구인 위치에 대한 이러한 요구는 진짜 물음을 감추고 있다: 만일 경건이 실재한다면, 그것은 어떤 종류의 실재성을 지녔는가? 만일 그것이 존재한다면, 그것으 어떻게 존재하는가?

2) 이러한 것들은 존재론적 지위에 대한 물음이고, 그러나 그것들은 관계들로서 비동일성이나 우선성을 듦으로써 답변될 것이 아니다. 왜냐하면 그것들은 꼴들, 그것들이 그러한 관계들 안에 존립할 것이라 추정되는 그러한 꼴들에 대한 물음들이기 때문이다. 그것들은 엄밀히 말해서 주어진 어떤 꼴이 무엇이냐에 대한 것이 아니라, 그것이 꼴이어야 할 어떤 것에 대한 물음들이다. Mr 리차드 로빈슨은 다음과 같이 지적한다:

중기 대화편들의 가장 대단한 단 하나의 혁신은 의심할 여지 없이 소크라테스가 그가 이전에 행했던 것처럼 특수한 '본질들' 또는 '꼴들'에 대해 어떠한지 묻는 대신에 이제는 '꼴들'의 전체에 대해서 이야기하기 시작했고, 그가 어떤 특수한 '꼴'에 대해서든 그러한 만큼 일반적 '꼴'의 본성에 대해 고려하게 된다는 것이다.

중기 대화편들은 일반성의 새로운 단계로 상승한다: 그것들은 그것이 꼴일 그러한 어떤 것을 묻고, 그 물음에 두 세계의 원리를 가지고 답한다. 『에우튀프론』과 다른 초기 대화편들에서 그 물음은 대답되기는커녕 일어나지도 않은 채로 남는다.

3) 그것은 일어나지 않은 채로 남겨지는데 왜냐하면 초기 대화편들에서 꼴들에 대한 이론은 비록 전문적이고 형이상학적인 이론이라 할지라도 본질적으로 일반 상식과 관련되기 때문이고, 그러므로 '그것은 무엇인가'라는 물음에 대하여 그 새로움을 감추는 자연스러움이 있기 때문이다. 우리 모두는 예를 들어 정확함과 같은 그런 어떤 것이 있다고 믿고, 그 정확함이 하나의 탁월함이라고 믿는다; 그리고, 이것을 믿는다면, 우리는 쉽사리 물음에 의해 탁월함인 그러한 어떤 것을 말하고자 시도하는 데에로 이끌릴 것이다. 소크라테스식 취급에 있어서, 이것은 고도로 복잡한 현존 전제, 정확함의 꼴이 있다는, 그리고 그것이 보편개념이며, 기준이고, 본질이라는 전제의 사용을 수반할 것이다; 만일 이 전제가 충분하다고 생각된다면, 정확함은 정확함의 예증들에 대해 우선할 것으로 그리고 그것들과 동일하지 않을 것으로 보일 것이다. 여전히, 그 전체 탐구는 무엇이 정확함인지를 결정하는 데에로 정향될 것이고, 그것이 있는지 아닌지로 향하지 않을 것이다. 논의의 진행은 모호함과 혼동으로부터 선명함으로의 이행으로 드러날 것이고, 보편에 속하는 정확함에 대한 숙고로서 드러나진 않을 것이다. 그리고 그러므로 탐구가 회의적 의심들에 의해 문제시되지 않은 채로 남는 한, 정확함과 같은 그러한 어떤 것이 있다는 상식적인 믿음은, 일반 상식이 소크라테스적 탐구에 의해 본질에 대한 숙고로 향해졌을 때조차, '그것은 어디에 있는가?'라는 물음을 제시하지 않을 것이다.

4) 그 이유는 '그것은 어디에 있는가?'라는 물음이, 그것이 존재론적 지위에 대한 설명을 요구할 때, 지적인 불만족의 표시라는 것이다. 정확함의 위치를 묻는 것은 무의미하다 - 무의미하다는 것은, 즉, 논박되는 그런 것이 있다는 무비판적 전제일 때까지 그렇다. 그리고 우리는 멈추도록 강제되고 설명을 내놓는다. 일반 상식은 그 설명을 요구하지 않을 것이다: 물론 정확함과 같은 그러한 것이 있다 - 정확한 모든 사람들을 보라. 소크라테스적 탐구가 그것을 요구하지도 않을 것이다: 『에우튀프론』에서 변증의 목표는 경건함의 꼴이 현존한다는 것을 증명하는 것, 또는 그것이 정의될 수 있다는 것이 아니다. 그러한 것들은 당연한 것으로 여겨진다. 목표는 오로지 그것을 정의하는 것이다 - 그것이 있는지 없는지가 아니라 그것이 무엇인지 말하는 것이다.

5) '그것은 무엇인가?'라는 물음과 구별되는 것으로서 '그것은 어디에 있는가?'라는 물음에 대한 대답은 꼴들의 현존이 전제되기 보다는 의심받을 바로 때에만, 오직 실재적 정의의 가능성, 그와 함께 변증의 추구가 당연시되기 보다는 도전받을 때 요청될 것으로 보인다. 이러한 물음은 『메논』에서 상기의 원리에 의해 제기된 선험적 앎의 문제에 의해 처음으로 플라톤의 정신에 강제되었다.


회의주의와 선험적 지식

1) 『메논』을 초중기 대화편으로 분류하는 것은 관습적이고 이것이 부당하지도 않다. 그 산문체 형식은 초기의 것과 구별할 수 없다; 그러나 그 저술의 상기와 영혼불멸에 대한 관심, 그것의 기하학으로부터 빌려온 가설적 방법론의 사용, 그리고 그것의 앎과 의견 사이의 구별은 『파이돈』과 『정체』에서 나중에 큰 부분을 차지하는 주제들에 대한 초기 진술들이다. 이것이 그러하기 때문에, 종종 『메논』이 기원전 387년 플라톤이 첫번째 이탈리아와 시칠리 여행으로부터 아테네로 돌아온 이후에 쓰여졌다고 생각되어 왔다; 이탈리아에 있는 동안, 그는 타렌툼의 아르퀴타스와 친밀한 교제를 나누고 있었다. 그는 피타고라스학파를 이끄는 당대의 수학자이자 철학자였다. 그러므로 『메논』이, 『파이돈』의 완성된 종합으로는 아직 소화되지 않은 새로운 물음들과 생각들에 대한 자극제를 시사하는 경계적 대화편이라 추측하는 것도 고려해 볼만 하다.

2) 『메논』은 여러 초기 대화편들이 시작한 것처럼 그렇게 '그것은 무엇인가?'라는 물음으로 시작한다. 메논은, 그것의 예시들을 구별하기 위해서가 아니라, 『에우튀프론』에서와 같이, 어떤 특성들이 그것과 관련되는지 결정하기 위해서 - 특히, 그것이 가르쳐질 수 있는지 아닌지, 또는 훈련을 통해 습득되는지 아닌지, 또는 본성에 의해 갖추어져 있는지 아닌지 결정하기 위해서, 탁월함이 무엇인지 말하도록 요청받는다. 초기 대화편들에서와 같이, 소크라테스는 재빨리 그의 응답을 당혹스러운 것으로 만들어 놓는다; 그러나 그 논의는 초기에 그러하였듯 거기에서 그치지 않는다. '그것이 무엇인가?'라는 물음에 답하는 데에 실패한 메논에게, 거기에는 그것을 물음에 있어서 아무런 요점도 없으리라 제안한다. 소크라테스는 그의 논증을 요약한다 (80e):

한 사람에게 그가 아는 것에 대해서나 그가 알지 못하는 것에 대해서 탐구해 들어가는 것은 불가능하다. 그는 그가 아는 것으로 탐구해 들어갈 수 없는데, 왜냐하면 그는 그것을 알기 때문이고, 탐구의 필요성이 전혀 없기 때문이다. 그는 그가 알지 못하는 것으로 탐구해 들어갈 수 없는데, 왜냐하면 그는 그가 탐구해 들어갈 것이 어떤 것인지 알지 못하기 때문이다.

이 역설은 『에우튀데모스』에서 도입되었던 배움에 대한 바보같은 역설들과 일부 유사성을 지니고, 소크라테스는 그것을 논쟁술의 일부라고 분류한다(80e). 그러나 그 역설을 소피스트주의라고 일축해 버리는 것은 실수일 것이다. 왜냐하면 소크라테스는 그 역설을 해소하기 위해 『에우튀데모스』에서 하듯 그것의 논리를 교정하고자 시도하지 않고, 오히려 상기의 원리를 도입하기 때문이다. 진지한 대답은 진지한 물음들을 시사한다.

3) 어째서 소크라테스가 그 역설을 심각하게 받아들이는지 그 이유는 찾기 어렵지 않다. F. H. 브레들리는 일찍이 다음과 같이 언급했다:

대상에 대한 결핍, 그리고 더욱이 대상에 대한 추구는 특정한 의미에서 그 대상에 대한 앎을 시사한다. 만일 한 사람이 언제 소유물이 획득되거나 획득되지 않는지 그가 절대로 말할 수 없으리라 생각했다면 그는 당연히 절대로 추구하지 않을 것이다. 그 추구 속에서 그리고 그 추구에 의해 그는 그 자신을 그 반대되는 가정에 맡긴다. 그리고 그 가정은 어느 정도 어떤 의미에서 있는 소유물에 달려 있다. 자연스럽게, 나는 처음부터 철학자가 그가 앞서 내놓는 명제들을 지닌다는 것을 말하지 않는다. 나는 그의 행위가 그가 가정하지 않는 한 아무런 의미도 없다고, 혹은 만일 요청받는다면 전제할 것이라고, 그가 명제들을 가졌을 때, 그는 그것들에 대해 판단할 수 있고, 그래서 그것들이 그에게 실재성의 이상적 소유물에 두는지 아닌지 말할 수 있다고 말한다. …그러므로 합리적인 철학에서 오로지 회의주의만이 진리가 지금까지 그리고 실제로 도달되었다는 것을 부정하는 데에 스스로 자신을 가둬두어야만 한다.

메논의 역설은 마라톤 전투의 정확한 일자, 또는 어제 아침식사의 식단에 대한 조사와는 거의 관련이 없다. 그러나 소크라테스적 변증에서 관련되는 그런 종류의 탐구에는 관련이 많다. 『에우튀프론』에서 경건의 정의에 대한 탐구를 고려해 보자. 에우튀프론은 무엇이 경건인지 알았는가? 그러면 그 탐구는 무의미한데, 왜냐하면 만일 그가 안다면 그는 말할 수 있고, 변증은 시작되기도 전에 성공했을 것이기 때문이다. 아마도 그래서 에우튀프론은 경건이 무엇인지에 대해 무지하다. 그러나 어째서 그러면 그에게 그것을 정의하는 것을 요청하는가? 무지하기 때문에, 그는 소크라테스적 물음에 대한 하나의 대답이 다른 대답보다 더 적절하다는 결정을 내리기 위한 아무런 근거도 가지지 못할 것이고, 그가 언제 옳은 대답에 닿았는지 혹은 잘못된 것을 내놓았는지 말할 아무런 기준도 가지지 못할 것이다: 변증은 그것이 진실 이후 응답자들의 고유한 노력들에 의존하기 때문에, 단지 어둠 속을 더듬거리게 되고, 탐구의 긴장은 끊겨 버린다.

4) 메논 역설의 압박은 꼴들이 기준들이라는 소크라테스의 전제에 의해 경감되기 보다는 오히려 증대된다. 경건한 것들을 점검함으로써는 경건에 대한 앎을 얻을 수 없는데, 왜냐하면 경건에 대한 앎이 없이는 어떤 것들이 경건한 것들인지 결정할 수 없기 때문이다. 에우튀프론은 그러므로 '그것은 무엇인가?'라는 물음에 대한 답변을 위해 그의 일상 세계를 고려해 볼 수 없다. 용이한 지적 직관에 어떤 쉬운 호소가 있지도 않은데, 그것은 탐구를 수반하고 그 역설은 어떻게 그 탐구가 가능한지에 대한 설명을 요하기 때문이다.

5) 소크라테스는 그 역설을 두 뿔 사이로 빠져나감으로써 해결한다. 그 역설은 명백한 앎과 절대적 무지 사이의 이분법을 전제한다. 그렇지만 배움과 탐구가 상기라고 가정해 보자; 만일 그것이 그렇다면, 탐구하는 것은 이미 함축적으로 알려진 어떤 것을 명확한 의식으로 가져오는 것이다.

6) 그것은 이 설명이 변증의 평범한 부분으로서가 아니라 탁월함이 무엇인지 말하고자하는 시도로서, 그러나 어떻게 변증이 가능한지를 설명하고자 하는 시도로서 - 즉, 어떻게 '탁월함이란 무엇인가?'라는 물음이 합리적으로 물어지고 대답될 것인지를 설명하고자 하는 시도로서 내놓아진다는 주의와 관련이 있다. 그 역설은 이 가능성을 시험함으로써 분명한 형이상학적 화두를 던진다. 변증은 실재적 정의를 목표로 한다; 그것은 꼴들의 현존을 전제한다. 만일 변증이 탐구의 한 형식으로서 불가능하다면, 그것들에 대한 아무런 앎도 없기 때문에, 그 변증의 탐구 대상들이 현존한다는 것을 뒷받침하기 위한 어떠한 훌륭한 근거도 없다.

7) 상기에 대해 주어진 그 정당화는 잠정적으로는 충분하다. 기하학에 무지한 한 노예가 그의 발 밑 모래에 새겨진 도형들과 지적 질의의 원조 이외에 다른 아무런 도움도 없이 상당히 어려운 정리의 진리를 상기해 내도록 만들어진다. 이것은 상기의 진리를 보여준다. 그러나 소크라테스는 그 문제에 있어서 망설인다: 그는 상기의 원리가 참이라고 확신하지 않고, '만일 우리가 우리가 그에 대해 무지한 바로 그것에 대해 탐구하는 것을 우리의 의무라 믿는다면, 우리는 더 나은 사람들이 될 것이다'(『메논』86b)라는 것만을 확신할 따름이다. 이 망설임의 이유는 찾기 어렵지 않다. 『메논』에는 어떻게 우리가 상기된 앎을 얻는지에 대해서도, 그 상기된 앎의 대상들의 지위는 무엇일지에 대해서도 아무런 분명한 설명이 없다. 대중적 신화의 차원에서를 제외하면 꼴들에 대해서든 선재하는 영혼에 대해서든 그것에 대해 던져진 '그것은 어디에 있는가?'라는 물음에 대해 아무런 대답도 없다. 하지만 그 물음은 만일 상기설이 참이라면 피할 수 없다. 『메논』에서의 상기설은 변증이 진행되는 것을 허용하기 위한 목적만을 제공한다. 그것은 그 자체로 임시적으로(ad hoc) 불충분하다.


상기와 결핍

1) 『파이돈』에서, 하나의 다른 정신, 사물들이 종결된다고 생각한 사람의 정신이 일어나고 있다. 거기에서 상기에 대한 논의는 『메논』의 논증에 대한 요약과 함께 시작하지만(『파이돈』73a), 우리가 보았듯 꼴들에 관련된 측면에서 감각적인 것들의 결핍에 기초한 새로운 설명으로 진행한다. 『메논』에서 소크라테스적 변증을 정당화하기 위한 필요에 의해 촉발된 상기의 원리는 두 세계 사이의 심연이 다리놓아지는 수단이 되었다.

2) 이 새로운 존재론은 회의주의적 의심들에 대한 하나의 해결책이다. 그것은 아마도 확실히 기하학에 의해 플라톤의 정신에 제시되었을 것이다. 프로타고라스는 아마도 그의 소실된 작품 『수학에 대하여』에서 기하학은 물리적 세계에 적용 불가능한 것이라고 주장했다: 자연 속에는 유사물들이 있지만, 기하학자의 넓이 없는 선과 길이 없는 점들에 대해 어떠한 물리적 등가물들도 없다. 프로타고라스는 이로부터 '기하학은 그렇게나 나쁘다'라고 추론하였다. 플라톤은 다른 결론을 이끌어 냈다. 기하학은 앎이며, 선명하고 확실하다. 만일 물리적 세계가 기하학의 조건들을 만족시키지 않는다면, 그만큼 물리적 세계가 나쁜 것이다. 직선자와 콤파스를 가지고 작업하는 기하학자는 모래 위에 구조들을 형성한다. 그러나 그의 도구들이 물리적일지라도, 그리고 그의 언어 - 플라톤은 오해의 소지가 있다고 생각했다 - 가 물리적 활동들의 언어일지라도, 그가 획득하는 진리는 물리적 대상들에 대한 진리가 아니다; 그의 구조물들은 오히려 그의 사유가 그것을 향해 정향된 그러한 것들의 상들, 부정확한 유사물들이다. 그래서 예를 들어 기하학적 선은 기하학적 원에 대해 하나의 점에서 그리고 오로지 그 한 점에서만 접선이다. 그러나 물리적 선은 물리적 원을 한 점에서가 아니라 직선의 부분과 호를 통해 건드린다. 곡선과 직선은 거리에 있어서 일치한다 - 기하학적 부조리이자 물리적 사실이다. 그래서 기하학의 정리들은 그것들이 그에 의해간접적으로 증명되는 그러한 모래 위의 구조물들에 대해서만 단발적으로 맞다. 이것은 그렇게, 기하학자의 도구들이 너무 무디기 때문이라거나 그의 작업들이 너무 어설프기 때문이 아니라, 그의 정리들이 견지되는 조건들이 그 자체로 물리적으로라기 보다는 기하학적으로 정의되기 때문이다; 물리적 대상들, 기하학적 서술들에 의해 그것들인 것으로 한정되는 그 대상들은 그러한 서술들에 의해 시사되는 조건들을 만족시키는 데에 실패한다. 감각적 도해들은 기하학적 관계들의 닮음꼴들이다 - 그 닮음꼴들이 우리에게 그 관계들을 상기시킨다. 그리고 그것들은 훌륭한 닮음꼴들인데, 다른 식으로는 그것들이 정리들을 발견함에 있어서나 증명들을 구성함에 있어서 유용하지 않을 것이기 때문이다. 그러나 그것들은 또한 결핍된 닮음꼴들이다. 그것들의 결핍은 정도의 일종이 아니라 종류의 하나이다; 물리적 도형들은 원칙적으로 기하학적 도형들의 대응물이 될 수 없는 유사물들이다.

3) 꼴들의 이론에 대한 이것의 관련성은 직접적이다. 『파이돈』에서 플라톤은 동등성의 감각적 예증들, 또한 비동등성의 예증들이기도 한 그 예증들은 그러므로 결핍되게 동등성과 유사하다고 전제하고, 이 판단이 비교에 의존하기 때문에, 우리는 동등성 그 자체에서 그 자체에 대한 우선적인 앎을 가졌어야만 한다고 전제한다. 이 논증이 의존하는 토대가, 기하학에 있어서 물리적 도형들의 유사한 결핍성에 의해, 최소한 부분적으로라도 제시되었어야만 한다.

4) 유비는 등가가 아니다. 물리적 원들과 기하학적 원들의 관계는 어떤 종류의 원들과 원형성의 관계와는 다르다. 그러나 기하학이 플라톤에게 있어서 형이상학을 해명해 주었고, 상기에 의해 제기된 존재론적 지위의 문제들에 대해 지대한 영향을 끼치는 대답을 제시했다는 사실은 남는다.


5. 결론

1) 1663년 하버드 대학에서 학위 수여식 직전에 학위 논문 한 편이 유포되었다. 오래 전 행복했던 때에 미친듯이 먹고 마시던 하버드 학위 수여식은 공식적 토론들에 의해 마침표가 찍혔다. 그 논문은 화제들을 제공했다. 그것은 논쟁을 위한 다음과 같은 명제를 제공했다. Ethica est vitiorum Emplastrum corrosivum. 거칠게 옮기자면 그것은 도덕 철학에 대한 연구가 악덕을 위한 허술한 고약이란 뜻이다. 중세 삼학(trivium; 문법, 논리학, 수사학)과 4과(quadrivium; 산술, 음악, 기하, 천문)의 윤곽들을 여전히 보존하고 있던 교육과정의 대학에 걸맞게, 문법 과학은 어느 하나 잊혀지지 않았다; 학생들은 Ha Ha He vox est bilaris bene Nota, 그러니까 Ha Ha He가 잘 알려진 웃음소리를 지시하는 표현인지 아닌지를 토론하도록 요구받았다. 그러나 무엇보다도 3학의 중심은 논리학이었고, 논리학은 보편개념들에 대한 고대의 고결한 문제들과 관련이 있는 한 명제 속에 기억되었다: Universalia sunt in se ἀειφανεῖς in re ἀφανεῖς Asterismi - '보편개념들은, 그 자체 내에서는 언제나 빛나는, 그러나 사물들 속에서는 보이지 않는 작은 별들이다.'

2) 물론 그 학위 논문은 가짜 학위 수여식을 벌이려고 작정을 한 학부 3학년 무리에 의해 퍼뜨려진 풍자물이었다. 그 날조차 학부생들은 시간 낭비에 기발한 재능을 가지고 학구적 예절에 대한 어떤 불경을 위해 합심하였다. 의심할 것 없이 그 모든 것은 그들의 학부에 중요한 원천이었다 - 그들의 학부의 여섯 전부는 그들의 우두머리인 찰스 촌시 총장과 함께였다.

3) 1663년 학부 3학년생들은 어디에서나 천체의 음악과 함께 언제나 빛나고 보이지 않는 그들의 별들의 빛을 연관지었을 것이다. 그들은 알았을 것이다, 로렌조가 그들보다 먼저 알았고 제시카에게 말했던 것처럼, '그대에게 보이는 가장 작은 별조차 그 궤적 안에서 천사와 같이 노래하지 않는 별이 없소, 어린 눈을 한 천사들에 따라 합창 하는 동안은.(셰익스피어 『베니스의 상인』, 4막 1장).' 그리고 그들은 소리를 가지고 있듯 바로 그렇게 빛을 가지고 있다고 추정하기 쉽다는 것을 알아차렸을 것이다. 작은 별들의 그 빛은 볼 수 없는 것이다, 그 별들의 운행의 음악이 들을 수 없듯이, 왜냐하면, '이 스러져 버릴 질척이는 육신이 그 빛을 지독하게 뒤덮어 가둔 동안은'(출처 상동), 우리는 그것을 볼 수 없기 때문이다.

4) 꽤 괜찮은 생각이다. 그러나 가짜 학위수여식에서의 그 무리는 까불거림을 증명한 듯하고, 당연히 일부 순전한 영혼에게는, 훌륭한 시를 훼손시켜가면서, 그 작은 별들의 빛은 사실 구경꾼의 영혼의 눈이 깜빡이는 것이었다고 주장하게 되는 일이 생긴다. 유명론자들은 거지 같이 언제나 우리 곁에 있다.

5) 이러한 회의주의는 물음의 특정한 분할을 제안한다. 물론 별들에 대한 물음이 있다; 그것은 철학자들과 논리학자들이 여전히 토론하고 있는 것이거나, 최소한 어째서 그들이 그것을 토론하지 않아야 하는지 토론하는 것이다. 그러나 또한 깜빡임에 대한 다양한 물음들도 있다 - 그것을 누가 가졌는지에 대해, 그리고 어째서 그가 그것을 가졌는지에 대해, 언제 그러한지에 대해. 내가 이 책에서 논의했던 것이 그러한 한 가지 물음일 것이다.

6) 나는 주장했다. - 아마 이 점에 대해서는 되풀이할 필요도 없을 것이다. - 플라톤의  초기 대화편들은, 그리고 특히나 『에우튀프론』은 꼴들에 대한 이론을 담고 있다는 것, 그리고 더 나아가서 그 이론은 『파이돈』과 『정체』에서 발견되는 그 꼴들에 대한 이론과 동일시되는 것이 아니라는 것. 초기 대화편들 내에서 꼴들에 대한 이론은 그 꼴들의 탐구에 속하는 대상들로서 존재들의 구분되는 계층이 현존한다고 전제하는 소크라테스적 변증의 토대이다. 그렇지만 중기 대화편들에서 꼴들에 대한 이론은 초기 대화편들이 보여줄 수 있는 그 어떤 것과도 달리 존재론적 지위에 대한 하나의 설명을 산출하기 위해 제공된다; 그 설명이 답변으로 여겨지는 그러한 물음들은 소크라테스적 변증 안에서 일어나는 물음들이 아니라, 소크라테스적 변증에 대해서 던져지는 물음들이다 - 특히, 실재적 정의들에 대한 탐구와 같은 그 변증의 가능성에 대해서. 이러한 물음들은 탐구의 역설에 의해 『메논』에서 처음으로 발생하고 상기의 원리에 의해 해소된다; 그래서 변증의 수행은 처음으로 선험적 지식의 소유를 암시하는 듯이 보인다. 『메논』에서 임시적으로 제공된 이러한 주장은 『파이돈』과 『정체』에서 결핍과 실재성의 정도에 대한 하나의 새로운 존재론에 기초된다.

7) 중기 대화편들에서 발견되는 꼴들에 대한 이론은 그래서 초기 대화편들에서 발견되는 이론과 동일한 것도 아니고, 별개의 한 가지 것도 아니다. 다르지 않다. 왜냐하면 그것은 초기 이론을 일부로서 탐고 있기 때문이다. 같지 않다. 그것은 초기 대화편들이 유발시키지 않은 화두들로 정향되기 때문이다. 그러나 만일 우리가 『에우튀프론』에서 플라톤의 깜빡임의 가득한 광채를 발견하지 않는다면, 우리는 알 수 있는 플라톤적 반짝임을 확실히 찾을 것이다. 그리고 아마도 그것으로 충분할 것이다.


-蟲-

P.S 아, 초벌번역 정말 개떡이지만 어쨌든 하나 끝.
4. 꼴들에 대한 이론의 존재론적 지위와 발전


1) 만일 꼴들이 존재하는 것이라면, 그것들이 어떻게 존재하는지, 또는 그것들의 존재론적 지위는 무엇인지 묻는 것은 합당하다. 중기 대화편들에서, 특히 『파이돈』과 『정체』에서, 꼴들과 그것들의 사례들 사이의 관계는 근본적 의존과 근본적 분리에 속하는 하나로 해석된다: 그 내용물들이 시간에 휩쓸리지 않는 새로운 세계가 발견되었고, 그 세계는, 감각적 현상계에 대해 덜한 실재에 대해 더한 실재가 서듯, 실체들이 그림자들과 반영물들에 마주서듯 그렇게 마주서서, 존재하는 모든 것의 토대가 된다. 체르니스 교수는 중기 대화편들의 존재론을 경탄스럽게 간추려 낸다:

현상적 세계, 전체로서도 그 부분들 모두에 있어서도 지속적으로 과정 속에 있는 그 세계는 앎의 대상인 실재일 수 없다. 인간 행위, 정신적 활동, 물리적 작용의 판이하게 다른 현상들은 각기 그리고 모두 오로지 현상적 과정 외부에 행위의 표준들, 과정의 경계들, 앎의 객관적 상관물들인 실재적 실체들이 존재한다는 전제에서만 설명될 수 있다. 이러한 실체들은 idea들이다….

이 설명은 체르니스 교수가 다른 어떤 곳에서 이데아들에 대한 이론의 철학적 경제성이라 불렀던 것을 기술한다; 그 이론이 많은 문들에 대한 하나의 열쇠이고, 동시에 윤리에 속하는 문제들에 대한 해결책이며, 형이상학이고, 인식론이기도 하다는 것이다.

2) 중기 대화편들에 있어서 이 이론의 존재는 그 뒤로 그림자를 드리웠다. 그것은 초기 대화편들에서 꼴들에 대한 이론을 찾은 학생들을 이 이론을 그 곳에서 찾도록 이끌었다. 그것은 초기 대화편들에서 이 이론을 찾는 데 실패한 학생들을 그 곳에 어떠한 이론이라도 있다는 걸 부정하도록 이끌었다.

3) 세 가지 주된 관점들이 주장되어 왔다. 하나는 초기와 중기 대화편들이 여기에서 동일하다는 것이다: 양쪽 모두에서, 꼴들은 존재하고 그것들을 지닌 사물들과 분리되어 있다; 『에우튀프론』에는 꼴들에 대한 이론이 있고, 그것은 본질적으로 『파이돈』과 『정체』에서 제기되고 후기에 『파르메니데스』에서 비판받은 동일한 이론이다. 두번째 관점은 초기 대화편들에서 꼴들이 있다손 치더라도, 그것들은 분리되어 있지 않은데, 사물들 '안에' 있고 그것들로부터 '떨어져' 있지는 않기 때문이라는 관점이다. 세번째 관점은 꼴들에 대한 언어가 초기 대화편들에서 발견된다 할지라도, 꼴들 그 자체는 없다는 것이다: 『파이돈』과 『정체』에서 제기되고 후기에 『파르메니데스』에서 비판받은 그 이론은 전혀 새로운 것이고, 플라톤은 『에우튀프론』을 썼을 당시 그것의 - 그 어휘를 제외하고는 -  어떤 부분도 고안하지 않았었다.

4) 또한 네번째 관점이 있다. 그 관점은 작금의 목적들을 위해 세번째 관점의 변형으로 다루어질 것이다. 그 관점은, 초기와 중기 대화편들 양쪽 모두에서, 꼴들이라고 불려왔던 것들이 단순히 언어적 술어들이고, 독립적 보편개념들이나 본질들이 아니며, 그래서 『에우튀프론』은 그것이 제기하지 않은 것에 대한 측면에서 『파이돈』과 『정체』에 동의한다는 것이다. 『파르메니데스』가 비판하지 않은 무엇이 현존하는지는 진술되지 않고 남는다.

5) 초기 대화편들에서 꼴들의 현존에 아무런 언급도 없다는 주장은 잘못이고, 더 진전된 논의 없이는 일축될 것이다. 초기 대화편들에서의 꼴들이 사물들 '안에' 있는지 그것들로부터 '떨어져' 있는지, '내재적'인지 '초월적인지'에 대한 물음은 여전히 남는다. 이 물음에 대해 아리스토텔레스의 증언이 종종 결정적인 것으로 생각되어 왔다. 비록 검토를 통해 그것이 결정적이지 않다는 것이 보여질 테지만 말이다.


아리스토텔레스 분리에 대하여

1) 아리스토텔레스는 『형이상학』에서 플라톤의 사유를 소크라테스의 사유와 다음과 같은 방식으로 구분한다:

소크라테스는… 윤리적 문제들에 전념하고 있었으며 전체로서의 본성의 세계를 도외시하고 있었지만, 이러한 윤리적 문제들에서 보편개념을 탐색하고 있었고, 정의에 대해서 처음으로 생각하게 되었다; 플라톤은 그의 가르침을 받아들였지만, 감각적인 사물들이 아니라 또 다른 종류의 실체들에 적용된 문제들을 주장했다 - 이러한 이유로, 공통 정의는 어떠한 감각적인 사물에 대해서도 그에 대한 정의일 수 없는데, 왜냐하면 그것들은 언제나 변하고 있을 것이기 때문이라고 주장했다. 이런 다른 종류의 사물들을 그래서 그는 이데아들이라 불렀고 감각적인 것들을 그는 이 이데아들 이후에, 이 이데아들에 대한 관계 때문에 명명되는 모든 것들이라 말했다; 다(多)는 그것들과 같은 이름을 가진 이데아들에 참여함으로써 존재했기 때문이다.

요점은 뒤에 다른 방식으로 놓인다:

그러나 소크라테스가 인격의 탁월함들에 몰두하고 있었을 때, 그리고 그것들과 관련하여 처음으로 보편적 정의의 문제를 불러일으키기 시작했을 때 … [그는] 보편개념들이나 정의들을 따로 떨어져 존재하도록 만들지 않았다; 그렇지만 그들[플라톤주의자들]은 그것들에 분리된 현존을 부여하였고, 이것이 그들이 이데아들이라 부른 종류의 것들이다.

그리고 다시:

그들[플라톤주의자들]은 감각세계에서 특수들이 유동상태에 있으며 그것들 중 그 어느 것도 유지되지 않는다고, 그러나 보편개념들은 이러한 것들로부터 떨어져 있고 그것들과 상이한 어떤 것이라고 생각했다. 그리고 소크라테스는 우리가 우리의 앞선 논의에서 이야기했듯이, 그의 정의들에 대한 논거로써 이 이론에 추진력을 보태 주었다. 그러나 그는 개별자들로부터 보편개념들을 분리시키지 않았다; 그리고 이 점에서, 그것들을 분리시키지 않음에 있어서 그는 옳게 생각했다. 이것은 그 결론들로부터 명백하다; 보편개념 없이 앎을 얻는 것은 불가능하지만, 그 분리는 이데아들에 대해 고려함과 함께 일어나는 반대들의 원인이기 때문이다. 그렇지만 만일 감각적이고 일시적인 실체들 이외에 어떤 실체들일 것들이 있다면, 그것들이 분리될 수 있어야만 한다는 것을 필연적인 것으로 다루는 그의 후계자들은, 다른 아무것도 가지지 않았으나, 이러한 보편적으로 서술되는 실체들에 분리된 현존을 부여했고, 그래서 보편개념들과 개별자들은 거의 같은 종류의 것이라는 것이 뒤따랐다.

이데아들이 χωριστά, '분리되어 있는 것들이었다'라고 말함으로써 아리스토텔레스는 무엇을 의미하는가? 그는 무엇보다도 그것들이 그것들의 예시들과 수적으로 구분되는 것으로서 분리되어 있었다는 것을 말한다 - 즉, 그 자체로 개별적이고 수에 있어서 하나인 것으로서 말이다: 플라톤주의자들은, '동시에 이데아들을 보편개념으로 만들면서 다시 그것들을 분리될 수 있는 것으로서 그리고 개별자들로서 다룬다'. 이것은 꼴들의 이론에 대한 아리스토텔레스의 비판에서 중요한 전제인데, 왜냐하면 아리스토텔레스는 보편개념은 개별자일 수 없다고, 다자에 대해 서술되는 것은 그 자체로는 많은 것들 중의 하나인 것일 수 없다고 생각했기 때문이다: '개별적이며 수적으로 하나인 사물들은 예외 없이 어떤 주어에 대해서도 이야기될 수 없다.'

2) 그렇지만 수적 구별은 분리의 필요조건이지 충분조건은 아니다. 『범주론』1a 24-25에서 아리스토텔레스는 '"주어에 있어서'라는 말로 내가 의미하는 것은 어떤 것 안에 있는, 그러나 일부분으로서 있지는 않은, 그리고 그것이 안에 있는 어떤 것으로부터 따로 떨어져 (χωρίς) 존재할 수 없는 것이다'라고 언급한다. 이 문서는, '아무것도 [개별적이며 수에 있어서 하나인 것을] 주어에 속해 현존하는 것으로부터 제한하지 않는다'(1b 7-8)라는 아리스토텔레스의 이후 언급과 마찬가지로, 분리는 수적 구별만큼이나 현존적 독립성 또한 시사한다고 주장한다. 이러한 독립성은 배분적인 것으로 해석되어야만 하고, 집합적으로 해석되어서는 안 된다; 왜냐하면 '주어 안에서'라는 말로 아리스토텔레스는 단지 '어떤 주어와 떨어져 존재할 수 없는'이 아니라 '그것이 속한 특정 주어로부터 떨어져 존재할 수 없는'을 의미하기 때문이다. 그래서 이데아들이 분리되어 있다고 주장함에 있어서 아리스토텔레스는 그것들이 개별자들이라고, 그리고 그것들이 어떤 주어진 사례들에 대해서도 독립적으로 존재한다고 말하는 것이다. 이데아의 현존은 그것의 사례들의 현존조건이고, 그렇게 정의된 분리는 비대칭적 관계성을 필요로 한다. 그것은 존재론적 우선성이다:

어떤 것들은 우선적이라고 불리고 후부적이라고 불리는데 본성과 실체의 측면에서 그러하다. 즉 다른 것들 없이 있을 수 있는 것들과, 반면에 그것들 없이는 있을 수 없는 그 외의 다른 것들 - 플라톤이 처음 사용했던 구별이다.

플라톤이 이데아들을 분리시켰다고 주장함으로써, 그래서, 아리스토텔레스는 이데아들이 수적으로 그것들의 예시들과 구분되고, 그것들의 예시들로부터 독립적으로 존재하며, 그것들의 예시들에 존재론적으로 우선한다고 말하는 것이다. 이에 대해 아리스토텔레스는 τὸ εἶδος τὸ ἔνον에 대한 그 자신의 이론을 반대했다. 그 이론은 물질적 실체의 꼴은 그것이 꼴인 것의 꼴과 수적으로 구분되지도 독립적이지도 않다는 것이다.


소크라테스와 분리

1) 플라톤이 이데아들을 분리시켰다고 주장함에 있어서 아리스토텔레스는 물론 옳았다. 그러나 그는 소크라테스가 그것들을 분리시키지 않았다고 주장함으로써 무엇을 생각하였는가? 이것은 틀림없이 소크라테스가 그것들을 그것들의 예시들과 구별하지 않았다거나, 그가 그것들을 그것들의 예시들에 독립적이거나 우선하는 것으로 간주하지 않았다는 것을 의미한다.

2) 역사적 소크라테스에 대해 이것이 참인지 아닌지는 여기에서 문제가 되지 않는다; 우리의 뜻대로 그 증거를 가지고, 그 문제는 결정할 수 있기가 어렵다. 그러나 역사적 소크라테스에 대한 아리스토텔레스의 지식은 아마도 주로 초기 대화편들에서의 플라톤의 묘사로부터 도출되었고, 그러한 이유로 소크라테스가 이데아들을 분리시키지 않았다는 그의 주장은 종종 초기 대화편들에 대한 해석으로 다루어져 왔다. 만일, 그래서, 우리가 역사적 소크라테스를 초기 플라톤적 소크라테스와 동일시하는 것이 용인된다면, 그 물음은 초기 대화편들에 대한 이 해석이 정확한지 아닌지 하는 것이다.

3) 아리스토텔레스가 소크라테스가 이데아들을 분리시켰다는 것을 부정함에 있어서 소크라테스가 꼴들을 그것들의 예시들과 전혀 구별하지 않았다는 것을 의미한다고 생각할 훌륭한 이유가 있다. 『형이상학』에서 소크라테스의 정의들에 대한 탐색을 언급하고 나서, 나는, 아리스토텔레스가 계속해서 '플라톤은 그의 가르침을 받아들였지만, 그 문제는 감각적 사물들이 아니라 다른 종류의 실체들에 적용되었고 - 이런 이유로, 보편적 정의는 항상 변화하고 있을 그 어떤 감각적 사물에 대한 정의일 수도 없다고 주장했다'고 첨언한다. 이것은 소크라테스가 정의의 대상들을 감각적인 것들과 동일시했다는 것을 시사한다. 그것은 그가 꼴들을 그것들의 예시들과 구별하지 않았다고 말하는 또 다른 방식이다.

4) 그러나 소크라테스는 분명히 초기 대화편들에서 감각적인 것들을 정의하고 있지 않다: 도덕적 꼴들은 어떠한 감각적 예시들도 가지지 않는다는 사실로부터 완전히 별개로, 꼴들과 그것들의 예시들의 비동일성은 정의를 목표로 하는 모든 초기 대화편들 각각의 변증에 의해 전제되는데, 왜냐하면 정의는 우리가 보았듯 대상에 대한 설명이기 때문이고, 그 대상은 보편개념이기 때문이다. 비동일성은 또한 꼴들이 본질들이며 원인들, 그것들에 의해 사물들이 그것들인 바의 것들인 그 본질들과 원인들이라는 사실에 의해, 그리고 꼴들은 그 꼴들을 지니는 사물들과 지니지 않는 사물들이 무엇인지 결정하기 위한 기준들이라는 사실에 의해 시사된다. 비동일성이 암묵적 전제일 따름인 것만은 아니다: 『에우튀데모스』300e-301a에서, 그것은 하나의 사실로 진술된다. 디오뉘소도루스가 그 곳에서 소크라테스에게 아름다운 것들은 아름다움과 같은지 혹은 다른지 물을 때, 소크라테스는 비록 그것들에 어떤 아름다움이 현전한다 하더라도, 그것드은 아름다움 그 자체와 다르다고 답한다.

5) 아마도 그래서 아리스토텔레스는 소크라테스가 꼴들을 그것들의 예시들로부터 구별했다는 것을 부정하고 있는 것으로서가 아니라, 이것을 허용하고 그것들의 구별이 수적이었다는 것을, 꼴들이 그것들의 고유한 지위에 있으리란 것을 부정하는 것으로 이해되어야 할 것이다. 그렇지만 이것이 아리스토텔레스의 관점이었다는 아무런 문헌 증거도 없고, 그가 그것을 주장했으리라 추정하기도 어렵다. 그것은 소크라테스가 아리스토텔레스가 그 자신의 것이라 알았던 비동일화된 꼴의 관념: 꼴은 있는 것이지만 개별자도 아니고 수적으로 하나도 아닌 것이라는 관념에 소크라테스가 도달했었음을 시사한다. 물론 한 사물이 수에 있어서 하나일 수 있지만 아직은 아니라고 주장하는 것은 역설이고, 아리스토텔레스 자신은 그가 오로지 아카데메이아의 일원이었던 동안 일어난 술어와 관여의 문제, 특히, 생각하기로, 『파르메니데스』(131a-c)와 『필레보스』(15a-c)에서 플라톤에 의해 진술된 관여의 딜레마의 문제에 대해 오랜 숙고의 결론으로 그 역설을 받아들이도록 이끌렸다. 그러나 소크라테스는 꼴들이 개별적이지 않을 것이라 생각하기 어려웠을 것이다. 의심할 나위 없이 이것을 증명하는 문헌들이 나타나기는 어려울 것이다: 한 사람이 일반적으로 그에게 한 번도 떠오른 적 없는 관념들의 결여에 대해 증명하지는 않으니까. 그러나 꼴들의 현존을 전제하는 초기 대화편들의 구절들은 또한 습관적으로 그것들의 수적 하나임을 전제한다; 다른 대화편들을 볼 것도 없이 『프로타고라스』를 보면, 올바름은 거기에서 어떤 것이고(πρᾶγμα τι, 330c; 330d, 352d를 참조하라), 탁월함은 하나인 것이며(ἓν τι, 329c), 만일 탁월함들이 서로 구별된다면, 각각은 그것 자체의 특유한 본성과 실재성을 가질 것이다(τις ἴδιος οὐσία, 349b). 초기 대화편들에서 꼴들의 현존에 대한 언급은 그것들의 개별성에 대한 언급을 필요로 한다.

6) 아리스토텔레스가 소크라테스가 이데아들을 분리시켰다는 것을 부정함에 있어서 소크라테스가 꼴들을 그것들의 예시들로부터 구별했다는 의미하진 않았지만, 그가 그것들을 그것들의 예시들에 독립적인 것으로나 우선하는 것으로 간주하지 않았다는 것을 의미했을 것인지 아닌지 고려하는 것이 남는다. 다시 한 번 아리스토텔레스가 이렇게 말하는 것이라는 아무런 문헌적 증거도 없다; 그의 현대 주석가들과 달리, 그는 소크라테스적 꼴이 그 예시들 안에 '내재적'이라고 주장하지 않는다. 그러나 만일 아리스토텔레스가 이러한 말을 하는 게 아니라면, 그는, 적어도 초기 대화편들에 관한 한에서, 실수를 저지르게 되었던 것이다; 왜냐하면 꼴들과 그것들의 예시들의 비동일성은 거기에서 꼴들의 그것들의 예시들에 대한 우선성과 관련되기 때문이다. 인식론적으로, 꼴들에 대한 앎은 그 꼴들을 가진 어떤 것들에 대한 앎에 우선하고, 또한 우연히 그것들과 관계된 어떤 특성들에 대한 앎에 우선한다. 존재론적으로, 꼴들의 우선성은 그것들이 어떤 것들이 그것들인 바의 것들이도록 하는 본질들이고 원인들이라는 사실에 의해 시사된다; 그것들의 현존은 그것들의 예시들의 현존조건이다. 그 우선성은 현존적 독립성을 시사한다. 만일 아버지를 고발하는 에우튀프론의 행위가 경건하다면, 경건으로서 그 행위의 현존은 경건함의 꼴, 그에 의해 그 행위가 경건한 바로 그 꼴의 현존에 의존한다; 경건함의 그 꼴이 그것의 예시를 위해 아버지를 고소하는 에우튀프론의 행위가 경건하다는 것에 의존한다고 생각하는 것은 그저 기괴한 생각일 따름일 것이다.

7) 요약하자면: 아리스토텔레스는 소크라테스가 이데아들을 분리시켰다는 것을 부정함에 있어서 아마도 소크라테스가 꼴들을 그것들의 감각적 예시들로부터 구별하지 않았다는 것을 말하고 있는 것일 터이다. 그가 소크라테스가 꼴들을 구별했지만 그것들을 개별자들로 구별하지는 않았다거나, 소크라테스가 꼴들을 구별했지만 그것들의 예시들에 독립적이고 우선하는 것으로 간주하지는 않았다는 것을 말하고 있을 것 같지는 않다. 그러나 이러한 것들 중 어느 것이 아리스토텔레스가 말한 것이든, 초기 대화편들에 적용된 그의 선언은 착오이다. 그가 제안하는 분리의 모든 기준들에 의해 - 비동일성, 개별성, 독립성과 우선성 - 꼴들은 후기 대화편들에서처럼 초기 대화편들에서 '분리된 것'으로 있다.


아리스토텔레스의 권위

1) 이 결론은 플라톤에 대한 주석가로서 아리스토텔레스의 권위에 대단한 신뢰를 두는데 익숙하게 되어 있는 그런 사람들 - 그들은 많기도 하고 - 에 의해 의문을 제기받을 것이다. 그래서, 예를 들어 Mr J. E. 레이븐은 『에우튀프론』6d-e가 두드러지게 꼴들의 이론에 대한 진술처럼 생각된다고 주장한 뒤, '이 물음에 대한 우리의 고대 증거의 모든 단편들 중 가장 중요하고 권위있는 것'에 기초하여 이러한 관념을 기각한다:

『형이상학』Μ, 1078b, 30에서 아리스토텔레스는 아주 많은 말을 적는다: '그러나 소크라테스가 그의 보편개념들을 분리될 수 있는 것으로도 간주하지 않고 그의 정의들도 그렇게 간주하지 않는 데에 반하여, 그들(즉 플라톤과 그의 추종자들)은 분리된 현존을 그것들에 귀착시켰고 이런 층위의 실재들에 이데아들이라는 이름을 부여하였다'. 내가 앞서 철학의 역사에 대한 소크라테스의 두 가지 기여는 귀납적 추리와 보편적 정의였다는 취지로 인용했던 구절에 거의 즉각적으로 뒤따르는 이 구절은 더 앞선 문장과 마찬가지로 사실에 대한 단순한 진술이다. 그러므로 이 예시에서 우리가 타당하게 여러 다른 문맥들에서 주장할 수 있듯이 그의 후계자들의 관점들에 대한 아리스토텔레스의 비판이 그 자신의 원칙들에 대해 앞선 사상가들에게서 그가 발견할 수 있는 어떤 예견들을 봄에 대한 그의 공언된 반대에 의해 무효화된다고 주장하는 것은 아무 소용이 없다. 여기에서 우리는 내가 말하는 것과 같이, 편견에 사로잡힌 비판이 아니라 사실에 대한 노골적인 진술을 가진다. 더욱이 그것은 다름 아닌 아리스토텔레스가 알 수 있는 모든 기회를 가졌다는 사실이다. 20여년이라는 최고조의 시기 동안 그는 아카데메이아에서 플라톤의 문하에 있었다. 그 시간 동안 플라톤과 소크라테스의 관계가 논의되지 않았다거나 아리스토텔레스가 이런 간략하지만 중대한 문장들을 썼을 때 실수를 저질렀으리라고는 생각할 수조차 없다.

이것을 『에우튀프론』6d-e의 해석에 적용함에 있어서 Mr. 레이븐은 아리스토텔레스의 역사적 소크라테스에 대한 설명이 또한 초기 대화편들의 소크라테스에 대해서도 지속된다고 전제한ㄷ; 그리고 만일 그러한 대화편들이 소크라테스에 대한 아리스토텔레스의 정보에 있어서 주요 원천이었다면, 이것은 불합리하다. 아직 그로써 아리스토텔레스의 주장들이 참이라는 것은 뒤따르기 어렵고, 만일 그것들이 실수라면, 그것들이 실수라는 건 아마도 생각해 볼 만할 것이다.

2) 플라톤의 문하에 있었던 아리스토텔레스가 그를 오해했을 수 없다는 논증은 무엇으로 이루어져야 하는가? 그것은 체르니스 교수에 의해 그 영향력이 유력하게 평가받아온 하나의 오래된 논증이다.

확실히 20여년 동안 플라톤의 학생이었으면서, [아리스토텔레스가] 그를 오해했을 수 없다는 그 논증에는 아무런 타당성도 없는데, 왜냐하면 우리는 다른 물음들에 대해서 플라톤의 직계 제자들이 그의 의견들의 본성과 취지에 대해 의견이 갈렸고 그래서 우리는 한 직계 제자의 해석이 오로지 그 이유만으로 옳다고 주장하는 것을 멈추게 된다는 암시들을 가지기 때문이다.

크세노크라테스와, 그럴 듯이 보이듯, 헤르모도로스는 모두 아리스토텔레스의 설명들을 가지고 변화에 대한 관점들에 설명들을 제시했다. 놀랄 일도 아니다. 철학자들의 문하생들이 그들의 선생들에게 이해를 의지했으리라 주장하기 위해, 누군가는, 경험에 더 잘 기초된 주먹구구식과 같이, 그 문하생이 철학자로서 혼자 힘으로 더욱 독창적일 수록, 그에 대한 선생의 초기 영향력은 더욱 지배적이고, 결국 오해될 것이기 더욱 쉽다는 주장을 반대할 것이다: 새로운 상황들, 특히 어렵게 획득된 시점은 새롭고 가끔은 왜곡된 관점을 산출한다. 아리스토텔레스는 말할 필요도 없이 가장 독창적인 제자였다; 플라톤은 말할 것도 없이 그의 사유에 대한 지배적 영향력이었다.

3) 그래서 아마도 아리스토텔레스가 플라톤에 대해 그가 편향된 비판으로부터 구분되는 것으로서 '단순히 사실에 대한 진술'을 형성하고 있을 때 실수했다는 것만이 생각됨직한 것이다. 이 구분은 새로운 것도 아니고 그것의 반박도 그렇다:

반드시 [아리스토텔레스의] 보고들은 그의 해석들과 구별되는 것으로서 의심할 여지 없는 증언으로 간주되어야만 하는가? 그런 것은 A. E. 테일러와 그에 앞서, 우리는 아리스토텔레스가 우리에게 플라톤이 특정한 어떤 것을 말했다고 할 때에는 그를 믿어야만 하지만 플라톤이 의미했던 어떤 것이나 한 원칙의 역사적 기원이었던 어떤 것을 우리에게 말할 때에는 그렇지 않다고 말한 존 버넷에 의해 채용된 타당한 구별이다. 이러한 말끔한 구별의 타당성은, 그렇지만 가장 좋은 의도들에도 불구하고 증인의 증언이 사실에 대한 것으로서 사실에 대한 그의 고유한 무의식적 해석에 깊이 영향받는다는 것을 가르쳐주는일반적인 경험에 의해 상당히 손상된다. 구별의 적용가능성은 특히 아리스토텔레스의 경우에서 손상된다. 그는, 플라톤에 대한 설명을 의존했던 폰 슈타인공조차 그의 보고들과 그의 비판은 서로에 의해 편견을 가지게 된다고 주장했던 보고, 해석, 그리고 비판을 그렇게 뒤얽히게 만들었다. …더욱이, 다른 학자들은 아리스토텔레스는 그가 해석한 그 철학자의 원칙의 필연적 함축이라고 간주한 것을 드물지 않게 철학자의 입에 집어 넣는다고, 그리고 그는 우리의 의도에 종사하는 플라톤적 대화편들의 단지 해석들만이 아니라 잘못된 보고들을 제시한다고 주장해 왔다.

무엇보다도, 소크라테스가 이데아들을 분리시키지 않았다는 아리스토텔레스의 주장이 '사실에 대한 단순한 진술'이라고 말하는 것은 무슨 뜻인가? 플라톤이나 소크라테스의 다른 동료들로부터의 인용문이란 것인가? 이에 대한 아무런 증거도 없다. 그것이 초기 대화편들이나 아카데메이아적 전통에 대한 해석이 아니라는 것인가? 이러한 것들 중 어느 것에도 아무런 증거도 없다. 그리고 그런 증거를 찾는다는 것은 사실을 아직 뒷받침되어야 하는 해석으로부터 구별해내기 위한 근거를 요구할 것이다. 그래서 그것은 '사실에 대한 순전한 진술'이란 표현에 대해 할당될 의미가, 순전함에 대한 고려와 별개로, 단지 그 진술이 참이라는 것인 듯이 보일 것이다. 그것이 Mr. 레이븐이 확립시키고자 시도하고 있는 점이기 때문에, 그의 논증은 하나의 요청(라틴어 petitio)이다.

4) 아리스토텔레스는 의심할 것도 없이 플라톤의 문하생이었다. 그러나 그 다음으로 아리스토텔레스가 제공하는 플라톤에 대한 증언의 질을 고려해 보자. 이것은 아마도 『형이상학』(Ⅰ, ⅵ장)에서 플라톤의 원칙들에 대한 그의 첫번째 조사에 의해 견본의 목적들에 대해 묘사될 것이다.

5) 아리스토텔레스는 이 장에서 네 가지 구별되는 종류의 증거들을 제공한다: (1) 그는 플라톤이 변화하는 감각적 대상들을 그것들이 그 안에 참여하는 불변하는 꼴들로부터 '분리시켰다'는 것을 증명한다; 그리고 플라톤의 다른 직계제자들의 증언만큼이나 대화편들이 그의 주장에 대한 충분한 증거를 제공한다는 것을 증명한다. (2) 그는 플라톤이 꼴들과 감각적인 것들 사이에 수학의 대상들인 '중간자들'의 계층을 상정했다는 것을 증명한다; 그리고 만일 이것이 대화편들에 의해 뒷받침되지 않는다면, 최소한 그것을 주장하는 것으로 그렇게 해석되었던 산재된 구절들이 있다는 것을 증명한다. (3) 그는 '하나'와 '큼과 작음' 또는 '둘'로부터 파생된 이데아적 수들의 현존에 대해 증명하고 이것이 대화편들에 의해 직접적으로 긍정되거나 부정될 수 없다는 것을 증명한다. (4) 마지막으로, 그는 플라톤이 설명에 있어서 형상인과 질료인만을 사용했다는 것을 증명하고, 함축적으로 그의 사유에서 운동인과 목적인이 어떤 역할을 가진다는 것을 부정한다. 즉, 아리스토텔레스는 『파이드로스』와 『법률』에서 영혼을 자기운동으로 정의했고, 그것이 우주의 모든 다른 움직임의 원천일 것이라 주장했던 그 사람은 그의 철학 안에서 운동인을 위한 어떠한 자리도 가지고 있지 않았다고 주장한다. 그는 『파이돈』에서 감각적 대상들이 꼴들과 닮고자 애쓴다고 말했던, 그리고 『정체』에서 좋음을 현존과 가지성의 제 1 원리로 정의했던, 그리고 『티마이오스』에서 세계는 이성적 설계자의 작품이라고 단언했던 그 사람은 그의 철학에서 목적인을 위한 그 어떤 자리도 가지지 않았다고 주장한다. 이러한 증언은 기본적인 결함을 겪는다: 그것은 믿기 어려울 정도이다.

6) 믿을 수 없을 만치 먹음직스러운 것을 만드는 방법들이 있다. 예를 들어, 누군가는 플라톤의 그 글을 버릴 것이다. 아마도 '대중적인' 플라톤은 많든 적든 그의 참된 견해들인 듯이 호도하는 대화편들을 다중(틀림없이 『파르메니데스』를 즐겼던)을 위해 펴냈을 것이고, 반면에 '비전적인' 플라톤은 그의 더 내밀하고 진전된 이론들, '기록되지 않은 원리들'을 아카데메이아의 그의 제자들을 위해 따로 마련해 두었을 것이다. 우리는 그래서 아리스토텔레스에 기대서 비전적 플라톤은 『티마이오스』에서 성스런 장인을 선언하는데, 왜냐하면 비전적 소크라테스는 그것이 그 무리들에게 좋다고 생각했기 때문이다.

7) 이러한 접근은 최소한 한 가지 훌륭한 장점을 가진다: 일관되게 적용된다면, 플라톤 문헌에 대한 어떠한 호소도 그것을 반박할 수 없다. 이 사실의 잇점을 알아차린 그런 사람들은, 플라톤의 대화편들이 세계를 움직인 사유의 요체를 담고 있지만, 아리스토테레스의 증언으로부터 그의 믿음들의 재현은, 철학적으로 능란한 손들에 의해 떠맡아질 때조차, 일반적으로 어떤 횡설수설한 모습을 가진다는 더 나아간 사실에 의해 단념시켜질 것 같지 않다. 그렇지만 물론 아리스토텔레스의 증언이 극단적으로 부정확하다고, 프라톤 사유의 가장 근본적인 측면들에 대해서 부정확하다고 전제하는 것은 선호할만 하다. 그의 증언이 독립적 통제에 대항해 측정될 수 없을 때, 그것은 플라톤의 관점들에 대해 불충분한 근거이다.

8) 이로부터 만일 플라톤이 '기록되지 않은 원칙들'을 아카데메이아에서 선언했다면, 우리는 지금 아리스토텔레스의 증언에 기초해 그러한 원칙들이 무엇이었는지 결정할 수는 없다는 것이 뒤따를 것이다. 이것은 물론 '아리스토텔레스가 플라톤에 대해 대화편들로부터 입증될 수 없다고 말한 모든 것이 순수한 오해이거나 와전'이라고 말하는 것이 아니다. 오히려 그것은 아리스토텔레스가 플라톤에 대해 대화편들로부터 입증될 수 없다고 말한 모든 것이 순수한 오해나 와전이 아닐 것이라 알려질 수 없다고 말하는 것이다. 그 문제는 근거가 있다.

9) 이것은 아리스토텔레스가 고의적으로 또는 의식적으로 플라톤을 와전시켰다는 것을 말하는 것이 아니다; 그 관점은 터무니없다. 그러나 아리스토텔레스는 철학자로서, 헤겔과 같이, 앞선 철학자들의 작업을 변증적으로, 그의 고유한 관점들의 바런을 위한 디딤돌들로 사용했다; 그리고, 다시 한 번 헤겔처럼, 그는 그의 선구자들의 작업을 단지 그가 이제 단언함을 통해서 획득해 냈던 진리에 대한 예측으로만 간주하는 경향이 있었다. 이러한 모든 것은 의심할 여지 없이 정당하고 적절하다. 철학자들은 언제나 그들의 주제에 속한 역사를 그 주제의 발전을 위한 매개물로 사용해 왔고, 그들의 선구자들을 그들의 논증에 대한 당파로 만들어 왔다. 이러한 전략이 가치있는 철학을 산출해 낼 때 - 그것이 아리스토텔레스의 경우 확실히 그러하였듯이 - 그 전락은 추구할 가치가 있다. 그러나 이러한 방식으로 진행하는 것은 거의 불가피하게 역사적으로 건전하지 못한, 가끔 말도 안 되는 해석들로 이끄는데, 왜냐하면 그것은 사유의 더 앞선 기획들이 생경한 물음들과 전제들의 배경에 맞서 보여지기를 요구하기 때문이다. 이것은 어째서 아리스토텔레스의 증언이 그렇게 자주 그것이 지니는 그러한 특성을 지니는지의 이유이다; 그것이 다른 식이었더라면 그가 독창적인 철학자는 아니었을 것이라 추정할지도 모른다.

10) 어째서 아리스토텔레스의 증언이 플라톤에 대한 해석에 있어서 그렇게나 자주 과도한 무게를 부여받아 왔는지에 대해서는 다양한 이유들이 있다. 하나는 다방면에서 고전적 전통의 내구력이다. 플라톤에 대한 신플라톤주의 해석을 허용하는 사람들은 아리스토텔레스의 증언으로 그들의 관점들에 대한 증거로 가치있게 여길 것인데, 왜냐하면 아리스토텔레스의 눈을 통해 읽힌 플라톤은 그 안에 플로티누스에 속한 많은 것을 지니기 때문이다. 신플라톤주의 해석은 일반적으로 영어권 학자들 사이에서 널리 지지받지 않았다. 그들은 그 해석의 암시된 신비주의를 불신했다. 그것은 아직까지도 학문적 전통의 가장 오래된 부분인 주장이다. 중세와 르네상스 동안, 그것이 '플라톤주의'라는 말이 의미하는 것이었고, 부분적으로는, 의심할 것도 없이 헤겔과 독일 낭만주의의 유입으로 인하여, 그것은 유럽 학문의 상당부분에 있어서 주된 주제로 남는다. 아리스토텔레스의 증언에 대한 그것의 취급은 그 자체를 수 세대에 걸쳐 확립시켰고, 외관상으로는, 그것 자체의 독립적 생명력을 전개시켰다.

11) 두번째 이유는 특권화된 접근방식의 원칙이다: 플라톤이 아카데메이아에서 강의하였다는 것, 아리스토텔레스가 그에게서 들었다는 것, 그리고 아리스토텔레스의 증언은 그가 들었던 것에 대해, 그것의 모든 결점들과 더불어, 우리가 가진 전부라는 것이 전제된다. 이것이 의존하는 증거는 아리스토크세노스의 단 하나의 구절이다. 그 구절은 플라톤의 좋음에 대한 강의를 들으러 왔던 청중을 묘사한다:

그들 모두는 세상이 좋음이라 부르는 것들 부와 건강 또는 힘의 일부 또는 다른 것을 그 강의로부터 얻으리라는 확신을 가지고 왔다; - 결국, 일부 비범한 운명의 선물. 그러나 그들이 플라톤의 추론이 수학이었음을 알아차렸을 때 - 수들, 기하학, 천문학 - 그리고 결국에는 하나의 좋음이 있다는 결과에 대해 그들의 환멸이 완전했다고 생각된다. 그 결과는 다른 이들이 그것을 비방하는 동안 그것을 조소하였던 것이다.

그 강의는 공적으로 언급되었다: 아카데메이아의 구성원들은 그들이 얻을 것에 대해 준비되어 있었을 것이다. 그리고, 인기이는 반응이 주어졌다면, 그것은 만일 플라톤이 그것들 중 다수를 제공했을지라도 참으로 놀라운 것이었을 것이다. 그러나 어떤 반어적인 것이 있다. 그들의 건강을 증진시키고 그들의 근육들을 강하게 하며 빨리 부를 이루기 위해 온 청중에게 수학과 형이상학에 대해 강연하는 플라톤의 모습에서, 그 고유한 재미를 지니는 그런 반어법 말이다.

12) 종종 아리스토텔레스가 이데아 수들의 이론에 대한 플라톤의 지식을 획득했다고 주장되어 오곤 하였다. 그러나 플라톤은 물론 만일 그가 그러한 사유들을 후기 대화편들에 밝히지 않았다면 형이상학에 대한 그의 가장 심오한 사유들을 수학과 변증에 훈련되어 있지 않았던 대중적 청중들에게 밝히지 않았을 것이다 - 대화편들은 일반 일반적인 출판물로서는 지나치게 전문적이고 어렵게, 그 자신의 아카데메이아 구성원들을 고려하여 쓰였다. 이 강연으로 소급하는 '기록되지 않은 원칙들'은 어떤 상식적인 인간이 무지한 대중에게 알릴 종류의 것이 아니다 - 만일 그들이 한 상식적인 사람이 좌우간 그들에게 퍼뜨릴 수 있을 그러한 종류의 사람들이라면 더욱 그러하다. 그러나 이렇게 말하는 것은 플라톤이 궁극적인 형이상학적 관점들을 강연들을 통해 설명했다는 그 어떠한 증거도 전혀 없다고 말하는 것인데, 왜냐하면 아리스토크세노스는 오로지 우리가 지니는 강연에 대한 직접적인 증거만을 제공하기 때문이다.

13) 그러나 플라톤의 사유에 대한 제일의 목격자로서 아리스토텔레스를 사용함에 있어서 또 다른 더욱 파급력 있는 작업요소가 있다. 이것은 외관상 근절할 수 없는 심리주의이다. 그것은 철학의 역사에 대한 연구를 전염시킨다. 플라톤의 그 제자는 플라톤의 사유들에 대한 발견을 그의 반대로서 지닌다 - 즉, 그의 품어진 신념들이다. 그러나 플라톤이 썼던 것과 독립적인 증거의 또 다른 원천이 있다 - 즉 아리스토텔레스의 증언이다. 플라톤이 기록하였던 것이 이따금 그가 믿었다고 아리스토텔레스가 말하는 것과 부딪치기 때문에, 어떻게든 그 분란을 판단하는 것은 역사가의 작업이다. 이러한 부분적으로 모순되는 증거의 두 분류들에 무게를 두는 것은 균형을 깨려는 것이기 때문이다: 역사적 사실, 즉 플라톤이 실제로 믿었던 어떤 것을 표현하는 그 균형. 우리는 그래서, 그것의 재건이 많은 즐거움과 재미를 제공할 그런 비전적 원리들을 믿는 우리의 길에 이어서 진척되어 있다.

14) 그러나 무엇보다도 처리의 또 다른 방식을 선택하는 것이 가능하다. 플라톤의 문헌을 어떤 다른 것에 대한 증거로서가 아니라 그 자체로 역사적 이해의 제 1 대상으로서 다루는 것이 가능하다. 탐구의 목적은 그래서 일련의 문헌자료들을 해석하는 것이고, 그것들의 권위에 대해 간직된 신념들을 헤아리는 것은 아니다. 물론 그 문헌들이 그러한 신념들에 대한 의존할 만한 색인이라고 가정하는 것은 합리적이다; 그러나 무엇보다도 그 관련이 우연적이고, 해석이 관련된 한에서 중요치 않다. 만일 플라톤이 속으로는 유명론자, 무신론자, 불멸에 대한 회의주의자, 그리고 쾌락주의자였다면, 또 이제는 알려지지 않은 어떤 알 수 없는 동기로 썼던 대화편들을 아직 쓰고 있던 중이었다면, 이것은 그가 썼고 개인적으로 아주 조금도 믿지 않았던 어떤 것에 대한 적절한 해석으로 바뀔 것이다: 한 사람이 그가 믿지 않는 것을 말할 때, 우리는 여전히 완전하게 그가 말한 그러한 것을 잘 이해할 것이다. 그래서 플라톤이 그가 표현하지 않았던 신념들, 또는 그가 표현한 것에 반대되는 신념들을 가지고 있었는지 아닌지에 대한 물음은 예언의 예술에 속하는 기술을 가진 이들에게 남겨질 것이다; 그 역사가는 더욱 합리적으로 자기 자신을 문헌과 그것의 의미에 대한 해석연구로 제한할 것이다. 만일 탐구가 이런 식으로 해석된다면, 그것은플라톤에 대한 해석의 문제에서 아리스토텔레스의 증언을 플라톤 문헌의 증거보다 선호하는 자기-언급적 모순이다.

-蟲-
본질인과성

1) 꼴들은, 로크의 말을 빌리자면, 실재적 본질이며 명사적 본질이 아니다. 꼴들은 실재적인데 왜냐하면 그것들이 명사적으로 이루어질 수 없기 때문이다: 명칭들이나 단어들의 일상적 의미들에 대한 탐구는 그 탐구가 꼴들에 대한 탐구인 경우를 제외하고는 꼴들에 대한 탐구가 아니다. 그러나 그것들은 또한 그것들이 실제 사물들인 바의 그러한 것이기도 하기 때문이다. 로크는 다음과 같이 본다:

본질은 그것이 바로 그것인 바의 것인 어떠한 것이든 간에 그것의 존재에 대해 취해질 것이다. 그리고 그래서 실재는 내재적이며, 그러나 본질적으로는 일반적으로 알려지지 않고, 사물들의 발견 가능한 특질들이 의존하는 사물들의 구조는 그 사물들의 본질로 불릴 것이다. 이것은, 그것의 형성으로부터 명백하듯이, 그 단어의 적절한 의미이다; 그 기본 표기법에 있어서 적합하게 존재함을 의미하는 esentia.

로크의 설명에 의해, 본질은 '어떤 것의, 그로써 어떤 것이 그것인 바의 것인 존재'이다. 아리스토텔레스는 『형이상학』에서, 개별자의 실체적 꼴을 그 개별자의 실체와 동일시하면서, 본질을 본질을 가진 것의 존재와 동일시하면서 이러한 주장을 내놓았다. 그래서 그는 로크가 물려받았던 그 전통을 기초하였다.

2) 그러나 플라톤의 초기 대화편들에 있어서 꼴들은 그 있음에 대해 그 꼴들이 꼴들인 것의 있음이 아니다. 하나의 있음인 보편개념은 다수성의 있음이 될 수 없다 - 정확하게 아리스토텔레스가 왜 보편개념들로부터 실체적 꼴을 구별하는 데에로 이끌렸는지의 이유이다. 『에우튀프론』은 경건이 어떤 주어진 경건한 사물이나 경건해 보이는 행위의 있음이라고 시사하지 않는다; 그것은 오로지 경건이 그에 의해 경건한 것들이 경건한 것들인 그러한 것임을 시사할 따름이다. 그것은 또 다른 아리스토텔레스적 어휘를 조금 빌리자면, 경건이 원인이라는 것을 시사한다.

3) 『대히피아스』에는 이 전제를 명백하게 만드는 논증이 있다. 소크라테스는 히피아스가 올바름 어떤 것이다(ἔστι τι τοῦτο)라는 것, 그리고 이것이 지혜에 있어서도 마찬가지로 참이라는 것에 동의하도록 이끄는데, 왜냐하면, '올바른 것과 지혜로운 것 등등의 그러한 것들은 만일 그것들이 어떠한 것이 아니라면 그것들에 의한 그러한 것이 아닐 것이기 때문이다'. 아름다운 것들은 아름다움에 의해 아름답기 때문에, 히피아스는 아름다움도 어떠한 것이며, 그것들이 다른 말들이나 생각들 또는 개념들에 의해 아름다워지지는 않는다는 것에 동의할 수밖에 없다.

4) 이 논증은 분명히 『대히피아스』에만 놓이지만, 그것은 정의를 목표로 하는 모든 각각의 초기 대화편들에 의해 가정되는데, 왜냐하면 꼴들이 본질들이라는 그 전제는 변증에 본질적이기 때문이다: 꼴들은 정확히 보편개념들이고 표준들인데 왜냐하면 그것들이 그에 의해 사물들이 그러한 바의 것들인 것이기 때문이다.

5) 플라톤이 매우 자주 꼴들과 그 꼴들을 지닌 사물들 사이의 관계를 특징화하기 위해 인과적 언어를 사용하는 이유는 꼴들이 본질들이기 때문이다. 『뤼시스』(221c)에서 소크라테스는 '원인이 파괴될 때, 그 원인이 그에 대한 존재를 지속시키는 원인인 그러한 것은 물론 있을 수가 없다'고 지적한다. 경건은 이러한 의미에서 원인이다: 경건의 현존은 경건한 것들이 경건한 것들인 한에서 그것들의 현존에 필수적이다.

6) 초기 대화편들은 꼴들과 그 꼴들의 예시들 사이의 관계를 묘사하기 위한 다양한 은유들을 사용한다. 꼴들은 그 예시들 '안에' 있어야 할 것으로, 또는 그 예시들'에 현전하는' 것으로, 혹은 그것들에 '덧붙여져'야 할 것으로, 아니면 그것들'에' 있어야 할 것으로 이야기된다. 그 예시들은 다른 한편으로 꼴들을 '가지거나' '허용하거나' '취하거나' '나누어 가진다'. 이러한 은유들 전부 일상언어에 기초된다. 그것들은 정말로 평범하게 그리스어에서 추상적 명사들과 함께 사용될 것이다. 영어에서 한 행위에 용기가 있다(there is courage in an action)고, 또는 올바름의 현전이 상황을 개선시킬 것이라고, 어떤 사람이 다른 사람보다 더 훌륭한 인내의 도량을 지닌다고 말하는 것과 똑같이 말이다.

7) 은유들로서, 그리고 친숙한 그리스어로서, 이러한 용어들은 소피스트적 논쟁들의 재료를 제공했다. 예를 들어 『에우튀데모스』에서는 디오니소도루스가 아름다운 것이 아름다움과 다른 것인지 어떤지를 묻는다. 소크라테스가 잠시 망설인 후 아름다운 것들은 아름다움 자체(αὐτὸ τὸ καλόν)와 다른 것들이고, 그러나 그 아름다운 것들 각각은 그 안에 현전하는 어떤 아름다움(κάλλος τι)을 지닌다고 말할 때, 디오니소도루스는 재빨리 주도권을 잡아챈다. 그는 다음과 같이 말한다. '그러면 소 한 마리가 당신 곁에 있다면, 당신은 소로군요. 그리고 내가 당신 곁에 있으니, 당신은 디오니소도루스고요'(301a). 소크라테스는 '맙소사'라고 말한다. 당연하다. 그러나 그 물음은 하찮은 것이 아니다: '현전'은 은유 - 혹은 최소한 많은 용례들을 지닌 단어 - 이고 잘 다룰 방법은 그것의 의미에 달려 있다.

8) 문제가 되고 있는 '현전'은 특수한 종류의 것이다. 『뤼시스』에서 소크라테스는 어떻게 나쁨이 좋지도 나쁘지도 않은 어떤 것에 현전할 것인지 설명하는 일을 떠맡고, 그가 의미하는 것을 명확하게 하려는 그의 논의를 갑작스럽게 중단한다. '일부 사물들은 그 사물들에 현전하는 그러한 것 그 자체일 그런 것들이다; 다른 것들은 그렇지 않다.' 만일, 예를 들어 금발이 하얗게 칠해진다면, 흼은 그 머리카락에 현전할 것이지만, 그 머리카락은 그 자체로 흰 것은 아닐 터이다. 그러나 나이가 들어 머리카락이 흴 적에, 흼은, 그 머리카락이 그래서 하얗게 될 것이기 때문에, 다른 의미에서 머리칼에 현전할 것이다. 앞서의 경우, 흼은 머리카락에 현전한다. 뒤의 경우, 머리카락은 '흼의 현전에 의해 하얗다'.

9) 어떤 것이 흴 때 그 의미에서 흼이 그것에 현전하는 그러한 특별한 의미를 설명하기 위해, 소크라테스는 도구적 여격에 의존한다. 이러한 장치는 초기 대화편들에서 빈번하게 나타난다. 경건한 것들은 경건함에 의해 경건하고, 사려있는 자들은 사려에 의해 사려있으며, 아름다운 것들은 아름다움에 의해 아름답다. 아름다움은, 『대히피아스』(300a)에 따르자면 아름다움 것들을 아름답도록 만든다(ποιεῖν). 이러한 표현들은 일상어법과 결부된다. 이온은 그가 호메로스를 기릴 수 있음이 앎에 의한(도구적 여격) 것이라고 생각한다(『이온』541e). 소크라테스는 우리가 건강에 의해(ὑπό와 동작주의 속격) 유익해진다고 생각한다(『크리톤』280a). 사려는 만일 그 사려가 그들에게 현전하는 그러한 자들에게 좋다면 좋은 것이고, 아니라면 나쁜 것이다(『카르미데스』161a).

10) 그러나 비록 이러한 표현들이 일상언어에 결부된다 할지라도, 그것들은 종종 소크라테스에 의해 비일상적인 방식으로 사용된다. Ποιεῖν은 πάσχειν에 속하는 규칙동사이고, πάσχειν은 규칙적으로 '자격지어짐'을 의미할 것이다; 그러나 ποιεῖν은 '자격을 둠'의 의미로 규칙적으로 사용되지는 않는다. 더욱이, 지혜가 사람들을 복되게 만든다고 말하는 것과 지혜가 사람들을 지혜롭게 만든다고, 또는 지혜가 그에 의해 사람들이 지혜로워지는 것이라고 말하는 것 사이에는 주목할만한 논리적 차이가 있다.

11) 이러한 표현들은 인과적이다 - 흔히 당구공들의 부딪침과 연관되는 의미에서가 아니라 아리스토텔레스가 형상적 원인에 대해 말하는 의미와 유사한 의미에서 -. 아름다움은 정확히 이런 의미에서 아름다운 것들의 원인이다: 아름다움의 현존은 다른 것들의, 그것들이 아름다운 것들인 한에서 현존의 필연적 조건이다. 『대히피아스』에서 소크라테스가 사람들이 아름답다고 말하는 사물들은 오로지 아름다움 자체가 현존해야만 아름다우며, 히피아스가 아름다움을 아름다운 처녀로 정의했을 때, 그 이유로 아름다운 것들이 아름다운 그러한 어떤 것(δι᾿ ὅ)이 현존한다고 주장한 것은 그런 이유에서이다.

12) 그것들이 그에 의해 어떤 것들이 바로 그러한 것들인 바의 그러한 본질들이라는 의미에서, 그래서, 꼴들은 원인들이다. 그러하기 때문에, 『파이돈』(100c)에서 소크라테스는 아름다운 것들이 '다른 어떤 이유도 아니라 바로' 그것들이 아름다움에 참여하기 때문에 아름답다고 주장하며, 케베스에게 그가 이러한 종류의 원인(αἰτία)을 허용하는지 묻는다. F. M. 콘포드는 이 구절을 다음과 같이 설명한다:

'다른 어떤 이유도 아니라 바로 그 이유'라는 구절은 … 모호하다. '이유'는 '설명'(αἰτία의 관습적 용법)을 의미할 것이다. 그 전제는 그래서 '이 장미는 아름답다'라는 명제가 '이 장미는 아름다움에 참여한다'는 진술과 동등하다고 단언할 것이다: 나는 말들의 그 형식을 바꾸어 말함으로써 대체하고 그렇게 설명할 수 있다. 그러나 플라톤은 명제의 분석이 아니라 일치하는 사실에 대한 분석을 말하고 있는 것처럼 보인다. 그 이론은 그래서 이 사실이 (1)특정한 가시적인 것, 이 장미, (2)아름다움(Beautiful or Beauty), 그리고 (3)우리가 '~에 참여한다'를 대체할 수 있는, '~는 ~이다'로 표현되는 그 둘 사이의 관계라고 불러야 할 어떤 것으로 구성된다고 단언할 것이다. 그러나 다시 한 번 우리는, 지금까지, 오로지 하나의 설명만을 가진다: 이 장미가 아름답다는 사실은 이 장미가 아름다움에 참여한다는 사실과 똑같은 것이다. 우리는 그 사실을 존립하게 할 어떤 원인에 대해서도 아무것도 배우지 않는다. 어느 관점에서든 우리는 한 명제에 대한 분석이나 하나의 사실에 대한 분석만을 가질 따름이고, 참인 진술의 이유나 그 사실의 현존에 대한 원인을 가지지는 않는다.

첫번째 주장은 제외될 것이다. 당신은 말을 바꿈으로써 오로지 본래의 단어들이 의미하는 것에 대해 무지한 누군가에게 하나의 명제를 설명할 수 있을 뿐이고, '이 장미는 아름답다'라는 것을 이해하는 데에 실패한 그 누구도 그 표현이 '이 장미는 아름다움에 참여한다'를 의미한다고 이야기되어야 한다는 데에 대해서 분명하게 해줄 것을 찾지는 못할 것이다. αἰτία가 말 바꿈에 대해 사용될 수 있었다는 것을 보여주는 어떤 고대의 증거가 있는 것도 아니다. '다른 어떤 이유도 아니라 바로 그 이유'가 말 바꾸기를 시사하리라는 주장은 그래서 불충분하다.

13) 콘포드가 '일치하는 사실에 대한 분석'이라고 부른 것은 여전히 남아 있다. 그러나 만일 이 장미가 아름답다는 사실이 이 장미는 아름다움에 참여한다는 사실과 똑같은 사실이라면, 그가 인정하듯, 어떤 식으로 분석이 말 바꾸기와 다른지는 불분명하다. 어떤 경우든, 이것은 플라톤의 핵심을 놓친다. '이 장미는 아름답다'와 '이 장미는 아름다움의 꼴에 참여한다'는 실로 등가의 진술들이다 - 만일 꼴들에 대한 이론이 참이라면 말이다. 그러나 그 이론은 정확히 사실들의 구조에 있어서 존재론적 우선성에 대한 이론이지, 명제 등가들에 대한 이론이 아니다. 아름다움의 꼴의 현존은 이 장미를 아름답다고 부르기 위한 필요충분조건이다: 그 사물에 세례를 내리는데 그것에 있어서 그 특징 때문이고,(그 특징 자체와 구별되는 것으로서) 그 세례의 정당함을 보장하는 그 특징의 현존 때문이다. 꼴 자체는 그렇지만 오로지 장미의 아름다움에 대한 필요조건일 따름이다: 그것은 아름다움이 만일 그것이 현존하지 않는다면 장미가 아름다운 것으로서 현존하지 않았을 것이라는 점에서 그 아름다움의 원인이다. 그 꼴은, 꼴의 현존과 구별되는 것으로서 그 장미의 아름다움에 대한 충분조건이 아닌데, 왜냐하면 그 꼴은그 장미에 있어서 그 장미의 아름다움의 고유한 현존에 대한 충분조건은 아니기 때문이다: 그것은 아름다움의 현존뿐만 아니라 토양의 조건, 그리고 기후, 정원사의 기술에도 의존하기 때문이다. 여기에서 충분조건은 본질인과만이 아니라 작용인 또한 필요로 한다.

14) 그래서 꼴들은 그것들이 그것들에 의해 사물들이 그러한 것들인 바의 것들이라는 의미에서 원인들이다. 그것들은 그러므로 세계의 운행에 영향을 미친다. 만일 그것들이 존재하지 않았더라면, 세계는 세계가 그 세계인 어떤 것이 아니었으리란 의미에서 말이다.


지시의 재고

1) 초기 대화편들은 의미에 대한 지시적 이론을 전제하는데, 추상적 명사들 또는 그것들 중 일부가 대상들을 명명한다는 의미에서 그러하다. 이로부터 초기 대화편들이 이제는 이전보다 덜 널리 주장되는 그 명제, 어떤 것을 기술한다는 것은 서술을 명명하는 것을 필요로 한다는 명제를 전제한다는 것이 뒤따르진 않는다. 이 마지막 명제는 일반적으로 추가적인 명제와 관련되어 왔다: 잘 구성된(혹은 잘 분석된) 문장들의 구조는 반드시 문장이 표현하는 그러한 사실들에 대해 일대일 혹은 2방향 유일 상응에 존립해야만 한다는 명제이다. 잘 구성된 문장들은 술어들을 포함하기 때문에, 그 문장들이 표현하는 사실들은 반드시 유사-술어들을 포함해야만 한다; 그러나 그러한 유사 술어들이 보편개념들이나 본질들이라고 추정하는 것은 더 나아간 단계이고, 그러한 술어들은 그것들을 명명한다.

2) 이것은 플라톤이 주장하지 않은 명제이다. 초기 대화편들은 더욱이 그것과 관련된다고 거의 이야기될 수가 없다; 그 대화편들은 특정한 어떤 것들이 무엇인지에 대한 물음과 관련되지, 어떻게 그리고 어떤 방식으로 사물들이 사물들에 대해 이야기될 것인지에 대한 물음과는 관련이 없다. 그러나 만일 우리가 『파이돈』을 본다면, 우리는 어떤 것을 기술하는 일이 하나의 기술을 이름하는 것이 아니라 서술된 그것을 명명하는 것을 필요로 한다는 것을 알아차릴 것이다. '꼴들의 각각은 현존하고, 다른 것들은, 그것들이 그 꼴들의 제 몫을 가지게 되기 때문에, 그것들 이후에 이름지어진다'라는 것 때문이다. 이것은 소크라테스를 올바르다고 기술하는 것은 그에게 올바르다고 명명하는 것이고, 좌우간 올바름을 명명하는 것은 아니라고 강력하게 주장한다; 그 이름이 비록, 올바르게 적용된다면, 소크라테스가 올바름에 참여하기 때문에 적용한다 할지라도 말이다.

3) '올바른'과 '올바름' 사이의 관계는 아리스토텔레스가 아카데메이아적 구별이었을 것을 사용하면서, 동근어라고 불렀던 것의 한 예시이다: '사물들이 상이한 어미들로 어떤 것으로부터 그것들의 이름을 획득할 때, 그것들은 동근어들이라 불린다. 그래서, 예를 들어 문법학자(grammarian)는 그의 이름을 문법(grammar)로부터 획득하고, 용감한 자들은 그들의 것을 용감으로부터 취한다.' 그래서 소크라테스를 올바르다고 명명함에 있어서 우리는 그를 동원어적으로 명명한다: 그는 그의 이름을 올바름으로부터 얻는데, 어미상의 차이를 가지고 그렇게 한다. 그러나 소크라테스가 참여하고 있는 꼴의 이름은 소크라테스가 그 꼴로부터 얻은 이름과 문법적 어미에 있어서 다를 필요가 없고, 플라톤의 고유한 사용에 따르고 소크라테스가 어원적으로 명명된다고 말하는 것이 더 나을 것이다. 만일 꼴들에 대한 이론이 참이라면, 그래서 '소크라테스는 올바르다'와 '소크라테스는 올바름에 참여한다'는 등가 명제들이다; 서로가 서로를 수반한다. 그러나 첫번째 것은 동일한 것에 대한 두 가지 이름들을 포함하는데, 하나는 어원이고, 반면에 다른 하나는 두 가지 것들에 대해 두 가지 이름들을 포함하고, 그 중 어느 것도 어원이 아니다. 두 경우 모두에서, 의미는 지시를 필요로 한다; 지시 대상들은 그렇지만 어원이 그것들의 관련에 근거한다 하더라도 구별된다.

4) 종과 유들은 근원적인 반면에 어원적으로 그것들을 도입하는 서술들은 파생적이라는 것이 이 관점의 결론이다: 용감한 자들은 용감으로부터 이름지어지지만, 그 역도 성립한다. 그러나 무엇보다도 왜 이것이 그러해야 하는가? 왜 우리는 종과 유들을 논리적(혹은 어쩌면 비논리적) 구조로, 기술들 그 자체인 의미의 근원적 매개물로 간주하지 말아야 하는가? 중기 대화편들에서 이에 대한 플라톤의 주된 답변은 일대다(一對多) 논증이어 왔다. 그 논증에 대한 전제들은 현대적인 차림으로 존 오스틴에 의해 경탄스럽게 진술되어왔다 - 그는 물론 그 논증의 결론에 동의하진 않았다:

만일 우리가 좌우간 언어를 통해 성취하는 그런 종류의 소통일 것이 있다면, 의사전달자('화자')가 '마음대로' 만들어낼 수 있고 또 피(彼)의사전달자('청자')가 관찰할 수 있는 어떤 종류의 상징들의 축적이 있어야만 한다: 이러한 것들이 '말'이라 불릴 것인데, 물론 비록 그것들이 우리가 보통 말이라 불러야 하는 것과 같은 어떤 것일 필요는 없다 할지라도 그렇다 - 그것들은 신호기(信號旗) 따위의 것일 터이다. 또한 말과는 다른 어떤 것, 말이 그에 대한 소통을 위해 사용될 것인 어떤 것이 있어야만 한다: 이것은 '세계'라 불릴 것이다. 어째서 세계가 말을 포함하지 않아야 하는지에 대해서는, 실제 진술 그 자체, 그에 대해 어떤 특수한 경우이든 세계에 대하여 이루어지는 것인 진술 자체의 면을 제외한 모든 면에서 아무런 이유도 없다. 더욱이, 세계는 유사성들과 부동성들(하나 없이는 다른 하나도 있을 수 없는)을 내보여야만 한다(우리는 관찰해야만 하고): 만일 모든 것들이 절대적으로 다른 어떤 것과도 구별될 수 없거나 완전히 다른 어떤 것과도 다르다면, 말할 아무것도 없을 것이다. 그리고 궁극적으로 (작금의 목적들을 위해서 - 물론, 마찬가지로 만족될 다른 조건들도 있다) 두 가지 관습적 방식들이 있어야만 한다: 세계에서 발견되는 상황, 사물, 사건 등의 유형들을 가지고 말과 관계하는 기술적인 관습적 방식들(= 문장들). 세계에서 발견되는 역사적 상황들 따위를 가지고 말과 관련하는 지시적인 관습적 방식들(= 명제들).

오스틴은 보통 일상언어철학자라고 불린다; 그러나 그는 어떤 표어들 아래에서도 행진하지 않았고 어떤 장단에도 맞추지 않았으며, 만일 언어가 그 세계에 대한 것일 터라면 세계가 어떠해야만 하는지 기술하는 위의 문단은, 만일 내가 실수하는 게 아니라면, 가장 투명한 형이상학의 단편이다. 오스틴은 주석에서 계속해서 첨언한다:

'~를 가진 일종의 ~이다'라는 것은 '~을 가진 그런 표준적 사태들과 충분히 유사한 ~이다'라는 것을 의미한다. 그래서, 참된 하나의 사태일 하나의 진술은 자연적 관계인 특정한 다른 것들과 유사해야만 하고, 그러나 또한, 더 이상 순수하게 자연적 관계는 아닌 동일한 '서술' 가치에 대해 충분히 유사해야만 한다. '이것은 붉다'라고 말하는 것은 '이것은 그런 것들과 닮았다'고 말하는 것과 같지 않고, '이것은 붉다고 불리는 그런 것들과 닮았다'라고 말하는 것조차 똑같은 것이 아니다. 사물들이 유사하다거나 또는 심지어 '정확하게' 유사하다는 것조차 나는 문자 그대로 이해할 터이지만, 그것들이 같다는 것을 나는 문자 그대로 알 수 없다 - 그것들을 똑같은 색이라 부름에 있어서 관습은 그것들이 그렇다고 이야기되는 색에 주어질 이름에 대한 관습적 선택에 부가적으로 필요한 것이다.

아마도 그럴 것이다. 그러나 그래서 유사성과 부동성은 상이한 측면들에서 획득하는 관계들이다. 크리켓 공과 야구공은 형태에 있어서 유사하고 색에 있어서 부동하다; 우리는 단순히 그것들이 이것이 그러하다는 측면들을 구별함이 없이 유사하고도 부동하다고 진술할 수는 없다. 만일 그래서 우리가, 세계 속의 사물들 사이에서 우리가 이야기하는 방식과 우리가 좌우간 이야기한다는 그 사실에 대해 독립적으로 획득하는 유사성과 부동성이 자연적 관계들이라고 받아들인다면, 관계들이 그 덕분에 획득하는 그러한 특징들 - 형태적 특징들, 색채적 특징들 등 - 이 언어적 관습의 문제들이라고 추정하는 것은 거의 이치에 맞기 어려워 보인다. 만일 그 관계들이 '자연적'이라면, 그것은 그 덕분에 그것들이 유지하는 그러한 특징들도 동일하게 그렇다는 것으로 드러날 것이다. 주의해야 할 그러한 특징들은 기술들이 아니다: 우리는 크리켓 공과 야구공이 모두 구형이라고 말하지만, 그것들이 서로 구체의 측면에서 유사하다고 말하지 않고, 구체 혹은 구형일 것임의 측면에서 유사하다고 말한다. 우리는 그 사물을 기술한다; 그러나 그 기술은 기술되는 그 사물의 특징 덕분에 유지된다. 이것은, 내가 보기에, 용감한 자들이 용기로부터 이름지어지고 그 역은 아니라고 말하는 이유이다.

5) 이러한 언급들은 물론 유사성들과 부동성들의 세계를 발견하고 특징들의 현존을 추론해내는 사람들과 동일한 세계를 발견하고 추론을 거부하는 사람들 사이의 그 문제를 조정하지는 않는다. 그러나 그 언급들은 아마도 최소한 플라톤의 경우 그 추론이 의지하는 그러한 근거들을 보여주는 어떤 것을 한다. 그렇지만 초기든 중기든 대화편들에서 유사성들에 대한 인식으로부터 도출된 꼴들에 대한 우리의 앎은 없다는 것을 주의해야 한다; 그것은 오로지 우리가 사실상 유사성이 획득한다고 확신할 표준으로서의 꼴의 사용을 통해서만 있기 때문이다.

-蟲-
환원성

1) 초기 대화편들은 꼴들이 현존한다고 주장되거나 암시되는 단언들로 수놓아져 있다; 그리고 그러한 단언들은 단순한 언어의 문제로 치부될 것이 아니다. 그러나 이에 대한 증거가 아직 그 결론을 반대하는 학자들에게 오래 전부터 알려져 왔기 때문에, 그 문제를 더 밀고 나가 보는 것이 좋을 지도 모르겠다.

2) 누군가는 이런 논증을 그려볼 수도 있겠다. 그 저술은 의심의 여지 없이 꼴들이 현존한다고 단언한다; 그러나 그 저술은 그런 주장을 필요로 하지 않는다. 초기 대화편들 내에서 꼴들에 대해 이야기하는 것은, 추정되는 존재론적 관련이 사실 암암리에 어쨌든 언어적 관련이 있음을 발견했다는 점에서, 피할 수 있다. 플라톤은 - 누군가는 그 논증이 연이어지는 것을 상상할 것이다 - 실제로 언어에 대한 사실들인 항목들을 세계에 대한 사실들로 표현하기 위해, 담화의 구체적인 방식에 대한 의심할 것 없는 불가피한 편애라고 할지라도, 운이 나빴다. 소크라테스적 변증은 사물들의 본성에 대한 분석으로서 제공된다; 그러나 그것은, 결국, 피할 수 있는 담화방식이다. 꼴들의 현존을 주장하거나 시사하는 진술들은 논리적 여항 없이 꼴들의 현존을 주장하지 않는 진술들로 환원가능하거나 분석가능하다. 본질에 대한 물음들은, 플라톤 자신조차 동의했을 것과 마찬가지로, 의미에 대한 물음들이고, 의미에 대한 물음들은 당연히 언어적이다. 적절하게 이해된 소크라테스적 변증은 - 그리고 플라톤은 의심할 것도 없이 적절하게 그것을 이해하지 않았는데 - 세계를 향해서가 아니라 말들의 의미를 향해 정향된다. 결국, 꼴들이 존재한다고 말하는 것은 짐작컨데 단어들이 의미를 가진다는 것 그리고 탐구를 통해 우리는 그러한 의미를 분명하게 만들 것이라 기대할 것이라는 점을 말하는 것이다. 꼴들이 보편개념들이라고 말하는 것은 상이한 문맥들에서 동일한 의미를 가지는 단어들이 있다고 말하는 것이고, 그 단어들이 우리가 나타내는 것들을 묘사한다고 말하는 것이다. 꼴들이 표준들이라고 말하는 것은 한 서술의 의미를 이해한다는 것이 그 서술이 해당하는 상황들을 동일화할 수단을 얻는 것이라고 말하는 것이다. 꼴들이 본질들이라고 말하는 것은 한 서술의 의미에 대한 모든 각각의 설명이 그 의미인 것을 정확하게 표현하지는 않는다고 말하는 것이다. 소크라테스의 물음, '경건이란 무엇인가?'란 물음은 요약하자면 '"경건"이란 단어가 뜻하는 것은 무엇인가?'라는 물음으로 만든다. 그러므로, 초기 대화편들에 꼴들에 대한 존재론적 언질이 있는지에 대한 문제는 이중적이고, 이중의 대답을 허용한다. 만일 그 물음이 그러한 대화편들의 본문이 꼴들의 현존을 긍정하는지 아닌지에 대한 것이라면, 그 대답은 긍정한다는 것이다. 만일 그 물음이 플라톤이 상술하고 있는 위치의 논리가 꼴들의 현존을 요청하느냐는 것이라면, 그 대답은 요청하지 않는다는 것이다.

3) 그래서 우리는 소크라테스의 물음, '경건이란 무엇인가?'라는 물음이 '"경건"이란 단어가 의미하는 것은 무엇인가?'라는 물음으로 변환될 수 있는지 결정하게 된다. 그것은 이러한 대체가 순환 없이는 이루어질 수 없다는 것으로 밝혀진다.

4) 몇몇 상황들 아래에서 '"경건"이란 단어가 의미하는 것은 무엇인가?'라는 그 물음은 사전적일 것이고, 프로디쿠스나 아니면 사전에 의해 대답될 물음일 것이다. 그러나 단순하게, '경건이란 무엇인가?'라고 묻는 와중에 소크라테스는 문자상의 동의어들에 대한 탐구에 종사하지 않는다. 에우튀프론조차 소크라테스가 경건이 신성함이라는 답으로 만족해야 한다고는 생각하지 않는다.

5) 다시, '"경건"이란 단어가 의미하는 것은 무엇인가?'라는 물음은 '실물 지시적 정의', 예시에 호소하는 정의에 의해 대답될 것이다. 그러나 예시가 구체적인 예증으로서 협소하게 구성되는지, 아니면 요청되는 특징을 드러내 보이는 일반적 경우로서 더 넓게 구성되는지, '그것은 무엇인가?'라는 물음에 대한 이런 종류의 답변은 초기 대화편에서 그러한 답변이 나타나는 어디에서고 거부된다: 에우튀프론은 경건을 '바로 지금 내가 하고 있는 일', 또는 더욱 일반적으로, 살인자들과 성물 절도범들을 고소하는 것으로 정의할 수 없다.

6) 다시금, '"경건"이란 단어가 의미하는 것은 무엇인가?'라는 물음은 아마도 경건의 구별적 특징들을 제공함으로써, '경건'이라는 단어의 적용의 기준들 혹은 사용의 규칙들을 제공함으로써 답변될 것이다. 그러나 구별적 특징의 관념이 가장 강한 의미에서 - 전체이자 유일하며 항상된 것(id quod omni et soli et semper), 그 용어를 적용하기 위한 필요조건이자 충분조건 -  이 물음은 '경건이란 무엇인가?'라는 그 물음에 등가가 아닌데, 왜냐하면 구별적 특징이나 대체 가능한 기준의 제공은 후자의 물음을 대답되지 않은 채로 남겨둘 것이기 때문이다. 에우튀프론이 경건을 모든 신들이 사랑하는 것으로 정의할 때, 소크라테스는 이 특징이 경건한 모든 것들이자 유일하게 경건한 것들을 구별하는 데에 시패한다는 것을 보여주는 일을 떠맡지 않는다; 그는 맹목적으로 이것을 받아들이거나, 최소한 그것의 가능성을 용인한다. 그는 그가 쉽사리 했었을 것 처럼, 그러한 특징이 경건의 예증들을 동일화함에 있어서 어떤 실질적 사용에 속한 것일지 아닐지 하는 물음을 던지는 것도 아니다. 오히려 그는 에우티프론이 우연히 경건의 πάθος를, 경건한 것들에 대해 우연히 참인 어떤 것을 발견했을지라도, 그가 경건의 οὐσία를, 경건의 본성과 실재성을 보여준 것은 아니라고 주장한다. 구별적 특징의 제공은 그래서 '경건이란 무엇인가?'라는 물음을 대답되지 않은 채로 남겨둔다.

7) 그러나 '"경건"이란 단어가 의미하는 것은 무엇인가?'라는 물음에 대한 또 다른 종류의 답변이 있다. 그것은 불분명한 답변이지만, 당장의 목적들을 위해서 중요한 한 가지 답변이다. 그것은 '경건'의 의미가 경건이라는 것이다.

그래서, 경건이란 것은 무엇인가 기원하는 것인가?
정확히 그렇다.

 8) 초기 대화편들에 있어서 꼴들에 대한 물음들은 단어들의 의미에 대한 물음들로 환원될 수 없는데, 왜냐하면 단어들의 의미에 대한 첫번째 물음은 꼴들에 대한 물음들로 바뀌기 때문이다. 만일 비환원성이 존재론적 관련의 기준일 것이라면, 초기 대화편들에는 꼴들에 대한 존재론적 관련이 있다.

9) 이러한 관련을 제공하는 의미이론은 더 탐구되어야 할 필요가 있다. 그러나 독자는 여전히 끈질긴 의심을 계속 지닐 것이다. 윌리엄 제임스는 일찍이 가상적 불길이 (당신이 그에 대해 말할 수 있는 그런 종류의 회피 가능한 것이 아니라) 실제 장작개비들을 태우지 않을 그런 종류의 불길이라고 언급했었다. 이것은 『소피스트』(247e)에서 존재론적 관련에 대한 플라톤의 고유한 기준, 그가 신들과 거인들에 의해 비슷하게 공유되었다고 생각했던 기준을 반영한다: 그것은 존재의 표징이 능력이라는 것이고, 작용하거나 작용받을 그러한 능력이라는 것이다(πάθημα ἢ ποίημα ἐκ δυνάμεώς τινος, 248b). 차이는, 차이이려면, 차이를 만들어야 한다. 만일 꼴들이 존재하는 것이라면, 우리는 그것들이, 말하자면 단지 거기에 놓이는 것이 아니라, 진실로 작용할 것, 세계의 운행에 영향을 줄 것을 기대할 것이다. 그것들은 이러한 것을 수행한다; 우리는 초기 대화편들에 있어서 꼴들이 원인들임을 알아차릴 것이기 때문이다.


본질과 지시

1) 만일 '경건'이란 단어의 의미가 경건이라면, 그리고 경건이 꼴이나 본질이라면, 그 단어와 그 어떤 것 사이에 자리하는 관계가 명칭이나 이름 붙임에 속하는 하나의 관계라고 추정하는 것은 합리적이다. 초기 대화편들은 이러한 관점을 뒷받침한다. 『프로타고라스』에서, 소크라테스는 프로타고라스가 방금 이름붙인(ὠνομάσατε) 어떤 것(πρᾶγμα)이 정의로운지 정의롭지 않은지 묻는다(330c); 나중에, 탁월함이 하나인지 혹은 여럿인지에 대한 물음은 '지혜', '올바름', '사려', '용기', 그리고 '경건'이 상이한 것들을 위한 이름들(ὀνόματα)인지 아니면 동일한 것을 위한 이름들인지 탐구하는 형식을 취한다. 초기 대화편들은 의미에 대한 지시적 이론을 전제한다: 그 대화편들은 추상적 명사들, 또는 그것들 중 일부가 이름들임을 전제한다.

2) 콰인 교수는 다음과 같이 지적했다. '의미는 그것이 지시대상과 분리되고 그 단어와 결합될 때 본질이 되는 어떤 것이다.' 반대로, 본질은 그것이 단어와 분리되고 대상과 결합될 때 의미가 되는 어떤 것이다. 그래서 의미되는 어떤 것은, 최소한 때때로라도, 단순히 표현이라는 면에서가 아니라, 다소간 세계의 질서에 알맞는 내용의 일부라는 면에서, 그러나 그 자체로 그 본성이 발견에 있는 그러한 세계의 질서 속에서 하나의 항목이 된다. 우리는 세계를 불완전하게 이해하기 때문에, 우리는 우리의 말들의 의미를, 혹은 그 중의 일부를 불완전하게 이해한다; 또 다른 방식으로 보자면, 우리가 일상적으로 하나의 표현으로써 의미하는 어떤 것, 그것의 공공연하거나 또는 명백한 내용과 우리가 만일 그 표현이 지시하는 본질을 이해했다면 의미해야 할 그러한 어떤 것 사이에는 차이가 있다. 에우튀프론은 분명 그가 일상적으로 경건을 통해 의미하는 어떤 것을 말할 수 있다; 그는 살인자들과 성물 절도범들을 고소하는 것을 말한다; 숙고를 거쳐서, 그는 신들에 의해 사랑받는 어떤 것을 말한다; 더욱 숙고한 끝에, 모든 신들에 의해 사랑받는 어떤 것을 말한다. 그러나 이것들 중 아무것도 경건의 본질을 진술하지 않는다. 그의 이해력과 그의 의도 사이에, 그가 경건에 대해 제시할 수 있는 설명과 그가 설명하고자 시도하고 있는 본질 사이에 차이가 있다. 만일 이따금 의미한다는 것이 이름하는 것이라면, 그리고 이름지어지는 것이 본질이라면, 의미되는 것은 종종 주어지는 것이 아니라, 목표일 것이다.

3) 만일 때때로 의미함이 본질을 수반한다면, 그리고 우리의 본질에 대한 앎이 종종 불완전하다면, '제각기 모든 것이 그것인 것으로 질서지어져 있다'는 것은 이상 언어에 대해 참이 아닐 것인데, 왜냐하면 단어들을 지배하는 사용규칙들이 필연적으로 그 단어들이 그에 의해 통제되어야 하는 규칙들인 것은 아니기 때문이다. 아마도 해결책은 본질들을 부정하는 것일 것이다:

누군가는 거듭해서 사물의 본성에 속하는 윤곽을 추적하고 있다고, 그리고 단순히 우리가 그를 통해 본성을 바라보는 그 틀의 주변을 추적하고 있다고 생각한다. 한 장면이 우리를 달아나지 못하도록 붙잡고 있었다. 그리고 우리는 그 밖으로 나갈 수 없었는데, 왜냐하면 그것이 우리의 언어와 우리에게 변치 않고 그것을 반복하는 듯이 보이는 언어에 자리하기 때문이다. 철학자들이 단어를 사용하고 그 사물의 본질을 파악하고자 노력할 때, 누군가는 반드시 그 자신에게 물어야 한다: 그 단어가 도대체 실제로 그것의 본래 집인 언어 게임 안에서 이런 방식으로 사용되는가? - 우리가 행하는 것은 단어들을 그 형이상학적 사용으로부터 우리 일상의 사용으로 되돌려 가져오는 것인가?

비트겐슈타인은, 그가 반대하고 있던 것을 아주 잘 알았다. 만일 이 관점이 참이라면, 소크라테스적 변증이 기대고 있는 그 전제들은 거짓이다. 그래서 탁월함의 단일성에 대한 전형적인 소크라테스적 주장들도 거짓이다: 탁월함이 앎이라는 주장, 용기가 지혜라는 주장, 경건이 올바름이라는 주장. 그 말들은 일상적인 방식으로 사용되지 않는다 - 현대 영어에서만큼이나 그 이상으로 고대 그리스어에서. 프로타고라스는 사람은 지혜롭지 않고도 용맹할 수 있으리라 주장함에 있어서, 그의 언어 사용 규칙들을 보여주고 있었다. 소크라테스의 언어적으로 독특한 사물들의 결합은, 그것이 만약 언어에 대한 단순한 전제적 입법일 것이 아니라면, 본질의 관념으로부터 분리되어 이해할 수 없는 것이다.


본질과 일상언어

1) 소크라테스적 변증은 단어들의 일상적 사용을 탐구하는 것이 아니라, 단어들의 사용이 근거하는 것들에 있어서 특징들을 탐구해 들어가는 것이다. 한 단어의 사용은 의심할 나위 없이 그 단어를 지시하는 특징과 연관된다; 만일 우리가 언제나 사물들의 본성에 대해 명확하지 않다면, 그로써 우리가 완전한 암흑 속에 있다는 것이 두따르진 않는다. 그러나 본질과 사용 사이의 관련은 경험적이다: '기준'이라는 용어의 제한적 의미에서, 단어가 지시하는 본질은 그것을 적용하기 위한 기준이고, 유일하게 신뢰 가능한 기준이다.

2) 이러한 관점에 대한 하나의 반박이 있다. 우리는 우리의 어머니 품에서 말하는 것을 배웠고, 말하는 방법을 아는 것은 일반적인 용어들을 사용하는 방법을 아는 것이다; 그렇지만 일반적 용어들을 사용하는 방법을 아는 것은 그 언어의 다른 발화자들이 확실하게 동의할 명백한 경우들이 그 용어들의 예시들임을 예증할 수 있는 것이다. 일반적 용어를 사용할 줄 안다는 것은 물론 그 용어의 사용을 위한 기준을 제시할 수 있다는 것과 다르다; 그러나 그 다음으로 우리는 그 사용을 관찰함으로써 기준들을 발견할 수 있다. 그래서, 우리는 종종 주어진 한 행위가 경건하다는 것을 '경건'이란 단어에 제공할 기준을 제시할 수 없어도 완벽하게 잘 알 수도 있다; 그리고 더 나아가서, 우리는 종종 기준보다는 사례가 제시됨을 통해서 어떤 것이 그것이 경건일 그러한 어떤 것을 더 잘 배울 것이다. 소크라테스적 변증은 그러므로 오해된다. 의미에 대한 탐구에 있어서 사례에 호소하는 것을 허용하기를 반대한다는 데에 이어서 그것은 실수이다; 같은 식으로 의미는 사례들이 결정될 수 있기에 앞서 반드시 고정되어야만 한다고 전제함에 있어서도 그것은 실수이다. 이러한 실수들 때문에, 소크라테스적 변증은, 의미에 대한 충분한 기준이 발견됐을 때 결정에 대한 아무런 건전한 수단도 가지지 못하고 부유하게 된다. 그리고 이 사실은 어째서 초기 대화편들의 변증이 그렇게나 일관되게 실패로 끝나는지에 대한 설명으로 잘 받아들여질 것이다. 게임의 규칙들은 아무도 그 게임에서 승리할 수 없게끔 그렇게 정해져 있다.

3) 확실히 이러한 반대에 핵심이 있다. 에우튀프론은 사례에 호소해서도 그가 사용하는 언어의 발화자들이 이상적으로 그 용어를 가지고 이해했던 어떤 것에 호소해서도 경건을 정의할 수 없다. 그러면 그는 도대체 그것을 어떻게 정의할 수 있는가? 여기에는 앎에 대한 분명한 문제가 하나 있다. 때마침, 『메논』에서 플라톤이 탐구의 역설로 진술하였던, 그리고 우리가 보게 될 것과 같이 상기의 원칙으로 해결했던 한 문제가 있다.

4) 그러나 만일 앎의 문제가 분명하다면, 또한 그것은 분명 실재의 문제인데, 왜냐하면 경건일 그러한 어떤 것은 경건의 사례들을 제시함으로써 정의될 수는 없다는 소크라테스의 전제는 그저 옳을 뿐인게 아니라 확실하게 옳기 때문이다. 만일 a가 F의 한 예라면, 그리고 만일 그에 대해 F일 그러한 어떤 것을 a가 정의한다고 주장하는 이유라면, 그리고 만일 b가 F의 한 예라면, b일 것은 a일 것이다; 그리고 a는 F일 그러한 어떤 것을 정의하므로, F일 것은 a일 것이다. 우리는 그래서 번갈아서 a는 F의 예시가 아닌데, 왜냐하면 그것은 F일 그러한 어떤 것이기 때문이라고 말하거나, F는 그 자신의 예시이자 유일한 예시라고 말할 것이다. 만일 우리가 'F'를 대체하고, 그에 따라 '경건일 그러한 어떤 것'은 '경건'이라는 형요사를 대신하는 것이 아니라 '경건'이라는 추상명사를 대신한다는 것은 분명해 진다면, 그 특이함의 원천은 여기에서 분명해진다. 무엇이 이러저러한 용어로 경건한 것을 경건으로 정의하는 것을 의미할 것인가?

5) 동일한 고려들이 경건의 본성을 알지 못하고서는 주어진 어떤 것이든 그것이 경건하다는 것을 아는 것은 불가능하다는 소크라테스의 주장을 대신한다. '나는 a가 F임을 안다'는 명제와 '나는 F일 그러한 어떤 것을 모른다'는 명제는 양립할 수 없다. 만일 내가 a는 F라는 것을 안다면, 나는 a가 F일 그러한 어떤 것의 일례라는 것을 안다; 만일 내가 F일 그러한 어떤 것을 모른다면, 나는 a가 그것의 한 예시일 그러한 어떤 것을 모르고, 만일 내가 a가 그것의 일례일 그러한 어떤 것을 모른다면, 나는 a가 F의 한 예시라는 것을 모르며, 그러므로 a가 F라는 것을 모른다. F일 그러한 어떤 것이 a가 F인지 아닌지를 결정하기 위한 기준이라는 것이 뒤따르는데, 오로지 a가 그 특징에 대한 하나의 예시인 그 특징에 대한 앎을 통해서만 a가 그 특징의 일례라고 누군가 결정할 수 있는 것이기 때문이다. 그래서 경건일 그러한 어떤 것은 주어진 어떤 것이 경건하다는 앎에 선행한다.

6) 소크라테스적 변증에 대한 이러한 방어가 수작을 부린 것이라고(주사위를 조작하다) 주장할지 모른다. 그 주장은 어떤 것이 무엇임을 정의하는 것과 아는 것에 관련된 관념들 - 그것이 정확하게 소크라테스적 실수인 - 에 달려 있다. 철학적 탐구의 고유한 목적은 정의가 아니라 사용의 분석이다. 『에우티프론』에서 세워진 그런 종류의 탐구는 그런 분석이 아니고, 이것이 그렇기 때문에, 그것은 일부 제한적 의미에서 (의심할 바 없이 신실하게) 기만이다.

7) 그러나 기만에 대한 그 주장은 두 가지 방식으로 작용한다. 『에우튀프론』의 변증이 특별히 다루고자 시도하는 문제를 고려해 보자: 도덕적 판단에 있어서 분쟁의 해결. 우리는 사례에 호소함으로써 도덕적 용어들의 사용을 위한 기준에 도달해야 한다고 제안된다. 하지만 실천적인 문제로서, 기준의 결여는 사례들이 빗나갈 때 정확하게 감지된다. 사례들의 비일관성은 기준의 비일관성에 대한 조짐이다: 만일 당신의 기준이 나의 사례를 예외로 한다면, 나는 나의 사례를 거부할 것이나, 나는 마찬가지로 당신의 기준도 거부할 것이다. - 많은 것이 의심의 여지 없이 누구의 소가 뿔에 받혔는지에 달려 있을 것이다(모순 여부). 만일 도덕전 논증에서의 호소만이 사례들로부터 추출된 사용기준일 것이라면, 에우튀프론과 그의 아버지는 각자 논리적으로 난공불락의 위치에 있다. 도덕적 분쟁을 해소할 궁극적 방법이란 크롬웰 류의 담론에 의지하는 외에 아무것도 없다 - 우리들의 원칙을 교황의 입김과 두드림들로 정통이라 증명하는 것이다.

8) 그러나 아마도 이러한 전망은 지나치게 암울한 것이다. 도덕적 의견 불일치, 그것이 아마도 주장될 것인데, 그것은 오로지 경계 경우들에서만 발생하고, 분명한 사례들에 호소함으로써 해소될 것이다. 우리는, 그것을, 모든 F들이 깔끔하게 그 안에 자리잡히고 모든 비(非)F들이 정확하게 묘사된 원 없이 위치되는, 일종의 객체화된 벤다이어그램으로 상상하면서, 태평스럽게 F인 것들의 계층에 대해 말한다. 진실은 수학적 언어와 구별되는 것으로서 일상 언어에서 가장 일반적인 용어가 그것들을 그 안에서는 그것들의 적용 조건들이 불분명한 반영(半影)으로 이끈다는 것이다: 사례는 분명히 F인 것과 분명히 F 아닌 것 사이에 그 안에서 말하는 것이 무엇인지 알기 어려운 범위, 준연속체가 있다는 그런 방식에서 사례에 그림자를 드리운다. 이것은 컵이다, 그리고 저것은 그릇이다, 그러나 여기에는 이러저러하게 부르거나 아무것으로도 부르지 않거나 그 둘 모두로 부를 어떤 것이 있다. 여전히, 우리는 의심스러운 경우들을 판단하기 위해서 경우들을 분명하게 하기를 기대할 것이고, 경우들을 그것들이 무엇인지에 대해 분명하게 인식하지 못하는 누구라도 그가 그것들을 논의하고 있는 그 언어를 이해하지 못한다. 만일 당신이 내가 지금 그로부터 커피를 마시고 있는 이것이 컵이라고 생각하지 않는다면, 당신은 '컵'이란 단어가 의미하는 어떤 것을 모른다. 그래서 마찬가지로 도덕적 용어들도 동일하다.

9) 이것이 참이라면 그건 즐거울 일일 것이다. 불행하게도, 그렇지는 않다. 도덕적 용어들은 일찍이 플라톤이 지적하였듯 '사람들에게 눈에 띄게 만드는 어떠한 상도 가지지 않는다.' 그렇지만 그것은 아마도 경험적 용어들과 함께일 것이다, 선악의 문제, 옳고 그름의 문제들, 한 사람의 분명한 경우가 두번째 사람에게 반영을 드리울 수 있고 세번째 사람을 덮으며, 이것이 단순히 주변부에 관한 것만이 아니라 도덕적 추론이 그것을 가지고 다루어야만 할 가장 핵심적 문제들에 관한 것이라는 특이점을 가진다. 이러한 점을 사례들을 들어 주장하는 것은 아마도 불필요할 것이다 - 다행스럽게도 -; 비참하고 소란한 세기(이 책은 1970년대에 출판되었다)에, 독자는 그 스스로 감당하도록 남겨질 것이다. 그러나 누군가는 아마도 탁월함이 앎이라는 것이 역설이리라거나 혹은 그렇지 않으리라고 할지라도, 악이 실어증이라고 생각하는 것은 단순히 터무니없는 소리라고 지적하는 모험을 할지도 모른다. 만일 선과 악에 대한 물음들이 사람들이 언어로 하는 게임들에 대한 물음들이 아니라면, 또는 태도나 관습 혹은 취향에 대한 물음들이 아니라면, 도덕적 앎과 같은 어떤 것이 있다거나 혹은 있을지도 모른다고 한다면, 앎이 본질에 대한 식별에 존립한다는 『에우튀프론』의 주장은, 그것이 틀렸을지도 모른다 할지라도, 확실히 기만은 아니다. 다루어져야 할 문제들을 정면으로 마주함이 없이 거부하는 것은 어리석은 일일 것이다.


단 하나의 이름 이름지어지는 단 하나의 것(Unum Nomen Unum Nominatum)

1) 지시적 의미 이론들은 종종 특유한 필연적 귀결, unum nomen unum nominatum(the sole name the sole named)을 가져오는 것으로 생각되어 왔다. 그리고 다양한 비판자들은 그들이 초기 대화편들에 있어서 이러한 전제를 감지했다고 생각해 왔다. Mr 리차드 로빈슨은 '만일 소크라테스의 물음(X는 무엇인가?)이 답변을 허용하는 정당한 물음이라면 … 우리는 반드시 X라는 단어가 단일한 뜻을 가진다고 전제해야만 한다'고 말했다. 로스 교수 또한 소크라테스적 물음에서 이러한 함축을 발견하고, 다음과 같이 추론한다. '플라톤은 이름의 의미에서 모호함의 가능성을 알아차리고 있었다. 그러나 보이기로는 그가 이것을 다소 드물게 일어나는 것으로 간주했고, 겉보기에 가장 순전한 이름이 가질 의미조차도 변화하는 그 의미의 음영을 완전히 알아차리지는 못하고 있었다.' 그러나 이것은 잘못된 추론에 기인한다. 소크라테스는 '경건이란 무엇인가?'와 같은 물음들을 묻고 답할 수 있다고 전제한다; 그는 더 나아가 경건이 하나의 꼴이나 본질이라고 전제한다; 그러나 이러한 전제들은 '경건'이나 '경건함'과 같은 단어들이 상이한 의미로 사용된다거나 사용될 수 있다거나 하는 물음에 아무런 관련도 없다. 그 잘못된 추론은 소크라테스는 다른 것들 중에서 우리가 같은 이름으로 상이한 것들을 부르는 것과 그렇게 불리는 것들로 똑같은 것을 의미하는 것이 어떻게 그러한지 설명하기 위해 경건이 본질이라고 전제한다는 흐릿한 참을 가장한다. 이것은, 만일 그것이 일상언어에 대한 주장을 의미하는 것으로 생각된다면, 각 단어에 대해 하나의 유일하게 정확한 의미가 있다고, 그리고 모호함은 불가능하다는 것으로 생각된다면, umun nomen unum nominatum을 수반할 수가 없다.

2) 그것은 로스 교수와 Mr 로빈슨이 추정하는 것처럼 절대의미와 모호의미 사이에 엄격한 괴리를 시사하지도 않는다. 우리는 의미에서의 같음과 의미에서의 다름이 배타적으로 그리고 철저하게 양자택일적이라 생각하기 쉽다. 만일 의미가 지시라면, 그 결론은 보편개념들에 대한 날카로운 이론이다. 그 이론에서 각각의 중요한 단어는정확하게 같은 방식으로 동일한 보편개념을 도입하거나, 서로 다른 보편개념을 도입한다. 그러나 이것은 의미들이 다를지도 모르지만 그럼에도 불구하고 공통된 중심 주위로 우위성과 후위성의 질서에서 조직화된다는 사실을 무시한다. 그래서, 예를 들어 사람들이 올바르고 행위들이 올바르다. 그러나 소크라테스는 『정체』(Ⅳ, 443e-444a)에서 올바른 행위들은 사람들 안에 영혼의 조건으로서 올바름을 산출하도록 지키거나 돕는 그러한 것들이라고 주장한다; 행위에 적용되는 것으로서 '올바름'이란 단어의 의미는 반드시 사람에게 적용되는 의미로 정의되어야만 한다는 것이 뒤따른다. 그래서 그 형용사는 우선 사람에 대한 의미로 사용되고, 파생적 의미에서 행위에 대해 사용되며, 이것은 절대의미도 모호의미도 아니며, 어원유래적이다. 그러나 그것은 동일한 꼴, 기본 사용과 파생 사용 양자에서 도입되는 동일한 꼴이다; 그것은 단순히 다른 방식으로 도입될 따름이다. 이것은 플라톤이 초기 대화편들에서 어원유래에 대한 사실을 인지하였다고 주장하는 것이 아니지만, 그럼에도 불구하고, 사실은, 이것이 그렇다는 강력한 암시가 있다. 그렇지만 그것은 꼴들에 대한 이론과 용어들의 어원유래적 사용 사이에 어떠한 양립불가능성도 없다고 시사하는 것이다. 그리고, 우리가 보게 될 것처럼, 중기 대화편들에서 꼴들 그 자체는 특정 종류의 어원들에 속하는 우선 지시되는 것들이 된다.

3) 만일 umun nomen unum nominatum이라는 꼬리표가 좌우간 초기 대화편들에 적용되는 것이라면, 그것은 일상 언어에 대한 주장으로 이해되어서는 안 되고, 어원을 제외하는 것으로 이해되어서도 안 된다. 그것은 언어의 개선을 위한 이상, 어휘의 조화가 세계의 질서에 부합한다는 요구로서 가장 잘 받아들여진다. 『크라튈로스』에서 하나의 이름은 베틀의 북과 유사한 도구인 것으로 이야기된다. 날실과 씨실을 가르면서 북으로 베를 짠다; 바로 그렇게, 사물들을 그것들이 그것들인 것에 따라 나누면서 이름을 가지고 가르친다(388b-c). 그리고 보통의 아무개에 의해서가 아니라, 만들어지거나 부숴질 수 있는 어떤 모형이 아니라 북 그 자체의 꼴을 보는 목수에 의해서 북이 만들어지는 것처럼, 그와 마찬가지로 이름들은 아무개가 아니라 작명가에 의해, '모든 공예가들 중 가장 드문' 그 기술자에 의해 주어져야 한다(389a); 작명가는 각각의 이름을 그것의 적절한 사용에 맞추면서, 주어진 이름들 속에서 이름 그 자체의 꼴을 볼 것이다(389d). 그리고 목수의 작품의 가치를 직조공이 판단할 것과 같이, 그렇게 변증가는 이름들의 부여자에 의한 작품의 가치를 판단할 것이다(390c). 플라톤주의는, 일상언어에 대한 그 태도에 있어서, 내재적으로 수정주의적이다.

4) 이름들을 부여하는 이러한 사업은 umun nomen unum nominatum에 관련되는데, 그렇지만, 이제껏 제시된 어떤 것과도 다른 의미에서 그러하다. 『메논』(71e 이후로)에서, 메논이 탁월함을 정의하도록 요구받을 때, 그는 그 각각이 고유한 정의를 지닌 탁월함들의 목록으로 응답한다: 사내의 탁월함은 폴리스를 돌보는 것이고, 친구들에게 유익하게 하는 것이며, 적들에게 해를 주는 것이다; 여성의 탁월함은 그녀의 남편에게 명령하고 또 그 남편에게 따르는 것이다; 그리고 아이들, 노예들, 노인들에 대한 다른 탁월함들도 있다. 소크라테스는 이러한 답변을 거부한다: 메논은 그에게 한 무리의 탁월함들을 주었고, 반면에 소크라테스는 그 모든 것들에 있어서 동일한 하나의 것, 하나의 탁월함에 대한 설명을 요구했다. 이것은 물론 단 하나의 것이 있다는 것을, 더욱이 'x의 탁월함'과 'y의 탁월함' 등에 대해 다양한 '~의 탁월함들'에 공통된 '탁월함'이 있다는 것을 전제한다. 다른 식으로 보자면, '탁월함'은 총괄적 단어도 아니고, 불완전한 표현도 아니다: 그것은 본질의 이름이다. 이것은 우리가 사내들의 탁월함과 여성들의 탁월함에 대해 말할 때, 어째서 우리가 어원들도 단순히 얼버무리는 것도 사용하지 않고 있는지에 대한 설명으로 취해질 수 있다; 그러나 『메논』과 『테아이테투스』 모두에서 그것은 오히려 보편성의 더 높고 더욱 흥미로운 단계를 지시하는 것으로 취해진다. 자신의 일을 알고 있는 작명가는 짐작컨데 그것을 마음 속에 품을 것이다.


본질인과성

1) 꼴들은, 로크의 말을 빌리자면, 실재적 본질이며 명사적 본질이 아니다. 꼴들은 실재적인데 왜냐하면 그것들이 명사적으로 이루어질 수 없기 때문이다: 명칭들이나 단어들의 일상적 의미들에 대한 탐구는 그 탐구가 꼴들에 대한 탐구인 경우를 제외하고는 꼴들에 대한 탐구가 아니다. 그러나 그것들은 또한 그것들이 실제 사물들인 바의 그러한 것이기도 하기 때문이다. 로크는 다음과 같이 본다:

본질은 그것이 바로 그것인 바의 것인 어떠한 것이든 간에 그것의 존재에 대해 취해질 것이다. 그리고 그래서 실재는 내재적이며, 그러나 본질적으로는 일반적으로 알려지지 않고, 사물들의 발견 가능한 특질들이 의존하는 사물들의 구조는 그 사물들의 본질로 불릴 것이다. 이것은, 그것의 형성으로부터 명백하듯이, 그 단어의 적절한 의미이다; 그 기본 표기법에 있어서 적합하게 존재함을 의미하는 esentia.

로크의 설명에 의해, 본질은 '어떤 것의, 그로써 어떤 것이 그것인 바의 것인 존재'이다. 아리스토텔레스는 『형이상학』에서, 개별자의 실체적 꼴을 그 개별자의 실체와 동일시하면서, 본질을 본질을 가진 것의 존재와 동일시하면서 이러한 주장을 내놓았다. 그래서 그는 로크가 물려받았던 그 전통을 기초하였다.

2) 그러나 플라톤의 초기 대화편들에 있어서 꼴들은 그 있음에 대해 그 꼴들이 꼴들인 것의 있음이 아니다. 하나의 있음인 보편개념은 다수성의 있음이 될 수 없다 - 정확하게 아리스토텔레스가 왜 보편개념들로부터 실체적 꼴을 구별하는 데에로 이끌렸는지의 이유이다. 『에우튀프론』은 경건이 어떤 주어진 경건한 사물이나 경건해 보이는 행위의 있음이라고 시사하지 않는다; 그것은 오로지 경건이 그에 의해 경건한 것들이 경건한 것들인 그러한 것임을 시사할 따름이다. 그것은 또 다른 아리스토텔레스적 어휘를 조금 빌리자면, 경건이 원인이라는 것을 시사한다.

3) 『대히피아스』에는 이 전제를 명백하게 만드는 논증이 있다. 소크라테스는 히피아스가 올바름이 어떤 것이다(ἔστι τι τοῦτο)라는 것, 그리고 이것이 지혜에 있어서도 마찬가지로 참이라는 것에 동의하도록 이끄는데, 왜냐하면, '올바른 것과 지혜로운 것 등등의 그러한 것들은 만일 그것들이 어떠한 것이 아니라면 그것들에 의한 그러한 것이 아닐 것이기 때문이다'. 아름다운 것들은 아름다움에 의해 아름답기 때문에, 히피아스는 아름다움도 어떠한 것이며, 그것들이 다른 말들이나 생각들 또는 개념들에 의해 아름다워지지는 않는다는 것에 동의할 수밖에 없다.

4) 이 논증은 분명히 『대히피아스』에만 놓이지만, 그것은 정의를 목표로 하는 모든 각각의 초기 대화편들에 의해 가정되는데, 왜냐하면 꼴들이 본질들이라는 그 전제는 변증에 본질적이기 때문이다: 꼴들은 정확히 보편개념들이고 표준들인데 왜냐하면 그것들이 그에 의해 사물들이 그러한 바의 것들인 것이기 때문이다.

5) 플라톤이 매우 자주 꼴들과 그 꼴들을 지닌 사물들 사이의 관계를 특징화하기 위해 인과적 언어를 사용하는 이유는 꼴들이 본질들이기 때문이다. 『뤼시스』(221c)에서 소크라테스는 '원인이 파괴될 때, 그 원인이 그에 대한 존재를 지속시키는 원인인 그러한 것은 물론 있을 수가 없다'고 지적한다. 경건은 이러한 의미에서 원인이다: 경건의 현존은 경건한 것들이 경건한 것들인 한에서 그것들의 현존에 필수적이다.

6) 초기 대화편들은 꼴들과 그 꼴들의 예시들 사이의 관계를 묘사하기 위한 다양한 은유들을 사용한다. 꼴들은 그 예시들 '안에' 있어야 할 것으로, 또는 그 예시들'에 현전하는' 것으로, 혹은 그것들에 '덧붙여져'야 할 것으로, 아니면 그것들'에' 있어야 할 것으로 이야기된다. 그 예시들은 다른 한편으로 꼴들을 '가지거나' '허용하거나' '취하거나' '나누어 가진다'. 이러한 은유들 전부 일상언어에 기초된다. 그것들은 정말로 평범하게 그리스어에서 추상적 명사들과 함께 사용될 것이다. 영어에서 한 행위에 용기가 있다(there is courage in an action)고, 또는 올바름의 현전이 상황을 개선시킬 것이라고, 어떤 사람이 다른 사람보다 더 훌륭한 인내의 도량을 지닌다고 말하는 것과 똑같이 말이다.

7) 은유들로서, 그리고 친숙한 그리스어로서, 이러한 용어들은 소피스트적 논쟁들의 재료를 제공했다. 예를 들어 『에우튀데모스』에서는 디오니소도루스가 아름다운 것이 아름다움과 다른 것인지 어떤지를 묻는다. 소크라테스가 잠시 망설인 후 아름다운 것들은 아름다움 자체(αὐτὸ τὸ καλόν)와 다른 것들이고, 그러나 그 아름다운 것들 각각은 그 안에 현전하는 어떤 아름다움(κάλλος τι)을 지닌다고 말할 때, 디오니소도루스는 재빨리 주도권을 잡아챈다. 그는 다음과 같이 말한다. '그러면 소 한 마리가 당신 곁에 있다면, 당신은 소로군요. 그리고 내가 당신 곁에 있으니, 당신은 디오니소도루스고요'(301a). 소크라테스는 '맙소사'라고 말한다. 당연하다. 그러나 그 물음은 하찮은 것이 아니다: '현전'은 은유 - 혹은 최소한 많은 용례들을 지닌 단어 - 이고 잘 다룰 방법은 그것의 의미에 달려 있다.

8) 문제가 되고 있는 '현전'은 특수한 종류의 것이다. 『뤼시스』에서 소크라테스는 어떻게 나쁨이 좋지도 나쁘지도 않은 어떤 것에 현전할 것인지 설명하는 일을 떠맡고, 그가 의미하는 것을 명확하게 하려는 그의 논의를 갑작스럽게 중단한다. '일부 사물들은 그 사물들에 현전하는 그러한 것 그 자체일 그런 것들이다; 다른 것들은 그렇지 않다.' 만일, 예를 들어 금발이 하얗게 칠해진다면, 흼은 그 머리카락에 현전할 것이지만, 그 머리카락은 그 자체로 흰 것은 아닐 터이다. 그러나 나이가 들어 머리카락이 흴 적에, 흼은, 그 머리카락이 그래서 하얗게 될 것이기 때문에, 다른 의미에서 머리칼에 현전할 것이다. 앞서의 경우, 흼은 머리카락에 현전한다. 뒤의 경우, 머리카락은 '흼의 현전에 의해 하얗다'.

9) 어떤 것이 흴 때 그 의미에서 흼이 그것에 현전하는 그러한 특별한 의미를 설명하기 위해, 소크라테스는 도구적 여격에 의존한다. 이러한 장치는 초기 대화편들에서 빈번하게 나타난다. 경건한 것들은 경건함에 의해 경건하고, 사려있는 자들은 사려에 의해 사려있으며, 아름다운 것들은 아름다움에 의해 아름답다. 아름다움은, 『대히피아스』(300a)에 따르자면 아름다움 것들을 아름답도록 만든다(ποιεῖν). 이러한 표현들은 일상어법과 결부된다. 이온은 그가 호메로스를 기릴 수 있음이 앎에 의한(도구적 여격) 것이라고 생각한다(『이온』541e). 소크라테스는 우리가 건강에 의해(ὑπό와 동작주의 속격) 유익해진다고 생각한다(『크리톤』280a). 사려는 만일 그 사려가 그들에게 현전하는 그러한 자들에게 좋다면 좋은 것이고, 아니라면 나쁜 것이다(『카르미데스』161a).

10) 그러나 비록 이러한 표현들이 일상언어에 결부된다 할지라도, 그것들은 종종 소크라테스에 의해 비일상적인 방식으로 사용된다. Ποιεῖν은 πάσχειν에 속하는 규칙동사이고, πάσχειν은 규칙적으로 '자격지어짐'을 의미할 것이다; 그러나 ποιεῖν은 '자격을 둠'의 의미로 규칙적으로 사용되지는 않는다. 더욱이, 지혜가 사람들을 복되게 만든다고 말하는 것과 지혜가 사람들을 지혜롭게 만든다고, 또는 지혜가 그에 의해 사람들이 지혜로워지는 것이라고 말하는 것 사이에는 주목할만한 논리적 차이가 있다.

11) 이러한 표현들은 인과적이다 - 흔히 당구공들의 부딪침과 연관되는 의미에서가 아니라 아리스토텔레스가 형상적 원인에 대해 말하는 의미와 유사한 의미에서 -. 아름다움은 정확히 이런 의미에서 아름다운 것들의 원인이다: 아름다움의 현존은 다른 것들의, 그것들이 아름다운 것들인 한에서 현존의 필연적 조건이다. 『대히피아스』에서 소크라테스가 사람들이 아름답다고 말하는 사물들은 오로지 아름다움 자체가 현존해야만 아름다우며, 히피아스가 아름다움을 아름다운 처녀로 정의했을 때, 그 이유로 아름다운 것들이 아름다운 그러한 어떤 것(δι᾿ ὅ)이 현존한다고 주장한 것은 그런 이유에서이다.

12) 그것들이 그에 의해 어떤 것들이 바로 그러한 것들인 바의 그러한 본질들이라는 의미에서, 그래서, 꼴들은 원인들이다. 그러하기 때문에, 『파이돈』(100c)에서 소크라테스는 아름다운 것들이 '다른 어떤 이유도 아니라 바로' 그것들이 아름다움에 참여하기 때문에 아름답다고 주장하며, 케베스에게 그가 이러한 종류의 원인(αἰτία)을 허용하는지 묻는다. F. M. 콘포드는 이 구절을 다음과 같이 설명한다:

'다른 어떤 이유도 아니라 바로 그 이유'라는 구절은 … 모호하다. '이유'는 '설명'(αἰτία의 관습적 용법)을 의미할 것이다. 그 전제는 그래서 '이 장미는 아름답다'라는 명제가 '이 장미는 아름다움에 참여한다'는 진술과 동등하다고 단언할 것이다: 나는 말들의 그 형식을 바꾸어 말함으로써 대체하고 그렇게 설명할 수 있다. 그러나 플라톤은 명제의 분석이 아니라 일치하는 사실에 대한 분석을 말하고 있는 것처럼 보인다. 그 이론은 그래서 이 사실이 (1)특정한 가시적인 것, 이 장미, (2)아름다움(Beautiful or Beauty), 그리고 (3)우리가 '~에 참여한다'를 대체할 수 있는, '~는 ~이다'로 표현되는 그 둘 사이의 관계라고 불러야 할 어떤 것으로 구성된다고 단언할 것이다. 그러나 다시 한 번 우리는, 지금까지, 오로지 하나의 설명만을 가진다: 이 장미가 아름답다는 사실은 이 장미가 아름다움에 참여한다는 사실과 똑같은 것이다. 우리는 그 사실을 존립하게 할 어떤 원인에 대해서도 아무것도 배우지 않는다. 어느 관점에서든 우리는 한 명제에 대한 분석이나 하나의 사실에 대한 분석만을 가질 따름이고, 참인 진술의 이유나 그 사실의 현존에 대한 원인을 가지지는 않는다.

첫번째 주장은 제외될 것이다. 당신은 말을 바꿈으로써 오로지 본래의 단어들이 의미하는 것에 대해 무지한 누군가에게 하나의 명제를 설명할 수 있을 뿐이고, '이 장미는 아름답다'라는 것을 이해하는 데에 실패한 그 누구도 그 표현이 '이 장미는 아름다움에 참여한다'를 의미한다고 이야기되어야 한다는 데에 대해서 분명하게 해줄 것을 찾지는 못할 것이다. αἰτία가 말 바꿈에 대해 사용될 수 있었다는 것을 보여주는 어떤 고대의 증거가 있는 것도 아니다. '다른 어떤 이유도 아니라 바로 그 이유'가 말 바꾸기를 시사하리라는 주장은 그래서 불충분하다.

13) 콘포드가 '일치하는 사실에 대한 분석'이라고 부른 것은 여전히 남아 있다. 그러나 만일 이 장미가 아름답다는 사실이 이 장미는 아름다움에 참여한다는 사실과 똑같은 사실이라면, 그가 인정하듯, 어떤 식으로 분석이 말 바꾸기와 다른지는 불분명하다. 어떤 경우든, 이것은 플라톤의 핵심을 놓친다. '이 장미는 아름답다'와 '이 장미는 아름다움의 꼴에 참여한다'는 실로 등가의 진술들이다 - 만일 꼴들에 대한 이론이 참이라면 말이다. 그러나 그 이론은 정확히 사실들의 구조에 있어서 존재론적 우선성에 대한 이론이지, 명제 등가들에 대한 이론이 아니다. 아름다움의 꼴의 현존은 이 장미를 아름답다고 부르기 위한 필요충분조건이다: 그 사물에 세례를 내리는데 그것에 있어서 그 특징 때문이고,(그 특징 자체와 구별되는 것으로서) 그 세례의 정당함을 보장하는 그 특징의 현존 때문이다. 꼴 자체는 그렇지만 오로지 장미의 아름다움에 대한 필요조건일 따름이다: 그것은 아름다움이 만일 그것이 현존하지 않는다면 장미가 아름다운 것으로서 현존하지 않았을 것이라는 점에서 그 아름다움의 원인이다. 그 꼴은, 꼴의 현존과 구별되는 것으로서 그 장미의 아름다움에 대한 충분조건이 아닌데, 왜냐하면 그 꼴은그 장미에 있어서 그 장미의 아름다움의 고유한 현존에 대한 충분조건은 아니기 때문이다: 그것은 아름다움의 현존뿐만 아니라 토양의 조건, 그리고 기후, 정원사의 기술에도 의존하기 때문이다. 여기에서 충분조건은 본질인과만이 아니라 작용인 또한 필요로 한다.

14) 그래서 꼴들은 그것들이 그것들에 의해 사물들이 그러한 것들인 바의 것들이라는 의미에서 원인들이다. 그것들은 그러므로 세계의 운행에 영향을 미친다. 만일 그것들이 존재하지 않았더라면, 세계는 세계가 그 세계인 어떤 것이 아니었으리란 의미에서 말이다.


지시의 재고

1) 초기 대화편들은 의미에 대한 지시적 이론을 전제하는데, 추상적 명사들 또는 그것들 중 일부가 대상들을 명명한다는 의미에서 그러하다. 이로부터 초기 대화편들이 이제는 이전보다 덜 널리 주장되는 그 명제, 어떤 것을 기술한다는 것은 서술을 명명하는 것을 필요로 한다는 명제를 전제한다는 것이 뒤따르진 않는다. 이 마지막 명제는 일반적으로 추가적인 명제와 관련되어 왔다: 잘 구성된(혹은 잘 분석된) 문장들의 구조는 반드시 문장이 표현하는 그러한 사실들에 대해 일대일 혹은 2방향 유일 상응에 존립해야만 한다는 명제이다. 잘 구성된 문장들은 술어들을 포함하기 때문에, 그 문장들이 표현하는 사실들은 반드시 유사-술어들을 포함해야만 한다; 그러나 그러한 유사 술어들이 보편개념들이나 본질들이라고 추정하는 것은 더 나아간 단계이고, 그러한 술어들은 그것들을 명명한다.

2) 이것은 플라톤이 주장하지 않은 명제이다. 초기 대화편들은 더욱이 그것과 관련된다고 거의 이야기될 수가 없다; 그 대화편들은 특정한 어떤 것들이 무엇인지에 대한 물음과 관련되지, 어떻게 그리고 어떤 방식으로 사물들이 사물들에 대해 이야기될 것인지에 대한 물음과는 관련이 없다. 그러나 만일 우리가 『파이돈』을 본다면, 우리는 어떤 것을 기술하는 일이 하나의 기술을 이름하는 것이 아니라 서술된 그것을 명명하는 것을 필요로 한다는 것을 알아차릴 것이다. '꼴들의 각각은 현존하고, 다른 것들은, 그것들이 그 꼴들의 제 몫을 가지게 되기 때문에, 그것들 이후에 이름지어진다'라는 것 때문이다. 이것은 소크라테스를 올바르다고 기술하는 것은 그에게 올바르다고 명명하는 것이고, 좌우간 올바름을 명명하는 것은 아니라고 강력하게 주장한다; 그 이름이 비록, 올바르게 적용된다면, 소크라테스가 올바름에 참여하기 때문에 적용한다 할지라도 말이다.

3) '올바른'과 '올바름' 사이의 관계는 아리스토텔레스가 아카데메이아적 구별이었을 것을 사용하면서, 동근어라고 불렀던 것의 한 예시이다: '사물들이 상이한 어미들로 어떤 것으로부터 그것들의 이름을 획득할 때, 그것들은 동근어들이라 불린다. 그래서, 예를 들어 문법학자(grammarian)는 그의 이름을 문법(grammar)로부터 획득하고, 용감한 자들은 그들의 것을 용감으로부터 취한다.' 그래서 소크라테스를 올바르다고 명명함에 있어서 우리는 그를 동원어적으로 명명한다: 그는 그의 이름을 올바름으로부터 얻는데, 어미상의 차이를 가지고 그렇게 한다. 그러나 소크라테스가 참여하고 있는 꼴의 이름은 소크라테스가 그 꼴로부터 얻은 이름과 문법적 어미에 있어서 다를 필요가 없고, 플라톤의 고유한 사용에 따르고 소크라테스가 어원적으로 명명된다고 말하는 것이 더 나을 것이다. 만일 꼴들에 대한 이론이 참이라면, 그래서 '소크라테스는 올바르다'와 '소크라테스는 올바름에 참여한다'는 등가 명제들이다; 서로가 서로를 수반한다. 그러나 첫번째 것은 동일한 것에 대한 두 가지 이름들을 포함하는데, 하나는 어원이고, 반면에 다른 하나는 두 가지 것들에 대해 두 가지 이름들을 포함하고, 그 중 어느 것도 어원이 아니다. 두 경우 모두에서, 의미는 지시를 필요로 한다; 지시 대상들은 그렇지만 어원이 그것들의 관련에 근거한다 하더라도 구별된다.

4) 종과 유들은 근원적인 반면에 어원적으로 그것들을 도입하는 서술들은 파생적이라는 것이 이 관점의 결론이다: 용감한 자들은 용감으로부터 이름지어지지만, 그 역도 성립한다. 그러나 무엇보다도 왜 이것이 그러해야 하는가? 왜 우리는 종과 유들을 논리적(혹은 어쩌면 비논리적) 구조로, 기술들 그 자체인 의미의 근원적 매개물로 간주하지 말아야 하는가? 중기 대화편들에서 이에 대한 플라톤의 주된 답변은 일대다(一對多) 논증이어 왔다. 그 논증에 대한 전제들은 현대적인 차림으로 존 오스틴에 의해 경탄스럽게 진술되어왔다 - 그는 물론 그 논증의 결론에 동의하진 않았다:

만일 우리가 좌우간 언어를 통해 성취하는 그런 종류의 소통일 것이 있다면, 의사전달자('화자')가 '마음대로' 만들어낼 수 있고 또 피(彼)의사전달자('청자')가 관찰할 수 있는 어떤 종류의 상징들의 축적이 있어야만 한다: 이러한 것들이 '말'이라 불릴 것인데, 물론 비록 그것들이 우리가 보통 말이라 불러야 하는 것과 같은 어떤 것일 필요는 없다 할지라도 그렇다 - 그것들은 신호기(信號旗) 따위의 것일 터이다. 또한 말과는 다른 어떤 것, 말이 그에 대한 소통을 위해 사용될 것인 어떤 것이 있어야만 한다: 이것은 '세계'라 불릴 것이다. 어째서 세계가 말을 포함하지 않아야 하는지에 대해서는, 실제 진술 그 자체, 그에 대해 어떤 특수한 경우이든 세계에 대하여 이루어지는 것인 진술 자체의 면을 제외한 모든 면에서 아무런 이유도 없다. 더욱이, 세계는 유사성들과 부동성들(하나 없이는 다른 하나도 있을 수 없는)을 내보여야만 한다(우리는 관찰해야만 하고): 만일 모든 것들이 절대적으로 다른 어떤 것과도 구별될 수 없거나 완전히 다른 어떤 것과도 다르다면, 말할 아무것도 없을 것이다. 그리고 궁극적으로 (작금의 목적들을 위해서 - 물론, 마찬가지로 만족될 다른 조건들도 있다) 두 가지 관습적 방식들이 있어야만 한다: 세계에서 발견되는 상황, 사물, 사건 등의 유형들을 가지고 말과 관계하는 기술적인 관습적 방식들(= 문장들). 세계에서 발견되는 역사적 상황들 따위를 가지고 말과 관련하는 지시적인 관습적 방식들(= 명제들).

오스틴은 보통 일상언어철학자라고 불린다; 그러나 그는 어떤 표어들 아래에서도 행진하지 않았고 어떤 장단에도 맞추지 않았으며, 만일 언어가 그 세계에 대한 것일 터라면 세계가 어떠해야만 하는지 기술하는 위의 문단은, 만일 내가 실수하는 게 아니라면, 가장 투명한 형이상학의 단편이다. 오스틴은 주석에서 계속해서 첨언한다:

'~를 가진 일종의 ~이다'라는 것은 '~을 가진 그런 표준적 사태들과 충분히 유사한 ~이다'라는 것을 의미한다. 그래서, 참된 하나의 사태일 하나의 진술은 자연적 관계인 특정한 다른 것들과 유사해야만 하고, 그러나 또한, 더 이상 순수하게 자연적 관계는 아닌 동일한 '서술' 가치에 대해 충분히 유사해야만 한다. '이것은 붉다'라고 말하는 것은 '이것은 그런 것들과 닮았다'고 말하는 것과 같지 않고, '이것은 붉다고 불리는 그런 것들과 닮았다'라고 말하는 것조차 똑같은 것이 아니다. 사물들이 유사하다거나 또는 심지어 '정확하게' 유사하다는 것조차 나는 문자 그대로 이해할 터이지만, 그것들이 같다는 것을 나는 문자 그대로 알 수 없다 - 그것들을 똑같은 색이라 부름에 있어서 관습은 그것들이 그렇다고 이야기되는 색에 주어질 이름에 대한 관습적 선택에 부가적으로 필요한 것이다.

아마도 그럴 것이다. 그러나 그래서 유사성과 부동성은 상이한 측면들에서 획득하는 관계들이다. 크리켓 공과 야구공은 형태에 있어서 유사하고 색에 있어서 부동하다; 우리는 단순히 그것들이 이것이 그러하다는 측면들을 구별함이 없이 유사하고도 부동하다고 진술할 수는 없다. 만일 그래서 우리가, 세계 속의 사물들 사이에서 우리가 이야기하는 방식과 우리가 좌우간 이야기한다는 그 사실에 대해 독립적으로 획득하는 유사성과 부동성이 자연적 관계들이라고 받아들인다면, 관계들이 그 덕분에 획득하는 그러한 특징들 - 형태적 특징들, 색채적 특징들 등 - 이 언어적 관습의 문제들이라고 추정하는 것은 거의 이치에 맞기 어려워 보인다. 만일 그 관계들이 '자연적'이라면, 그것은 그 덕분에 그것들이 유지하는 그러한 특징들도 동일하게 그렇다는 것으로 드러날 것이다. 주의해야 할 그러한 특징들은 기술들이 아니다: 우리는 크리켓 공과 야구공이 모두 구형이라고 말하지만, 그것들이 서로 구체의 측면에서 유사하다고 말하지 않고, 구체 혹은 구형일 것임의 측면에서 유사하다고 말한다. 우리는 그 사물을 기술한다; 그러나 그 기술은 기술되는 그 사물의 특징 덕분에 유지된다. 이것은, 내가 보기에, 용감한 자들이 용기로부터 이름지어지고 그 역은 아니라고 말하는 이유이다.

5) 이러한 언급들은 물론 유사성들과 부동성들의 세계를 발견하고 특징들의 현존을 추론해내는 사람들과 동일한 세계를 발견하고 추론을 거부하는 사람들 사이의 그 문제를 조정하지는 않는다. 그러나 그 언급들은 아마도 최소한 플라톤의 경우 그 추론이 의지하는 그러한 근거들을 보여주는 어떤 것을 한다. 그렇지만 초기든 중기든 대화편들에서 유사성들에 대한 인식으로부터 도출된 꼴들에 대한 우리의 앎은 없다는 것을 주의해야 한다; 그것은 오로지 우리가 사실상 유사성이 획득한다고 확신할 표준으로서의 꼴의 사용을 통해서만 있기 때문이다.

-蟲-
로고스와 우시아

1) 만일 앞서 말한 설명이 건전하다면, 초기 대화편들 내에서 그런 종류의 논증은 고려함에 있어서 실재적이고 명사적이지 않을 것이다: 그것은 한 대상에 대한 설명이지 한 단어에 대한 설명이 아니다.

2) 그 대상이 적어도 그것이 유와의 관계에서 존립하는 종일 때에는 단순하기 때문에, 아리스토텔레스가 정의될 것과 정의함 사이에서 통용된다는 것을 고수하였던 그러한 관련은 획득하지 않는다: 그 종은 결합된 부분들로서 그것의 유와 종차와 함께 완전히 동일하지는 않다. 그러나 이러한 종류의 같음이 통용되지 않는다고 말하는 것은 어떤 종류의 같음도 통용되지 않는다고 그렇게 부정하는 것은 아니다: 『에우튀프론』10d, 13에서 신들에 의해 사랑받는 것은 경건에 대한 정의로서는 거부되는데 왜냐하면 그 둘이 다르기 때문이고(ἕτερον; 10e, 9에서 ταὐτόν이라는실에 견주어 반대로), 이러한 거부는 15c, 2에서 반복된다(οὐ ταὐτόν … ἀλλ᾿ ἕτερα ἀλλήλων).

3) 여기에는 어떤 종류의 같음이 필요한가? 그것은 외연의 동일성, 또는 진리치 보존(salve veritate) 대체성이 아닌데, 설령 신들에 의해 사랑받는 그러한 것들 모두가 그리고 오로지 그러한 것들만이 경건하거나 경건할 수 있다 할지라도, 이것은 단순히 πάθος를 진술할 뿐 οὐσία를 진술하지 않기 때문이다. 그것이 개념적 동일성인 것도 아니다: 에우튀프론은, 경건을 생각함에 있어서, 그렇게 함으로써 경건의 정의를 생각하지는 않는다. 동의어의 같음도 아니다.

4) 정의는 대상에 대한 설명이다. 그래서 아마도 정의에 있어서 필요한 같음은 이를테면 서술적 동일성, 말하자면 샛별과 태백성의 동일성에 대한 것이어야 하거나, 또는 - 그 예시에서 필요한 경험적 우연성을 피하기 위해 -  일곱번째 정수와 네번째 소수의 동일성에 대한 것이어야 한다. 그러나 이러한 종류의 동일성은 두 가지 한정적 서술들이 동일한 대상에 의해 충족된다는 것을 시사하는 반면에, 실재적 정의에 있어서 정의될 것은 대상에 대한 서술이 아니고, 대상 그 자체이다. 그 비유는 만일 서술적 구절이 명사에 의해 대체된다면 더욱 근접하다: '금성은 샛별이다' 또는 '7은 네번째 소수이다'; 무엇보다도 유일한 서술로 정의를 생각하는 것이 타당하다. 그러나 이것도 오독이다. 지혜는, 의심의 여지 없이, 탁월함이다; 그러나 '…는 탁월하다'라는 말이 소크라테스를 서술하는과 같은 의미에서 '…은(는) 탁월함이다'라는 말이 지혜를 서술하한다고 주장하는 것은 잘못인데, 왜냐하면 첫번째 구절에서 그 관계는 유에 대한 종의 관계이고, 두번째 구절에서 그 관계는 한 특성에 대한 한 특성의 예시 관계이기 때문이다. 그것은 한정적 서술에 있어서 필요한 후자의 관계이다.

5) 아마도 정의에서 필요한 종류의 동일성은 그것 자체의 유(sui generis)라고 말하는 것이 가장 간단할 것이다: 그것은 어떤 것을 포함하는 유와 그것을 그 유 속에서 다른 종들로부터 지적해 내는 종차를 진술함으로써 그 종인 어떤 것을 말할 때 필요한 그런 동일성이다. 종들은 단순하기 때문에, 종차는 종들에 대한 구별의 구성요소는 아니지만, 그 구별의 결과이다; 정의에 있어서 종차는 사유근거이고, 존재근거가 아니며, 그러므로 정의에서 필요한 동일성은 대상들의 동일성이 아니다. 정의의 단일성 문제는 일어나지 않는다. 아마도 정의에 대한 이러한 관점을 위한 최선의 - 그리고 ὁρίζειν과 ὅρος의 대다수 함축에 부합하는 하나의 -  비유는 사상(寫像)의 비유이다: 정의한다는 것은 한 종의 위치를 그 종을 포함한 유 속에서 정하는 것이다. 만일 종이 농장이라면, 유를 규정함의 목표는 그 농장이 위치하는 마을을 지시하는 것일 터이다; 차이를 규정함의 목표는 농장의 경계들을 측량하는 것이 아니라, 오래 전 조사된 경계선들의 현존을 밝혀내는 것일 것이다. 정의에서 필요한 같음은 말하자면, 유일한 위치의 같음이다: 정의될 것은 정확하게 정의하는 것 안에서 사상된 그 위치를 점유한다.

6) 이러한 방식에서 생각되는 최근류와 종차를 통한 정의는 형이상학에서의 전제를 필요로 한다. 그것은 꼴들이, 서로에 대해 포함하는 것에 포함되는 것으로서, 포함되는 것에 포함하는 것으로서 존립하면서, 계층적으로 정렬된다는 것이다. 이러한 관계는 내적이어야 할 것으로 드러나거나, 상호 구성적이어야 할 것으로 드러난다: 종들은 그것들이 만일 그것들을 포함하는 어떤 것에 의해 포함되지 않았다면 그러한 것들이 아니었리라는 점에서 그것들의 특성을 그것들의 유들에 의존한다; 올바름은, 만일 그것이 탁월함이 아니었다면, 올바름이 아니었을 것이다. 반대로, 탁월함은, 그것의 한 종이 올바름이 아니었더라면, 탁월함이 아니었을 것이다: 유는 그것의 현존과 특징을 그것의 종에 의존한다. 전체는 그것의 부분들에 의존하여 전체인 그것일 것이다. 플라톤은 그 점에 대해 조금 덜 명확하지만, 아마도 유들에 대한 정의는, 종들에 대한 정의와 달리, 구성 요소들로의 분석을 필요로 할 것이다.

7) 이것은 플라톤적 유들에 대한 함축의 풍부함을 설명한다. 아리스토텔레스적 논리학에 있어서, 유들은 종들로부터의 추상들이다: 소크라테스와 플라톤은 사람이라는 것에 속하는 공통된 특징을 가지고, 사람과 말은 동물이라는 것에 속하는 공통된 특징을 가진다. 그 유는 여기에서 단지 다양성에 대한 공통된 요소일 따름이고, 그것의 종들보다 더 '추상적'일 따름이다. 그러나 만일 정의가 사상에 대해 동종적이라면, 만일 유들이 부분들을 포함하는 전체들이라면, 유는 공통된 특징이 아니라, 마치 그것의 현존이 그것의 요소들 혹은 부분들의 현존에 의해 전제하기도 하고 전제되기도 하는 하나의 체계이다. 그것은 이것이 그래서 유는, 함축의 상대적으로 결여되었음보다는 오히려 함축을 잔뜩 실은 것으로서, 결핍되었다기 보다는 풍부한 것으로 생각되기 때문이다.

8) 하나의 체계로서, 유는 한 종류의 폐쇄를 지니고 있어야만 하고, 그 체계를 다른 체계들로부터 또는 다른 종류의 제한들로부터 갈라놓는 단일성을 지니고 있어야만 한다. οὐσία와 πάθος 사이의 구별은 한 사물에 본질적인 것과 그 사물이 단지 가지게 되었을 뿐인 특징 사이의 구별을 수반한다. 올바름 또는 탁월함에 대한 경건의 관계와 모든 신들에 의해 사랑받는 것에 대한 경건의 관계 사이에는 차이가 있다. 만일 '그것은 무엇인가?'라는 물음이 본질적 정의에 있어서 내적 성질을 시사한다면, 그러므로 그것은 꼴에 대한 모든 설명들이 본질을 진술하는 것은 아님을 시사하는 것이다. 그것은 즉 그 관계의 내적 성질만큼이나 외적 성질도 시사한다.

9) 유들은 체계들이기 때문에, 그리고 종들은 그 유들의 요소들이기 때문에, 종들에 대한 정의들은 동일성(정체성)에 대한 일상적 진술들이라기 보다는 오히려 사상들이다. 그러한 사상을 구축하기 위해, 또는 사용하기 위해서, 일반적 언어에도 개별적 사물들과 행위들에도 호소할 수 없다: 꼴들에 대한 지식은 지적 직관을, 존재하는 사물들의 본성을 꿰뚫는 직접적 통찰을 요청할 것이다. 실재적 정의의 진리는 오직 그에 대한 정의가 설명인 그러한 진리를 파악함으로써만 파악될 수 있다.

10) 만일 이러하다면, 정의와 직관은 상보적일 것이다. 지적 직관은 실례로 해석되는 것이 아니고, 그렇지만 '알고 있음에 의한 앎'에 속하는 난해한 것이다; 말하자면 어떤 종류의 설명도 제공함이 없이 푸른 하늘의 한 조각을 알고 있을 수 있는데, 왜냐하면 감각은 지적으로 노력할 것이 없기 때문이다. 그러나 꼴들에 대한 직관은 반대로 설명을 요구하기 때문이다; 변증은 단순히 그냥 보는 것이 아니라 보려고 하는 것, 이전에 불충분한 설명 속에서 흐릿하게 보였던 어떤 것을 충분한 설명 속에서 명확하게 보려고 하는 것을 필요로 한다. 시야는 진술들에 대한 검토에 의해 확보된다. 미리 앎에 의한 앎에 대한 지적 직관의 비교는 지적 직관의 대상이 주어진 것이 아니라 하나의 목표라는 사실을 도외시한다.

11) 일부 철학자들은 소크라테스적 변증의 이 지점에서 압박을, 직관과 논리 사이의 긴장을 발견할 것을 주장해 왔다. R. G. 콜링우드는 다음과 같이 썼다.

그렇지만 상당히 높게 플라톤의 철학적 성취들은 평가받고, 최고에 못 미치는 어떤 가치에 있어서 그것들을 평가하는 것은 스스로, 그의 방법론에 대한 이론이 그 자신에 의해 철학과 수학 사이에서 확립되는 충분히 깊은 구별로 이끄는 것에 대한 실패를 통하여 결함있는 것으로 인정되어야만하는, 철학자가 아님을 인정하는 것이 될 것이다. 그 결과는 그의 방법론이 철학을 둘로 분열시키는 것이다: 하나는 재기 발랄한 억지 이론의 식상한 낭비이고, 다른 하나는 궁극적 실재에 대한 직관적 시야이다. 이러한 첫번째 것이 두번째 것으로의 경로라는 것은 여러 세대들에 속하는 경험에 의해 플라톤이 그들의 길잡이로 취해졌다는 것에 대한 보증이 될 것이다; 그러나 설령 그렇다 할지라도, 우리는, 사실들에 권위에서 받아들여질 사실들에 대한 탐구가 아니라, 그 조명 속에서 사살들이 이해될 그러한 개념들에 대한 탐구 속에서, 철학적 탐구에 매진한다; 그리고 이러한 것들을 플라톤이 우리에게 주진 않았다.

이러한 비판은 감각 지각의 틀을 관통하는 지적 직관을 강요함으로부터, 지적인 것으로서, 그 직관이 판단을 통해 지속한다는 것을 알아차리는 데에 실패함으로부터 귀결한다. 지적 통찰에 속하는 한 행위는 하나의 진술이 아니라, 진술 속에서 표현 가능한 하나의 내용을 필요로하고, 그로써 비평에 적합하다. 한 진술은 지적 통찰에 속하는 하나의 행위가 아니라, 그것은 그를 통해 통찰이 표현되는 그러한 것이다 - 그 자신에 대해서조차. 안다는 것은 설명을 제공할 수 있다는 것이다; 설명을 제공할 수 있다는 것은 안다는 것이다.

12) 그러나 이러한 지점에 도달했다는 것은 초기 대화편들을 뛰어 넘었다는 것이다.


3. 꼴들의 현존

1) 소크라테스는 『에우튀프론』에서 경건의 꼴을 존재하는 것으로 다루고, 그와 에우튀프론은 그 꼴의 본성을 발견해 보기로 했다. 현존에 대한 이러한 존재는 다른 대화편들에서도 발견된다. 예를 들어『프로타고라스』에서 소크라테스는 올바름이 어떠한 것인지 혹은 그게 아무것도 아닌지(πρᾶγμα τι ἢ οὐδὲν πρᾶγμα) 묻고, 프로타고라스가 그것이 분명 어떠한 것이라고 그에게 동의할 때, 소크라테스는 올바름이란 게 어떠한 종류의 것인지에 대한 탐구를 계속한다. 더 나아가 경건이 올바름과 같은 것인지 아닌지에 대해 의견의 차이는 있다 할지라도, 경건이 어떠한 것임은 동의된다. 프로타고라스는 더 먼저 소크라테스에 의해 탁월함이 단일한 것이라고 말하도록 이끌렸다; 그는 나중에 각각의 탁월함은 그것의 고유하게 특유한 본성과 실재성을 가진다고 말할 것이다. 사실, 『프로타고라스』의 변증은 정확히 현존의 문제로 돌아선다(349b-c):

물음은 바로 이것이다: '지혜', '사려', '용기', '올바름', '경건'이 동일한 어떤 것에 대한 다섯 가지 서로 다른 이름들인지 아닌지, 또는 그러한 이름들의 각각에 대해 그에 특유한 특정한 본성과 실재성(οὐσία)을, 각 경우에, 다른 어떤 그런 종류에도 속하지 않는, 그것의 고유한 기능을 지니고 있는 어떤 것을, 거기에서 답하는지 아닌지. 이제, 프로타고라스, 당신은 그러한 이름들의 각각이 그것에 특유한 뭔가에 대해 답한다고 말했다. 설명 그것들 모두가 탁월함의 부분들이라 하더라도 말이다.

여기에서 그 물음은, 어떤 탁월함들이 있는가 하는 것이 아니다. 그 물음은 여러 탁월함들이 있는지 아니면 오직 하나의 탁월함만이 있는지 하는 것이다. 『에우튀프론』에서와 비슷하게, 수학, 기하학, 천문학에 대한 플라톤의 철학에 중요한 구절로서, 그리고 수학자들이 찾아다닐 사람들로 이야기되는데, 왜냐하면 그들이 도해만을 만드는 것이 아니라, τὰ ὄντα, 실재성들을 발견하기 때문이다. 탐색자들로서, 그들은 그들이 잡은 것을 사용하지 않지만, 그들의 사냥감을 적절히 사용하기 위해 변증론자들에게로 돌려야만 한다 (290b-c). 다시, 소크라테스의 비유들 대다수는 존재론적 가정 없이는 무의미해질 것이다 - 예를 들어, 탁월함과 『라케스』에서 도출되는 시야 사이의 유비와 같은 것들 말이다.

만일 우리가 그것을 부가하는 것으로 그것이 부가된 것을 더 좋게 만드는 것이라면 어떤 것이든 그에 대해 안다면, 그리고 나아가서, 만일 우리가 그것을 덧붙일 수 있다면, 분명히 우리는 우리가 어떻게 가장 좋게 그리고 가장 쉽게 그것을 얻을지에 대해서 상담자들이 될 그러한 것 자체인 것을 알아야만 한다. 혹시 당신은 이해하지 못하는가? 이런 식으로 보자. 만일 우리가 두 눈에 덧붙여진 시야는 그것이 덧붙여진 그것을 더욱 좋게 만든다는 것을 안다면, 그리고 나아가, 만일 우리가 그 눈에 그것을 더할 수 있다면, 틀림없이 우리는 시야의 본질적 본성을 알고, 그에 대해 우리가 어떻게 그것이 가장 훌륭하게 그리고 가장 쉽게 취해지는지에 대한 조언자들이 되는 것이다; 만일 누군가 시각이나 청각의 본질적 본성인 그러한 것 자체를 알지 못한다면, 상담자나 의사로서 눈과 귀의 문제에 있어서 적임자이기 어려울 것이다. …그러나 그것이 그러한 것처럼, 라케스, 여기 우리의 두 친구들이 이제 어떻게 탁월함이 그들을 더 좋게 만들기 위해 그들의 영혼들에 덧붙여질지에 대해 상담하기 위해 우리를 초대한다. 그래서 처음으로 필요한 것은 탁월함의 본질적 본성을 아는 것이다. 확실히, 만일 우리가 탁월함이 무엇인지 모른다면, 우리는 어떻게 가장 훌륭하게 그것을 획득할지에 대해 조언자들이 되기 어려울 것이다.

시각은 눈에 대한 것이기 때문에, 그리고 청각은 귀에 대한 것이기 때문에, 그래서 탁월함은 영혼에 대한 것이다. 시각과 청각은 있는 것들이기 때문에, 그래서 탁월함은 있는 것이다.

2) 이런 현존에 대한 가정은 변증에 있어서 하나의 역할을 담당한다; 더욱 정확하게, 그것은 그 핵심을 설명한다. 변증의 목표는 실재적 정의이다; 현존적 전제는 정확하게 실재적 정의가 발견되리라는 전제이다. 경건에 대한 참된 설명은 임의적이거나 주관적이지 않고, 에우튀프론이나 혹은 다른 누구라도 우연히 그의 마음에 떠오른 것으로서, 또는 그러한 방식에서 그가 사용하는 말들로서 그러한 내용에 기초하지 않는다: 참된 설명은 사물들의 본성에 대한 설명이다.


언어와 현존

1) 이러한 해석은 초기 대화편들 속에 꼴들의 현존에 대한 그 어떤 형이상학적 관련도 없다는, 그리고 꼴들에 대해 '단순하게 언어적인 문제'가 있을 따름이라고 말하는 공통되게 주장되는 관점과 갈등을 일으킨다. 그러한 관점은 언어와 언어를 사용해 주장하는 어떤 것 사이의 혼동인 듯하다. 무사이 여신들은 그들의 현전에 의해 도취된 시인에게, 비프스테이크의 붉음에 드문 일몰의 붉음을 비교하도록 영감을 줄 것이다; 그 여신들은 그 시인이 그의 존재론에 따라 일몰들과 비프스테이크들에 붉음을 더했다는 믿음을 가지고 영감을 주거나 그로써 영감을 주진 않는다. 그러나 만일 같은 시인이, 더 도취되지 않은 분위기에서, 일몰들과 비프스테이크들뿐만 아니라 곳간들, 소방차들, 러시아인들, 그런 비슷한 모든 것들에 공통된 특징의 본성을 이야기하게 되길 바란다고 설명하면서, 붉음이란 무엇인지 묻는 것이었다면; 만일 그가 계속해서, 그가 그것인 어떤 것을 배웠을 때 그가 그것을 실제로 붉은 어떤 것을 그렇지 않은 어떤 것으로부터 구별해내기 위한 표준으로서 그것을 사용할 것을 기대했다는 것을, 그리고 그가 그것의 αὐσία, 본성과 실재성을 진술할 그것에 대한 적절한, 그리고 최근류와 종차를 통해 표현되는 설명을 기대했다는 것을 더했다면 - 요약해서, 만일 그가 실재적 정의에 대한 규칙을 마련하고 그의 탐구에 있어서 그 규칙들을 따랐더라면, 무사이 여신들이 그에게 내렸던 그 영감이 시적이라기 보다는 형이상학적이었지는 않았는지, 그리고 그가 이제 세계가 있는 방식과 그 방식이 담고 있는, 일상언어나 일반적 상식이 보여줄 수 있는 그 어떤 것도 훨씬 넘어선 어떤 것에 대한 관점을 지니지는 않는지 의심하기 시작해야 한다.

2) 이에 대한 남은 반대는, 만일 소크라테스의 물음들이 사실은 꼴들에 대한 형이상학적 전제를 필요로 했다면, 그의 응답자들은 그것들에 대답할 수 없었을 것이라는 것이다. 물론 에우튀프론과 같은, 그 밖에 다른 누구든, 변증론자가 아닌 사람은 경건의 ἰδέα가 있다는 전제를 이의 없이 받아들여야 한다는 것은 의미가 있고 또 주장될 것이다. 확실히 플라톤 자신이 도덕에 있어서 인습주의자로 그려냈던 프로타고라스는 주저 없이 올바름'은 어떠한 것이다'라는 것에 동의하게끔 된다는 것은 의미심장하다. 프로타고라스가 그로써 추상적 대상들의 존재론을 수용한다고 말하기는 어려웠다; 그의 동의는 오히려 언어의 일상적 사용들에 의해 유도되었을 성싶다. 냉소적이거나 혹은 그 반대로 부족한 우리 모두는 무엇보다도 올바름과 같은 그러한 것이 있다고 믿는다.

3) 그 반대는 추상물들을 인격화하는 고대 그리스의 경향을 지적함으로써 강화될지 모른다.

대부분의 그리스 산문작가들에게서 추상적 명사들은 드물게 동사들의 주어가 된다; 평범한 행위자들은 인간존재들이다. 그럼에도 불구하고, 수많은 예외들이 있다. …추상적 주어의 사용이 의인화의 발상을 - 상이한 경우들에 있어서 - 얼마나 수반하는지 말하는 것은 때로 어렵다. 아마도 고대 그리스인은 대답에 대해 당황했을 것이다. 의심할 나위 없이 의인화가 나타나는 곳에서 얼마나 많은 의인화가 종교적이거나 극적인 암시들을 일으키는지 결정하는 것도 쉽지 않다. 추상적 주어는 그리스에서 언제나, 또는 거의 언제나 최소한 의인화의 어떤 색조를 띤다. 우리는, 얼마나 완전하게 우리 고유의 언어에서 추상적 주어가 의인화하는 능력을 상실했는지 보기 위해 오직 크세노폰의 ἀναγκάζει με καὶ ταῦτα ὁμολογεῖν ἡ ἐμὴ φαυλότης,라는 경구(『메모라빌리아』, Ⅳ, ii, 39)를 영어 'candour compels me to admit(정직이 나에게 인정하도록 강제한다)'라는 말과 비교해야 한다. 그러므로 그러한 구절들을 영어로 번역함에 있어서, 아마도 부분적으로는 첫 대문자들의 18세기 사용을 되살림으로써 마주칠 심각한 어려움이 일어난다.

그러나 의인화는 형이상학을 수반하기 어렵다 - candour가 Candour라고 할지라도.

4) 데니스톤이 여기에서 고찰하고 있는 추상 개념들은 잠(Sleep), 밤(Night), 그리고 탈선(Digression) 같은 명사들이다. 의인화가 형용사적 추상어들에서 'the holy' 또는 'holiness'처럼 관사나 접미사와 관련되는지 보는 일은, 비록 그것이 그렇다 할지라도 덜 쉽다. 그러나 그것이 그렇지 않다 할지라도, 여전히 초기대화편들에서 나타나는 것과 같은 그런 보편개념들의 실체화는 진정한 존재론적 관련도 수반하지 않는다는 것이 주장될 것이다. 우리는 유니콘이 존재한다고 생각하지 않으면서 유니콘은 어떤 종류의 것인지 물을 것이기 때문에, 그래서, 우리가 경건이 존재한다고 추정함이 없이 경건이 어떠한 종류의 것인지 물을 것이라고 주장될 것이다. 우리는 어떤 그리스 신이 있다고 추정하지 않고 몇몇 그리스 신들은 간통을 했다고 주장할 것이기 때문에, 그래서 우리는 어떤 탁월함들이 있다고 생각하지 않고 여러 탁월함들이 있다고 주장할 것이다. 일상적 언어에서 - 일상적인 근대 영어뿐만 아니라 일상적인 고대 그리스어에서도 - 어떤 의미에서 어떤 것이 '있다(이다)'고 말할 것이다 - εἶναι πως; 그러나 그래서 εἶναι πως는 εἶναι와 상당히 거리가 멀다. 그것은 '어떤 의미에서'의 한 가지 것일 것이다; 그것은 실제 세계에서 하나의 요소일 또 다른 것이다. 그래서, 말하자면 올바름의 현존에 대한 일상적 언어나 공통 감각의 관련이 Mr Pickwick에 대한 그것의 관련과 훨씬 더 동등하다고 주장하는 것은 비합리적이지 않다. 물론 Mr Pickwick와 같은 그런 사람이 있다. - 그는 일찍이 Sam Weller와 함께 선거들을 위해 Eatanswill로 여행했다. 그러나 Pickwick적인 여행들은 Pcikwick적인 올바름과 같다. 비록 각각이 그것들의 고유한 방식에서 흥미로 가득 찬다 하더라도, 그것들은 존재론적으로 불분명하다. 어째서 이것은 또한 초기 대화편들에 있어서 꼴들에 대한 이야기에 대해 참이 아니어야 하는가?

5) 그 물음은 쉽사리 답해진다. 일상적인 그리스어에서 '추상적 실체들'에 대한 관련 또는 용인에 의해 꼴들의 현존에 대한 초기 대화편들의 관련이 측정되리라고 추정할 아무런 이유도 없다. 『에우튀프론』에서 경건은 보편개념, 표준, 본질로 다루어진다. 이러한 취급은 전문적이며 공식적이다; 일상적 언어는 이런 식으로 생각되는 추상들의 현존에 관련되지 않는다. 프로타고라스가 정의와 같은 그런 것이 있다고 생각하는 중에 그로써 그것을 꼴로 생각하지는 않았다는 것은 사실이다; 그러나 그래서 그는 올바름이 무엇인지 몰랐다. 에우튀프론이 초반에 경건이 하나의 ἰδέα임을 수용했다는 것은 참이다; 그가 그것을 얼마나 조금 이해하는지는 대화의 남음 부분에서의 변증이 보여준다. 초기 대화편들에 있어서 꼴들에 대한 그 관련은 '단순히 언어에 속하는 문제'가 아니다. 그것은 형이상학에 속하는 문제이다 - 본질의 형이상학 말이다.

6) 그러나 만일 소크라테스가 형이상학에 있어서의 이론을 갖춘다면, 어떻게 그가 그것을 일상인들과의 대화에 있어서 적용하는 것 - 더욱이 당연한 것으로 받아들이는 것 - 이 가능하다는 것인가? 그 대답은, 비록 꼴들에 대한 이론이 나아가 형이상학적 이론이라 할지라도, 그것은 또한 본질적으로, 진부한 단어의 한 의미로, 의미에 대한 이론임으로써, 공통 감각(상식)과 연관되어 있다. '당신은 무엇을 사려라고 말하는가?' 소크라테스가 카르미데스에게 묻는다. '왜냐하면 당신은 그리스어를 할 줄 아니까, 당신은 의심할 것도 없이 그것이 당신에게 어떠한 것으로 보이는지 말할 수 있다.' 크리티아스가 오로지 그를 논박하기만을 위해서 논증할 것을 소크라테스에게 떠넘길 때, 소크라테스는 대답한다:

만일 내가 당신을 논박하는 데에 나의 최선을 다 한다면, 내가 모르는 어떤 것을 내가 안다고 생각하는 것이 어떤 지점에서 내 주의를 빠져나간다면 그러지 못하도록 그렇게 나 자신의 말들이 뜻하는 어떤 것을 검토하는 것보다, 내가 다른 어떤 동기로 그렇게 한다는 것을 당신은 어떻게 믿는가?

평범한 사람들은 소크라테스가 그들에게 그들이 뜻하는 것이 정확하게 무엇이었는지 묻기 시작한 오래 전부터경건에 대해 말해 왔다. 그의 탐구는 경건에 대한 정밀함에 의해 구별되었다. 그는 경건의 동의어들, 혹은 경건의 사례들, 또는 경건의 구별되는 특징들을 원한 것이 아니라, 경건의 본질에 대한 분석을 원했다. 그의 물음은 공통 감각(상식)이 그 자신의 고유한 방책들에 기대어 물을 그러한 것이기 어렵다. 그러나 그것은 일반 상식이 확실히 이끌릴 물음이고, 『에우튀프론』의 변증은, 소크라테스가 에우튀프론으로 하여금 그의 물음의 실질적 본성을 보도록 만드는 내내 노력한 것과 같이, 사실상 그러한 인도의 기록이다. 변증의 진행은 '경건 같은 그러한 것이 이다'라는 응답자의 순진한 현존 전제에서, 만일 변증이 성공적이라면, 경건의 본질이 있다는, 그리고 그것이 정의될 수 있다는 고도로 정교화된 현존 전제에 대한 그 응답자의 수락으로 이행하는 것을 필요로 한다. 그러나 만일 후자의 주장이 참이라면, 그 이행은 연속적이다: 본질에 대한 관련은 우리의 일상적 단어 사용에 잠재적이기 때문이다. 경건의 본질은 '경건'이란 단어가 의미하는 그것이다; 우리가 본질을 이해하지 못하는 수준에서, 우리는 우리의 단어들이 의미하는 바를 이해하지 못한다.

환원성

1) 초기 대화편들은 꼴들이 현존한다고 주장되거나 암시되는 단언들로 수놓아져 있다; 그리고 그러한 단언들은 단순한 언어의 문제로 치부될 것이 아니다. 그러나 이에 대한 증거가 아직 그 결론을 반대하는 학자들에게 오래 전부터 알려져 왔기 때문에, 그 문제를 더 밀고 나가 보는 것이 좋을 지도 모르겠다.

2) 누군가는 이런 논증을 그려볼 수도 있겠다. 그 저술은 의심의 여지 없이 꼴들이 현존한다고 단언한다; 그러나 그 저술은 그런 주장을 필요로 하지 않는다. 초기 대화편들 내에서 꼴들에 대해 이야기하는 것은, 추정되는 존재론적 관련이 사실 암암리에 어쨌든 언어적 관련이 있음을 발견했다는 점에서, 피할 수 있다. 플라톤은 - 누군가는 그 논증이 연이어지는 것을 상상할 것이다 - 실제로 언어에 대한 사실들인 항목들을 세계에 대한 사실들로 표현하기 위해, 담화의 구체적인 방식에 대한 의심할 것 없는 불가피한 편애라고 할지라도, 운이 나빴다. 소크라테스적 변증은 사물들의 본성에 대한 분석으로서 제공된다; 그러나 그것은, 결국, 피할 수 있는 담화방식이다. 꼴들의 현존을 주장하거나 시사하는 진술들은 논리적 여항 없이 꼴들의 현존을 주장하지 않는 진술들로 환원가능하거나 분석가능하다. 본질에 대한 물음들은, 플라톤 자신조차 동의했을 것과 마찬가지로, 의미에 대한 물음들이고, 의미에 대한 물음들은 당연히 언어적이다. 적절하게 이해된 소크라테스적 변증은 - 그리고 플라톤은 의심할 것도 없이 적절하게 그것을 이해하지 않았는데 - 세계를 향해서가 아니라 말들의 의미를 향해 정향된다. 결국, 꼴들이 존재한다고 말하는 것은 짐작컨데 단어들이 의미를 가진다는 것 그리고 탐구를 통해 우리는 그러한 의미를 분명하게 만들 것이라 기대할 것이라는 점을 말하는 것이다. 꼴들이 보편개념들이라고 말하는 것은 상이한 문맥들에서 동일한 의미를 가지는 단어들이 있다고 말하는 것이고, 그 단어들이 우리가 나타내는 것들을 묘사한다고 말하는 것이다. 꼴들이 표준들이라고 말하는 것은 한 서술의 의미를 이해한다는 것이 그 서술이 해당하는 상황들을 동일화할 수단을 얻는 것이라고 말하는 것이다. 꼴들이 본질들이라고 말하는 것은 한 서술의 의미에 대한 모든 각각의 설명이 그 의미인 것을 정확하게 표현하지는 않는다고 말하는 것이다. 소크라테스의 물음, '경건이란 무엇인가?'란 물음은 요약하자면 '"경건"이란 단어가 뜻하는 것은 무엇인가?'라는 물음으로 만든다. 그러므로, 초기 대화편들에 꼴들에 대한 존재론적 언질이 있는지에 대한 문제는 이중적이고, 이중의 대답을 허용한다. 만일 그 물음이 그러한 대화편들의 본문이 꼴들의 현존을 긍정하는지 아닌지에 대한 것이라면, 그 대답은 긍정한다는 것이다. 만일 그 물음이 플라톤이 상술하고 있는 위치의 논리가 꼴들의 현존을 요청하느냐는 것이라면, 그 대답은 요청하지 않는다는 것이다.

3) 그래서 우리는 소크라테스의 물음, '경건이란 무엇인가?'라는 물음이 '"경건"이란 단어가 의미하는 것은 무엇인가?'라는 물음으로 변환될 수 있는지 결정하게 된다. 그것은 이러한 대체가 순환 없이는 이루어질 수 없다는 것으로 밝혀진다.

4) 몇몇 상황들 아래에서 '"경건"이란 단어가 의미하는 것은 무엇인가?'라는 그 물음은 사전적일 것이고, 프로디쿠스나 아니면 사전에 의해 대답될 물음일 것이다. 그러나 단순하게, '경건이란 무엇인가?'라고 묻는 와중에 소크라테스는 문자상의 동의어들에 대한 탐구에 종사하지 않는다. 에우튀프론조차 소크라테스가 경건이 신성함이라는 답으로 만족해야 한다고는 생각하지 않는다.

5) 다시, '"경건"이란 단어가 의미하는 것은 무엇인가?'라는 물음은 '실물 지시적 정의', 예시에 호소하는 정의에 의해 대답될 것이다. 그러나 예시가 구체적인 예증으로서 협소하게 구성되는지, 아니면 요청되는 특징을 드러내 보이는 일반적 경우로서 더 넓게 구성되는지, '그것은 무엇인가?'라는 물음에 대한 이런 종류의 답변은 초기 대화편에서 그러한 답변이 나타나는 어디에서고 거부된다: 에우튀프론은 경건을 '바로 지금 내가 하고 있는 일', 또는 더욱 일반적으로, 살인자들과 성물 절도범들을 고소하는 것으로 정의할 수 없다.

6) 다시금, '"경건"이란 단어가 의미하는 것은 무엇인가?'라는 물음은 아마도 경건의 구별적 특징들을 제공함으로써, '경건'이라는 단어의 적용의 기준들 혹은 사용의 규칙들을 제공함으로써 답변될 것이다. 그러나 구별적 특징의 관념이 가장 강한 의미에서 - 전체이자 유일하며 항상된 것(id quod omni et soli et semper), 그 용어를 적용하기 위한 필요조건이자 충분조건 -  이 물음은 '경건이란 무엇인가?'라는 그 물음에 등가가 아닌데, 왜냐하면 구별적 특징이나 대체 가능한 기준의 제공은 후자의 물음을 대답되지 않은 채로 남겨둘 것이기 때문이다. 에우튀프론이 경건을 모든 신들이 사랑하는 것으로 정의할 때, 소크라테스는 이 특징이 경건한 모든 것들이자 유일하게 경건한 것들을 구별하는 데에 시패한다는 것을 보여주는 일을 떠맡지 않는다; 그는 맹목적으로 이것을 받아들이거나, 최소한 그것의 가능성을 용인한다. 그는 그가 쉽사리 했었을 것 처럼, 그러한 특징이 경건의 예증들을 동일화함에 있어서 어떤 실질적 사용에 속한 것일지 아닐지 하는 물음을 던지는 것도 아니다. 오히려 그는 에우티프론이 우연히 경건의 πάθος를, 경건한 것들에 대해 우연히 참인 어떤 것을 발견했을지라도, 그가 경건의 οὐσία를, 경건의 본성과 실재성을 보여준 것은 아니라고 주장한다. 구별적 특징의 제공은 그래서 '경건이란 무엇인가?'라는 물음을 대답되지 않은 채로 남겨둔다.

7) 그러나 '"경건"이란 단어가 의미하는 것은 무엇인가?'라는 물음에 대한 또 다른 종류의 답변이 있다. 그것은 불분명한 답변이지만, 당장의 목적들을 위해서 중요한 한 가지 답변이다. 그것은 '경건'의 의미가 경건이라는 것이다.

그래서, 경건이란 것은 무엇인가 기원하는 것인가?
정확히 그렇다.

 8) 초기 대화편들에 있어서 꼴들에 대한 물음들은 단어들의 의미에 대한 물음들로 환원될 수 없는데, 왜냐하면 단어들의 의미에 대한 첫번째 물음은 꼴들에 대한 물음들로 바뀌기 때문이다. 만일 비환원성이 존재론적 관련의 기준일 것이라면, 초기 대화편들에는 꼴들에 대한 존재론적 관련이 있다.

9) 이러한 관련을 제공하는 의미이론은 더 탐구되어야 할 필요가 있다. 그러나 독자는 여전히 끈질긴 의심을 계속 지닐 것이다. 윌리엄 제임스는 일찍이 가상적 불길이 (당신이 그에 대해 말할 수 있는 그런 종류의 회피 가능한 것이 아니라) 실제 장작개비들을 태우지 않을 그런 종류의 불길이라고 언급했었다. 이것은 『소피스트』(247e)에서 존재론적 관련에 대한 플라톤의 고유한 기준, 그가 신들과 거인들에 의해 비슷하게 공유되었다고 생각했던 기준을 반영한다: 그것은 존재의 표징이 능력이라는 것이고, 작용하거나 작용받을 그러한 능력이라는 것이다(πάθημα ἢ ποίημα ἐκ δυνάμεώς τινος, 248b). 차이는, 차이이려면, 차이를 만들어야 한다. 만일 꼴들이 존재하는 것이라면, 우리는 그것들이, 말하자면 단지 거기에 놓이는 것이 아니라, 진실로 작용할 것, 세계의 운행에 영향을 줄 것을 기대할 것이다. 그것들은 이러한 것을 수행한다; 우리는 초기 대화편들에 있어서 꼴들이 원인들임을 알아차릴 것이기 때문이다.


본질과 지시

1) 만일 '경건'이란 단어의 의미가 경건이라면, 그리고 경건이 꼴이나 본질이라면, 그 단어와 그 어떤 것 사이에 자리하는 관계가 명칭이나 이름 붙임에 속하는 하나의 관계라고 추정하는 것은 합리적이다. 초기 대화편들은 이러한 관점을 뒷받침한다. 『프로타고라스』에서, 소크라테스는 프로타고라스가 방금 이름붙인(ὠνομάσατε) 어떤 것(πρᾶγμα)이 정의로운지 정의롭지 않은지 묻는다(330c); 나중에, 탁월함이 하나인지 혹은 여럿인지에 대한 물음은 '지혜', '올바름', '사려', '용기', 그리고 '경건'이 상이한 것들을 위한 이름들(ὀνόματα)인지 아니면 동일한 것을 위한 이름들인지 탐구하는 형식을 취한다. 초기 대화편들은 의미에 대한 지시적 이론을 전제한다: 그 대화편들은 추상적 명사들, 또는 그것들 중 일부가 이름들임을 전제한다.

2) 콰인 교수는 다음과 같이 지적했다. '의미는 그것이 지시대상과 분리되고 그 단어와 결합될 때 본질이 되는 어떤 것이다.' 반대로, 본질은 그것이 단어와 분리되고 대상과 결합될 때 의미가 되는 어떤 것이다. 그래서 의미되는 어떤 것은, 최소한 때때로라도, 단순히 표현이라는 면에서가 아니라, 다소간 세계의 질서에 알맞는 내용의 일부라는 면에서, 그러나 그 자체로 그 본성이 발견에 있는 그러한 세계의 질서 속에서 하나의 항목이 된다. 우리는 세계를 불완전하게 이해하기 때문에, 우리는 우리의 말들의 의미를, 혹은 그 중의 일부를 불완전하게 이해한다; 또 다른 방식으로 보자면, 우리가 일상적으로 하나의 표현으로써 의미하는 어떤 것, 그것의 공공연하거나 또는 명백한 내용과 우리가 만일 그 표현이 지시하는 본질을 이해했다면 의미해야 할 그러한 어떤 것 사이에는 차이가 있다. 에우튀프론은 분명 그가 일상적으로 경건을 통해 의미하는 어떤 것을 말할 수 있다; 그는 살인자들과 성물 절도범들을 고소하는 것을 말한다; 숙고를 거쳐서, 그는 신들에 의해 사랑받는 어떤 것을 말한다; 더욱 숙고한 끝에, 모든 신들에 의해 사랑받는 어떤 것을 말한다. 그러나 이것들 중 아무것도 경건의 본질을 진술하지 않는다. 그의 이해력과 그의 의도 사이에, 그가 경건에 대해 제시할 수 있는 설명과 그가 설명하고자 시도하고 있는 본질 사이에 차이가 있다. 만일 이따금 의미한다는 것이 이름하는 것이라면, 그리고 이름지어지는 것이 본질이라면, 의미되는 것은 종종 주어지는 것이 아니라, 목표일 것이다.

3) 만일 때때로 의미함이 본질을 수반한다면, 그리고 우리의 본질에 대한 앎이 종종 불완전하다면, '제각기 모든 것이 그것인 것으로 질서지어져 있다'는 것은 이상 언어에 대해 참이 아닐 것인데, 왜냐하면 단어들을 지배하는 사용규칙들이 필연적으로 그 단어들이 그에 의해 통제되어야 하는 규칙들인 것은 아니기 때문이다. 아마도 해결책은 본질들을 부정하는 것일 것이다:

누군가는 거듭해서 사물의 본성에 속하는 윤곽을 추적하고 있다고, 그리고 단순히 우리가 그를 통해 본성을 바라보는 그 틀의 주변을 추적하고 있다고 생각한다. 한 장면이 우리를 달아나지 못하도록 붙잡고 있었다. 그리고 우리는 그 밖으로 나갈 수 없었는데, 왜냐하면 그것이 우리의 언어와 우리에게 변치 않고 그것을 반복하는 듯이 보이는 언어에 자리하기 때문이다. 철학자들이 단어를 사용하고 그 사물의 본질을 파악하고자 노력할 때, 누군가는 반드시 그 자신에게 물어야 한다: 그 단어가 도대체 실제로 그것의 본래 집인 언어 게임 안에서 이런 방식으로 사용되는가? - 우리가 행하는 것은 단어들을 그 형이상학적 사용으로부터 우리 일상의 사용으로 되돌려 가져오는 것인가?

비트겐슈타인은, 그가 반대하고 있던 것을 아주 잘 알았다. 만일 이 관점이 참이라면, 소크라테스적 변증이 기대고 있는 그 전제들은 거짓이다. 그래서 탁월함의 단일성에 대한 전형적인 소크라테스적 주장들도 거짓이다: 탁월함이 앎이라는 주장, 용기가 지혜라는 주장, 경건이 올바름이라는 주장. 그 말들은 일상적인 방식으로 사용되지 않는다 - 현대 영어에서만큼이나 그 이상으로 고대 그리스어에서. 프로타고라스는 사람은 지혜롭지 않고도 용맹할 수 있으리라 주장함에 있어서, 그의 언어 사용 규칙들을 보여주고 있었다. 소크라테스의 언어적으로 독특한 사물들의 결합은, 그것이 만약 언어에 대한 단순한 전제적 입법일 것이 아니라면, 본질의 관념으로부터 분리되어 이해할 수 없는 것이다.


본질과 일상언어

1) 소크라테스적 변증은 단어들의 일상적 사용을 탐구하는 것이 아니라, 단어들의 사용이 근거하는 것들에 있어서 특징들을 탐구해 들어가는 것이다. 한 단어의 사용은 의심할 나위 없이 그 단어를 지시하는 특징과 연관된다; 만일 우리가 언제나 사물들의 본성에 대해 명확하지 않다면, 그로써 우리가 완전한 암흑 속에 있다는 것이 두따르진 않는다. 그러나 본질과 사용 사이의 관련은 경험적이다: '기준'이라는 용어의 제한적 의미에서, 단어가 지시하는 본질은 그것을 적용하기 위한 기준이고, 유일하게 신뢰 가능한 기준이다.

2) 이러한 관점에 대한 하나의 반박이 있다. 우리는 우리의 어머니 품에서 말하는 것을 배웠고, 말하는 방법을 아는 것은 일반적인 용어들을 사용하는 방법을 아는 것이다; 그렇지만 일반적 용어들을 사용하는 방법을 아는 것은 그 언어의 다른 발화자들이 확실하게 동의할 명백한 경우들이 그 용어들의 예시들임을 예증할 수 있는 것이다. 일반적 용어를 사용할 줄 안다는 것은 물론 그 용어의 사용을 위한 기준을 제시할 수 있다는 것과 다르다; 그러나 그 다음으로 우리는 그 사용을 관찰함으로써 기준들을 발견할 수 있다. 그래서, 우리는 종종 주어진 한 행위가 경건하다는 것을 '경건'이란 단어에 제공할 기준을 제시할 수 없어도 완벽하게 잘 알 수도 있다; 그리고 더 나아가서, 우리는 종종 기준보다는 사례가 제시됨을 통해서 어떤 것이 그것이 경건일 그러한 어떤 것을 더 잘 배울 것이다. 소크라테스적 변증은 그러므로 오해된다. 의미에 대한 탐구에 있어서 사례에 호소하는 것을 허용하기를 반대한다는 데에 이어서 그것은 실수이다; 같은 식으로 의미는 사례들이 결정될 수 있기에 앞서 반드시 고정되어야만 한다고 전제함에 있어서도 그것은 실수이다. 이러한 실수들 때문에, 소크라테스적 변증은, 의미에 대한 충분한 기준이 발견됐을 때 결정에 대한 아무런 건전한 수단도 가지지 못하고 부유하게 된다. 그리고 이 사실은 어째서 초기 대화편들의 변증이 그렇게나 일관되게 실패로 끝나는지에 대한 설명으로 잘 받아들여질 것이다. 게임의 규칙들은 아무도 그 게임에서 승리할 수 없게끔 그렇게 정해져 있다.

3) 확실히 이러한 반대에 핵심이 있다. 에우튀프론은 사례에 호소해서도 그가 사용하는 언어의 발화자들이 이상적으로 그 용어를 가지고 이해했던 어떤 것에 호소해서도 경건을 정의할 수 없다. 그러면 그는 도대체 그것을 어떻게 정의할 수 있는가? 여기에는 앎에 대한 분명한 문제가 하나 있다. 때마침, 『메논』에서 플라톤이 탐구의 역설로 진술하였던, 그리고 우리가 보게 될 것과 같이 상기의 원칙으로 해결했던 한 문제가 있다.

4) 그러나 만일 앎의 문제가 분명하다면, 또한 그것은 분명 실재의 문제인데, 왜냐하면 경건일 그러한 어떤 것은 경건의 사례들을 제시함으로써 정의될 수는 없다는 소크라테스의 전제는 그저 옳을 뿐인게 아니라 확실하게 옳기 때문이다. 만일 a가 F의 한 예라면, 그리고 만일 그에 대해 F일 그러한 어떤 것을 a가 정의한다고 주장하는 이유라면, 그리고 만일 b가 F의 한 예라면, b일 것은 a일 것이다; 그리고 a는 F일 그러한 어떤 것을 정의하므로, F일 것은 a일 것이다. 우리는 그래서 번갈아서 a는 F의 예시가 아닌데, 왜냐하면 그것은 F일 그러한 어떤 것이기 때문이라고 말하거나, F는 그 자신의 예시이자 유일한 예시라고 말할 것이다. 만일 우리가 'F'를 대체하고, 그에 따라 '경건일 그러한 어떤 것'은 '경건'이라는 형요사를 대신하는 것이 아니라 '경건'이라는 추상명사를 대신한다는 것은 분명해 진다면, 그 특이함의 원천은 여기에서 분명해진다. 무엇이 이러저러한 용어로 경건한 것을 경건으로 정의하는 것을 의미할 것인가?

5) 동일한 고려들이 경건의 본성을 알지 못하고서는 주어진 어떤 것이든 그것이 경건하다는 것을 아는 것은 불가능하다는 소크라테스의 주장을 대신한다. '나는 a가 F임을 안다'는 명제와 '나는 F일 그러한 어떤 것을 모른다'는 명제는 양립할 수 없다. 만일 내가 a는 F라는 것을 안다면, 나는 a가 F일 그러한 어떤 것의 일례라는 것을 안다; 만일 내가 F일 그러한 어떤 것을 모른다면, 나는 a가 그것의 한 예시일 그러한 어떤 것을 모르고, 만일 내가 a가 그것의 일례일 그러한 어떤 것을 모른다면, 나는 a가 F의 한 예시라는 것을 모르며, 그러므로 a가 F라는 것을 모른다. F일 그러한 어떤 것이 a가 F인지 아닌지를 결정하기 위한 기준이라는 것이 뒤따르는데, 오로지 a가 그 특징에 대한 하나의 예시인 그 특징에 대한 앎을 통해서만 a가 그 특징의 일례라고 누군가 결정할 수 있는 것이기 때문이다. 그래서 경건일 그러한 어떤 것은 주어진 어떤 것이 경건하다는 앎에 선행한다.

6) 소크라테스적 변증에 대한 이러한 방어가 수작을 부린 것이라고(주사위를 조작하다) 주장할지 모른다. 그 주장은 어떤 것이 무엇임을 정의하는 것과 아는 것에 관련된 관념들 - 그것이 정확하게 소크라테스적 실수인 - 에 달려 있다. 철학적 탐구의 고유한 목적은 정의가 아니라 사용의 분석이다. 『에우티프론』에서 세워진 그런 종류의 탐구는 그런 분석이 아니고, 이것이 그렇기 때문에, 그것은 일부 제한적 의미에서 (의심할 바 없이 신실하게) 기만이다.

7) 그러나 기만에 대한 그 주장은 두 가지 방식으로 작용한다. 『에우튀프론』의 변증이 특별히 다루고자 시도하는 문제를 고려해 보자: 도덕적 판단에 있어서 분쟁의 해결. 우리는 사례에 호소함으로써 도덕적 용어들의 사용을 위한 기준에 도달해야 한다고 제안된다. 하지만 실천적인 문제로서, 기준의 결여는 사례들이 빗나갈 때 정확하게 감지된다. 사례들의 비일관성은 기준의 비일관성에 대한 조짐이다: 만일 당신의 기준이 나의 사례를 예외로 한다면, 나는 나의 사례를 거부할 것이나, 나는 마찬가지로 당신의 기준도 거부할 것이다. - 많은 것이 의심의 여지 없이 누구의 소가 뿔에 받혔는지에 달려 있을 것이다(모순 여부). 만일 도덕전 논증에서의 호소만이 사례들로부터 추출된 사용기준일 것이라면, 에우튀프론과 그의 아버지는 각자 논리적으로 난공불락의 위치에 있다. 도덕적 분쟁을 해소할 궁극적 방법이란 크롬웰 류의 담론에 의지하는 외에 아무것도 없다 - 우리들의 원칙을 교황의 입김과 두드림들로 정통이라 증명하는 것이다.

8) 그러나 아마도 이러한 전망은 지나치게 암울한 것이다. 도덕적 의견 불일치, 그것이 아마도 주장될 것인데, 그것은 오로지 경계 경우들에서만 발생하고, 분명한 사례들에 호소함으로써 해소될 것이다. 우리는, 그것을, 모든 F들이 깔끔하게 그 안에 자리잡히고 모든 비(非)F들이 정확하게 묘사된 원 없이 위치되는, 일종의 객체화된 벤다이어그램으로 상상하면서, 태평스럽게 F인 것들의 계층에 대해 말한다. 진실은 수학적 언어와 구별되는 것으로서 일상 언어에서 가장 일반적인 용어가 그것들을 그 안에서는 그것들의 적용 조건들이 불분명한 반영(半影)으로 이끈다는 것이다: 사례는 분명히 F인 것과 분명히 F 아닌 것 사이에 그 안에서 말하는 것이 무엇인지 알기 어려운 범위, 준연속체가 있다는 그런 방식에서 사례에 그림자를 드리운다. 이것은 컵이다, 그리고 저것은 그릇이다, 그러나 여기에는 이러저러하게 부르거나 아무것으로도 부르지 않거나 그 둘 모두로 부를 어떤 것이 있다. 여전히, 우리는 의심스러운 경우들을 판단하기 위해서 경우들을 분명하게 하기를 기대할 것이고, 경우들을 그것들이 무엇인지에 대해 분명하게 인식하지 못하는 누구라도 그가 그것들을 논의하고 있는 그 언어를 이해하지 못한다. 만일 당신이 내가 지금 그로부터 커피를 마시고 있는 이것이 컵이라고 생각하지 않는다면, 당신은 '컵'이란 단어가 의미하는 어떤 것을 모른다. 그래서 마찬가지로 도덕적 용어들도 동일하다.

9) 이것이 참이라면 그건 즐거울 일일 것이다. 불행하게도, 그렇지는 않다. 도덕적 용어들은 일찍이 플라톤이 지적하였듯 '사람들에게 눈에 띄게 만드는 어떠한 상도 가지지 않는다.' 그렇지만 그것은 아마도 경험적 용어들과 함께일 것이다, 선악의 문제, 옳고 그름의 문제들, 한 사람의 분명한 경우가 두번째 사람에게 반영을 드리울 수 있고 세번째 사람을 덮으며, 이것이 단순히 주변부에 관한 것만이 아니라 도덕적 추론이 그것을 가지고 다루어야만 할 가장 핵심적 문제들에 관한 것이라는 특이점을 가진다. 이러한 점을 사례들을 들어 주장하는 것은 아마도 불필요할 것이다 - 다행스럽게도 -; 비참하고 소란한 세기(이 책은 1970년대에 출판되었다)에, 독자는 그 스스로 감당하도록 남겨질 것이다. 그러나 누군가는 아마도 탁월함이 앎이라는 것이 역설이리라거나 혹은 그렇지 않으리라고 할지라도, 악이 실어증이라고 생각하는 것은 단순히 터무니없는 소리라고 지적하는 모험을 할지도 모른다. 만일 선과 악에 대한 물음들이 사람들이 언어로 하는 게임들에 대한 물음들이 아니라면, 또는 태도나 관습 혹은 취향에 대한 물음들이 아니라면, 도덕적 앎과 같은 어떤 것이 있다거나 혹은 있을지도 모른다고 한다면, 앎이 본질에 대한 식별에 존립한다는 『에우튀프론』의 주장은, 그것이 틀렸을지도 모른다 할지라도, 확실히 기만은 아니다. 다루어져야 할 문제들을 정면으로 마주함이 없이 거부하는 것은 어리석은 일일 것이다.


단 하나의 이름 이름지어지는 단 하나의 것(Unum Nomen Unum Nominatum)


1) 지시적 의미 이론들은 종종 특유한 필연적 귀결, unum nomen unum nominatum(the sole name the sole named)을 가져오는 것으로 생각되어 왔다. 그리고 다양한 비판자들은 그들이 초기 대화편들에 있어서 이러한 전제를 감지했다고 생각해 왔다. Mr 리차드 로빈슨은 '만일 소크라테스의 물음(X는 무엇인가?)이 답변을 허용하는 정당한 물음이라면 … 우리는 반드시 X라는 단어가 단일한 뜻을 가진다고 전제해야만 한다'고 말했다. 로스 교수 또한 소크라테스적 물음에서 이러한 함축을 발견하고, 다음과 같이 추론한다. '플라톤은 이름의 의미에서 모호함의 가능성을 알아차리고 있었다. 그러나 보이기로는 그가 이것을 다소 드물게 일어나는 것으로 간주했고, 겉보기에 가장 순전한 이름이 가질 의미조차도 변화하는 그 의미의 음영을 완전히 알아차리지는 못하고 있었다.' 그러나 이것은 잘못된 추론에 기인한다. 소크라테스는 '경건이란 무엇인가?'와 같은 물음들을 묻고 답할 수 있다고 전제한다; 그는 더 나아가 경건이 하나의 꼴이나 본질이라고 전제한다; 그러나 이러한 전제들은 '경건'이나 '경건함'과 같은 단어들이 상이한 의미로 사용된다거나 사용될 수 있다거나 하는 물음에 아무런 관련도 없다. 그 잘못된 추론은 소크라테스는 다른 것들 중에서 우리가 같은 이름으로 상이한 것들을 부르는 것과 그렇게 불리는 것들로 똑같은 것을 의미하는 것이 어떻게 그러한지 설명하기 위해 경건이 본질이라고 전제한다는 흐릿한 참을 가장한다. 이것은, 만일 그것이 일상언어에 대한 주장을 의미하는 것으로 생각된다면, 각 단어에 대해 하나의 유일하게 정확한 의미가 있다고, 그리고 모호함은 불가능하다는 것으로 생각된다면, umun nomen unum nominatum을 수반할 수가 없다.

2) 그것은 로스 교수와 Mr 로빈슨이 추정하는 것처럼 절대의미와 모호의미 사이에 엄격한 괴리를 시사하지도 않는다. 우리는 의미에서의 같음과 의미에서의 다름이 배타적으로 그리고 철저하게 양자택일적이라 생각하기 쉽다. 만일 의미가 지시라면, 그 결론은 보편개념들에 대한 날카로운 이론이다. 그 이론에서 각각의 중요한 단어는정확하게 같은 방식으로 동일한 보편개념을 도입하거나, 서로 다른 보편개념을 도입한다. 그러나 이것은 의미들이 다를지도 모르지만 그럼에도 불구하고 공통된 중심 주위로 우위성과 후위성의 질서에서 조직화된다는 사실을 무시한다. 그래서, 예를 들어 사람들이 올바르고 행위들이 올바르다. 그러나 소크라테스는 『정체』(Ⅳ, 443e-444a)에서 올바른 행위들은 사람들 안에 영혼의 조건으로서 올바름을 산출하도록 지키거나 돕는 그러한 것들이라고 주장한다; 행위에 적용되는 것으로서 '올바름'이란 단어의 의미는 반드시 사람에게 적용되는 의미로 정의되어야만 한다는 것이 뒤따른다. 그래서 그 형용사는 우선 사람에 대한 의미로 사용되고, 파생적 의미에서 행위에 대해 사용되며, 이것은 절대의미도 모호의미도 아니며, 어원유래적이다. 그러나 그것은 동일한 꼴, 기본 사용과 파생 사용 양자에서 도입되는 동일한 꼴이다; 그것은 단순히 다른 방식으로 도입될 따름이다. 이것은 플라톤이 초기 대화편들에서 어원유래에 대한 사실을 인지하였다고 주장하는 것이 아니지만, 그럼에도 불구하고, 사실은, 이것이 그렇다는 강력한 암시가 있다. 그렇지만 그것은 꼴들에 대한 이론과 용어들의 어원유래적 사용 사이에 어떠한 양립불가능성도 없다고 시사하는 것이다. 그리고, 우리가 보게 될 것처럼, 중기 대화편들에서 꼴들 그 자체는 특정 종류의 어원들에 속하는 우선 지시되는 것들이 된다.

3) 만일 umun nomen unum nominatum이라는 꼬리표가 좌우간 초기 대화편들에 적용되는 것이라면, 그것은 일상 언어에 대한 주장으로 이해되어서는 안 되고, 어원을 제외하는 것으로 이해되어서도 안 된다. 그것은 언어의 개선을 위한 이상, 어휘의 조화가 세계의 질서에 부합한다는 요구로서 가장 잘 받아들여진다. 『크라튈로스』에서 하나의 이름은 베틀의 북과 유사한 도구인 것으로 이야기된다. 날실과 씨실을 가르면서 북으로 베를 짠다; 바로 그렇게, 사물들을 그것들이 그것들인 것에 따라 나누면서 이름을 가지고 가르친다(388b-c). 그리고 보통의 아무개에 의해서가 아니라, 만들어지거나 부숴질 수 있는 어떤 모형이 아니라 북 그 자체의 꼴을 보는 목수에 의해서 북이 만들어지는 것처럼, 그와 마찬가지로 이름들은 아무개가 아니라 작명가에 의해, '모든 공예가들 중 가장 드문' 그 기술자에 의해 주어져야 한다(389a); 작명가는 각각의 이름을 그것의 적절한 사용에 맞추면서, 주어진 이름들 속에서 이름 그 자체의 꼴을 볼 것이다(389d). 그리고 목수의 작품의 가치를 직조공이 판단할 것과 같이, 그렇게 변증가는 이름들의 부여자에 의한 작품의 가치를 판단할 것이다(390c). 플라톤주의는, 일상언어에 대한 그 태도에 있어서, 내재적으로 수정주의적이다.

4) 이름들을 부여하는 이러한 사업은 umun nomen unum nominatum에 관련되는데, 그렇지만, 이제껏 제시된 어떤 것과도 다른 의미에서 그러하다. 『메논』(71e 이후로)에서, 메논이 탁월함을 정의하도록 요구받을 때, 그는 그 각각이 고유한 정의를 지닌 탁월함들의 목록으로 응답한다: 사내의 탁월함은 폴리스를 돌보는 것이고, 친구들에게 유익하게 하는 것이며, 적들에게 해를 주는 것이다; 여성의 탁월함은 그녀의 남편에게 명령하고 또 그 남편에게 따르는 것이다; 그리고 아이들, 노예들, 노인들에 대한 다른 탁월함들도 있다. 소크라테스는 이러한 답변을 거부한다: 메논은 그에게 한 무리의 탁월함들을 주었고, 반면에 소크라테스는 그 모든 것들에 있어서 동일한 하나의 것, 하나의 탁월함에 대한 설명을 요구했다. 이것은 물론 단 하나의 것이 있다는 것을, 더욱이 'x의 탁월함'과 'y의 탁월함' 등에 대해 다양한 '~의 탁월함들'에 공통된 '탁월함'이 있다는 것을 전제한다. 다른 식으로 보자면, '탁월함'은 총괄적 단어도 아니고, 불완전한 표현도 아니다: 그것은 본질의 이름이다. 이것은 우리가 사내들의 탁월함과 여성들의 탁월함에 대해 말할 때, 어째서 우리가 어원들도 단순히 얼버무리는 것도 사용하지 않고 있는지에 대한 설명으로 취해질 수 있다; 그러나 『메논』과 『테아이테투스』 모두에서 그것은 오히려 보편성의 더 높고 더욱 흥미로운 단계를 지시하는 것으로 취해진다. 자신의 일을 알고 있는 작명가는 짐작컨데 그것을 마음 속에 품을 것이다.


본질인과성

1) 꼴들은, 로크의 말을 빌리자면, 실재적 본질이며 명사적 본질이 아니다. 꼴들은 실재적인데 왜냐하면 그것들이 명사적으로 이루어질 수 없기 때문이다: 명칭들이나 단어들의 일상적 의미들에 대한 탐구는 그 탐구가 꼴들에 대한 탐구인 경우를 제외하고는 꼴들에 대한 탐구가 아니다. 그러나 그것들은 또한 그것들이 실제 사물들인 바의 그러한 것이기도 하기 때문이다. 로크는 다음과 같이 본다:

본질은 그것이 바로 그것인 바의 것인 어떠한 것이든 간에 그것의 존재에 대해 취해질 것이다. 그리고 그래서 실재는 내재적이며, 그러나 본질적으로는 일반적으로 알려지지 않고, 사물들의 발견 가능한 특질들이 의존하는 사물들의 구조는 그 사물들의 본질로 불릴 것이다. 이것은, 그것의 형성으로부터 명백하듯이, 그 단어의 적절한 의미이다; 그 기본 표기법에 있어서 적합하게 존재함을 의미하는 esentia.

로크의 설명에 의해, 본질은 '어떤 것의, 그로써 어떤 것이 그것인 바의 것인 존재'이다. 아리스토텔레스는 『형이상학』에서, 개별자의 실체적 꼴을 그 개별자의 실체와 동일시하면서, 본질을 본질을 가진 것의 존재와 동일시하면서 이러한 주장을 내놓았다. 그래서 그는 로크가 물려받았던 그 전통을 기초하였다.

2) 그러나 플라톤의 초기 대화편들에 있어서 꼴들은 그 있음에 대해 그 꼴들이 꼴들인 것의 있음이 아니다. 하나의 있음인 보편개념은 다수성의 있음이 될 수 없다 - 정확하게 아리스토텔레스가 왜 보편개념들로부터 실체적 꼴을 구별하는 데에로 이끌렸는지의 이유이다. 『에우튀프론』은 경건이 어떤 주어진 경건한 사물이나 경건해 보이는 행위의 있음이라고 시사하지 않는다; 그것은 오로지 경건이 그에 의해 경건한 것들이 경건한 것들인 그러한 것임을 시사할 따름이다. 그것은 또 다른 아리스토텔레스적 어휘를 조금 빌리자면, 경건이 원인이라는 것을 시사한다.

3) 『대히피아스』에는 이 전제를 명백하게 만드는 논증이 있다. 소크라테스는 히피아스가 올바름이 어떤 것이다(ἔστι τι τοῦτο)라는 것, 그리고 이것이 지혜에 있어서도 마찬가지로 참이라는 것에 동의하도록 이끄는데, 왜냐하면, '올바른 것과 지혜로운 것 등등의 그러한 것들은 만일 그것들이 어떠한 것이 아니라면 그것들에 의한 그러한 것이 아닐 것이기 때문이다'. 아름다운 것들은 아름다움에 의해 아름답기 때문에, 히피아스는 아름다움도 어떠한 것이며, 그것들이 다른 말들이나 생각들 또는 개념들에 의해 아름다워지지는 않는다는 것에 동의할 수밖에 없다.

4) 이 논증은 분명히 『대히피아스』에만 놓이지만, 그것은 정의를 목표로 하는 모든 각각의 초기 대화편들에 의해 가정되는데, 왜냐하면 꼴들이 본질들이라는 그 전제는 변증에 본질적이기 때문이다: 꼴들은 정확히 보편개념들이고 표준들인데 왜냐하면 그것들이 그에 의해 사물들이 그러한 바의 것들인 것이기 때문이다.

5) 플라톤이 매우 자주 꼴들과 그 꼴들을 지닌 사물들 사이의 관계를 특징화하기 위해 인과적 언어를 사용하는 이유는 꼴들이 본질들이기 때문이다. 『뤼시스』(221c)에서 소크라테스는 '원인이 파괴될 때, 그 원인이 그에 대한 존재를 지속시키는 원인인 그러한 것은 물론 있을 수가 없다'고 지적한다. 경건은 이러한 의미에서 원인이다: 경건의 현존은 경건한 것들이 경건한 것들인 한에서 그것들의 현존에 필수적이다.

6) 초기 대화편들은 꼴들과 그 꼴들의 예시들 사이의 관계를 묘사하기 위한 다양한 은유들을 사용한다. 꼴들은 그 예시들 '안에' 있어야 할 것으로, 또는 그 예시들'에 현전하는' 것으로, 혹은 그것들에 '덧붙여져'야 할 것으로, 아니면 그것들'에' 있어야 할 것으로 이야기된다. 그 예시들은 다른 한편으로 꼴들을 '가지거나' '허용하거나' '취하거나' '나누어 가진다'. 이러한 은유들 전부 일상언어에 기초된다. 그것들은 정말로 평범하게 그리스어에서 추상적 명사들과 함께 사용될 것이다. 영어에서 한 행위에 용기가 있다(there is courage in an action)고, 또는 올바름의 현전이 상황을 개선시킬 것이라고, 어떤 사람이 다른 사람보다 더 훌륭한 인내의 도량을 지닌다고 말하는 것과 똑같이 말이다.

7) 은유들로서, 그리고 친숙한 그리스어로서, 이러한 용어들은 소피스트적 논쟁들의 재료를 제공했다. 예를 들어 『에우튀데모스』에서는 디오니소도루스가 아름다운 것이 아름다움과 다른 것인지 어떤지를 묻는다. 소크라테스가 잠시 망설인 후 아름다운 것들은 아름다움 자체(αὐτὸ τὸ καλόν)와 다른 것들이고, 그러나 그 아름다운 것들 각각은 그 안에 현전하는 어떤 아름다움(κάλλος τι)을 지닌다고 말할 때, 디오니소도루스는 재빨리 주도권을 잡아챈다. 그는 다음과 같이 말한다. '그러면 소 한 마리가 당신 곁에 있다면, 당신은 소로군요. 그리고 내가 당신 곁에 있으니, 당신은 디오니소도루스고요'(301a). 소크라테스는 '맙소사'라고 말한다. 당연하다. 그러나 그 물음은 하찮은 것이 아니다: '현전'은 은유 - 혹은 최소한 많은 용례들을 지닌 단어 - 이고 잘 다룰 방법은 그것의 의미에 달려 있다.

8) 문제가 되고 있는 '현전'은 특수한 종류의 것이다. 『뤼시스』에서 소크라테스는 어떻게 나쁨이 좋지도 나쁘지도 않은 어떤 것에 현전할 것인지 설명하는 일을 떠맡고, 그가 의미하는 것을 명확하게 하려는 그의 논의를 갑작스럽게 중단한다. '일부 사물들은 그 사물들에 현전하는 그러한 것 그 자체일 그런 것들이다; 다른 것들은 그렇지 않다.' 만일, 예를 들어 금발이 하얗게 칠해진다면, 흼은 그 머리카락에 현전할 것이지만, 그 머리카락은 그 자체로 흰 것은 아닐 터이다. 그러나 나이가 들어 머리카락이 흴 적에, 흼은, 그 머리카락이 그래서 하얗게 될 것이기 때문에, 다른 의미에서 머리칼에 현전할 것이다. 앞서의 경우, 흼은 머리카락에 현전한다. 뒤의 경우, 머리카락은 '흼의 현전에 의해 하얗다'.

9) 어떤 것이 흴 때 그 의미에서 흼이 그것에 현전하는 그러한 특별한 의미를 설명하기 위해, 소크라테스는 도구적 여격에 의존한다. 이러한 장치는 초기 대화편들에서 빈번하게 나타난다. 경건한 것들은 경건함에 의해 경건하고, 사려있는 자들은 사려에 의해 사려있으며, 아름다운 것들은 아름다움에 의해 아름답다. 아름다움은, 『대히피아스』(300a)에 따르자면 아름다움 것들을 아름답도록 만든다(ποιεῖν). 이러한 표현들은 일상어법과 결부된다. 이온은 그가 호메로스를 기릴 수 있음이 앎에 의한(도구적 여격) 것이라고 생각한다(『이온』541e). 소크라테스는 우리가 건강에 의해(ὑπό와 동작주의 속격) 유익해진다고 생각한다(『크리톤』280a). 사려는 만일 그 사려가 그들에게 현전하는 그러한 자들에게 좋다면 좋은 것이고, 아니라면 나쁜 것이다(『카르미데스』161a).

10) 그러나 비록 이러한 표현들이 일상언어에 결부된다 할지라도, 그것들은 종종 소크라테스에 의해 비일상적인 방식으로 사용된다. Ποιεῖν은 πάσχειν에 속하는 규칙동사이고, πάσχειν은 규칙적으로 '자격지어짐'을 의미할 것이다; 그러나 ποιεῖν은 '자격을 둠'의 의미로 규칙적으로 사용되지는 않는다. 더욱이, 지혜가 사람들을 복되게 만든다고 말하는 것과 지혜가 사람들을 지혜롭게 만든다고, 또는 지혜가 그에 의해 사람들이 지혜로워지는 것이라고 말하는 것 사이에는 주목할만한 논리적 차이가 있다.

11) 이러한 표현들은 인과적이다 - 흔히 당구공들의 부딪침과 연관되는 의미에서가 아니라 아리스토텔레스가 형상적 원인에 대해 말하는 의미와 유사한 의미에서 -. 아름다움은 정확히 이런 의미에서 아름다운 것들의 원인이다: 아름다움의 현존은 다른 것들의, 그것들이 아름다운 것들인 한에서 현존의 필연적 조건이다. 『대히피아스』에서 소크라테스가 사람들이 아름답다고 말하는 사물들은 오로지 아름다움 자체가 현존해야만 아름다우며, 히피아스가 아름다움을 아름다운 처녀로 정의했을 때, 그 이유로 아름다운 것들이 아름다운 그러한 어떤 것(δι᾿ ὅ)이 현존한다고 주장한 것은 그런 이유에서이다.

12) 그것들이 그에 의해 어떤 것들이 바로 그러한 것들인 바의 그러한 본질들이라는 의미에서, 그래서, 꼴들은 원인들이다. 그러하기 때문에, 『파이돈』(100c)에서 소크라테스는 아름다운 것들이 '다른 어떤 이유도 아니라 바로' 그것들이 아름다움에 참여하기 때문에 아름답다고 주장하며, 케베스에게 그가 이러한 종류의 원인(αἰτία)을 허용하는지 묻는다. F. M. 콘포드는 이 구절을 다음과 같이 설명한다:

'다른 어떤 이유도 아니라 바로 그 이유'라는 구절은 … 모호하다. '이유'는 '설명'(αἰτία의 관습적 용법)을 의미할 것이다. 그 전제는 그래서 '이 장미는 아름답다'라는 명제가 '이 장미는 아름다움에 참여한다'는 진술과 동등하다고 단언할 것이다: 나는 말들의 그 형식을 바꾸어 말함으로써 대체하고 그렇게 설명할 수 있다. 그러나 플라톤은 명제의 분석이 아니라 일치하는 사실에 대한 분석을 말하고 있는 것처럼 보인다. 그 이론은 그래서 이 사실이 (1)특정한 가시적인 것, 이 장미, (2)아름다움(Beautiful or Beauty), 그리고 (3)우리가 '~에 참여한다'를 대체할 수 있는, '~는 ~이다'로 표현되는 그 둘 사이의 관계라고 불러야 할 어떤 것으로 구성된다고 단언할 것이다. 그러나 다시 한 번 우리는, 지금까지, 오로지 하나의 설명만을 가진다: 이 장미가 아름답다는 사실은 이 장미가 아름다움에 참여한다는 사실과 똑같은 것이다. 우리는 그 사실을 존립하게 할 어떤 원인에 대해서도 아무것도 배우지 않는다. 어느 관점에서든 우리는 한 명제에 대한 분석이나 하나의 사실에 대한 분석만을 가질 따름이고, 참인 진술의 이유나 그 사실의 현존에 대한 원인을 가지지는 않는다.

첫번째 주장은 제외될 것이다. 당신은 말을 바꿈으로써 오로지 본래의 단어들이 의미하는 것에 대해 무지한 누군가에게 하나의 명제를 설명할 수 있을 뿐이고, '이 장미는 아름답다'라는 것을 이해하는 데에 실패한 그 누구도 그 표현이 '이 장미는 아름다움에 참여한다'를 의미한다고 이야기되어야 한다는 데에 대해서 분명하게 해줄 것을 찾지는 못할 것이다. αἰτία가 말 바꿈에 대해 사용될 수 있었다는 것을 보여주는 어떤 고대의 증거가 있는 것도 아니다. '다른 어떤 이유도 아니라 바로 그 이유'가 말 바꾸기를 시사하리라는 주장은 그래서 불충분하다.

13) 콘포드가 '일치하는 사실에 대한 분석'이라고 부른 것은 여전히 남아 있다. 그러나 만일 이 장미가 아름답다는 사실이 이 장미는 아름다움에 참여한다는 사실과 똑같은 사실이라면, 그가 인정하듯, 어떤 식으로 분석이 말 바꾸기와 다른지는 불분명하다. 어떤 경우든, 이것은 플라톤의 핵심을 놓친다. '이 장미는 아름답다'와 '이 장미는 아름다움의 꼴에 참여한다'는 실로 등가의 진술들이다 - 만일 꼴들에 대한 이론이 참이라면 말이다. 그러나 그 이론은 정확히 사실들의 구조에 있어서 존재론적 우선성에 대한 이론이지, 명제 등가들에 대한 이론이 아니다. 아름다움의 꼴의 현존은 이 장미를 아름답다고 부르기 위한 필요충분조건이다: 그 사물에 세례를 내리는데 그것에 있어서 그 특징 때문이고,(그 특징 자체와 구별되는 것으로서) 그 세례의 정당함을 보장하는 그 특징의 현존 때문이다. 꼴 자체는 그렇지만 오로지 장미의 아름다움에 대한 필요조건일 따름이다: 그것은 아름다움이 만일 그것이 현존하지 않는다면 장미가 아름다운 것으로서 현존하지 않았을 것이라는 점에서 그 아름다움의 원인이다. 그 꼴은, 꼴의 현존과 구별되는 것으로서 그 장미의 아름다움에 대한 충분조건이 아닌데, 왜냐하면 그 꼴은그 장미에 있어서 그 장미의 아름다움의 고유한 현존에 대한 충분조건은 아니기 때문이다: 그것은 아름다움의 현존뿐만 아니라 토양의 조건, 그리고 기후, 정원사의 기술에도 의존하기 때문이다. 여기에서 충분조건은 본질인과만이 아니라 작용인 또한 필요로 한다.

14) 그래서 꼴들은 그것들이 그것들에 의해 사물들이 그러한 것들인 바의 것들이라는 의미에서 원인들이다. 그것들은 그러므로 세계의 운행에 영향을 미친다. 만일 그것들이 존재하지 않았더라면, 세계는 세계가 그 세계인 어떤 것이 아니었으리란 의미에서 말이다.


지시의 재고

1) 초기 대화편들은 의미에 대한 지시적 이론을 전제하는데, 추상적 명사들 또는 그것들 중 일부가 대상들을 명명한다는 의미에서 그러하다. 이로부터 초기 대화편들이 이제는 이전보다 덜 널리 주장되는 그 명제, 어떤 것을 기술한다는 것은 서술을 명명하는 것을 필요로 한다는 명제를 전제한다는 것이 뒤따르진 않는다. 이 마지막 명제는 일반적으로 추가적인 명제와 관련되어 왔다: 잘 구성된(혹은 잘 분석된) 문장들의 구조는 반드시 문장이 표현하는 그러한 사실들에 대해 일대일 혹은 2방향 유일 상응에 존립해야만 한다는 명제이다. 잘 구성된 문장들은 술어들을 포함하기 때문에, 그 문장들이 표현하는 사실들은 반드시 유사-술어들을 포함해야만 한다; 그러나 그러한 유사 술어들이 보편개념들이나 본질들이라고 추정하는 것은 더 나아간 단계이고, 그러한 술어들은 그것들을 명명한다.

2) 이것은 플라톤이 주장하지 않은 명제이다. 초기 대화편들은 더욱이 그것과 관련된다고 거의 이야기될 수가 없다; 그 대화편들은 특정한 어떤 것들이 무엇인지에 대한 물음과 관련되지, 어떻게 그리고 어떤 방식으로 사물들이 사물들에 대해 이야기될 것인지에 대한 물음과는 관련이 없다. 그러나 만일 우리가 『파이돈』을 본다면, 우리는 어떤 것을 기술하는 일이 하나의 기술을 이름하는 것이 아니라 서술된 그것을 명명하는 것을 필요로 한다는 것을 알아차릴 것이다. '꼴들의 각각은 현존하고, 다른 것들은, 그것들이 그 꼴들의 제 몫을 가지게 되기 때문에, 그것들 이후에 이름지어진다'라는 것 때문이다. 이것은 소크라테스를 올바르다고 기술하는 것은 그에게 올바르다고 명명하는 것이고, 좌우간 올바름을 명명하는 것은 아니라고 강력하게 주장한다; 그 이름이 비록, 올바르게 적용된다면, 소크라테스가 올바름에 참여하기 때문에 적용한다 할지라도 말이다.

3) '올바른'과 '올바름' 사이의 관계는 아리스토텔레스가 아카데메이아적 구별이었을 것을 사용하면서, 동근어라고 불렀던 것의 한 예시이다: '사물들이 상이한 어미들로 어떤 것으로부터 그것들의 이름을 획득할 때, 그것들은 동근어들이라 불린다. 그래서, 예를 들어 문법학자(grammarian)는 그의 이름을 문법(grammar)로부터 획득하고, 용감한 자들은 그들의 것을 용감으로부터 취한다.' 그래서 소크라테스를 올바르다고 명명함에 있어서 우리는 그를 동원어적으로 명명한다: 그는 그의 이름을 올바름으로부터 얻는데, 어미상의 차이를 가지고 그렇게 한다. 그러나 소크라테스가 참여하고 있는 꼴의 이름은 소크라테스가 그 꼴로부터 얻은 이름과 문법적 어미에 있어서 다를 필요가 없고, 플라톤의 고유한 사용에 따르고 소크라테스가 어원적으로 명명된다고 말하는 것이 더 나을 것이다. 만일 꼴들에 대한 이론이 참이라면, 그래서 '소크라테스는 올바르다'와 '소크라테스는 올바름에 참여한다'는 등가 명제들이다; 서로가 서로를 수반한다. 그러나 첫번째 것은 동일한 것에 대한 두 가지 이름들을 포함하는데, 하나는 어원이고, 반면에 다른 하나는 두 가지 것들에 대해 두 가지 이름들을 포함하고, 그 중 어느 것도 어원이 아니다. 두 경우 모두에서, 의미는 지시를 필요로 한다; 지시 대상들은 그렇지만 어원이 그것들의 관련에 근거한다 하더라도 구별된다.

4) 종과 유들은 근원적인 반면에 어원적으로 그것들을 도입하는 서술들은 파생적이라는 것이 이 관점의 결론이다: 용감한 자들은 용감으로부터 이름지어지지만, 그 역도 성립한다. 그러나 무엇보다도 왜 이것이 그러해야 하는가? 왜 우리는 종과 유들을 논리적(혹은 어쩌면 비논리적) 구조로, 기술들 그 자체인 의미의 근원적 매개물로 간주하지 말아야 하는가? 중기 대화편들에서 이에 대한 플라톤의 주된 답변은 일대다(一對多) 논증이어 왔다. 그 논증에 대한 전제들은 현대적인 차림으로 존 오스틴에 의해 경탄스럽게 진술되어왔다 - 그는 물론 그 논증의 결론에 동의하진 않았다:

만일 우리가 좌우간 언어를 통해 성취하는 그런 종류의 소통일 것이 있다면, 의사전달자('화자')가 '마음대로' 만들어낼 수 있고 또 피(彼)의사전달자('청자')가 관찰할 수 있는 어떤 종류의 상징들의 축적이 있어야만 한다: 이러한 것들이 '말'이라 불릴 것인데, 물론 비록 그것들이 우리가 보통 말이라 불러야 하는 것과 같은 어떤 것일 필요는 없다 할지라도 그렇다 - 그것들은 신호기(信號旗) 따위의 것일 터이다. 또한 말과는 다른 어떤 것, 말이 그에 대한 소통을 위해 사용될 것인 어떤 것이 있어야만 한다: 이것은 '세계'라 불릴 것이다. 어째서 세계가 말을 포함하지 않아야 하는지에 대해서는, 실제 진술 그 자체, 그에 대해 어떤 특수한 경우이든 세계에 대하여 이루어지는 것인 진술 자체의 면을 제외한 모든 면에서 아무런 이유도 없다. 더욱이, 세계는 유사성들과 부동성들(하나 없이는 다른 하나도 있을 수 없는)을 내보여야만 한다(우리는 관찰해야만 하고): 만일 모든 것들이 절대적으로 다른 어떤 것과도 구별될 수 없거나 완전히 다른 어떤 것과도 다르다면, 말할 아무것도 없을 것이다. 그리고 궁극적으로 (작금의 목적들을 위해서 - 물론, 마찬가지로 만족될 다른 조건들도 있다) 두 가지 관습적 방식들이 있어야만 한다: 세계에서 발견되는 상황, 사물, 사건 등의 유형들을 가지고 말과 관계하는 기술적인 관습적 방식들(= 문장들). 세계에서 발견되는 역사적 상황들 따위를 가지고 말과 관련하는 지시적인 관습적 방식들(= 명제들).

오스틴은 보통 일상언어철학자라고 불린다; 그러나 그는 어떤 표어들 아래에서도 행진하지 않았고 어떤 장단에도 맞추지 않았으며, 만일 언어가 그 세계에 대한 것일 터라면 세계가 어떠해야만 하는지 기술하는 위의 문단은, 만일 내가 실수하는 게 아니라면, 가장 투명한 형이상학의 단편이다. 오스틴은 주석에서 계속해서 첨언한다:

'~를 가진 일종의 ~이다'라는 것은 '~을 가진 그런 표준적 사태들과 충분히 유사한 ~이다'라는 것을 의미한다. 그래서, 참된 하나의 사태일 하나의 진술은 자연적 관계인 특정한 다른 것들과 유사해야만 하고, 그러나 또한, 더 이상 순수하게 자연적 관계는 아닌 동일한 '서술' 가치에 대해 충분히 유사해야만 한다. '이것은 붉다'라고 말하는 것은 '이것은 그런 것들과 닮았다'고 말하는 것과 같지 않고, '이것은 붉다고 불리는 그런 것들과 닮았다'라고 말하는 것조차 똑같은 것이 아니다. 사물들이 유사하다거나 또는 심지어 '정확하게' 유사하다는 것조차 나는 문자 그대로 이해할 터이지만, 그것들이 같다는 것을 나는 문자 그대로 알 수 없다 - 그것들을 똑같은 색이라 부름에 있어서 관습은 그것들이 그렇다고 이야기되는 색에 주어질 이름에 대한 관습적 선택에 부가적으로 필요한 것이다.

아마도 그럴 것이다. 그러나 그래서 유사성과 부동성은 상이한 측면들에서 획득하는 관계들이다. 크리켓 공과 야구공은 형태에 있어서 유사하고 색에 있어서 부동하다; 우리는 단순히 그것들이 이것이 그러하다는 측면들을 구별함이 없이 유사하고도 부동하다고 진술할 수는 없다. 만일 그래서 우리가, 세계 속의 사물들 사이에서 우리가 이야기하는 방식과 우리가 좌우간 이야기한다는 그 사실에 대해 독립적으로 획득하는 유사성과 부동성이 자연적 관계들이라고 받아들인다면, 관계들이 그 덕분에 획득하는 그러한 특징들 - 형태적 특징들, 색채적 특징들 등 - 이 언어적 관습의 문제들이라고 추정하는 것은 거의 이치에 맞기 어려워 보인다. 만일 그 관계들이 '자연적'이라면, 그것은 그 덕분에 그것들이 유지하는 그러한 특징들도 동일하게 그렇다는 것으로 드러날 것이다. 주의해야 할 그러한 특징들은 기술들이 아니다: 우리는 크리켓 공과 야구공이 모두 구형이라고 말하지만, 그것들이 서로 구체의 측면에서 유사하다고 말하지 않고, 구체 혹은 구형일 것임의 측면에서 유사하다고 말한다. 우리는 그 사물을 기술한다; 그러나 그 기술은 기술되는 그 사물의 특징 덕분에 유지된다. 이것은, 내가 보기에, 용감한 자들이 용기로부터 이름지어지고 그 역은 아니라고 말하는 이유이다.

5) 이러한 언급들은 물론 유사성들과 부동성들의 세계를 발견하고 특징들의 현존을 추론해내는 사람들과 동일한 세계를 발견하고 추론을 거부하는 사람들 사이의 그 문제를 조정하지는 않는다. 그러나 그 언급들은 아마도 최소한 플라톤의 경우 그 추론이 의지하는 그러한 근거들을 보여주는 어떤 것을 한다. 그렇지만 초기든 중기든 대화편들에서 유사성들에 대한 인식으로부터 도출된 꼴들에 대한 우리의 앎은 없다는 것을 주의해야 한다; 그것은 오로지 우리가 사실상 유사성이 획득한다고 확신할 표준으로서의 꼴의 사용을 통해서만 있기 때문이다.

-蟲-
꼴들의 영역의 구조

1) 만일 종의 유에 대한 관계가 꼴들의 구조에 기초된다면, 그 구조는 반드시 계층적이어야만 한다.

2) 이것은 널리 받아들여지는 관점이다; 그러나 그 관점은 반대에 부딪쳤다. 예를 들어 체르니스 교수는 그것이 틀렸다고 주장한다:

다른 어떤 것에 대해서도 전체에 대한 구성부분으로서 또는 종에 대한 유로서 어떠한 이데아도 없다. …플라톤은 어디에서도 이데아들 사이에 유와 종의 구별을 만들지 않는다; 그러나 아리스토텔레스가 유라고 부른 것, 종차라고 부른 것, 그리고 종이라고 부른 것은 그에게 모든 별개의 관념적 단일체들이다, 각기 다른 것들의 현존을 수반하거나 다른 것들과 양립 가능한 측면들을 지닌 다른 것들 보다는 가기 다른 것들, 그러나 남김없이 다른 것들로 분석될 수는 없는 독립적 본성인. 플라톤은 그래서 분절을 사용하여 이데아들의 세계에서의 존재론적 계층을 산출할 생각을 할 수는 없었다. 『소피스트』와 『정치가』, 분절에 대한 안내서로 여겨지게 된 그 두 저술들은 그가 그 분절을 가지고 오히려 발견적 학습법을 의미했고, 한정된 이데아를 탐색함이 용이하도록 하는 수단을, 다른 이데아들로부터 그 이데아를 구별하는 것을, 그리고 그것의 함축들과 동일성을 의미했다는 것과 그가 그 분절을 가지고 이데아, 이데아의 기원, 혹은 그것의 구성 요소들의 '구조'를 묘사하는 것으로 상상하지 않았다는 것을 보여준다.

체르니스 교수는 일부 꼴들이 어떠한 부분들도 가지지 않는 것으로서 단순하다는 것뿐만 아니라, 모든 꼴들이 이러한 방식으로 단순하다는 것을, 그리고 꼴들의 영역이 균일하다는 것을 또한 주장한다. 그의 의견들은 후기 대화편들을 겨냥하고 있고, 명확하게 『소피스트』에서와 같은 종류의 집단에 속하는 원칙을 겨냥한다; 그러나 그것들은 초기 대화편들의 해석으로서도 잘 맞는다.

2) 이러한 관점에 반대되는 그 저술들은 '그것은 무엇인가?'라는 물음에 대한 답이 유나 혹은 그에 대해 꼴이 검토 아래에서 일부인 그러한 전체를 명시할 것임을, 또는 그 꼴이 부분들을 포함하는 전체임을 진술하거나 시사하는 모든 저술들이다. 그러한 저술들은 : 『에우튀프론』 11e-12a, 『프로타고라스』 329c와 그 이후, 『메논』 74-6, 『라케스』 190c-d, 199e이다. 이러한 구절들 모두 유 아래에 종을 포함하는 가능성을 암시하고, 그래서 꼴들의 계층적 배열을 암시한다. 그것들은 꼴들의 영역이 균일하지 않고 단층화되어 있음을 시사한다.

3) 체르니스 교수는 이러한 저술들, 그리고 후기 대화편들의 다른 저술들이 그 저술들의 의도가 발견적이라는 점을 제쳐두고 세워진다고 제안한다: 그 저술들은 형이상학적이라기보다는 방법론적으로 전제를 반성한다. 그러나 탐구의 방법은 그 탐구의 핵심-문제의 본성에 적합한 것이거나 적합해야 하는 것이다. 그리고 만일 꼴들이 상호에 대해 종과 유처럼 자리한다고 가정하는 것이 발견의 의도에 유용하다면, 어째서 이것이 그러한지에 대한 설명의 가능성에 대해 책임이 있다. 확실한 대답은 그 전제가 꼴들의 실재적 구조에 답한다는 것이다. 체르니스 교수는, 이러한 대답에 반대하면서, 그 대신에 아무것도 내놓지 않는다. 하지만 그 거부는 방법론과 설명을 요하는 반대 사이의 동일척도불가능성을 시사한다: 사실상 아무런 유들도 그리고 어떠한 계층도 없을 때, 어떻게 꼴들이 유들 아래에서 포함가능한 것들이고 발견적 기능을 제공하도록 계층적으로 배열된다는 전제가 가능한가?

4) 더 큰 어려움도 있다. 초기 대화편들에서, 플라톤은 ποῖόν τι의 물음보다 τί의 물음이 우선한다거나 οὐσία와 πάθος 사이에 구별이 있다고 주장한다. 그러나 만일 체르니스 교수가 제안하듯 오로지 꼴들 사이에서의 관계들만이 차이, 포함, 양립의 관계들이라면, 이러한 구별은 도출될 수 없다. 왜냐하면 그래서 만일 꼴 F가 그것의 함축들과 양립들에 대해 동일하지 않다면, 'F는 무엇인가?'라는 물음은 답해질 수 없기 때문이다: F는 오로지 함축들과 양립들의 관계들에 의해서만 특징지어질 수 있고, 그러한 관계들 중 특정한 것들이 다른 것들보다 더 본질적인 것임을 뒷받침할 어떠한 근거도 없다. 그러므로 οὐσία를 πάθος로부터 구별하기 위한 어떠한 근거도 없다. 다른 한편으로, 만일 F가 그것의 포함들과 양립들에 대해 동일시된다면, 후자는 그것이 F일 그러한 것이 될 것이다; 그리고 만일 그것들 중의 어떠한 것들이든 그것들이 그것들인 것보다 결핍하고 있거나 다른 것이라면, F는 그것이 그러한 것인 바의 것이 아닐 것이고, 그러나 그것이 그러한 것인 바의 것과는 다른 것일 것이다. 즉, 동일성은 '내적 관계들'로 이끈다. 그리고 양립가능성과 함축이 모든 각각의 꼴을 다른 모든 각각의 꼴들에 관련시키기 때문에, F는 본질적으로 다른 모든 각각의 꼴과 관련될 것이다: 진리는 전체일 것이다. 스페우시푸스는 나중에 이러한 단계에 매우 근접하게 나아갔던 것으로 드러난다; 그러나 플라톤은, 최소한 초기 대화편들에서는, 그렇지지 않은데, 왜냐하면 πάθος와 οὐσία 사이에 구별이 있다는 것을 그 단계는 부정하기 때문이다.

5) 만일 '그것은 무엇인가?'라는 물음이, 소크라테스가 물었고 또 그것이 대답되는 것을 보려고 하였던 것처럼 물어지고 또 대답될 것이라면, 그리고 만일 ποῖόν τι의 물음들로부터 구별이 유지될 것이라면, 꼴들 사이에서 포함들과 양립가능성 그리고 그것들의 부정들의 관계들 보다 더 나아간 어떤 관련이 요구될 것이다. 그러한 관련의 본성은 『메논』에서 올바름은 탁월함 자체가 아니지만 하나의 탁월함이라는 주장에 의해, 『라케스』에서 용기는 전체로서의 탁월함에 속하는 한 부분이라는 주장에 의해, 『에우튀프론』에서 경건은 올바름의 한 부분이라는 주장에 의해 제시된다. 그 관련은 종과 유 사이의 관련이다: 한 부분의 본성을 이해하기 위해, 우리는 반드시 그 부분의 위치를 그것을 한 부분으로 하는 전체 속에서 정립해야만 한다.


탁월함의 단일성

1) 탁월함들이 유 속에 그것들을 정립시킴으로써 한정가능하다는 관점은 또 다른 초기 대화편 『프로타고라스』와 충돌하는 듯 보인다. 그곳에서 소크라테스는 탁월함이 하나라고 주장한다. 이 명제는 플라톤의 생애 마지막에 그의 생각을 차지한 것이었고, 그것은 종종 탁월함들이 동일하다는 의미로 받아들여졌다. 특히, 『프로타고라스』의 교훈은 모든 탁월함들은 그것들이 앎이나 지혜로 환원될 수 있다는 점에서 동일하다는 것으로 생각되어 왔다. 그리고 탁월함이 앎이라는 주장은 탁월함에 대한 소크라테스적 정의로 고수되어 왔다. 솔직히, 그것은, 종과 유의 비동일성을 시사하는 최근류와 종차를 통한 정의가 아니다.

2) 『프로타고라스』에서 앎의 우선성은 단호하게 내세워진다(352b-c):

대부분의 사람들은 앎이 명령의 힘도 권위도 능력도 없다고 생각한다; 앎을 가질지는 모르지만 그 앎은 제압당할 수 있다고 생각한다 - 분노에 의해, 그리고 기쁨과 고통에 의해, 가끔은 사랑에 의해, 그리고 자주 두려움에 의해서 - 비록 앎이 어떤 가난한 노예에게 있다고 할지라도, 격정들에 의해 제멋대로 끌려다닐 것과 같이 말이다. 그것이 당신의 관점 아닌가, 프로타고라스? 아니면 당신은 오히려 앎이 아름다움과 능력, 그리고 그 무엇도 꺾을 수 없는 것에 속하는 어떤 것이라는 데에 동의할 텐가; 나쁨으로부터 좋음을 구별해 내기만 하면, 지상의 그 무엇도 그러한 앎을 가진 자를 그 앎 - 그의 조력자로 충분한 지혜 - 과 반대로 행동하도록 강제할 수 없다는 것에 동의하겠는가?

이 구절은 의심할 나위 없이 소크라테스 그 자신의 믿음을 대변한다: 탁월함은 앎이다. 그 누구도 제 뜻으로 또는 알면서도 악을 행하지는 않는다는 점에서 말이다.

3) 만일 앎이 탁월함을 수반한다면, 그리고 탁월함이 앎을 수반한다면, 그렇게 함으로써 탁월함과 앎이 하나이고 같은 것이라는 점이 따르지는 않는다. 동치는 동일성이 아니다. 그러나 이러한 동치는 더 넓은 논증의 부분으로 나타난다. 그 논증에서 소크라테스는 프로타고라스가 탁월함들이 동일한 것에 대한 모든 이름들이라는, '용기', '올바름', '사려', '경건', '지혜', '탁월함'은 동의어들이라는(349b-c) 점을 받아들이도록 이끌고자 시도한다. 이러한 관점에서, 『프로타고라스』가 탁월함들이 동일하다는 것, 그리고 앎으로 정의될 수 있다는 것을 의미한다고 해석되어야 한다는 것은 놀라운 일이 아니다.

4) 하지만 아직 이러한 방식에는 어려움들이 있다. 우선, 그 설명이 탁월함과 앎을 동일시한다 할지라도, 그것은 능력의 동일성을 부당하게 취하는 것이다. 만일 그 탁월함들의 명칭들이 동의어들이라면, 그 탁월함들이 지혜로 화원될 수 있다고 이야기될 수 있는지 어떤지, 지혜를 포함한 그 탁월함들이 경건이나 올바름이나 다른 어떤 것으로 환원될 수 있다고 이야기될 수 있는지 아닌지, 그것은 임의적이다. 물론, 탁월함은 앎이나 지혜이다; 그러나 그래서, 탁월함은 용기나 경건, 또는 당신이 원하는 무엇이든, 동의어에 대해 대칭적인 관계이다.

5) 그 동일화 방식에는 두번째 어려움도 있다. 소크라테스는 『프로타고라스』에서 탁월함들이 구별된다는 프로타고라스의 주장에 반대하여 네 가지 논증들을 내놓는다. 첫번째(330b-331b)는 올바름과 경건의 단일성을 주장한다; 두번째(332a-333b)는 사려와 지혜의 단일성을; 세번째(333d-334a)는 그 결론에 도달하지 않고 중단되는데, 어쨌든 올바름과 사려의 단일성을 주장할 것이었다. 거기에서 남은 대화편을 통해 중단 없이 추적되는 그 물음이 다시 말해진 다음(439b-d) 긴 중간극이 따른다. 마지막 논증에 있어서 소크라테스는 용기와 지혜의 단일성을 증명할 것을 자임한다(349d-360e). 만일 탁월함들의 단일성이 탁월함들의 동일성을 의미하는 것으로 취해진다면, 이러한 논증들 각각은 논리적 오류이고, 그것들 중 일부는 노골적으로 그리고 창피스럽도록 오류를 범한 것이다. 이것은 그 문답의 고비를 넘긴다. 프로타고라스는 지적인 사람으로서 도처에서 묘사되고, 말들을 다루는 방식들에 있어서 지혜롭지만, 소크라테스의 논증들을 다룰 줄 모른다. 그의 실패는 물론 말의 단련에 있어서 기술 부족 탓이 아니다; 그것은 소피스트에게서 기대하기 어려운 일이다. 그것은, 비록 그가 탁월함을 가르치리라 주장할지라도, 탁월함이 무엇인지 모른다는 그 사실에 기인한다. 소크라테스의 논증들은, 만일 그 논증들이 공허한 오류들에 불과하다면, 이러한 결론을 가리키는 쪽으로 그 논증들이 행한다고 의미되는 것으로서 이바지하기 어려울 것이다.

6) 다른 초기 대화편들에서와 마찬가지로, 『프로타고라스』도 실패로 끝나고, 그것은 익숙한 이유 때문이다: 소크라테스와 프로타고라스는 먼저 탁월함이 무엇인지 결정함이 없이는 탁월함이 가르쳐질 수 있는지에 대해 동의하지 않는다. 이 점은 추론할 것으로 남겨지지 않는다: 그것은 그 문답의 변증에 대한 명쾌한 요약정리를 되보여준다 (360e-361c):

나는 이렇게 말했다. 나는 다른 어떤 이유도 아니라 그것이 어떻게 탁월함에 관련한 것들을 가지고 바로 그것인지, 탁월함이 그 자체로 무엇인지 밝혀내려는 바람 때문에 이러한 것들을 묻는다. 왜냐하면 나는 만일 그것이 한 번만 보여진다면 당신과 내가 논의한 물음, 탁월함이 가르쳐질 수 있는지 아닌지가 말끔하게 정리되리란 것을 알기 때문이다. 그렇기 때문에, 그 논증의 결과는 그것이 사람이었던 것처럼 우리를 조롱하고 비난하는 것처럼 보일지도 모른다; 만일 그것이 혀를 가졌다면, 아마도 이렇게 말할 것이다. '당신들 둘 다 뭐 이런 바보들이 있나, 소크라테스와 프로타고라스. 분명히 탁월함이 가르쳐질 수 없다고 말한 소크라테스 당신은 지금 바로 그 반대의 것을 설득하려 들고 있다, 모든 것 - 올바름도 사려도 그리고 용기도 - 은 앎임을 보여주려고 애쓰면서, 그걸 보여줌으로써 탁월함이 가르쳐질 수 있다 는 것이 따라나오는 것을 말이다. 만일 그것이 프로타고라스가 주장하려 애쓰는 것처럼 앎과 다른 것이라면, 분명히 그것은 가르쳐질 수 없다; 그러나 이제, 만일 그것이 당신 주장처럼 전적으로 앎일 것이라 증명하는 것이 된다면, 소크라테스, 만일 그것이 가르쳐질 수 없다면 더욱이 이상한 일일 것이다. 다른 한편으로 여기에서 프로타고라스가 조금 전에 탁월함이 가르쳐질 수 있다고 전제했는데 지금은, 다름 아닌 앎을 피함으로써, 그것이 무엇보다도 최소한 가르쳐질 수만은 없을 것이도록 그렇게, 그 반대를 주장하는 것처럼 보인다.' 자, 프로타고라스, 내가 주장된 이 모든 것을 앞뒤로 볼 때, 나의 바람은 그것을 말끔하게 정리하는 것이오: 나는 '탁월함이란 무엇인가'라는 물음을 향해 진격해야 할 것 같고 또 공격해야 할 것 같소. 그리고 나서 다시 되돌아와 그것이 가르쳐질 수 있을지 없을지를 물어야 할 듯 싶소.

『프로타고라스』의 결론에서, 소크라테스와 프로타고라스 모두 탁월함이 가르쳐질 수 있는지에 대한 물음에 대해서 자신들의 입장들을 뒤집어 버렸다. 이러한 뒤얽힘은 변증적 반어법의 단편이다. 그 목적은 탁월함이 무엇인지 밝혀내는 일의 필연성을 변호하는 것이 아니라 그것을 드러내 보이는 것이다.

7) 『프로타고라스』는 『메논』에서 그 짝이 되는 조각을 가진다. 소크라테스가 메논에게 탁월함이 가르쳐질 수 있는지에 대해 질문을 받을 때, 그는 전제를 가지고 그 문제를 정착시킬 것을 최종적으로 떠맡는다. 그는 만일 탁월함이 앎이라면 그것은 가르쳐질 수 있다(87a-c)고 가정한고; 그것이 앎이라고 주장한다(87c-89a); 그리고 그것은 가르쳐질 수 없는데, 왜냐하면 그에 대한 선생들이 없기 때문이라고 주장한다(89c-96c); 그리고 그것이 '다이몬적인 분배에 의해, 이해 없이' 사람들 속에 현존한다고 결론내린다(100a). 즉, 소크라테스는 함축을 전제하고, 그 함축의 선행하는 것을 긍정하고, 그것의 결론을 부정하며, 회의주의로 끝을 맺는다. 이러한 구조는 플라톤이 그의 독자들을 그의 전제들 사이에서 선택하도록 할 작정이었다면 무의미할 것이다: 문제의 핵심은 어디에나 놓여 있다. 소크라테스와 메논은 탁월함이 무엇인지 먼저 밝혀내지 않고 탁월함이 가르쳐질 수 있는지 아닌지에 대한 물음을 해결하고자 시도했다. 그들은 아포리아(당황)로 끝을 맺었고, 심지어 기하학자들로부터 빌려온 새로운 가정의 방법조차 그들을 구해내지 못한다(86e-87a). 그러나 또, 기하학자들은 그들의 용어들을 정의할 수 있다; 소크라테스와 메논은 할 수 없다. 이것은, 그래서, 그들의 실패의 교훈이다. 정의는 필수적이라는, 이야기하기 위해 이야기하는 중인 그 어떤 것을 아는 것은 좋은 일이라는 익숙한 소크라테스적 교훈이다. 그래서 『메논』은 또한 변증적 역설의 한 조각이다: 그 극적인 구조는 정의가 필수적임을 선언하는 것이 아니라 드러내 보이는 것을 의미하게 된다. 그것은 탁월함이 앎이라는 것과 같은 그런 도덕적 결론들을 권고하는 것에 관심이 두어지는 것이 아니라, 그 아래에서 그러한 결론들만이 달성될 수 있는 그러한 조건을 지시하는 데에 관심이 두어지는 것이다.

8) 만일 이것이 『프로타고라스』와 『메논』에 대한 옳은 설명이라면, 『프로타고라스』의 전하고자 하는 바가 탁월함과 앎의 단일성이라는 관점에 반대하여 강력하게 영향력을 끼치는 것이다. 그 둘 중 어떤 대화편도 이러한 문제에 헌신하지 않고, 두 대화편 모두 그 대화편들의 변증적 중심점으로서, '탁월함이란 무엇인가?'라는 물음에 답하는 일의 필수불가결함 - 즉 그것을 정의하는 일의 필연성 - 을 강조하며 결론에서는 그 물음이 대답되지 않은 채로 남겨졌다는 것을 주장한다.

9) 『프로타고라스』에서 탁월함의 단일성에 관한 문제에 대한 플라톤의 설명이, 그 방식에서 탁월함이 하나일 그러한 대안적 방식들을 예상한다는 것은 보여주어야 할 것으로 남는다. 프로타고라스는 다음과 같은 명제들에 동의하도록 이끌린다 (329c-330b): 탁월함은 올바름, 사려, 경건, 지혜, 용기라는 여러 부분들과 함께 하나의 전체이다; 이러한 부분들은 구별되고, 같은 것에 대한 다른 이름들이 아니다; 이러한 부분들은 오로지 크기에서만 차이가 나는 황금의 일부 같은 부분들이 아니고, 두 눈과 두 귀 처럼 얼굴의 부분들과 같은 부분들이다. 그것들은 그 자체로서나 그것들의 능력이나 기능에 있어서가 아니라(οὔτε αὐτὸ οὔτε ἡ δύναμις αὐτοῦ, 330b) 서로 그러한 종류의 것(οἷον)에 속하는 것들이다; 탁월함의 그 부분들은 구별될 뿐만 아니라, 사람이 용기 있으나 올바르지 못하거나 올바르되 지혜롭지 못할 것이 가능하고, 그래서 그 사람은 탁월함 전부를 가지지 못하고서도 탁월함들의 일부를 가질 것이라는 점에서, 분리될 수도 있다.

10) 다음으로 프로타고라스는 탁월함이 여러 부분들과 함께하는 하나의 전체임을 긍정함에 있어서, 복합된 반박을 내놓고 있다: 그는 탁월함의 상이한 부분들이 그것들 각각의 그 자체에 있어서 동일한 것들임을 부정하고 있다; 그는 또한 그 부분들이 능력이나 기능에 있어서 동일하다는 것도 부정하고 있다. 그래서 탁월함들이 하나일 두 가지 방법들이 있다.

11) 탁월함들이 그것들 각각의 그 자체에 있어서 다르다고 말하는 것은 아마도 그것들이 상이한 정의들을 가진다고 마하는 것일 터이다. 그렇지만 탁월함이 기능에 있어서 다르거나 같다고 말하는 것이 무엇을 의미하는지 분명하지 않다. 탁월함의 기능은 무엇인가?

12) 그런 물음은 『프로타고라스』 어디에서도 직접적으로 대답되지 않는다. 탁월함의 기능은 탁월함 그 자체가 아닌데, 왜냐하면 그것은 얼굴에 대한 예시를 무관한 것으로 만들기 때문이다: 두 눈과 두 귀는 그것들의 기능들인 보는 것과 듣는 것으로 차이가 난다. 다시, 두 가지 덕들은 그것들이 동일하다거나(ταὐτόν) 아주 유사하다(ὁμοιότατον)는 것조차 긍정할 것도 없이 서로 그와 같은 그러한 종류(οἷον)에 속한다는 것을 긍정할 수 있다. 그래서, 한 단어의 '기능'이 그 단어의 의미 또는 능력이라 할지라도, 그리고 올바름이 '올바름'의 의미라 할지라도, 올바름의 기능은, '올바름'의 기능으로부터 구별되는 것으로서, 올바름은 아니다.

13) 만일 탁월함의 기능에 대한 관념이 이해될 것이라면, 우리는 그 표현이 그 안에서 사용되는 그러한 논증으로 시선을 돌려야만 한다. 프로타고라스가 탁월함들이 그 자체로서나 그것들의 기능으로서나 동일하다는 것을 부정(330b)하는 근거, 그리고 나중에 그 탁월함들이 서로 간에 그와 같은 그러한 종류의 것들이라는 것을 부정(330e-331c)하는 근거는 탁월함들이 분리가능하다는 것이다(329e; 349d참조). 그리고 이것은 탁월함들이 서로 만나도록 한다는 것, 경건의 현존이 올바름의 현존을 수반한다는 것, 또는 용기의 현존이 지혜의 현존을 수반한다는 것을 부정하는 것이다. 그 추론은 명확해 보인다: 탁월함의 기능은 그 기능을 도입하는 어떤 것 속에 놓인다; 만일 탁월함들이 기능에 있어서 상이하다면, 그것들은 분리가능하다; 만일 분리가능하다면, 기능에 있어서 상이하다.

14) 이것은 그 논증의 양식에 걸맞는다. 프로타고라스는 탁월함들이 그것들 자체에 있어서 그것들의 기능에 있어서도 모두 다르다고 주장했다; 이 명제는 만일 그 명제가 탁월함들이 그것들 자체에 있어서나 또는 그것의 기능에 있어서 동일하다는 것이 보여질 수 있다면 틀린 것으로 증명될 것이다. 만일 탁월함들이 정의에 있어서 하나라면, 그것들은 기능에 있어서도 하나이다. 그러나 만일 그것들이 기능에 있어서 하나라면 정의에 있어서도 하나라는 것이 뒤따를 방법은 없다. 사람이 올바름 없이 경건할 수 없다거나 경건함 없이 올바를 수 없다고, 또는 경건과 올바름이 서로를 도입한다고 말하는 것은 올바름의 정의와 경건의 정의가 같다고 말하는 것과는 다르다. 탁월함들은 만일 분리가능하다면 구별가능하다; 만일 탁월함들이 구별가능하다면, 그것들이 분리가능하다는 것이 뒤따르진 않는다. 만일 탁월함의 부분들이 황금의 부분들과 같은 그런 부분들이 아니라면, 그 자체에 있어서 그것들의 기능에 있어서 모두 상이한, 얼굴의 부분들과 같은 그런 부분들이라는 것이 뒤따르진 않는다. 즉, 탁월함의 특수한 경우에, 황금과 얼굴에 대한 그 예시들은 부적당하다는 것일 수 있고, 황금도 얼굴도 밝혀내지 못하는 정의에 있어서 차이와 기능에 있어서의 동일성이 있을 수도 있다는 것일 수 있다.

15) 그 논증에 대해서는 이 만큼이면 됐다. 그러나 그 변증적 상황은 여전히 더욱 복잡하다. 만일 소크라테스가 프로타고라스가 탁월함들이 분리가능하다고 추정함에 있어서, 사람들이 올바름 없이 단지 지혜롭거나 용감할 수 있다고 주장함에 있어서 틀렸다는 것을 보여줄 수 있다면, 그는 탁월함들이 그 자체에서도 그것들의 기능에 있어서도 다르다는 그 주장을 반박할 것이고, 탁월함들이 동일한 의미에서 하나임을 보여줄 것이다. 그러나 이것은 탁월함들이 어떤 면에서 하나인지를 보여주는 것은 아니다: 탁월함들이 정의에 있어서 하나인지, 아니면 오로지 그것들이 서로 도입하고 분리될 수 없다는 의미에서만 하나라는 것인지 말이다.

16) 『프로타고라스』는 이 물음에 대해 세심하게 고안된 모호함을 유지한다. 그것은 대안들 사이에서의 선택이 아니고, 프로타고라스 그 자신은 그 논증에 대한 그 대안들의 관계를 인지하지 못한다. 이것은 놀랄 것도 없다. 탁월함들이 정의에서 하나인지 기능에서 하나인지 혹은 양자에 있어서 하나인지 묻는 것은 탁월함이 무엇인지를 암시함으로써 묻는 것이다. 그 물음은 오로지 대화의 중국에서야 명확하게 된다(361b-c). 거기에서 소크라테스는 그가 고려하는 중에 내내 그것을 가지고 있었다고 시사한다. 그러나 프로타고라스는, 그가 탁월함은 가르칠 수 있는 것이라고 자신있게 주장했고(318a-319a), 그리고 탁월함이 무엇인지 알지 못했던 자로서, 소크라테스적 엘렝코스(논박술)가 바로 그것을 가르치기 위해 계획된 그러한 그 자신의 무지에 대한 확신을 갖기 전까지는, 탁월함의 본성으로 파고드는 탐구에 유익하게 착수할 수 없었다.

17) 요약하면: 『프로타고라스』는 탁월함들이 하나일 수 있는 두 가지 길을 예상한다 - 말하자면, 정의에 있어서의 동일성이나 같음, 그리고 기능에 있어서의 상호 함축 또는 같음이 그것이다. 이것은 그 주제에 대한 다른 대화편들의 논증 양식에 부합한다. 그에 대해 『메논』은 탁월한 예시를 제공한다:

우리는 일반적으로 이렇게 단언할 것이다: 사람에게 있어서, 다른 모든 것들은 영혼에 의존하지만, 영혼에 속하는 것들은 그것들의 좋음을 지혜에 의존한다고; 이러한 설명으로, 유익한 것은 지혜이고, 우리는 탁월함이 유익하다고 말했다. …그래서 우리는 탁월함이 지혜이고, 전체로서나 부분에 있어서나 그러하다고 말하고 있다.

이러한 논증은 지혜와 탁월함이 동일함을 보여주지 않는다; 그것은 사실상 탁월함들이 구분된다는 것을 전제한다. 그것은 오히려 지혜가 탁월함에 대해 필연적인 것이고, 그러므로 충분한 것이며, 조건이며, 다른 탁월함들을 수반하고 또 그것들에 의해 수반된다는 것을 보여준다.

18) 이후 『정체』에서 플라톤은 영혼에 대한 분석에 기초된 탁월함들에 대한 설명을 제공했다. 그 설명은 탁월함들의 단일성을 상호 함축을 통해 긍정한다. 영혼은 그 부분들 각각이 적합한 기능을 수행할 때 비로소 올바를 수 있다; 절제는, 각 부분이 지배해야 하고 따를 것에 동의할 때; 지혜는 이성적 부분이 그 적절한 기능을 수행하고 지배할 때, 용기는, 기백의 부분이 그 지배자의 명령에 있어서 훈련될 때 생긴다(『정체』Ⅳ, 442b-d). 그 탁월함들은 여기에서 명백하게 정의상으로 다르다; 그만큼 명백하게, 탁월함들은 기능에 있어서 하나이다 - 그것들은 서로를 도입하고, 서로로부터 따로 떨어져 존재할 수 없다. 용기는 지혜를 수반하고 그것에 의해 수반된다; 지혜는 올바름을 수반하고 또 그에 의해 수반된다; 올바름은 절제를 수반하고 그에 의해 수반된다. 소크라테스가 결론에 있어서 '우리가 지금 논증에 있어서 이 높이에 올랐으니, 내가 보기에, 산 정상으로부터 보는 것처럼, 탁월함의 단일한 꼴 하나가 있는 것으로 보인다'라고 언급해야 한다는 것은 놀라운 일이 아니다.

19) 탁월함의 단일성에 대한 소크라테스적 원칙은 그래서 탁월함이 여러 부분들과 함께 하나의 전체라는 주장과, 그리고 탁월함들이 최근류와 종차를 통해 정의될 수 있다는 주장과도 양립불가능한 것이 아니다. 그렇지만 그것은 부분들과 부분들 사이의 관계, 그리고 부분들과 전체 사이의 관계가 동시함축적이라는 것을 시사한다.

로고스와 우시아

1) 만일 앞서 말한 설명이 건전하다면, 초기 대화편들 내에서 그런 종류의 논증은 고려함에 있어서 실재적이고 명사적이지 않을 것이다: 그것은 한 대상에 대한 설명이지 한 단어에 대한 설명이 아니다.

2) 그 대상이 적어도 그것이 유와의 관계에서 존립하는 종일 때에는 단순하기 때문에, 아리스토텔레스가 정의될 것과 정의함 사이에서 통용된다는 것을 고수하였던 그러한 관련은 획득하지 않는다: 그 종은 결합된 부분들로서 그것의 유와 종차와 함께 완전히 동일하지는 않다. 그러나 이러한 종류의 같음이 통용되지 않는다고 말하는 것은 어떤 종류의 같음도 통용되지 않는다고 그렇게 부정하는 것은 아니다: 『에우튀프론』10d, 13에서 신들에 의해 사랑받는 것은 경건에 대한 정의로서는 거부되는데 왜냐하면 그 둘이 다르기 때문이고(ἕτερον; 10e, 9에서 ταὐτόν이라는실에 견주어 반대로), 이러한 거부는 15c, 2에서 반복된다(οὐ ταὐτόν … ἀλλ᾿ ἕτερα ἀλλήλων).

3) 여기에는 어떤 종류의 같음이 필요한가? 그것은 외연의 동일성, 또는 진리치 보존(salve veritate) 대체성이 아닌데, 설령 신들에 의해 사랑받는 그러한 것들 모두가 그리고 오로지 그러한 것들만이 경건하거나 경건할 수 있다 할지라도, 이것은 단순히 πάθος를 진술할 뿐 οὐσία를 진술하지 않기 때문이다. 그것이 개념적 동일성인 것도 아니다: 에우튀프론은, 경건을 생각함에 있어서, 그렇게 함으로써 경건의 정의를 생각하지는 않는다. 동의어의 같음도 아니다.

4) 정의는 대상에 대한 설명이다. 그래서 아마도 정의에 있어서 필요한 같음은 이를테면 서술적 동일성, 말하자면 샛별과 태백성의 동일성에 대한 것이어야 하거나, 또는 - 그 예시에서 필요한 경험적 우연성을 피하기 위해 -  일곱번째 정수와 네번째 소수의 동일성에 대한 것이어야 한다. 그러나 이러한 종류의 동일성은 두 가지 한정적 서술들이 동일한 대상에 의해 충족된다는 것을 시사하는 반면에, 실재적 정의에 있어서 정의될 것은 대상에 대한 서술이 아니고, 대상 그 자체이다. 그 비유는 만일 서술적 구절이 명사에 의해 대체된다면 더욱 근접하다: '금성은 샛별이다' 또는 '7은 네번째 소수이다'; 무엇보다도 유일한 서술로 정의를 생각하는 것이 타당하다. 그러나 이것도 오독이다. 지혜는, 의심의 여지 없이, 탁월함이다; 그러나 '…는 탁월하다'라는 말이 소크라테스를 서술하는과 같은 의미에서 '…은(는) 탁월함이다'라는 말이 지혜를 서술하한다고 주장하는 것은 잘못인데, 왜냐하면 첫번째 구절에서 그 관계는 유에 대한 종의 관계이고, 두번째 구절에서 그 관계는 한 특성에 대한 한 특성의 예시 관계이기 때문이다. 그것은 한정적 서술에 있어서 필요한 후자의 관계이다.

5) 아마도 정의에서 필요한 종류의 동일성은 그것 자체의 유(sui generis)라고 말하는 것이 가장 간단할 것이다: 그것은 어떤 것을 포함하는 유와 그것을 그 유 속에서 다른 종들로부터 지적해 내는 종차를 진술함으로써 그 종인 어떤 것을 말할 때 필요한 그런 동일성이다. 종들은 단순하기 때문에, 종차는 종들에 대한 구별의 구성요소는 아니지만, 그 구별의 결과이다; 정의에 있어서 종차는 사유근거이고, 존재근거가 아니며, 그러므로 정의에서 필요한 동일성은 대상들의 동일성이 아니다. 정의의 단일성 문제는 일어나지 않는다. 아마도 정의에 대한 이러한 관점을 위한 최선의 - 그리고 ὁρίζειν과 ὅρος의 대다수 함축에 부합하는 하나의 -  비유는 사상(寫像)의 비유이다: 정의한다는 것은 한 종의 위치를 그 종을 포함한 유 속에서 정하는 것이다. 만일 종이 농장이라면, 유를 규정함의 목표는 그 농장이 위치하는 마을을 지시하는 것일 터이다; 차이를 규정함의 목표는 농장의 경계들을 측량하는 것이 아니라, 오래 전 조사된 경계선들의 현존을 밝혀내는 것일 것이다. 정의에서 필요한 같음은 말하자면, 유일한 위치의 같음이다: 정의될 것은 정확하게 정의하는 것 안에서 사상된 그 위치를 점유한다.

6) 이러한 방식에서 생각되는 최근류와 종차를 통한 정의는 형이상학에서의 전제를 필요로 한다. 그것은 꼴들이, 서로에 대해 포함하는 것에 포함되는 것으로서, 포함되는 것에 포함하는 것으로서 존립하면서, 계층적으로 정렬된다는 것이다. 이러한 관계는 내적이어야 할 것으로 드러나거나, 상호 구성적이어야 할 것으로 드러난다: 종들은 그것들이 만일 그것들을 포함하는 어떤 것에 의해 포함되지 않았다면 그러한 것들이 아니었리라는 점에서 그것들의 특성을 그것들의 유들에 의존한다; 올바름은, 만일 그것이 탁월함이 아니었다면, 올바름이 아니었을 것이다. 반대로, 탁월함은, 그것의 한 종이 올바름이 아니었더라면, 탁월함이 아니었을 것이다: 유는 그것의 현존과 특징을 그것의 종에 의존한다. 전체는 그것의 부분들에 의존하여 전체인 그것일 것이다. 플라톤은 그 점에 대해 조금 덜 명확하지만, 아마도 유들에 대한 정의는, 종들에 대한 정의와 달리, 구성 요소들로의 분석을 필요로 할 것이다.

7) 이것은 플라톤적 유들에 대한 함축의 풍부함을 설명한다. 아리스토텔레스적 논리학에 있어서, 유들은 종들로부터의 추상들이다: 소크라테스와 플라톤은 사람이라는 것에 속하는 공통된 특징을 가지고, 사람과 말은 동물이라는 것에 속하는 공통된 특징을 가진다. 그 유는 여기에서 단지 다양성에 대한 공통된 요소일 따름이고, 그것의 종들보다 더 '추상적'일 따름이다. 그러나 만일 정의가 사상에 대해 동종적이라면, 만일 유들이 부분들을 포함하는 전체들이라면, 유는 공통된 특징이 아니라, 마치 그것의 현존이 그것의 요소들 혹은 부분들의 현존에 의해 전제하기도 하고 전제되기도 하는 하나의 체계이다. 그것은 이것이 그래서 유는, 함축의 상대적으로 결여되었음보다는 오히려 함축을 잔뜩 실은 것으로서, 결핍되었다기 보다는 풍부한 것으로 생각되기 때문이다.

8) 하나의 체계로서, 유는 한 종류의 폐쇄를 지니고 있어야만 하고, 그 체계를 다른 체계들로부터 또는 다른 종류의 제한들로부터 갈라놓는 단일성을 지니고 있어야만 한다. οὐσία와 πάθος 사이의 구별은 한 사물에 본질적인 것과 그 사물이 단지 가지게 되었을 뿐인 특징 사이의 구별을 수반한다. 올바름 또는 탁월함에 대한 경건의 관계와 모든 신들에 의해 사랑받는 것에 대한 경건의 관계 사이에는 차이가 있다. 만일 '그것은 무엇인가?'라는 물음이 본질적 정의에 있어서 내적 성질을 시사한다면, 그러므로 그것은 꼴에 대한 모든 설명들이 본질을 진술하는 것은 아님을 시사하는 것이다. 그것은 즉 그 관계의 내적 성질만큼이나 외적 성질도 시사한다.

9) 유들은 체계들이기 때문에, 그리고 종들은 그 유들의 요소들이기 때문에, 종들에 대한 정의들은 동일성(정체성)에 대한 일상적 진술들이라기 보다는 오히려 사상들이다. 그러한 사상을 구축하기 위해, 또는 사용하기 위해서, 일반적 언어에도 개별적 사물들과 행위들에도 호소할 수 없다: 꼴들에 대한 지식은 지적 직관을, 존재하는 사물들의 본성을 꿰뚫는 직접적 통찰을 요청할 것이다. 실재적 정의의 진리는 오직 그에 대한 정의가 설명인 그러한 진리를 파악함으로써만 파악될 수 있다.

10) 만일 이러하다면, 정의와 직관은 상보적일 것이다. 지적 직관은 실례로 해석되는 것이 아니고, 그렇지만 '알고 있음에 의한 앎'에 속하는 난해한 것이다; 말하자면 어떤 종류의 설명도 제공함이 없이 푸른 하늘의 한 조각을 알고 있을 수 있는데, 왜냐하면 감각은 지적으로 노력할 것이 없기 때문이다. 그러나 꼴들에 대한 직관은 반대로 설명을 요구하기 때문이다; 변증은 단순히 그냥 보는 것이 아니라 보려고 하는 것, 이전에 불충분한 설명 속에서 흐릿하게 보였던 어떤 것을 충분한 설명 속에서 명확하게 보려고 하는 것을 필요로 한다. 시야는 진술들에 대한 검토에 의해 확보된다. 미리 앎에 의한 앎에 대한 지적 직관의 비교는 지적 직관의 대상이 주어진 것이 아니라 하나의 목표라는 사실을 도외시한다.

11) 일부 철학자들은 소크라테스적 변증의 이 지점에서 압박을, 직관과 논리 사이의 긴장을 발견할 것을 주장해 왔다. R. G. 콜링우드는 다음과 같이 썼다.

그렇지만 상당히 높게 플라톤의 철학적 성취들은 평가받고, 최고에 못 미치는 어떤 가치에 있어서 그것들을 평가하는 것은 스스로, 그의 방법론에 대한 이론이 그 자신에 의해 철학과 수학 사이에서 확립되는 충분히 깊은 구별로 이끄는 것에 대한 실패를 통하여 결함있는 것으로 인정되어야만하는, 철학자가 아님을 인정하는 것이 될 것이다. 그 결과는 그의 방법론이 철학을 둘로 분열시키는 것이다: 하나는 재기 발랄한 억지 이론의 식상한 낭비이고, 다른 하나는 궁극적 실재에 대한 직관적 시야이다. 이러한 첫번째 것이 두번째 것으로의 경로라는 것은 여러 세대들에 속하는 경험에 의해 플라톤이 그들의 길잡이로 취해졌다는 것에 대한 보증이 될 것이다; 그러나 설령 그렇다 할지라도, 우리는, 사실들에 권위에서 받아들여질 사실들에 대한 탐구가 아니라, 그 조명 속에서 사살들이 이해될 그러한 개념들에 대한 탐구 속에서, 철학적 탐구에 매진한다; 그리고 이러한 것들을 플라톤이 우리에게 주진 않았다.

이러한 비판은 감각 지각의 틀을 관통하는 지적 직관을 강요함으로부터, 지적인 것으로서, 그 직관이 판단을 통해 지속한다는 것을 알아차리는 데에 실패함으로부터 귀결한다. 지적 통찰에 속하는 한 행위는 하나의 진술이 아니라, 진술 속에서 표현 가능한 하나의 내용을 필요로하고, 그로써 비평에 적합하다. 한 진술은 지적 통찰에 속하는 하나의 행위가 아니라, 그것은 그를 통해 통찰이 표현되는 그러한 것이다 - 그 자신에 대해서조차. 안다는 것은 설명을 제공할 수 있다는 것이다; 설명을 제공할 수 있다는 것은 안다는 것이다.

12) 그러나 이러한 지점에 도달했다는 것은 초기 대화편들을 뛰어 넘었다는 것이다.

-蟲-
2. 실재적 정의

1) '그것은 무엇인가?'라는 물음은 초기 대화편들의 변증 속에서 확정적이고 기술적인 의미를 지닌다. 그 물음에 대한 대답은 꼴을 명시해야 한다 - 즉, 보편개념이고 표준이며 본질이다. 『에우튀프론』은 그 설명이 반드시 그 꼴이 부분인 어떤 전체를, 그리고 그 전체의 어떤 부분이 그 꼴인지 진술해야만 한다고 제안한다: '경건이란 무엇인가?'라는 물음에 답하는 것은 최근류와 종차로써 경건을 정의하는 것이다.

2) 이 관점에서 그런 종류의 정의는 물론 실재적 정의이며 명사적 정의가 아니다; 그 정의는 어떤 것들의 진리치인 말들에 대한 정의가 아니라 그에 대한 말들이 참인 그러한 것들의 본성에 대한 정의이다. 실재적 정의는 말들이 어떻게 사용되어야 하는지에 대한 조건들이나 어떻게 그 말들이 실제로 사용되는지에 대한 보고가 아니라 본질에 대한 분석이다. 그것이 본질분석이기 때문에, 실재적 정의는 조건들로서가 아니라 참 또는 거짓이다. 예를 들어 올바름이 탁월함이라는 것은 참이고, 그것이 양이라는 것은 거짓이며, 만일 'is'가 종들과 유들에 관련하는 개념정의적 'is'로서 해석된다면 계속 그러할 것이다.

3) 실재적 정의에 대한 공통된 관념은 그러한 정의가 단순하게 명사적 정의에 현존 주장을 더한 것이라는 관념이다. 그래서 예를 들어 삼각형의 실재적 정의는 다음과 같을 것이다: '삼각형은 "분명한 세 각을 가진 도형"이고, 삼각형들이 있다'. 밀은 지난 세기(19세기)에 이것을 주장했고, 일부 해석자들에 따르자면, 그것은 또한 아리스토텔레스의 관점이었기도 하다. 그래서 토마스 히쓰 경은 다음과 같이 기록하였다:

아리스토텔레스가 강조하고자 더욱 고심한 정의들과 정의된 사물의 현존이나 현존하지 않음에 대해 아무것도 주장하지 않는 정의는 아무런 관련이 없다. 아리스토텔레스의 정의는 '어떤 것이 무엇인가'라는 물음에 대한 답변이고, 그 어떤 것이 그것이라고 이야기하지 않는다. 정의된 다양한 것들의 현존은 각 학문에서 기본적인 몇몇 것들, 그것들의 현존이 증명될 수 없고 각 학문의 제 1 원리들 중에서 전제되어야만 하는 것들에 대한 경우를 제외하고는 입증되어야 한다; 예를 들어 기하학에서 점들과 선들은 있는 것으로 전제되어야만 하지만, 다른 모든 것들의 현존은 입증되어야만 한다.

히쓰 경은 아리스토텔레스의 이론을 계속해서 연결한다. 그 이론은 아마도 구조가 현존의 증명으로 기능하는 유클리드 『원론』에서 유클리드의 실행을 가지고 일반적으로 알려져 있는 것보다는 아리스토텔레스 당대의 수학에 더욱 밀접하게 관련될 것이다.

일반적으로 말해서, 유클리드의 정의들과 그 정의들에 대한 그의 사용은 정의들 그 자체는 정의된 것들의 현존에 대해 아무것도 말하지 않는다는, 하지만 그것들 각각의 현존은 입증되거나 ('원칙들'의 경우에는) 전제되어야만 한다는 아리스토텔레스의 원칙과 일치한다. 아리스토텔레스는 기하학에서 점들과 선들의 현존만이 반드시 전제되어야만 하는 것이고, 나머지 것들의 현존은 증명되는 것들이라고 말한다. 따라서 유클리드의 첫번째 세 가지 가정들은 직선들과 원들을 작도할 수 있는 가능성을 언명한다(『원론』에서 사용된 직선들을 제외한 '선들'만). 다른 것들은 정의되고 그 후에 작도되며 존재하는 것으로 증명된다: 예를 들어 1권 정의 20에서 등변 삼각형이 의미하는 것이 무엇인지 설명된다; 그리고 (1권 1) 그 등변 삼각형을 작도할 것이 제안되고, 작도되었을 때, 그것은 정의와 일치하는 것으로 증명된다. 사각형이 정의될 때(1권 정의 22) 그러한 것이 정말로 존재하는지 어떤지 하는 물음은 1권 46에서 정의를 만족시키는 것으로 증명되기 전까지 열려 있다. 직각(1권 정의 10, 1권 11)과 평행(1권 정의 23, 1권 27-9)을 가지고도 비슷하다. 단순한 추정과 상상을 배제시키는데에 굉장한 주의를 요한다. 이름들에 대한 주관적 정의로부터 사물들에 대한 객관적 정의로의 이행은 기하학에서 구조들(가정된 제 1원리들)을 수단으로, 다른 학문들에 있어서 경험을 통해 이루어지듯이 그렇게 이루어진다.

정의와 현존에 대한 이러한 설명은 의심할 것도 없이 아카데미아에서 알려져 있었다. 그리고 아리스토텔레스는 그것을 거기에서 배웠을 것이다. 그 뿌리들은, 특정한 본성의 어떤 것에 대한 현존을 전제하도록 허용되고, 그 가정을 지지하거나 거부하도록 허용되는, 플라톤의 가설들에 의한 연구방법에 있을 것이다. 아리스토텔레스는 단일한 것과 단일한 것이 존재한다는 사실은 다른 것이다'라고 주장한 첫번째 사람이 아니었다.

4) 실재적 정의에 대한 이러한 관점은 또한 현존과 근대 논리학에서 자주 제시되는 분석성 사이의 관계에 대한 설명에 밀접하게 부합한다. 정의들, 혹은 더욱 정확하게 말해서 그 참값이 유일하게 정의들에만 기대는 명제들은 분석적이고, 그들의 용어들로 의미하는 바에 의해 참이며, 그것의 반박들은 자기모순적이다. 그래서, 예를 들어'암여우(vixen)들은 암컷인 여우들이다'라는 것은 정확하게 참인데 왜냐하면 '암여우'라는 표현은 '암컷 여우'라는 표현과 같은 뜻이기 때문이다: 그것을 입증하기 위해, 우리는 세계가 아니라 우리가 그 속에서 세계에 대해 이야기하는 언어를 본다. 그러나 어떠한 분석적 명제도 분석적인 것으로서 현존 주장을 시사하지 않는다: 이것을 가정하는 것은 현존이 술어적 표현의 의미에 대해 부분일 수 있다는 것, 그리고 현존이 술어라는 것을 가정하는 것이 될 것인데, 반면 사실 그것은 술어들에 대한 적용을 명시하는 양화사이다. 그래서 '암여우'가 의미하는 것은 암여우들이 있다는 것을 시사하지 않고 시사할 수도 없다: 그러한 정보의 단편을 위해서 우리는 우리가 세계에 대해 말하는 언어가 아니라 그 세계를 보아야만 한다.

5) 실재적 정의가 단순하게 명사적 정의에 현존 주장을 더한 것이라는 관점은 그 이면에 수세기에 걸친 수학적이고 철학적인 전통의 부담과 동시대 논리학의 부담 모두를 가진다. 그 관점은 그 관점이 『에우튀프론』과 다른 초기 대화편들에서 전제된 관점이 아니라는 것을 보기 위한 지적 상상력의 활동을 요구한다.

6) 그리고 분명히 그건 아니다. 『에우튀프론』의 변증 절차는 '경건'이란 단어가 의미하는 것이 무엇인지를 발견하기 위한 시도로 대변될 수는 없다. 추가적으로 '경건'이 어떤 것에든 해당되는지 아닌지를 밝혀내기 위한 시도와 결부되기 때문이다. 왜냐하면 우선 '그것은 무엇인가?'라는 물음을 답하기 위한 필요조건들은 명사적 정의의 필요조건들이 아니기 때문이다: 명사적 정의들은 말들에 대한 정의들이지 표준들 또는 본질들에 대한 정의들이 아니고, 어째서 그것들이 최근류와 종차를 통한 것이기를 요구받아야 하는지 아무런 훌륭한 이유도 없다. 그리고 둘째로 초기 대화편들 내에서 존재적 의미는 증명되는 것이 아니라 당연시된다: 소크라테스와 에우튀프론은 경건한 것들이 있다고  전제하고, 오로지 무엇이 그것들의 본성인지만을 묻는다; 그리고 현존에 대한 이러한전제는 '그것은 무엇인가?'라는 물음이 최초에 사례들 - 그 물음이 제기된 모든 초기 대화편들 각가에서 이야기하는 것 - 에 호소함으로써 대답되는 모든 초기대화편들에서 이루어진다.

7) 이건 다른 식으로 요약될 수 있을 것이다. 꼴은 '실체시된 의미'가 아닌데, 예를 들어 '염소사슴'과 '직각원' 같이 의미하는 바는 있지만 어떠한 적용도 가지지 않는 표현들이 있을 수도 있기 때문이다. 꼴은 '가능성', 또는 화이트헤드주의의 영원적 객체가 아닌데, 예컨데 유니콘들처럼 아무것도 만족시킬 수 없는 가능성들이 있을 수 있기 때문이다. 꼴은 오히려 무엇인 어떤 것의 본성이고, 그러므로 존재적 의미는 그 어떤 것의 바로 그 관념 안에 세워진다; 아무것도 가지지 않는 꼴들이 있는지에 대한 물음은 혼동에 기초하는 것이다.

8) 소크라테스적 변증에 대해서 현존이란 말하자면 주어진 것이다: 그 목적은 변증의 본성을 관통하는 것이고, 그러한 관통은 정의 속에서 표현될 것이다. 만일 이러한 관점이 일반화되고, 윤리학에 대한 물음들로부터 옮겨져서 전체 세계에 대한 물음들로 활동한다면, 정의는 어떤 것에 대한 모든 앎의 토대가 될 것이다.

9) 초기 대화편들 내에서 실재적 정의는 대상 - 탐구의 목표 역할을 하는 대상에 대한 설명이다. 그리고 이러한 관계에서 만일 그러한 정의가 단어에 대한 정의가 아니라면, '개념'에 대한 정의 또한 아닐 것이라는 것은 아마도살펴볼 가치가 있을 것이다. 에우튀프론의 경건에 대한 개념 또는 일상적인 아테나인들이 '경건'이란 단어를 가지고 일상적으로 이해하는 어떤 것은 경건의 본성과 본질에 대해 불충분하다; 이것은 엘렝코스, 변증적 반박술에 이 보여주는 것이다. 우리의 일상적 개념들의 꼴에 대한 관계는 어떤 의미에서 목적론적이다: 경건의 꼴은 짐작컨데 에우튀프론이 만일 그의 말들이 의미하는 것을 충분히 이해했더라면 그 '경건'이란 말을 가지고 이해했을 어떤 것이다. 『프로타고라스』는 가능성으로서 그가 '경건'이란 단어를 가지고 그가 '올바름'이나 '용기', '사려', '지혜' 그리고 '탁월함'이라는 다른 단어들로 의미하는 것과 정확하게 똑같은 것을 의미했을 것임을 알아차릴 거라고 말한다. 비록 이러한 단어들의 이상적 사용에 있어서 이 단어들의 검토가 오직 이것이 역설이었다는 것만을 말할 것이라 하더라도 말이다. 우리가 일상적으로 한 단어를 가지고 이해하는 어떤 것은 우리가 그걸 가지고 필연적으로 그렇게 이해해야만 하는 어떤 것은 아니고, 꼴들과 우리의 일상적 개념들 사이의 그 구분은 실재적 정의의 결핍과 어려움 모두를 설명한다. 사물들의 본성과 사물들의 본성에 대한 우리들의 관념들을 무분별하게 동일시하는 것을 막기 위해서는 상당한 훈련과 많은 숙고가 요구된다.

최근류와 종차

1) 전통적인 논리학 교과서들에 따르면 정의는 많은 종들에 공통된 하나의 유를 언급함으로써 수행되고, 다른 모든 것들로부터 검토 하에 그 종들을 구별해 내는 차이점이다. 예를 들어 아리스토텔레스에게서 그러하고 로스 교수는 초기 대화편들 내에서도 또한 그렇다고 주장했다:

['용기란 무엇인가?'와 같은 그런 물음들에 의해서] 용기는 요소들로 나뉘어질 수 있는 복합적인 것임이 암시된다; 만일 용기가 그러하지 않다면, 그 물음은 … 오직 용기는 용기라는 것만이 참인 응답일 바보같은 물음이 될 것이기 때문이다. 때때로 그[소크라테스]가 제시하는 이러한 형식의 물음들에 대한 응답들은 원칙적으로 그가 아리스토텔레스가 명시적으로 수행해 나아가던 것과 같이 정의는 최근류와 종차를 통한 분석이라는 가정을 하고 있었음을 보여준다.

정의가 유들과 종차를 통해 진행하는 것이라는 그 주장은 『에우튀프론』에 의해 확인된다. 소크라테스의 그 문제에 대한 논의는 두 부분으로 나뉘어 들어간다. 첫번째 부분은 정의가 유를 제공해야만 한다고 제안한다: 그 부분은 경건이 올바름의 한 부분인지, 그리고 올바름은 경건의 한 부분이 아닌지 어떤지를 묻는다. 그것은 스타시누스(고대 그리스의 신화적 인물, 시인)가 주장하게 된 것처럼 삼가는 마음은 두려움의 한 부분인 반면에 두려움은 삼가하는 마음의 한 부분이 아니라는 것과 같다. 소크라테스는 다음으로 그것을 뭐라 이름붙이지 않고서 종차의 관념을 도입하는 데에로 나아간다; 그 논증에서의 이 새로운 단계는 명백하게 '다음과 같은 점을 숙고하라'는 그의 요구에 의해 표시된다(12d, 5): 만일 경건이 올바름의 부분이라면, 그 다음으로 밝혀지는 것은, 짝수가 수에 있어서 서로 나뉘어 떨어지지 않는 측면들이 아니라 똑같이 나뉘어 떨어지는 두 측면들을 지닌 그러한 부분인 것처럼, 그것이 어떤 부분인가 하는 것이다.

2) 초기 대화편들의 정의에 대한 모든 사례들이 최근류와 종차를 제공하지는 않는다. 예를 들어 『라케스』(192b)에서는 재빠름이 '짧은 시간에 꽤나 많은 것을 해내는 능력'으로 정의되고, 『메논』(75b)에서는 처음에 도형이 '언제나 빛깔을 뒤따르는 것'으로 정의된다. 어느 정의도 『에우튀프론』의 요구들을 만족시키지 않는다. 그러나 『메논』은 그 도형을 '한 입체의 경계'로 한정함으로써 도형에 대한 정의를 개선시켜 나아간다. 그것은 최근류와 종차를 제공한다; 그래서 아마도 고르기아스의 방식으로 제공되는, '시각에 맞추어질 수 있는 형태의 발산'으로서 색을 정의하는(76d) 그럴싸한 가짜 정의도 제공한다.

3) 『에우튀프론』에서 정의에 대한 소크라테스의 설명이 그 문제에 대한 그의 최종적인 설명인지 아닌지, 또는 그의 의견들이, 에우튀프론을 이끌어 가는 수단으로서 부분적으로는 변증적인 것으로 의미되는지는 불확실한 것으로 남는다. 후자의 관점에 대한 증거가 있다. 만일 삼가는 마음이 두려움의 부분인 것처럼 경건이 올바름의 부분이라면, 그래서, 에우튀프론의 설명에 의해, 두려움이 삼가는 마음보다 넓은 것처럼 올바름은 경건보다 넓다; 즉, 경건은 올바름을 수반하는 반면, 올바름은 경건을 수반하는 것은 아니다. 이것은 『프로타고라스』에서 주장되고 다른 초기 대화편들에서 암시적인, 탁월함은 정의에 있어서 모든 탁월함들과 동일하다는 의미에서나, 그 탁월함들이 상호적으로 관련이 있다는 의미에서나 하나의 탁월함이라는 저 유명한 소크라테스적 명제와 상충한다. 그러므로 그것은 아마도 최소한 특히 탁월함의 경우에는, 『에우튀프론』에 있어서 정의에 대한 소크라테스의 설명이 자격을 부여함 없이, 그리고 수정을 요구하지 않고 받아들였을 것은 아니라는 것일 터이다: a가 b의 부분이라고 말하는 것이 b가 a보다 더 넓은 영역들에서 발견된다는 것을 언제나 수반할 필요는 없다.

4) 이것이 『에우튀프론』이 정의를 최근류와 종차를 통한 것으로 그린다는 사실을 대체하지는 않는다. 그러나 『에우튀프론』은 정의될 것이 반드시 '요소들로 분석될 수 있는 복합적인 것'이어야 한다는 로스 교수의 주장을 뒷받침하지도 않는다. 이 제안은 그것이, 예를 들어 경건이 요소들로 구성되지만, 그 요소들이 무엇인지 말하는 것이 불가능한 경우, 정의가 독립적일 수 있다는 의미에서 단순한 그런 일련의 항목들에 있어서 반드시 한정해야만 한다는 것을 그 제안이 암시한다거나, 또는 정의가 불가능하고 그에 있어서 그 어떤 것도 무엇이라고 말할 수 없는 경우 종결할 수 없는 무한퇴행을 암시한다는, 그런 일반적 반대에 대해 책임이 있다. 로스 교수의 해석에 대한 원문의 뒷받침도 없다; 소크라테스의 공간적 은유들은 공간들의 가분성을 제안하는 어떤 방식과도 중첩하지 않는다; 삼가는 마음이 있는 그 어디에서나 두려움이 있지만 두려움이 있는 곳 어디에서나 삼가는 마음이 있는 것은 아니라는 사실로부터 도출된 그 추론은, 두려움이 삼가는 마음의 부분이 아니지만 삼가는 마음은 두려움의 일부라는 것이 아니다; 삼가는 마음과 두려워하는 것을 나누지 않지만, 삼가는 마음을 검토함에 있어서 그것이 두려움을 수반한다는 것은 안다. 최근류를 가지고 그러하듯 종차를 가지고서도 마찬가지이다. 주어진 종차의 사례는 짝수가 '나뉘어 떨어지지 않는 두 쪽을 가지는 것이 아니라 똑같이 나뉘어 떨어지는 두 쪽을 갖는 수'라는 것이다. 즉, 자체적으로 둘로 나뉠 수 있는 수라는 것이다. 2는 그 자체로 짝수이기 때문인데, 그렇지만, 이 차이는 짝수의 현존에 속하는 필연적 귀결을 표현하는 것이지, 짝수의 구조 속에서 하나의 요소를 표현하는 것은 아니다. 정의는 정의될 것이 반드시 '요소들로 분석될 수 있는 복합적인 것'이어야 한다는 것을 시사한다는 가정은 『에우튀프론』이라는 글 위에 정의에 대한 아리스토텔레스주의자의 설명을 투영하였음을 표현한다. 이러한 투사는 실수이다.

아리스토텔레스적 유(類)들

1) '유'와 '종'의 일상적 함의는 아리스토텔레스적 함의이다. 아리스토텔레스가 생각하였던, 그의 가정들로부터 유래된 논리학적 전통의 침전물은, 유들은 내용에 있어서 종들보다 빈약하고, 그 추상적 계층에서 위로 향하는 이행은 공허를 향하는 이행이라는 것이다. 포르피리우스의 계통수에서 각각의 종은 최근, 하위, 그리고 최상위[summum] 유들을 수반하는데, 왜냐하면 그것들이 그것의 정의에 포함되기 때문이다. 그러나 어떠한 유도 그 유 아래에 들어가는 종을 수반하지 않는다. 그래서, 사람은 동물을 수반하지만, 동물은 사람을 수반하지 않는다; 동물의 경우 생물을 수반하지만 그에 의해 수반되지는 않는다. 유는 종보다 더 적은 표지(標識, merkmal. de. pl.-e)들, 더 적은 특징들을 가진다; 그것은 개념적으로 더욱 빈약하다.

2) 상승은 공허를 향하고, 하강은 충족을 향한다. 유에 대한 종의 관계에 대한 이런 설명은 종종 논리학의 중립적 문제로 다루어졌다. 그러나 그것은 어떤 식으로도 중립적일 수 없는 절차인데 그것이 다름 아니라 형이상학을 포함하는 것으로 나타나게끔 만들어져있기 때문이다.

3) 종종 아리스토텔레스가 어느 날 그가 사는 세상을 바라 보았고 개별자들을 발견했다고 가정된다 - '실재에 대한 강렬한 감각'이라 불린 어떤 것을 지니고 있는 철학자들의 웅장한 노선에서 첫번째로 말이다. 어쩌면 그럴 것이다. 그러나 그가 오히려 자연적 종류들을 - 소크라테스가 아니라 사람에 대한, 짐말이 아니라 말에 대한 우선함을 - 찾고 있었고 발견했다는, 그리고 그가 그러한 종류들에 속하는 것들인 소크라테스와 짐말과 같은 그런 개별자들의 존재론적 우선성을 지지하도록 이끌렸던 것은 자연적 종류들에 대한 이러한 발견을 통해서라는 증거가 있다. 이러한 발견을 향한 도정은 플라톤만이 아니라 스페우시푸스도 포함한 아카데메이아에서 정의를 둘러싼 논란의 가시덤불을 통과했다. 그것은 제 2 실체의 구별적 표시 - 즉, 실체의 범주에 있어서 술부들 - 는 이름과 정의에 있어서 서술할 수 있는 것들이라는 『범주들』의 원칙이 되었다. 이러한 규칙, 기초적 실재들의 본성을 표현하는 용어들을 골라낼 수 있도록 하게 되어있는 그 규칙은 존재론에 묶인다.

제 2 실체에 속하는 종은 유보다 더욱 실체적인데, 왜냐하면 종은 제 1 실체에 더욱 가깝기 때문이다. 만일 제 1 시체에 대해 그것이 무엇인지 말하는 것이라면, 그것은 더욱 많은 정보를 주고 유보다 종을 부여하는 경향이 더 많은 것일 터이기 때문이다. 예를 들어 개별적인 사람에 대해 그가 사람이라고 말하는 것이 그는 동물이라고 말하는 것보다 더욱 많은 정보를 줄 것이다(왜냐하면 하나는 개별적 사람에 대해 더욱 특징적인 반면, 다른 하나는 더욱 일반적이기 때문이다); 그리고 개별적 나무에 대해 그것이 나무라고 말하는 것이 그것이 식물이라고 말하는 것보다 더욱 많은 정보를 줄 것이다. 나아가, 제 1 실체들은 다른 모든 것들에 대한 것이고 모든 다른 것들은 그 제 1실체들에 대해 술부일 수 있거나 그 제 1 실체들 속에 있기 때문에, 그 제 1실체들은 무엇보다도 실체들로 불리는 것이다. 그러나 제 1 실체는 다른 것들에 대해서 자리하는 것처럼, 그렇게 종도 유에 대해 자리한다: 종은 유에 대한 것이다(유들은 종에 대해 서술되지만, 종은 유들에 대해 상호적으로 서술되지 않기 때문에). 그러므로 이러한 이유로 마찬가지로 종은 유보다 더욱 실체적이다.

이러한 이행은, 철학사에서 처음으로, 일반화적 공허의 원칙으로 향한다. 그것은 또한 그것의 토대를 해명한다. 아리스토텔레스는 유에 대한 종의 관계를 술어적인 것으로, 그리고 술부를 비대칭적으로 이해한다: 유는 그것의 종에 대해 서술 가능하지만, 종도 종차도 유에 대해 서술 가능하지 않다: 사람은 동물이지만, 동물은 사람도 아니고 이족보행동물도 아니다. 유들이 종에 대해서 종이 개별자들에 대해 서는 것처럼 서기 때문이고, 개별자들은 종들보다 더욱 실체적이고, 종들은 유보다 더욱 실체적이기 때문이다. 소크라테스나 코리스쿠스처럼 그것들도 그에 대해 서술가능한 제 1 실체에 대해 대조적으로, 설령 종들과 유들이 둘 다 제 2 실체라 할지라도 말이다. 이 모든 것을 인정하면, 유들의 결핍이 즉각적으로 따른다: 상대적으로 덜 실체적인 유들은 상대적으로 더 실체적인 종들을 거의 포함할 수 없다시피 하다. 논리적으로 놓고 봤을 때, 유들은 종들을 수반할 수 없는데 왜냐하면 종차들이 그 종들의 유들에 대해 비본질적이어야만 하기 때문이고, 그 점에서 그 종들은 그것들의 유들에 대해 서술 가능하지 않다: 그 유는 그것의 종을 수반하지 않는데 왜냐하면 그것은 그 종의 분석에 있어서 오로지 한 요소일 따름이기 때문이다. 유의 종에 대한 관계에서 술어의 비대칭성은 수반의 비대칭성을 수반한다.

4) 『범주론』은 초기작이고, 아리스토텔레스의 어휘는 그의 정신이 성장에 따라 변화했다. 제 1 실체와 제 2 실체 사이의 대립은 후기에 폐기되었다; 제 1 실체는 질료, 꼴, 그리고 그것들의 합성으로 분해되어 들어갔다; 제 2 실체는 실체적 꼴과 보편적 꼴로 분해되었다. 그러나 유들이 종들보다 더욱 결핍되어 있다는 전제는 남았다. 『형이상학』에서, 은유로써, 유들은 종들에 대한 질료로서, 나아가 종차들이 상대적으로 꼴로서 서는 한정에 대한 잠재력이 된다.

5) 이러한 주장 또는 그와 비슷한 어떤 것은 가장 나중의 생각을 보편개념들에 채색하였다. 그것은 논리적 주어들로서, 이야기되는 것들에 속하는 우선성이고 어떤 것에 대해서도 그 자체로 이야기되지 않는 것들의 우선성인, 개별자들의 우선성을 전제한다; 그리고 그것은 술부들의 파생물, 다른 것들에 대해 이야기되는 것들을 전제한다. 이러한 것들은 플라톤적인 전제들이 아니다.


플라톤적 유들

1) 아리스토텔레스의 유들에 대한 설명은 초기 대화편들에 맞지 않는다. 우선, 그 설명이 비록 소크라테스의 어휘를 유지하고 있다 하더라도, 그 설명은 소크라테스의 은유를 뒤집는다. 소크라테스는 종이 유의 부분이라고 주장한다. 그 유는 전체이다. - 이 종 부분에 다른 부분들을 더하여 포함하는 그러한 전체이다. 그러나 만일 유들이 종들보다 더 결핍된 것이라면, 그 종은 전체인 종일 것이고, 그 종에 대한 유와 종차는 각각의 부분들일 것이다. 아리스토텔레스는 아카데메이아에서의 정의에 대한 이론들에 내재하는 문제들에 대한 숙고로써 은유의 이러한 역전을 지지하는 데에로 이끌렸다; 그러나 소크라테스, 처음으로 부분과 전체의 은유를 도입했던 그가 그것을 뒤집힌 것으로 도입했을 성 싶지는 않다.

2) 결핍된 유들과 초기 및 중기 대화편들을 조화시키는 그 방식에는 두번째 어려움이 있다. 그 어려움은 그 유들이 소크라테스가 그 곳에서 처음으로 고려한 용어의 종류들에 맞지 않는다는 것이다. 포르피리우스의 계통수는 위로 갈수록 좁아진다. 사람의 동물에 대한 관계나 동물의 생물에 대한 관계는 이것으로 충분하다. 그러나 짝수의 수에 대한 관계나 도형에 대한 원의 관계, 또는 탁월함에 대한 올바름의 관계에는 충분치 못하다. 왜냐하면 수의 현존이 짝수의 현존을 수반한다고 추정하는 것은 타당하고, 도형의 현존이 원형의 현존을 수반한다는 것도 타당하며, 탁월함의 현존이 하나의 탁월함으로서 올바름의 현존을 수반한다는 것도 타당하기 때문이다. 일반화적 공허의 원칙은, 만일 그것이 어떤 것에든 어울리고, 말이나 개와 같은 자연적인 종류들에 어울린다. 그것이 아리스토텔레스가 그 원칙을 지지했던 이유이다. 그러나 그 원칙은 수학적 용어들, 또는 도덕적 용어들에 어울리지 않고, 그러므로 그것은 초기 대화편들의 변증에 어울리지 않는다. 정의에 대한 관념에 대해 전형적인 그 원칙을 상정하는 것은 개념들의 실제 구조를 무시하는 것이다; 그리고 유들에 대한 그러한 관점이 그렇게나 오래, 또 그렇게나 널리 받아들여졌어야 하다는 것은 인간 정신에 대한 잘못된 추상의 능력을 위한 서글픈 증거이다.

3) 대신에 유들이 충족되거나 결핍되는 만큼 정의에 있어서 종차의 역할은 변화할 것이다. 만일 후자라면, 종차는 종에 대한 궁극적 한정이 될 것이다: '마지막 차이는 사물과 그 정의에 대한 실체일 것이다.' 종은 부분들로서 그것의 유와 종차를 포함할 것이고, 그것들이 바로 그것인 어떠한 것으로 결합한다는 의미에서, 정의상으로 그 부분들과 동일할 것이다. 그러나 만일 유들이 그 유들의 종들을 부분들로서 포함하는 전체들로 간주된다면, 종차는 종차는 상이한 역할을 해야만 한다; 왜냐하면, 전체는 그 자체 그것의 부분들로 구성되기 때문에, 전체에 덧붙혀져 부분을 이룰 어떠한 한정도 없기 때문이다. 다른 방식으로 놓고 보자면, 종차는 그것을 수단으로 삼아 한 종이 동일한 유 속에서 다른 종들과 구별될 그러한 특징이 될 것이다; 그러나 그 특징은 그 종차의 구성요소인 것이라기 보다는 오히려 구별의 귀결일 것이다; 그것은 인식근거(ratio cognoscendi)일 것이고, 존재근거(ratio dssendi)는 아닐 것이다. 그래서, 예를 들어 둘로 나뉠 수 있음은 홀수와 짝수 사이의 본질적 차이이다; 그렇지만 그 차이는 홀수와 짝수 사이의 구별을 구성하지는 않고, 오히려 그러한 구별의 필연적 귀결이다.


한정가능성과 단순성

1) 어느 초기 대화편에도 탁월함에 대한 만족스러운 정의는 아무것도 없다. 이로부터 종종 암시되어 온 것은 실재적 정의가 어렵다는 것이 아니라, 그건 말할 것도 없고, 그 정의가 불가능하다는 것이다: 원칙적으로 꼴들은 정의될 수 없다.

2) 만족스러운 정의들이 결론에 도달되지 않는다는 사실로부터 그 정의들이 달성될 수 없다고 추론하는 것은 논리적 오류이다; 만일 플라톤과 소크라테스가 꼴들은 정의될 수 없는 것이라고 믿었다면, 어째서 그들이 꼴들이 있다고 전제되는 변증적 탐구 방법을 지지해야 했는지 그 이유도 설명될 수 없는 것이다. 만일 꼴들이 정의될 수 없다면, '그것은 무엇인가?'라는 물음은 원칙적으로 대답될 수 없다. 즉, 그것은 말도 안 되는 물음이다. 하지만 그러면 어째서 그것이 물어져야 했는가? 어째서 소크라테스는 『에우튀프론』14c에서 그것이 답해질 수 있다고 암시해야 하는가? 그리고 왜 그는 그 물음이 대답되기 전까지는 예를 들어 ποῖόν τι에 속하는 물음들과 같은 다른 여러 물음들에 대한 대답들이 알려질 수 없다고 생각했어야 하는가? - '탁월함은 무엇인가?'라는 물음은 '탁월함은 가르쳐질 수 있는가?'라는 물음에 앞선다는 것이다. 초기 대화편의 몇 안 되는 주장들은 꼴들이 정의될 수 없다는 주장으로서 아주 좋지 못하게 놓인다.

3) 그러나 만일 꼴들이 정의될 수 있다면, 꼴들의 정의가능성을 그것들의 단순성과 조화시키는 일에 대한 문제가 남는다.

4) 아리스토텔레스를 괴롭혔던 정의의 통일성에 대한 문제를 고려해 보자. S라는 종이 있다. 그 종은 G라는 유와 D라는 종차로 정의될 것이다. 즉, S는 G와 D이고, '~는~이다'는 한정적 동일성에 속하는 '~는 ~이다'인데, GD는 S인 바로 그것이기 때문이다. 그러나 이것은 역설을 낳는다: 정의될 것에서는 언급된 하나가 있고, 정의함에 있어서는 둘이 있기 때문이다. 그러나 어떻게 하나가 둘과 동일할 수 있는가? 어떻게 S는 B와 D일 수 있는가?

5) 만일 정의를 고려함에 있어서 단지 명사적일 따름이라면, 이 문제는 흥미로울 것도 없을 것이다. 그 '~는 ~이다'는 동의어에 대한 '~는 ~이다'일 것이고, 한 복잡한 표현이 그 표현에 대해 상대적으로, 언어적으로 더 단순한 또 다른 표현과 동의적이지 않아야 할 이유는 없다. 그러나 여기에서 고려 중인 정의는 명사적인 것이 아니라 실재적인 것이고, 그 정의가 산출한 역설도 또한 실재적이다. 아리스토텔레스는 『형이상학』에서 그 역설을 존재하는 것은 가능성과 현실성 사이에서 이중적이라는 그의 원칙으로써 해결하고자 시도했다. 종과 유 그리고 종차는 존재하는 모든 것들이다; 그렇지만 유는 그 자체로 오로지 한 종류의 질료나 가능성, 그에 대해 종차가 상대적으로 현실성으로서 그리고 꼴로서 마주서는 그러한 것이다.

6) 이런 해법의 이점이 뭐든지 간에, 그건 플라톤의 초기 대화편들에는 부적절하다. 하나의 것인 꼴은 그것과 구별되는 다른 어떤 꼴과도 또는 일련의 꼴들과도 동일시될 수 없고, 그러므로 정의는 아리스토텔레스가 의도한 그런 종류의 동일성으로 표현할 수도 없다. 소크라테스는 한 탁월함에 대해 '그것은 무엇인가?'라는 물음을 던지는 일이 그 탁월함의 내적 존재가 복잡하고, 탁월함의 유를 포함하며, 그리고 그 복잡성 안에서 요소들로서 차이를 포함한다는 것을 수반한다고 일관성 있게 생각할 수 없었다. 정의에 대한 관계물, 종들은 단순한 단일체들이고, 정의의 단일성 문제는, 최소한 아리스토텔레스가 제시했던 것처럼은 일어나지 않는다.

7) 그러나 또 다른 문제가 생기지는 않는지는 물어볼 만한 일일 것이다. 만일 종들이 단순한 단일체들이라면, 도대체 어떻게 그것들은 정의될 수 있는가? 이러한 어려움은, 비록 널리 논의되어 왔다 할지라도, 현실보다 더욱 분명하다. 단순성과 복잡성은 상대적 용어들이다. 그 어떤 것도 완전히 단순하거나 완전하게 복잡하지는 않다; 그것은 주어진 국면들에서 단순하거나 복잡하고, 다른 측면들에서는 다른 식일 것이다. 만일 탁월함이 경건을 부분으로 하는 하나의 전체라면, 그리고 경건이 경건에 마주서는 탁월함으로서 그 어떠한 더 하위의 꼴에도 마주서지 않는다면, 부분들을 가진다는 측면에서 탁월함은 복잡하고 경건은 단순하다. 그러나 만일 종들의 꼴들이 이러한 측면에서 단순하다면, 그 꼴들이 모든 측면에서 단순하다는 것은 따라나오기 어렵다; 꼴들의 단순성을 확립시키는 바로 그 논증은 그 꼴들이 다른 꼴들과 부분과 전체, 종차, 수반, 양립가능성의 관계 속에 정립한다는 것을 시사한다. 꼴들은 창 없는 단자들이 아니고, 그것들에 대해 설명이 제시될 수 있다. 그 꼴들은 그것들을 포함하는 더 넓은 꼴들에 대한 그것들의 관계를 추적함으로써 정의할 수 있다. 유들인 그러한 더 넓은 꼴들은 부분들을 가진다는 측면에서 복잡하다; 그러나 다시 한 번, 유들이 이러한 측면에서 복잡하다는 사실로부터 그것들이 모든 측면에 있어서 복잡하다는 것이 뒤따르진 않는다; 그것은 아마도 유들이, 더 상위의 유들에 의한 그것들의 포함이라는 측면에서, 전체들이 그것들의 부분들의 합산보다 더 큰 그러한 전체들로서, 단순하다는 것일 터이다.


꼴들의 영역의 구조

1) 만일 종의 유에 대한 관계가 꼴들의 구조에 기초된다면, 그 구조는 반드시 계층적이어야만 한다.

2) 이것은 널리 받아들여지는 관점이다; 그러나 그 관점은 반대에 부딪쳤다. 예를 들어 체르니스 교수는 그것이 틀렸다고 주장한다:

다른 어떤 것에 대해서도 전체에 대한 구성부분으로서 또는 종에 대한 유로서 어떠한 이데아도 없다. …플라톤은 어디에서도 이데아들 사이에 유와 종의 구별을 만들지 않는다; 그러나 아리스토텔레스가 유라고 부른 것, 종차라고 부른 것, 그리고 종이라고 부른 것은 그에게 모든 별개의 관념적 단일체들이다, 각기 다른 것들의 현존을 수반하거나 다른 것들과 양립 가능한 측면들을 지닌 다른 것들 보다는 가기 다른 것들, 그러나 남김없이 다른 것들로 분석될 수는 없는 독립적 본성인. 플라톤은 그래서 분절을 사용하여 이데아들의 세계에서의 존재론적 계층을 산출할 생각을 할 수는 없었다. 『소피스트』와 『정치가』, 분절에 대한 안내서로 여겨지게 된 그 두 저술들은 그가 그 분절을 가지고 오히려 발견적 학습법을 의미했고, 한정된 이데아를 탐색함이 용이하도록 하는 수단을, 다른 이데아들로부터 그 이데아를 구별하는 것을, 그리고 그것의 함축들과 동일성을 의미했다는 것과 그가 그 분절을 가지고 이데아, 이데아의 기원, 혹은 그것의 구성 요소들의 '구조'를 묘사하는 것으로 상상하지 않았다는 것을 보여준다.

체르니스 교수는 일부 꼴들이 어떠한 부분들도 가지지 않는 것으로서 단순하다는 것뿐만 아니라, 모든 꼴들이 이러한 방식으로 단순하다는 것을, 그리고 꼴들의 영역이 균일하다는 것을 또한 주장한다. 그의 의견들은 후기 대화편들을 겨냥하고 있고, 명확하게 『소피스트』에서와 같은 종류의 집단에 속하는 원칙을 겨냥한다; 그러나 그것들은 초기 대화편들의 해석으로서도 잘 맞는다.

2) 이러한 관점에 반대되는 그 저술들은 '그것은 무엇인가?'라는 물음에 대한 답이 유나 혹은 그에 대해 꼴이 검토 아래에서 일부인 그러한 전체를 명시할 것임을, 또는 그 꼴이 부분들을 포함하는 전체임을 진술하거나 시사하는 모든 저술들이다. 그러한 저술들은 : 『에우튀프론』 11e-12a, 『프로타고라스』 329c와 그 이후, 『메논』 74-6, 『라케스』 190c-d, 199e이다. 이러한 구절들 모두 유 아래에 종을 포함하는 가능성을 암시하고, 그래서 꼴들의 계층적 배열을 암시한다. 그것들은 꼴들의 영역이 균일하지 않고 단층화되어 있음을 시사한다.

3) 체르니스 교수는 이러한 저술들, 그리고 후기 대화편들의 다른 저술들이 그 저술들의 의도가 발견적이라는 점을 제쳐두고 세워진다고 제안한다: 그 저술들은 형이상학적이라기보다는 방법론적으로 전제를 반성한다. 그러나 탐구의 방법은 그 탐구의 핵심-문제의 본성에 적합한 것이거나 적합해야 하는 것이다. 그리고 만일 꼴들이 상호에 대해 종과 유처럼 자리한다고 가정하는 것이 발견의 의도에 유용하다면, 어째서 이것이 그러한지에 대한 설명의 가능성에 대해 책임이 있다. 확실한 대답은 그 전제가 꼴들의 실재적 구조에 답한다는 것이다. 체르니스 교수는, 이러한 대답에 반대하면서, 그 대신에 아무것도 내놓지 않는다. 하지만 그 거부는 방법론과 설명을 요하는 반대 사이의 동일척도불가능성을 시사한다: 사실상 아무런 유들도 그리고 어떠한 계층도 없을 때, 어떻게 꼴들이 유들 아래에서 포함가능한 것들이고 발견적 기능을 제공하도록 계층적으로 배열된다는 전제가 가능한가?

4) 더 큰 어려움도 있다. 초기 대화편들에서, 플라톤은 ποῖόν τι의 물음보다 τί의 물음이 우선한다거나 οὐσία와 πάθος 사이에 구별이 있다고 주장한다. 그러나 만일 체르니스 교수가 제안하듯 오로지 꼴들 사이에서의 관계들만이 차이, 포함, 양립의 관계들이라면, 이러한 구별은 도출될 수 없다. 왜냐하면 그래서 만일 꼴 F가 그것의 함축들과 양립들에 대해 동일하지 않다면, 'F는 무엇인가?'라는 물음은 답해질 수 없기 때문이다: F는 오로지 함축들과 양립들의 관계들에 의해서만 특징지어질 수 있고, 그러한 관계들 중 특정한 것들이 다른 것들보다 더 본질적인 것임을 뒷받침할 어떠한 근거도 없다. 그러므로 οὐσία를 πάθος로부터 구별하기 위한 어떠한 근거도 없다. 다른 한편으로, 만일 F가 그것의 포함들과 양립들에 대해 동일시된다면, 후자는 그것이 F일 그러한 것이 될 것이다; 그리고 만일 그것들 중의 어떠한 것들이든 그것들이 그것들인 것보다 결핍하고 있거나 다른 것이라면, F는 그것이 그러한 것인 바의 것이 아닐 것이고, 그러나 그것이 그러한 것인 바의 것과는 다른 것일 것이다. 즉, 동일성은 '내적 관계들'로 이끈다. 그리고 양립가능성과 함축이 모든 각각의 꼴을 다른 모든 각각의 꼴들에 관련시키기 때문에, F는 본질적으로 다른 모든 각각의 꼴과 관련될 것이다: 진리는 전체일 것이다. 스페우시푸스는 나중에 이러한 단계에 매우 근접하게 나아갔던 것으로 드러난다; 그러나 플라톤은, 최소한 초기 대화편들에서는, 그렇지지 않은데, 왜냐하면 πάθος와 οὐσία 사이에 구별이 있다는 것을 그 단계는 부정하기 때문이다.

5) 만일 '그것은 무엇인가?'라는 물음이, 소크라테스가 물었고 또 그것이 대답되는 것을 보려고 하였던 것처럼 물어지고 또 대답될 것이라면, 그리고 만일 ποῖόν τι의 물음들로부터 구별이 유지될 것이라면, 꼴들 사이에서 포함들과 양립가능성 그리고 그것들의 부정들의 관계들 보다 더 나아간 어떤 관련이 요구될 것이다. 그러한 관련의 본성은 『메논』에서 올바름은 탁월함 자체가 아니지만 하나의 탁월함이라는 주장에 의해, 『라케스』에서 용기는 전체로서의 탁월함에 속하는 한 부분이라는 주장에 의해, 『에우튀프론』에서 경건은 올바름의 한 부분이라는 주장에 의해 제시된다. 그 관련은 종과 유 사이의 관련이다: 한 부분의 본성을 이해하기 위해, 우리는 반드시 그 부분의 위치를 그것을 한 부분으로 하는 전체 속에서 정립해야만 한다.


탁월함의 단일성

1) 탁월함들이 유 속에 그것들을 정립시킴으로써 한정가능하다는 관점은 또 다른 초기 대화편 『프로타고라스』와 충돌하는 듯 보인다. 그곳에서 소크라테스는 탁월함이 하나라고 주장한다. 이 명제는 플라톤의 생애 마지막에 그의 생각을 차지한 것이었고, 그것은 종종 탁월함들이 동일하다는 의미로 받아들여졌다. 특히, 『프로타고라스』의 교훈은 모든 탁월함들은 그것들이 앎이나 지혜로 환원될 수 있다는 점에서 동일하다는 것으로 생각되어 왔다. 그리고 탁월함이 앎이라는 주장은 탁월함에 대한 소크라테스적 정의로 고수되어 왔다. 솔직히, 그것은, 종과 유의 비동일성을 시사하는 최근류와 종차를 통한 정의가 아니다.

2) 『프로타고라스』에서 앎의 우선성은 단호하게 내세워진다(352b-c):

대부분의 사람들은 앎이 명령의 힘도 권위도 능력도 없다고 생각한다; 앎을 가질지는 모르지만 그 앎은 제압당할 수 있다고 생각한다 - 분노에 의해, 그리고 기쁨과 고통에 의해, 가끔은 사랑에 의해, 그리고 자주 두려움에 의해서 - 비록 앎이 어떤 가난한 노예에게 있다고 할지라도, 격정들에 의해 제멋대로 끌려다닐 것과 같이 말이다. 그것이 당신의 관점 아닌가, 프로타고라스? 아니면 당신은 오히려 앎이 아름다움과 능력, 그리고 그 무엇도 꺾을 수 없는 것에 속하는 어떤 것이라는 데에 동의할 텐가; 나쁨으로부터 좋음을 구별해 내기만 하면, 지상의 그 무엇도 그러한 앎을 가진 자를 그 앎 - 그의 조력자로 충분한 지혜 - 과 반대로 행동하도록 강제할 수 없다는 것에 동의하겠는가?

이 구절은 의심할 나위 없이 소크라테스 그 자신의 믿음을 대변한다: 탁월함은 앎이다. 그 누구도 제 뜻으로 또는 알면서도 악을 행하지는 않는다는 점에서 말이다.

3) 만일 앎이 탁월함을 수반한다면, 그리고 탁월함이 앎을 수반한다면, 그렇게 함으로써 탁월함과 앎이 하나이고 같은 것이라는 점이 따르지는 않는다. 동치는 동일성이 아니다. 그러나 이러한 동치는 더 넓은 논증의 부분으로 나타난다. 그 논증에서 소크라테스는 프로타고라스가 탁월함들이 동일한 것에 대한 모든 이름들이라는, '용기', '올바름', '사려', '경건', '지혜', '탁월함'은 동의어들이라는(349b-c) 점을 받아들이도록 이끌고자 시도한다. 이러한 관점에서, 『프로타고라스』가 탁월함들이 동일하다는 것, 그리고 앎으로 정의될 수 있다는 것을 의미한다고 해석되어야 한다는 것은 놀라운 일이 아니다.

4) 하지만 아직 이러한 방식에는 어려움들이 있다. 우선, 그 설명이 탁월함과 앎을 동일시한다 할지라도, 그것은 능력의 동일성을 부당하게 취하는 것이다. 만일 그 탁월함들의 명칭들이 동의어들이라면, 그 탁월함들이 지혜로 화원될 수 있다고 이야기될 수 있는지 어떤지, 지혜를 포함한 그 탁월함들이 경건이나 올바름이나 다른 어떤 것으로 환원될 수 있다고 이야기될 수 있는지 아닌지, 그것은 임의적이다. 물론, 탁월함은 앎이나 지혜이다; 그러나 그래서, 탁월함은 용기나 경건, 또는 당신이 원하는 무엇이든, 동의어에 대해 대칭적인 관계이다.

5) 그 동일화 방식에는 두번째 어려움도 있다. 소크라테스는 『프로타고라스』에서 탁월함들이 구별된다는 프로타고라스의 주장에 반대하여 네 가지 논증들을 내놓는다. 첫번째(330b-331b)는 올바름과 경건의 단일성을 주장한다; 두번째(332a-333b)는 사려와 지혜의 단일성을; 세번째(333d-334a)는 그 결론에 도달하지 않고 중단되는데, 어쨌든 올바름과 사려의 단일성을 주장할 것이었다. 거기에서 남은 대화편을 통해 중단 없이 추적되는 그 물음이 다시 말해진 다음(439b-d) 긴 중간극이 따른다. 마지막 논증에 있어서 소크라테스는 용기와 지혜의 단일성을 증명할 것을 자임한다(349d-360e). 만일 탁월함들의 단일성이 탁월함들의 동일성을 의미하는 것으로 취해진다면, 이러한 논증들 각각은 논리적 오류이고, 그것들 중 일부는 노골적으로 그리고 창피스럽도록 오류를 범한 것이다. 이것은 그 문답의 고비를 넘긴다. 프로타고라스는 지적인 사람으로서 도처에서 묘사되고, 말들을 다루는 방식들에 있어서 지혜롭지만, 소크라테스의 논증들을 다룰 줄 모른다. 그의 실패는 물론 말의 단련에 있어서 기술 부족 탓이 아니다; 그것은 소피스트에게서 기대하기 어려운 일이다. 그것은, 비록 그가 탁월함을 가르치리라 주장할지라도, 탁월함이 무엇인지 모른다는 그 사실에 기인한다. 소크라테스의 논증들은, 만일 그 논증들이 공허한 오류들에 불과하다면, 이러한 결론을 가리키는 쪽으로 그 논증들이 행한다고 의미되는 것으로서 이바지하기 어려울 것이다.

6) 다른 초기 대화편들에서와 마찬가지로, 『프로타고라스』도 실패로 끝나고, 그것은 익숙한 이유 때문이다: 소크라테스와 프로타고라스는 먼저 탁월함이 무엇인지 결정함이 없이는 탁월함이 가르쳐질 수 있는지에 대해 동의하지 않는다. 이 점은 추론할 것으로 남겨지지 않는다: 그것은 그 문답의 변증에 대한 명쾌한 요약정리를 되보여준다 (360e-361c):

나는 이렇게 말했다. 나는 다른 어떤 이유도 아니라 그것이 어떻게 탁월함에 관련한 것들을 가지고 바로 그것인지, 탁월함이 그 자체로 무엇인지 밝혀내려는 바람 때문에 이러한 것들을 묻는다. 왜냐하면 나는 만일 그것이 한 번만 보여진다면 당신과 내가 논의한 물음, 탁월함이 가르쳐질 수 있는지 아닌지가 말끔하게 정리되리란 것을 알기 때문이다. 그렇기 때문에, 그 논증의 결과는 그것이 사람이었던 것처럼 우리를 조롱하고 비난하는 것처럼 보일지도 모른다; 만일 그것이 혀를 가졌다면, 아마도 이렇게 말할 것이다. '당신들 둘 다 뭐 이런 바보들이 있나, 소크라테스와 프로타고라스. 분명히 탁월함이 가르쳐질 수 없다고 말한 소크라테스 당신은 지금 바로 그 반대의 것을 설득하려 들고 있다, 모든 것 - 올바름도 사려도 그리고 용기도 - 은 앎임을 보여주려고 애쓰면서, 그걸 보여줌으로써 탁월함이 가르쳐질 수 있다 는 것이 따라나오는 것을 말이다. 만일 그것이 프로타고라스가 주장하려 애쓰는 것처럼 앎과 다른 것이라면, 분명히 그것은 가르쳐질 수 없다; 그러나 이제, 만일 그것이 당신 주장처럼 전적으로 앎일 것이라 증명하는 것이 된다면, 소크라테스, 만일 그것이 가르쳐질 수 없다면 더욱이 이상한 일일 것이다. 다른 한편으로 여기에서 프로타고라스가 조금 전에 탁월함이 가르쳐질 수 있다고 전제했는데 지금은, 다름 아닌 앎을 피함으로써, 그것이 무엇보다도 최소한 가르쳐질 수만은 없을 것이도록 그렇게, 그 반대를 주장하는 것처럼 보인다.' 자, 프로타고라스, 내가 주장된 이 모든 것을 앞뒤로 볼 때, 나의 바람은 그것을 말끔하게 정리하는 것이오: 나는 '탁월함이란 무엇인가'라는 물음을 향해 진격해야 할 것 같고 또 공격해야 할 것 같소. 그리고 나서 다시 되돌아와 그것이 가르쳐질 수 있을지 없을지를 물어야 할 듯 싶소.

『프로타고라스』의 결론에서, 소크라테스와 프로타고라스 모두 탁월함이 가르쳐질 수 있는지에 대한 물음에 대해서 자신들의 입장들을 뒤집어 버렸다. 이러한 뒤얽힘은 변증적 반어법의 단편이다. 그 목적은 탁월함이 무엇인지 밝혀내는 일의 필연성을 변호하는 것이 아니라 그것을 드러내 보이는 것이다.

7) 『프로타고라스』는 『메논』에서 그 짝이 되는 조각을 가진다. 소크라테스가 메논에게 탁월함이 가르쳐질 수 있는지에 대해 질문을 받을 때, 그는 전제를 가지고 그 문제를 정착시킬 것을 최종적으로 떠맡는다. 그는 만일 탁월함이 앎이라면 그것은 가르쳐질 수 있다(87a-c)고 가정한고; 그것이 앎이라고 주장한다(87c-89a); 그리고 그것은 가르쳐질 수 없는데, 왜냐하면 그에 대한 선생들이 없기 때문이라고 주장한다(89c-96c); 그리고 그것이 '다이몬적인 분배에 의해, 이해 없이' 사람들 속에 현존한다고 결론내린다(100a). 즉, 소크라테스는 함축을 전제하고, 그 함축의 선행하는 것을 긍정하고, 그것의 결론을 부정하며, 회의주의로 끝을 맺는다. 이러한 구조는 플라톤이 그의 독자들을 그의 전제들 사이에서 선택하도록 할 작정이었다면 무의미할 것이다: 문제의 핵심은 어디에나 놓여 있다. 소크라테스와 메논은 탁월함이 무엇인지 먼저 밝혀내지 않고 탁월함이 가르쳐질 수 있는지 아닌지에 대한 물음을 해결하고자 시도했다. 그들은 아포리아(당황)로 끝을 맺었고, 심지어 기하학자들로부터 빌려온 새로운 가정의 방법조차 그들을 구해내지 못한다(86e-87a). 그러나 또, 기하학자들은 그들의 용어들을 정의할 수 있다; 소크라테스와 메논은 할 수 없다. 이것은, 그래서, 그들의 실패의 교훈이다. 정의는 필수적이라는, 이야기하기 위해 이야기하는 중인 그 어떤 것을 아는 것은 좋은 일이라는 익숙한 소크라테스적 교훈이다. 그래서 『메논』은 또한 변증적 역설의 한 조각이다: 그 극적인 구조는 정의가 필수적임을 선언하는 것이 아니라 드러내 보이는 것을 의미하게 된다. 그것은 탁월함이 앎이라는 것과 같은 그런 도덕적 결론들을 권고하는 것에 관심이 두어지는 것이 아니라, 그 아래에서 그러한 결론들만이 달성될 수 있는 그러한 조건을 지시하는 데에 관심이 두어지는 것이다.

8) 만일 이것이 『프로타고라스』와 『메논』에 대한 옳은 설명이라면, 『프로타고라스』의 전하고자 하는 바가 탁월함과 앎의 단일성이라는 관점에 반대하여 강력하게 영향력을 끼치는 것이다. 그 둘 중 어떤 대화편도 이러한 문제에 헌신하지 않고, 두 대화편 모두 그 대화편들의 변증적 중심점으로서, '탁월함이란 무엇인가?'라는 물음에 답하는 일의 필수불가결함 - 즉 그것을 정의하는 일의 필연성 - 을 강조하며 결론에서는 그 물음이 대답되지 않은 채로 남겨졌다는 것을 주장한다.

9) 『프로타고라스』에서 탁월함의 단일성에 관한 문제에 대한 플라톤의 설명이, 그 방식에서 탁월함이 하나일 그러한 대안적 방식들을 예상한다는 것은 보여주어야 할 것으로 남는다. 프로타고라스는 다음과 같은 명제들에 동의하도록 이끌린다 (329c-330b): 탁월함은 올바름, 사려, 경건, 지혜, 용기라는 여러 부분들과 함께 하나의 전체이다; 이러한 부분들은 구별되고, 같은 것에 대한 다른 이름들이 아니다; 이러한 부분들은 오로지 크기에서만 차이가 나는 황금의 일부 같은 부분들이 아니고, 두 눈과 두 귀 처럼 얼굴의 부분들과 같은 부분들이다. 그것들은 그 자체로서나 그것들의 능력이나 기능에 있어서가 아니라(οὔτε αὐτὸ οὔτε ἡ δύναμις αὐτοῦ, 330b) 서로 그러한 종류의 것(οἷον)에 속하는 것들이다; 탁월함의 그 부분들은 구별될 뿐만 아니라, 사람이 용기 있으나 올바르지 못하거나 올바르되 지혜롭지 못할 것이 가능하고, 그래서 그 사람은 탁월함 전부를 가지지 못하고서도 탁월함들의 일부를 가질 것이라는 점에서, 분리될 수도 있다.

10) 다음으로 프로타고라스는 탁월함이 여러 부분들과 함께하는 하나의 전체임을 긍정함에 있어서, 복합된 반박을 내놓고 있다: 그는 탁월함의 상이한 부분들이 그것들 각각의 그 자체에 있어서 동일한 것들임을 부정하고 있다; 그는 또한 그 부분들이 능력이나 기능에 있어서 동일하다는 것도 부정하고 있다. 그래서 탁월함들이 하나일 두 가지 방법들이 있다.

11) 탁월함들이 그것들 각각의 그 자체에 있어서 다르다고 말하는 것은 아마도 그것들이 상이한 정의들을 가진다고 마하는 것일 터이다. 그렇지만 탁월함이 기능에 있어서 다르거나 같다고 말하는 것이 무엇을 의미하는지 분명하지 않다. 탁월함의 기능은 무엇인가?

12) 그런 물음은 『프로타고라스』 어디에서도 직접적으로 대답되지 않는다. 탁월함의 기능은 탁월함 그 자체가 아닌데, 왜냐하면 그것은 얼굴에 대한 예시를 무관한 것으로 만들기 때문이다: 두 눈과 두 귀는 그것들의 기능들인 보는 것과 듣는 것으로 차이가 난다. 다시, 두 가지 덕들은 그것들이 동일하다거나(ταὐτόν) 아주 유사하다(ὁμοιότατον)는 것조차 긍정할 것도 없이 서로 그와 같은 그러한 종류(οἷον)에 속한다는 것을 긍정할 수 있다. 그래서, 한 단어의 '기능'이 그 단어의 의미 또는 능력이라 할지라도, 그리고 올바름이 '올바름'의 의미라 할지라도, 올바름의 기능은, '올바름'의 기능으로부터 구별되는 것으로서, 올바름은 아니다.

13) 만일 탁월함의 기능에 대한 관념이 이해될 것이라면, 우리는 그 표현이 그 안에서 사용되는 그러한 논증으로 시선을 돌려야만 한다. 프로타고라스가 탁월함들이 그 자체로서나 그것들의 기능으로서나 동일하다는 것을 부정(330b)하는 근거, 그리고 나중에 그 탁월함들이 서로 간에 그와 같은 그러한 종류의 것들이라는 것을 부정(330e-331c)하는 근거는 탁월함들이 분리가능하다는 것이다(329e; 349d참조). 그리고 이것은 탁월함들이 서로 만나도록 한다는 것, 경건의 현존이 올바름의 현존을 수반한다는 것, 또는 용기의 현존이 지혜의 현존을 수반한다는 것을 부정하는 것이다. 그 추론은 명확해 보인다: 탁월함의 기능은 그 기능을 도입하는 어떤 것 속에 놓인다; 만일 탁월함들이 기능에 있어서 상이하다면, 그것들은 분리가능하다; 만일 분리가능하다면, 기능에 있어서 상이하다.

14) 이것은 그 논증의 양식에 걸맞는다. 프로타고라스는 탁월함들이 그것들 자체에 있어서도 그것들의 기능에 있어서도 모두 다르다고 주장했다; 이 명제는 만일 그 명제가 탁월함들이 그것들 자체에 있어서나 또는 그것의 기능에 있어서 동일하다는 것이 보여질 수 있다면 틀린 것으로 증명될 것이다. 만일 탁월함들이 정의에 있어서 하나라면, 그것들은 기능에 있어서도 하나이다. 그러나 만일 그것들이 기능에 있어서 하나라면 정의에 있어서도 하나라는 것이 뒤따를 방법은 없다. 사람이 올바름 없이 경건할 수 없다거나 경건함 없이 올바를 수 없다고, 또는 경건과 올바름이 서로를 도입한다고 말하는 것은 올바름의 정의와 경건의 정의가 같다고 말하는 것과는 다르다. 탁월함들은 만일 분리가능하다면 구별가능하다; 만일 탁월함들이 구별가능하다면, 그것들이 분리가능하다는 것이 뒤따르진 않는다. 만일 탁월함의 부분들이 황금의 부분들과 같은 그런 부분들이 아니라면, 그 자체에 있어서도 그것들의 기능에 있어서도 모두 상이한, 얼굴의 부분들과 같은 그런 부분들이라는 것이 뒤따르진 않는다. 즉, 탁월함의 특수한 경우에, 황금과 얼굴에 대한 그 예시들은 부적당하다는 것일 수 있고, 황금도 얼굴도 밝혀내지 못하는 정의에 있어서 차이와 기능에 있어서의 동일성이 있을 수도 있다는 것일 수 있다.

15) 그 논증에 대해서는 이 만큼이면 됐다. 그러나 그 변증적 상황은 여전히 더욱 복잡하다. 만일 소크라테스가 프로타고라스가 탁월함들이 분리가능하다고 추정함에 있어서, 사람들이 올바름 없이 단지 지혜롭거나 용감할 수 있다고 주장함에 있어서 틀렸다는 것을 보여줄 수 있다면, 그는 탁월함들이 그 자체에서도 그것들의 기능에 있어서도 다르다는 그 주장을 반박할 것이고, 탁월함들이 동일한 의미에서 하나임을 보여줄 것이다. 그러나 이것은 탁월함들이 어떤 면에서 하나인지를 보여주는 것은 아니다: 탁월함들이 정의에 있어서 하나인지, 아니면 오로지 그것들이 서로 도입하고 분리될 수 없다는 의미에서만 하나라는 것인지 말이다.

16) 『프로타고라스』는 이 물음에 대해 세심하게 고안된 모호함을 유지한다. 그것은 대안들 사이에서의 선택이 아니고, 프로타고라스 그 자신은 그 논증에 대한 그 대안들의 관계를 인지하지 못한다. 이것은 놀랄 것도 없다. 탁월함들이 정의에서 하나인지 기능에서 하나인지 혹은 양자에 있어서 하나인지 묻는 것은 탁월함이 무엇인지를 암시함으로써 묻는 것이다. 그 물음은 오로지 대화의 중국에서야 명확하게 된다(361b-c). 거기에서 소크라테스는 그가 고려하는 중에 내내 그것을 가지고 있었다고 시사한다. 그러나 프로타고라스는, 그가 탁월함은 가르칠 수 있는 것이라고 자신있게 주장했고(318a-319a), 그리고 탁월함이 무엇인지 알지 못했던 자로서, 소크라테스적 엘렝코스(논박술)가 바로 그것을 가르치기 위해 계획된 그러한 그 자신의 무지에 대한 확신을 갖기 전까지는, 탁월함의 본성으로 파고드는 탐구에 유익하게 착수할 수 없었다.

17) 요약하면: 『프로타고라스』는 탁월함들이 하나일 수 있는 두 가지 길을 예상한다 - 말하자면, 정의에 있어서의 동일성이나 같음, 그리고 기능에 있어서의 상호 함축 또는 같음이 그것이다. 이것은 그 주제에 대한 다른 대화편들의 논증 양식에 부합한다. 그에 대해 『메논』은 탁월한 예시를 제공한다:

우리는 일반적으로 이렇게 단언할 것이다: 사람에게 있어서, 다른 모든 것들은 영혼에 의존하지만, 영혼에 속하는 것들은 그것들의 좋음을 지혜에 의존한다고; 이러한 설명으로, 유익한 것은 지혜이고, 우리는 탁월함이 유익하다고 말했다. …그래서 우리는 탁월함이 지혜이고, 전체로서나 부분에 있어서나 그러하다고 말하고 있다.

이러한 논증은 지혜와 탁월함이 동일함을 보여주지 않는다; 그것은 사실상 탁월함들이 구분된다는 것을 전제한다. 그것은 오히려 지혜가 탁월함에 대해 필연적인 것이고, 그러므로 충분한 것이며, 조건이며, 다른 탁월함들을 수반하고 또 그것들에 의해 수반된다는 것을 보여준다.

18) 이후 『정체』에서 플라톤은 영혼에 대한 분석에 기초된 탁월함들에 대한 설명을 제공했다. 그 설명은 탁월함들의 단일성을 상호 함축을 통해 긍정한다. 영혼은 그 부분들 각각이 적합한 기능을 수행할 때 비로소 올바를 수 있다; 절제는, 각 부분이 지배해야 하고 따를 것에 동의할 때; 지혜는 이성적 부분이 그 적절한 기능을 수행하고 지배할 때, 용기는, 기백의 부분이 그 지배자의 명령에 있어서 훈련될 때 생긴다(『정체』Ⅳ, 442b-d). 그 탁월함들은 여기에서 명백하게 정의상으로 다르다; 그만큼 명백하게, 탁월함들은 기능에 있어서 하나이다 - 그것들은 서로를 도입하고, 서로로부터 따로 떨어져 존재할 수 없다. 용기는 지혜를 수반하고 그것에 의해 수반된다; 지혜는 올바름을 수반하고 또 그에 의해 수반된다; 올바름은 절제를 수반하고 그에 의해 수반된다. 소크라테스가 결론에 있어서 '우리가 지금 논증에 있어서 이 높이에 올랐으니, 내가 보기에, 산 정상으로부터 보는 것처럼, 탁월함의 단일한 꼴 하나가 있는 것으로 보인다'라고 언급해야 한다는 것은 놀라운 일이 아니다.

19) 탁월함의 단일성에 대한 소크라테스적 원칙은 그래서 탁월함이 여러 부분들과 함께 하나의 전체라는 주장과, 그리고 탁월함들이 최근류와 종차를 통해 정의될 수 있다는 주장과도 양립불가능한 것이 아니다. 그렇지만 그것은 부분들과 부분들 사이의 관계, 그리고 부분들과 전체 사이의 관계가 동시함축적이라는 것을 시사한다.


로고스와 우시아

1) 만일 앞서 말한 설명이 건전하다면, 초기 대화편들 내에서 그런 종류의 논증은 고려함에 있어서 실재적이고 명사적이지 않을 것이다: 그것은 한 대상에 대한 설명이지 한 단어에 대한 설명이 아니다.

2) 그 대상이 적어도 그것이 유와의 관계에서 존립하는 종일 때에는 단순하기 때문에, 아리스토텔레스가 정의될 것과 정의함 사이에서 통용된다는 것을 고수하였던 그러한 관련은 획득하지 않는다: 그 종은 결합된 부분들로서 그것의 유와 종차와 함께 완전히 동일하지는 않다. 그러나 이러한 종류의 같음이 통용되지 않는다고 말하는 것은 어떤 종류의 같음도 통용되지 않는다고 그렇게 부정하는 것은 아니다: 『에우튀프론』10d, 13에서 신들에 의해 사랑받는 것은 경건에 대한 정의로서는 거부되는데 왜냐하면 그 둘이 다르기 때문이고(ἕτερον; 10e, 9에서 ταὐτόν이라는실에 견주어 반대로), 이러한 거부는 15c, 2에서 반복된다(οὐ ταὐτόν … ἀλλ᾿ ἕτερα ἀλλήλων).

3) 여기에는 어떤 종류의 같음이 필요한가? 그것은 외연의 동일성, 또는 진리치 보존(salve veritate) 대체성이 아닌데, 설령 신들에 의해 사랑받는 그러한 것들 모두가 그리고 오로지 그러한 것들만이 경건하거나 경건할 수 있다 할지라도, 이것은 단순히 πάθος를 진술할 뿐 οὐσία를 진술하지 않기 때문이다. 그것이 개념적 동일성인 것도 아니다: 에우튀프론은, 경건을 생각함에 있어서, 그렇게 함으로써 경건의 정의를 생각하지는 않는다. 동의어의 같음도 아니다.

4) 정의는 대상에 대한 설명이다. 그래서 아마도 정의에 있어서 필요한 같음은 이를테면 서술적 동일성, 말하자면 샛별과 태백성의 동일성에 대한 것이어야 하거나, 또는 - 그 예시에서 필요한 경험적 우연성을 피하기 위해 -  일곱번째 정수와 네번째 소수의 동일성에 대한 것이어야 한다. 그러나 이러한 종류의 동일성은 두 가지 한정적 서술들이 동일한 대상에 의해 충족된다는 것을 시사하는 반면에, 실재적 정의에 있어서 정의될 것은 대상에 대한 서술이 아니고, 대상 그 자체이다. 그 비유는 만일 서술적 구절이 명사에 의해 대체된다면 더욱 근접하다: '금성은 샛별이다' 또는 '7은 네번째 소수이다'; 무엇보다도 유일한 서술로 정의를 생각하는 것이 타당하다. 그러나 이것도 오독이다. 지혜는, 의심의 여지 없이, 탁월함이다; 그러나 '…는 탁월하다'라는 말이 소크라테스를 서술하는과 같은 의미에서 '…은(는) 탁월함이다'라는 말이 지혜를 서술하한다고 주장하는 것은 잘못인데, 왜냐하면 첫번째 구절에서 그 관계는 유에 대한 종의 관계이고, 두번째 구절에서 그 관계는 한 특성에 대한 한 특성의 예시 관계이기 때문이다. 그것은 한정적 서술에 있어서 필요한 후자의 관계이다.

5) 아마도 정의에서 필요한 종류의 동일성은 그것 자체의 유(sui generis)라고 말하는 것이 가장 간단할 것이다: 그것은 어떤 것을 포함하는 유와 그것을 그 유 속에서 다른 종들로부터 지적해 내는 종차를 진술함으로써 그 종인 어떤 것을 말할 때 필요한 그런 동일성이다. 종들은 단순하기 때문에, 종차는 종들에 대한 구별의 구성요소는 아니지만, 그 구별의 결과이다; 정의에 있어서 종차는 사유근거이고, 존재근거가 아니며, 그러므로 정의에서 필요한 동일성은 대상들의 동일성이 아니다. 정의의 단일성 문제는 일어나지 않는다. 아마도 정의에 대한 이러한 관점을 위한 최선의 - 그리고 ὁρίζειν과 ὅρος의 대다수 함축에 부합하는 하나의 -  비유는 사상(寫像)의 비유이다: 정의한다는 것은 한 종의 위치를 그 종을 포함한 유 속에서 정하는 것이다. 만일 종이 농장이라면, 유를 규정함의 목표는 그 농장이 위치하는 마을을 지시하는 것일 터이다; 차이를 규정함의 목표는 농장의 경계들을 측량하는 것이 아니라, 오래 전 조사된 경계선들의 현존을 밝혀내는 것일 것이다. 정의에서 필요한 같음은 말하자면, 유일한 위치의 같음이다: 정의될 것은 정확하게 정의하는 것 안에서 사상된 그 위치를 점유한다.

6) 이러한 방식에서 생각되는 최근류와 종차를 통한 정의는 형이상학에서의 전제를 필요로 한다. 그것은 꼴들이, 서로에 대해 포함하는 것에 포함되는 것으로서, 포함되는 것에 포함하는 것으로서 존립하면서, 계층적으로 정렬된다는 것이다. 이러한 관계는 내적이어야 할 것으로 드러나거나, 상호 구성적이어야 할 것으로 드러난다: 종들은 그것들이 만일 그것들을 포함하는 어떤 것에 의해 포함되지 않았다면 그러한 것들이 아니었리라는 점에서 그것들의 특성을 그것들의 유들에 의존한다; 올바름은, 만일 그것이 탁월함이 아니었다면, 올바름이 아니었을 것이다. 반대로, 탁월함은, 그것의 한 종이 올바름이 아니었더라면, 탁월함이 아니었을 것이다: 유는 그것의 현존과 특징을 그것의 종에 의존한다. 전체는 그것의 부분들에 의존하여 전체인 그것일 것이다. 플라톤은 그 점에 대해 조금 덜 명확하지만, 아마도 유들에 대한 정의는, 종들에 대한 정의와 달리, 구성 요소들로의 분석을 필요로 할 것이다.

7) 이것은 플라톤적 유들에 대한 함축의 풍부함을 설명한다. 아리스토텔레스적 논리학에 있어서, 유들은 종들로부터의 추상들이다: 소크라테스와 플라톤은 사람이라는 것에 속하는 공통된 특징을 가지고, 사람과 말은 동물이라는 것에 속하는 공통된 특징을 가진다. 그 유는 여기에서 단지 다양성에 대한 공통된 요소일 따름이고, 그것의 종들보다 더 '추상적'일 따름이다. 그러나 만일 정의가 사상에 대해 동종적이라면, 만일 유들이 부분들을 포함하는 전체들이라면, 유는 공통된 특징이 아니라, 마치 그것의 현존이 그것의 요소들 혹은 부분들의 현존에 의해 전제하기도 하고 전제되기도 하는 하나의 체계이다. 그것은 이것이 그래서 유는, 함축의 상대적으로 결여되었음보다는 오히려 함축을 잔뜩 실은 것으로서, 결핍되었다기 보다는 풍부한 것으로 생각되기 때문이다.

8) 하나의 체계로서, 유는 한 종류의 폐쇄를 지니고 있어야만 하고, 그 체계를 다른 체계들로부터 또는 다른 종류의 제한들로부터 갈라놓는 단일성을 지니고 있어야만 한다. οὐσία와 πάθος 사이의 구별은 한 사물에 본질적인 것과 그 사물이 단지 가지게 되었을 뿐인 특징 사이의 구별을 수반한다. 올바름 또는 탁월함에 대한 경건의 관계와 모든 신들에 의해 사랑받는 것에 대한 경건의 관계 사이에는 차이가 있다. 만일 '그것은 무엇인가?'라는 물음이 본질적 정의에 있어서 내적 성질을 시사한다면, 그러므로 그것은 꼴에 대한 모든 설명들이 본질을 진술하는 것은 아님을 시사하는 것이다. 그것은 즉 그 관계의 내적 성질만큼이나 외적 성질도 시사한다.

9) 유들은 체계들이기 때문에, 그리고 종들은 그 유들의 요소들이기 때문에, 종들에 대한 정의들은 동일성(정체성)에 대한 일상적 진술들이라기 보다는 오히려 사상들이다. 그러한 사상을 구축하기 위해, 또는 사용하기 위해서, 일반적 언어에도 개별적 사물들과 행위들에도 호소할 수 없다: 꼴들에 대한 지식은 지적 직관을, 존재하는 사물들의 본성을 꿰뚫는 직접적 통찰을 요청할 것이다. 실재적 정의의 진리는 오직 그에 대한 정의가 설명인 그러한 진리를 파악함으로써만 파악될 수 있다.

10) 만일 이러하다면, 정의와 직관은 상보적일 것이다. 지적 직관은 실례로 해석되는 것이 아니고, 그렇지만 '알고 있음에 의한 앎'에 속하는 난해한 것이다; 말하자면 어떤 종류의 설명도 제공함이 없이 푸른 하늘의 한 조각을 알고 있을 수 있는데, 왜냐하면 감각은 지적으로 노력할 것이 없기 때문이다. 그러나 꼴들에 대한 직관은 반대로 설명을 요구하기 때문이다; 변증은 단순히 그냥 보는 것이 아니라 보려고 하는 것, 이전에 불충분한 설명 속에서 흐릿하게 보였던 어떤 것을 충분한 설명 속에서 명확하게 보려고 하는 것을 필요로 한다. 시야는 진술들에 대한 검토에 의해 확보된다. 미리 앎에 의한 앎에 대한 지적 직관의 비교는 지적 직관의 대상이 주어진 것이 아니라 하나의 목표라는 사실을 도외시한다.

11) 일부 철학자들은 소크라테스적 변증의 이 지점에서 압박을, 직관과 논리 사이의 긴장을 발견할 것을 주장해 왔다. R. G. 콜링우드는 다음과 같이 썼다.

그렇지만 상당히 높게 플라톤의 철학적 성취들은 평가받고, 최고에 못 미치는 어떤 가치에 있어서 그것들을 평가하는 것은 스스로, 그의 방법론에 대한 이론이 그 자신에 의해 철학과 수학 사이에서 확립되는 충분히 깊은 구별로 이끄는 것에 대한 실패를 통하여 결함있는 것으로 인정되어야만하는, 철학자가 아님을 인정하는 것이 될 것이다. 그 결과는 그의 방법론이 철학을 둘로 분열시키는 것이다: 하나는 재기 발랄한 억지 이론의 식상한 낭비이고, 다른 하나는 궁극적 실재에 대한 직관적 시야이다. 이러한 첫번째 것이 두번째 것으로의 경로라는 것은 여러 세대들에 속하는 경험에 의해 플라톤이 그들의 길잡이로 취해졌다는 것에 대한 보증이 될 것이다; 그러나 설령 그렇다 할지라도, 우리는, 사실들에 권위에서 받아들여질 사실들에 대한 탐구가 아니라, 그 조명 속에서 사살들이 이해될 그러한 개념들에 대한 탐구 속에서, 철학적 탐구에 매진한다; 그리고 이러한 것들을 플라톤이 우리에게 주진 않았다.

이러한 비판은 감각 지각의 틀을 관통하는 지적 직관을 강요함으로부터, 지적인 것으로서, 그 직관이 판단을 통해 지속한다는 것을 알아차리는 데에 실패함으로부터 귀결한다. 지적 통찰에 속하는 한 행위는 하나의 진술이 아니라, 진술 속에서 표현 가능한 하나의 내용을 필요로하고, 그로써 비평에 적합하다. 한 진술은 지적 통찰에 속하는 하나의 행위가 아니라, 그것은 그를 통해 통찰이 표현되는 그러한 것이다 - 그 자신에 대해서조차. 안다는 것은 설명을 제공할 수 있다는 것이다; 설명을 제공할 수 있다는 것은 안다는 것이다.

12) 그러나 이러한 지점에 도달했다는 것은 초기 대화편들을 뛰어 넘었다는 것이다.

-蟲-
플라톤의 꼴들에 대한 초기 이론

서론

1)『에우튀프론』은 실패로 끝났다; 경건함의 어떤 정의(定義)도 진술되지 않고, 아무것도 함축되지 않는다. 그 대화의 참된 의미가 밝혀지도록 파헤쳐질 수 있는 아무런 '가면'도 없다; 그 대화는 그것의 의미를 그 표면에 지닌다.

2) 이렇게 말하는 것이 그 대화가 아무런 명확한 철학적 내용도 지니지 않는다는 것을 말하는 건 아니다. W.A.하이델은 일찍이 『에우튀프론』에 대해 '더 짧은 플라톤 대화편들 중 어느 것도 철학적 이론으로 향하는 암시들의 가치로 그 책과 비교될 수 없다'라고 언급했었고, 이것은 물론 진실이다. 그 대화편의 흥미로운 점은 그것의 변증법에 대한 산출에 놓이지 않는다, 아무런 산물도 없기 때문에; 그 흥미는 그 변증 자체에 자리한다. 그 변증은 그 변증의 작업을 규제하는 것들의 본질 또는 본성에 대한 가정들에 기초하여 나아간다; 그리고 만일 그 작업이 실패로 끝난다면, 경건에 대해 성취된 어떠한 정의도 없이, 실패의 시금석은 정확하게 그 탐구를 이끌었던 규칙들에 놓인다.

3) 그러한 규칙들, 그리고 그 규칙들이 근거하는 전제들은 형상들에 대한 이론(어떤 특정한 바로 그 이론은 아닌)을 형성한다. 그 이론은 명시적 진술보다 전제의 차원에서 훨씬 더 많이 작동하는, 하지만 그럼에도 불구하고 그 이론의 요소들이 어느 정도 정밀하게 배치되는 이론이다. 『에우튀프론』에서 소크라테스의 목적은 '무엇이 경건인가?'라는 물음의 답을 얻는 것이다. 그는 그의 탐구를 이어감에 있어서 경건에 대한 하나의 ἰδέα, 혹은 εἶδος, 하나의 꼴이 있다고, 그리고 이 꼴은 보편적이고, 모든 경건한 것들에 있어서 동일한 것이라고 생각한다(5d, 6d-e). 나아가 그는 그 꼴이 그로써 어떠한 것들이 경건하고 어떠한 것들은 그렇지 않은지 판단할 규준으로 사용될 것이라 가정한다(6e); <p67>그 꼴에 의해 또는 그 꼴의 덕(virtue) 안에서 경건한 것들이 경건하다고(6d); 그리고 그 꼴이 실질적이거나 본질적인 정의의 능력이라고 가정한다(11a, 12c-d). 이러한 전제들은 꼴들에 대한 하나의 이론을 구성한다.

4) 그 이론은 논리적이기도 하고 형이상학적이기도 하다. 논리적으로, 꼴들은 변증에 있어서 규제적 규칙의 역할을 한다: '그것은 무엇인가?'라는 물음에서 '그것'의 전항들로서 그것들은 소크라테스의 정의 탐색에 있어서 허용가능한 답변의 종류들을 결정하고, 더욱 중요한 것으로는, 허용될 수 없는 것들을 결정한다. 그 꼴들은 변증이 성공한 순간과 실패한 순간의 조건들을 한정한다. 형이상학적으로, 형상들은 세계의 진행에 영향을 미친다: 꼴들은 사물들의 실질적 본성들이고, 세계는 그 본성들이 그러한 것들이기 때문에 그러한 세계이다.

5) 플라톤의 이론에 대한 이런 두 가지 측면들은 실질적 정의의 관념에서 만난다. 변증은 규칙에 의해 통제되고, 사물들의 본성으로 향하게 된다. 변증의 규칙들은 그 변증의 목적에 의해 결정된다. 꼴들이 있다고 말하는 것은 실질적 정의가 추구되어야 한다고 말하는 것이다; 실질적 정의가 추구되어야 한다고 말하는 것은 꼴들이 있다고 말하는 것이다. 『에우튀프론』 안에서 꼴들에 대한 이론은 변증에 쌓아올린 조잡한 상부구조물이 아니다: 그 이론은 변증의 토대이다. 그 이론 없이, 변증은 그것이 사용하는 규칙들에 의해 수행되지 않았거나 그것이 의도했던 목표를 향해 작동하지 않았을 것이다. 소크라테스의 도덕 탐구는 실재에 대한 탐구이다.

6) 그 이론의 유사함에도 불구하고, 『에우튀프론』에서 가정된 꼴들에 대한 이론은 플라톤의 중기 대화편들에서 발견되는 그 이론들과 동일시되지 않는다. 버크는 일찍이 '아무도 밤과 낮의 경계 사이에 한 획을 그을 수 없다, 빛과 어둠이 분간할 수 있는 그 전체 위에 있기 전까지'라고 언급했었다. 이것은 초기 대화편들과 중기 대화편들에서도 그렇다. 한편으로는 『에우튀프론』과 다른 한편으로는『파이돈』 그리고 『정체』 사이의 그 차이는 아마도 빛과 어둠의 차이는 아닐 것이다. 하지만 그 차이는 받아들일 수 있을 만큼 분명하다. 중기 대화편들의 철학은 결부된 반대항들의 온상이다: 있음과 됨, 겉모습과 실재, 영속과 유전(流轉), 이성과 감각, 몸과 영혼, 육체와 정신. 이러한 반대항들은 결핍의 심연에 의해 나뉘어진 두 세계에 대한 존재론에 뿌리를 둔다. 더욱 실재적인 것이 덜 실재적인 것에 맞서고, 원형(原形)들이 그림자들과 반영물들에 맞서는 것처럼, 영원한 꼴들을 내용물들로 지닌 앎의 세계는 감각적이고 변화하는 내용물들을 지닌 사견의 세계에 맞선다. 가시적 세계는 그 세계에 속하는 결핍 안에서 알 수 없는 상(象)이고, 오로지 이성에 의해서만 파악되는 이해 가능한 세계의 상이다. <68p>만일 『에우튀프론』과 같은 초기 대화편들에 이러한 세계관의 씨앗들이 뿌려져 있다면, 그 씨앗들은 아직 그곳에서 수확되지 않았을 것이다.(67-69.2)


1. 변증의 규제적 원칙들로서 꼴들

1)『에우튀프론』의 중심물음인 '경건함은 무엇인가?'라는 물음은 다양한 방식으로 표현된다. 그 자체로, 그 물음은 고유하게 다의적이다: 그 물음은 그에 답하도록 요청되는 본질에 대해 하나의 예시인지 구별되는 특징인지 또는 본질에 대한 설명인지 규정하지 않는다. 그러나 소크라테스는 그 특징 자체(αὐτὸ τὸ εἶδος)에 의해 경건한 것들이 경건한 그러한 특징을 설명하도록 - 즉, τὴν ἰδέαν τίς ποτέ ἐστιν(6d-e), '이 특징인 그것'을 설명하도록 - 에우튀프론에게 요청하면서 이 물음을 더욱 명확하게 만든다.

보편개념들로서 꼴들

1) 경건의 ἰδέα 또는 꼴들을 요구하면서, 소크라테스는 모든 각각의 경건한 행위에 있어서 동일한 것(ταὐτόν, 5d)이 이야기되길 기대한다. 에우튀프론이 경건함의 의문스러운 예시들을 가지고 답할 때, 소크라테스는 에우튀프론의 예시들이 끝나지 않는다는 것을 상기하게 된다; 다른 것들도 마찬가지로 경건하니까, 그리고 소크라테스가 바란 것은 그것들 모두가 지니는(ἔχον, 5d) 그 꼴의 본성이기 때문에.

2) 논증의 이러한 흐름은 다른 대화편들에서 발견된다. 『라케스』(191e-192b)에서, 용기가 전쟁에서의 용기에 대한 예시에 의해 정의될 때, 소크라테스는 이것이 충분하지 못하다고 주장한다. 용기는 전쟁에서만이 아니라 바다에서의 위험들 속에서도, 질병과 가난, 그리고 정치에 있어서도 또한 발견되고, 소크라테스가 이야기되길 바란 것은 이러한 것들 전부에 있어서 동일한 것(ταὐτόν)이다. 『대(大) 히피아스』(300a-b)에서, 소크라테스는 만일 두 가지 서로 다른 것들이 아름답다면, 그것들은 반드시 그것들을 아름답게 만드는 그러한 어떤 동일한 것(τι τὸ αὐτό)을 가져야만 한다고, 그리고 이 공통된 것(τὸ κοινὸν τοῦτο)은 그 두 가지 것들에 있는 것이어야만 한다고 주장한다. 초중기 대화편에서 메논이 예시들의 목록을 제시함으로써 덕(탁월함)을 정의하고자 시도할 때, 소크라테스는 유추를 가지고 그를 바로잡는다. 당신이 벌이 무엇인지 질문받았다고 가정하자. 여러 서로 다른 종류의 벌들이 있지만, 그렇게 많은 벌들이 있다는 것은 그것들을 이름짓는 데에는 아무런 득될 것이 없다; 벌들은 벌들로서 다르지 않고, 그 물음은 모든 벌들이 나누어 가지는 공통된 특징에 대한 어떤 설명을 요구하였기 때문이다. 그래서 그 특징은 덕과 함께: '다양하고 무수한 덕들이 있겠지만,<69p> 그 덕들은 전부 그것들을 덕들이도록 만드는 하나의 특징(εἶδος)을 가지고, 그 특징은 덕이 무엇인지 말할 그 누구라도 그의 시선을 집중시켜야 할 그러한 것이다'(『메논』72c).

3) 보편개념으로서, 꼴들은 변증에 있어서 규제적 역할을 수행한다; 그 꼴들은 τί ἐστί, '그것은 무엇인가?'라는 물음들에 있어서 ἐστί의 전항들이고, 그 꼴들은 그러므로 그 물음의 본성을 상술하며, 그래서 그 물음에 분별있게 제시될 답변들의 범위를 제한한다. '경건은 무엇인가?' 혹은 '아름다움은 무엇인가?'라는 물음들은 단순하게 문법적 형식의 말로는, 경건이나 아름다움의 예시에 대한 요청일 수 있었다; 그리고 에우튀프론과 히피아스 모두 처음에는 바로 이러한 방식으로 소크라테스의 물음을 해석한다. 중성 형용사와 관사에 대비되는 것으로서 추상적 명사조차 이러한 의미에 대해 열려있다: 카르미데스는 사려의 예시를 듦으로써 무엇이 σωφροσύνη, 사려(중용, 절제)인지 말하려고 시도한다. 그러나 그것의 예시들 모두에 있어서 동일한 그런 ἰδέα 또는 꼴을, 그리고 예시들 모두가 가지는 어떤 것을 찾고 있는 소크라테스는 이것을 배제시켰다; 왜냐하면 경건과 사려와 아름다움은 보편개념들이고, 그것들이 무엇인지의 물음에 대한 대답들은 단지 그것들의 예시들을 제공하기만 할 수는 없기 때문이다.

규준들로서의 꼴들

1) 소크라테스는 그것으로써 어떤 것들이 경건하고 어떤 것들이 경건하지 않은지 말할 규준 혹은 모형으로 사용하기 위해 경건의 본성이 이야기되기를 바랐다. 경건의 꼴은 경건한 것들이 그와 같은 종류(τοιοῦτον)의 것(6e)이라는 그 사실에 대한 추론에 의해서 이 일에 적합하게 될 것이다. 유사하게, 뤼시스에서 나이와 함께 희어진 머리카락은 흼과 같은 동일한 종류(οἷον)의 것 - 그것에 지금 있는 것과 같은 동일한 종류의 것 - 이다(217c-d).

2) τοιοῦτος (라틴어 talis, of such kind)와 οἷος (라틴어 qualis, of the kind that, such as)라는 단어들은 각기 의문사 ποῖος, '어떤 종류의?'와 상관하는 지시 대명사와 부정 대명사이다. 그 대명사들의 사용은 꼴들의 예시들과 꼴들 사이에 어떤 종류의 유사성이 있다는 것을 시사한다. 그 시사점은 그 꼴이 규준 또는 παράδειγμα로 기술된다는 사실에 의해 뒷받침된다. 그러나 이 암시는 함축이 거의 아니다. 예를 들어 『고르기아스』460b에서 οἷον의 사용은 유사점이 아니라 아리스토텔레스가 나중에 paronymy(파생어, 동근어)라고 부를 어떤 것을 포함한다. 『프로타고라스』에서 οἷον의 사용은 동일성이나 상호 함축을 포함한다. 그리고 『파이드로스』246a에서 그 사용은 유사성과 대비된다. 이러한 단어들의 사용이 유동적이라는 것, 그리고 오직 특정한 문맥 안에서만 한정된다는 것은 사실이다. τοιοῦτον이 『에우튀프론』6e에서 유사성을 함축할 필요는 없다; 그리고 플라톤이 그의 마음대로 사용할 수 있는 유사성에 대한 풍부하고 다양한 어휘를 가지고 있었다 할지라도, 그는 초기 대화편들 그 어디에서도 꼴들의 예시들을 꼴들의 μιμήματα나 εἰκόνες, 또는 ὁμοιώματα라고 부르지 않고, 그러한 예시들이 꼴들과 맺는 관계들을 이러한 명사들의 어원들로부터 파생된 동사들을 가지고 묘사하지도 않는다.

3) 꼴들의 예시들이 꼴들과 유사하다는 주장은 그러니까 『에우튀프론』이라는 글에 의해 시사되지 않는다. 더욱이, 그건 비경제적이다. 그 글이 그런 주장 없이도 이해되기 때문이다. 경건의 꼴은 경건한 것들을 경건하지 않은 것들로부터 구별하기 위한 규준의 역할을 하는 것이다. 그러나 확실히 그 꼴이 이러한 기능을 수행하는 데에 적합한 이유는 그 꼴이 경건한 것들이 경건한 것들인 한에서 그것들의 본성이라는 것이다. 이러한 이유로 소크라테스는 만일 경건이 그 자체로 무엇인지 한 번 이해되기만 한다면 그러한 앎이 그 아는 자로 하여금 경건한 것들을 경건하지 않은 것들로부터 구별해낼 수 있도록 한다고 가정한다. 그래서 경건은 그것과 같은 그러한 종류의 것들을 - 즉, 그것의 사례들인 것들을- 간파해내기 위한 규준이다.

4) 이러한 해석은 그 문맥에 적합하다. 인식론적으로, 꼴들은 그 꼴들의 사례들을 간파하기 위한 규준들이다. 이러한 기능의 존재론적 근거(토대)는 예시들이 그 꼴을 가진다는 것(5d), 그리고 그 꼴은 바로 그 꼴에 의하여 그 사례들이 그것들인 그러한 것인 바의 것이라는 것이다(6d).

5) 꼴들이 규준이라는 소크라테스의 전제가 그 꼴들이 보편개념들이라는 전제로서 '그것은 무엇인가?'라는 그의물음 속에 직접적으로 깊숙히 박혀 있다는 것을 알아차리는 것은 중요하다. '경건이란 무엇인가?'라는 물음은 동일성에 대한 실천적인 문제에 의해 『에우튀프론』에서 촉구된다: 경건이 무엇인지 밝혀내는 것은 어떤 종류의 행위들이 경건하고 또 어떤 행위들이 그렇지 않은지를 알기 위해 중요하다. 무엇이 우정인지 알지 못하고서는 두 사람이 친구인지를(『뤼시스』223b), 혹은 아름다움이 무엇인지 모르는 채로 한 연설이 아름다운지를 확신할 수 없는 것처럼(『대(大)히피아스』286c-d), 무엇이 경건인지에 대한 앎이 없이는 한 주어진 행위가 경건한지 아닌지 알 수는 없다(6e, 9a-c, 15d-e). 그 꼴을 아는 것은 그 꼴의 사례들을 알기 위한 조건이다: 경건이 무엇인지 묻는 것은, 경건한 것들을 경건하지 않은 것들로부터 구분할 척도에 대한 앎을 요청하는 것이다.

6) 척도에 대한 이러한 요구는 단지 추상적 이해가 아니라 삶에 대한 올바른 질서지음을 향해 정향된 소크라테스의 변증에 본질적이다; 변증의 그 목표는 그러한 질서의 원칙들을 이해하는 것이고, 그 원칙들을 구체적인 사례들에서 확인할 수단을 얻는 것이다. 『정체』(1권, 344d-e)에서는 경건이 무엇인지 그리고 정의는 무엇이며 탁월함(덕스러움)은 무엇인지 아는 것이 중요하다고 주장되는데, 왜냐하면 삶을 꾸려나아감이 그러한 앎에 의존하기 때문이다. 『카르미데스』(175e)에서 젊은 카르미데스는 가장 사려깊은 것으로 이야기된다.; 그러나 사려에 대한 앎이 없다면, 그는 그의 삶에서 그 사려의 현존- 훗날 30인 참주들 중 하나가 되는 한 남자에게 있어서의 - 으로부터 아무런 이득도 갖지 못할 것이다. 만일 변증의 목적이 꼴을 정의하는 것이라면, 그 실천적 목표는 사물들 속에서 꼴의 식별일 것이다. '경건함 그 자체는 모든 각각의 행위들에 있어서 그 자체로 동일하지 않은가? 그리고 불경건함의 경우에는, 경건함과 전혀 반대로, 그 자체로서는 동일하지 않은가? 그리고 모든 각각의 불경할 것들은 불경건함이라는 점에서 단 하나의 특징을 지니지 않는가?'

7) 꼴들은 숱한 상이한 사례들에 공통적인 보편적 개념들이기 때문에 , 특정한 종류의 대답은 '그것은 무엇인가?'라는 물음에서 배제된다: 말하자면 예시에 의해, 즉 사례들을 인용함으로써 정의를 시도하는 그런 모든 대답들이 배제된다. 왜냐하면 꼴들은 규준들이기 때문이다. 또 다른 종류의 답변이 배제된다: 경건이나 사려나 아름다움이 어떤 식으로든 불경건이나 사려깊지 못함 또는 추함을 암시할 그런 종류의 답변들은 배제되는 것이다. 꼴들은 그것들의 반대항들에 의해서 한정되지 않는다.

8) 『에우튀프론』의 경우에서 첫번째 정의에 대한 소크라테스의 검토는 정확히 이 점에 주목한다. 경건은 단지 정의로움이 부정의함일 수 있는 만큼만 불경건일 수 있다. 그렇기 때문에, 그리고 신들에 의해 사랑받는 것이 또한 신들에 의해 미움받고 그래서 불경하기 때문에, 경건은 신들에 의해 사랑받는 것일 수 없다. 『정체』에서,  그러한 행위들이 때에 따라 올바를지 모르지만, 또 이따금 올바르지 못하다는 것은, 올바름에 대해서 제안된 진실을 말하고 빚을 갚는 것으로서의 정의의 충분한 반박이다: 만일 한 친구가 당신에게 무기들을 빌려주었고 그 다음에는 정신이 나갔다면, 그러한 친구가 그 무기들을 요구할 때 그것들을 되돌려주는 것은 옳지 못할 것이다; 그리고 미친 사람에게 진실만을 말하는 것도 옳지 못할 것이다. 그래서 진실을 말하고 빚진 것을 되돌려주는 것은 올바름의 정의가 아니다.

9) 꼴들이 그것들의 반대항들에 의해 한정될 수 없다는 그 원칙의 응용은 『대히피아스』에 있다. 히피아스가 아름다움을 아름다운 처녀로 정의하고자 할 때, 소크라테스는 이것이 너무 협소하다는 사실에 대해 그를 주의시킨다; 아름다운 처녀들 외에도 아름다운 암말들과 아름다운 리라들 그리고 아름다운 항아리들도 있다. 히피아스는 그 항아리에 대해서 반대한다. 그가 생각하기에 가치 없고 사소한 것은 아름다움으로 암말이나 여인에 전혀 비할 수 없다. 소크라테스는 무엇보다도 한 항아리는 그 종류에 있어서 아름다울 것이고, 아름다운 여인조차 여신에 비한다면 추할 것이라고 답한다. 그래서 아름다움 그 자체는 무엇인가? 그 물음은 상이한 비교들 속에서 추한 것보다 더 아름답지 않은 것들을 예증함으로써 답해질 수는 없다. 히피아스는 다음으로 아름다움이 황금이라고 제안한다; 그러나 소크라테스가 황금이나 황금의 있음이 아름답지 않은 상황들을 보여줌으로써 이것을 반박할 때, 히피아스는 다음과 같은 점을 알아차리기 시작한다: '나로서는 당신이 아름다움이란 그 누구에게도 그리고 그 어느 곳에서도 결코 추한 것으로는 드러나지 않을 그러한 종류의 어떤 것이어야만 한다는 답변을 찾고 있는 것으로 보이는군요.' 이에 대해 소크라테스는 대답한다. '바로 그거에요, 히피아스. 지금 당신은 아름답게(빼어나게) 나를 이해하네요.'(291d).

10) 꼴들이 그것들의 고유한 반대항들에 의해 한정되지 않는다는 원칙은 꼴들의 자기동일성과 직접적으로 연결된다. 소크라테스는 경건이 αὐτὸ αὑτῷ, 그 자체로 동일한 것이며, 그것의 반대, 불경건도 αὑτῷ ὅμοιον, 그것 자체와 같은(동일한 것)이라고 제안함으로써『에우튀프론』에서의 변증을 시작했다(5d). 그리고 그는 그와 같은 경건의 본성을 찾고 있다. 이러한 제안은 7a에서 경건이 불경건과 동일하지 않지만 그것의 정 반대항이라는 의미로 확장된다. 그리고 신들에 의해 사랑받는 것으로서 에우튀프론의 경건에 대한 정의에 대한 반박은 그래서 그 정의가 다음과 같은 원칙을 위배한다는 것을 시사하는 것으로 진술된다: 당신은 ὃ τυγχάνει ταὐτὸν ὃν ὅσιον τε καὶ ἄνοσιον, '어떤 동일한 것이 우연히 경건이면서도 또 불경건이기도 한 그러한 것'을 진술함으로써 경건이 무엇인지 말할 수는 없다(8a). 여기에서 전제는 만일 경건이 어떤 식으로든 불경이라면, 그것은 불경건의 반대항이 아닐 것이라는 것이고, 이 추론 아래에 깔린 직관적 원칙은 분명하다: 어떤 것 또는 그 어떤 것의 덕에 의해 그것들이 경건한 그러한 어떤 것은 그러한 경건이고, 그 자체로 불경건일 수 없다는 것이다. 꼴들의 자기동일성은 그 꼴들의 반대항들에 대한 단적인 배제를 함축하고, 그러한 배제는 직접적으로 규준으로서의 꼴의 사용과 관련된다. '경건이란 무엇인가?'라는 물음에 대해 어떤 식으로든 불경건인 어떤 것을 이름붙이는 것은 결코 참인 대답일 수 없다.


본질들로서의 꼴들

1) 아리스토텔레스의 설명에 의하면, 경건의 본질은 어떤 경건이 καθ᾿ αὑτό, 그러니까 그 자체 안에서 또 그 자체에 대한 것, 혹은 그러한 바의 것이어야 할 것이고, 어떤 것에 대한 설명은 정의여야 한다. 이것은 정확하게 플라톤의 상투어들인 'αὐτὸ τὸ…'와 '…αὐτὸ καθ᾿ αὑτό', 그러니까 '~인 것 자체'와 '오로지 그 자체로 …인 것'의 영향력이다.

2) 본질에 대한 관념의 본원들은 아낙사고라스에게 있다. 그의 체계의 중심 논제는 모든 것들 하나하나에 '큼'과 '작음' 처럼 대립항들의 부분들이 있다는 것이고, 오로지 정신만이 그 자체로 혼합되지 않고 현존한다는 것이다. 이러한 종류의 논제를 제시하는 것은 그 논제의 부정을 자극시키는 것이다. 만일 그 반대항들이 언제나 함께 뒤섞여 있다면, 무엇이 그것들에 대해 뒤섞이지 않은,
ἐφ᾿ ἑαυτοῦ, 오로지 그러한 것들 자체로서 그러한 것들 자체인 것과 같은 것
일까? 이러한 종류의 물음을 묻는 것은 『에우튀프론』이 경건에 대해 묻는 그런 종류의 물음으로의 긴 걸음을 내딛은 것이다. 강력하진 않지만 아리스토파네스에 역사적 소크라테스가 그러한 걸음을 디뎠다는 증거가 있다. 그는 'αὐτὸ καθ᾿ αὑτό'라는 상투어를 사용했었고 그러므로 생각컨데 그 뒤에 그런 사고가 놓여 있다는 증거 말이다. 그리고 크세노폰은 소크라테스가 '그것은 무엇인가?'라는 물음을 던졌다는 것을 증언한다.

그 자신은 결코 인간적 주제들을 토론하는데에 싫증을 내는 일이 없었다: 경건이란 무엇인가? 불경건이란 무엇인가? 아름다움은 무엇인가? 추함은 무엇인가? 고귀함은 무엇인가? 천함은 무엇인가? 정의로움과 부정의함은 무엇을 의미하는가? … 그리고 비슷한 다른 문제들, 어떤 것에 대한 앎은, 그가 말하듯, 귀족의 특권을 소유주에게 주었고, 반면에 그 앎을 결여한 그러한 자들은 아마도 응당 노예로서 비난받았을 것이다.

물론 이로부터 역사적 소크라테스가 '그것은 무엇인가?'라는 물음을 정확하게 『에우튀프론』에서 제안한 그 방식으로 한정된 것으로 의도했었다는 것이 따라 나오지는 않는다. 그러나 그 부정도 따라나오진 않는다. 최소한 그 대화편이 그것의 본질에 대한 탐구, 그 자체로서의 경건에 대한 정의에 대한 탐구에 있어서, 역사적 소크라테스의 심중을 어느 정도 정확하게 나타낸다는 것은 가능하다.

3) 본질로서, 꼴들은 변증에서 규제적 역할을 수행한다. 꼴들은 '그것은 무엇인가?'라는 물음에 대해 응답의 특정한 종류들을 문제 외적인 것으로 치부한다. - 말하자면, 꼴 자체와는 다른 것으로서, 단지 그 꼴을 지니는 것들을 특징짓는 특색 있는 징표를 제공할 뿐인 그러한 모든 대답들을 배제한다. 당신은 단순히 그것의 사례들을 구별짓기 위한 징표를 제공하는 것만으로는 어떠한 것이 그 자체의 특징이라고 말할 수 없다. 『에우튀프론』 10e-11a의 내용은 이것의 전형적인 예시이다. 경건이 모든 신들에 의해 사랑받는 다는 것은 아마 맞는 말일 것이다: 하지만 이러한 것은 오직 경건의 πάθος만을 진술할 뿐, 그것의 οὐσία를 진술하지 않는다. 다시금, 소크라테스는 뒤이어 만일 경건이 신들에게 만족스러울 어떤 것이라면 그것은 신들에 의해 사랑받는 것이 아닐 것이라고 추론한다(15b); 이러한 추론은 그 'is'가 특징 부여의 'is'가 아니라,
 πάθος로써가 아닌 οὐσία로써 대답하는
 정의의 'is'로 의미되지 않는 한 말이 되지 않는다. 본질에 대한 탐구는 변증의 본질에 속하는 것이다.

4) Mr.리처드 로빈슨은 '그것은 무엇인가?'라는 물음, 혹은 그가 부르듯 'X-는-무엇-인가?"라는 물음 속에 이중성이 있다고 주장한다.

한편으로 그것은
 그것을 가지고 어떠한 서술이라도 치환할 수 있기 때문에
 단지 X의 동치에 대한 탐구일 따름이다. 다른 한편으로 그것은
 다른 어떤 것들보다도 더욱 근본적인 방식으로 X로 생각되는 어떤 X의 특별한 하나의 동치이기 때문에
 이보다 더 협소하게 생각되는 어떤 것에 대한 탐구이다.

Mr. 로빈슨이 제기하는 그 이중성은 설령 소크라테스가 때때로 X의 본성이나 본질이 이야기되기를 기대한다 치더라도, 그는 다른 경우들에서 X의 치환 가능한 서술에, 다른 X들로부터 X 전부를 그리고 오직 X만을 드러낼 특징적 징표에 만족했다는 사실로 되어 있다. 이러한 이중성은 소크라테스가 본질과 동일성을 구별하지 않았기 때문에, 또는 그것들을 혼동했기 때문에 있는 것으로 생각된다.

5) Mr. 로빈슨의 한 가지 논증은 이에 대해 예시의 역할을 할 지지에 있어서 나타난다.

여러 구절들이 그(소크라테스)가 원한 모든 것은 그것으로 주어진 어떤 것에 대해서든 그것이 X인지 아닌지 판단할 모형의 역할을 할 징표라고 암시한다. 『에우튀프론』(6e)에서 그는 그의 목표를 바로 이 방식으로 묘사한다.

하지만 그는 그의 목표를 바로 이런 방식으로 묘사하지 않는다. 그는 그 목표를 오히려 경건의 τὴν ἰδέαν τίς ποτέ ἐστιν에 대한 배움의 방식으로 묘사한다. 그것은 그가 어떤 것들이 경건하고 또 어떤 것이 그렇지 않은지 판단하기 위한 규준이나 παράδειγμα로 사용할 것을 기대한 것이다. - '징표'에 대한 아무런 언급도 없다'. 그러한, 경건 그 자체인 규준에 대한 설명은 반드시 경건의 οὐσία와 그것의 본성, 그리고 실체를 진술해야만 한다. Mr. 로빈슨은 더욱 앞서서 그가 '추론에 의한 오역'이라고 불렀던 결정적 착오를 기술했었다; 소크라테스가 『에우튀프론』6e 에서 오직 특징적 징표만을 바란다는 그의 전제는 그러한 유의 종이다, 그것이 어떠한 함축에도 기초되지 않은 추론이라는 차이와 함께. 그 추론은 덜 만족스러운데 왜냐하면 Mr. 로빈슨은 소크라테스가 『메논』74d에서 '…X의 꼴, 많은 것들에 있어서 단 하나, 여러 X들 모두에 있어서 그 현존이 우리가 그것들을 모든 X들이라고 부른다는 사실에 의해 보증되는 그러한 단일한 동일성'을 추구하고 있다는 이유로 그가 본질을 추구하고 있다고 가정하기 때문이다. 마지막 절은 제쳐두고 - 사물들 안에 존재하는 어떤 것은 물론 우리가 그것들을 부르는 그 어떤 것에 의해 보증되지 않는다 - 이것은 『에우튀프론』6d-e에서 소크라테스의 목적에 대한 훌륭한 진술이다. 그러나 소크라테스가 비록 꼴을 찾고 있다 하더라도 본질을 찾고 있지는 않다는 한 구절로부터, 그리고 소크라테스가 본질을 찾고 있는데 그것은 그가 꼴을 찾고 있기 때문이라는 또 다른 한 구절로부터 누군가 추론해내도록 한다는 것은 불만족스러운 해석방법이다.

6) Mr. 로빈슨은 또한 소크라테스적 물음에 이중성이 있다는 논증을 다음과 같이 제시한다:


그것은 소크라테스가 자주 What-is-it 물음을 묻는 과정을 기술하기 위해 사용하는
한 단어, 즉 horizein에 의해 시사된다. 이 용어는, 경계석과의 본래적인 관련에 대한 감각을 결코 잊지 않으면서
, 그렇게 한정된 영역들에서의 경작지들이나 작물들을 묘사하는 어떤 방식도 없이,
 한 영역을 다른 영역과 구분하기 위한 표지를 세울 것을 제안하기 때문이다. 그리고 플라톤의 대화편들 속에서, 'distinguish(구분하다)' 또는 'mark off(구분선을 긋다)'라는 번역들은 'define(한정하다)'라는 번역어만큼, 혹은 그 이상으로 자주 적절하다.

그러나 그 물음은 소크라테스가 그의 물음에 대해 의도했던 특수한 종류의 구분짓기에 관한 것이다. 초기 대화편에서의 대답은 명확하다: 경계석은 본질이다.

7) 이러한 결론은 또한 꼴들이 표준들이라는 사실에 의해, 그리고 Mr. 로빈슨 자신이 주의를 기울였던 '그것은 무엇인가?'라는 물음이 지니는 그 외의 물음들에 대한 그 특유한 우선성에 의해 암시된다. 바로 무엇이 경건인지 모르고서는 어떤 것들이 경건한지 알 수 없는 것과 마찬가지로, 어떤 특성들이 경건과 관련되는지 결정할 수 있기에 앞서 경건이 무엇인지 알아야만 한다. X가 Y임을 알 수 있기 전에 X의 본성을 알아야만 하는 것이다. 그래서, 탁월함이 무엇인지 알기 전에는, 그 탁월함을 얻기 위해 어떻게 하는 것이 가장 좋은가를 누군와 논의할 수 없다(『라케스』190b-c). 올바름이 무엇인지 알기 전에는, 그것이 탁월함인지 아닌지, 또는 득이 되는 것인지 아닌지, 혹은 그것을 가진 자를 행복하게 해주는 것인지 어떤지 알 수 없다(『정체』Ⅰ, 354b-c). 소크라테스와 프로타고라스는 탁월함이 가르칠 수 있는 건지 아닌지에 대해, 혹은 그것이 앎인지 아닌지에 대해 동의할 수 없다(『프로타고라스』359e-361d). 소크라테스는 탁월함이 무엇인지 모르기 때문에, 그는 그것의 특성들이 무엇인지 (ὁποῖόν τι), 또는 특히, 그것이 연습에 의해서 가르쳐지거나 획득되는지 아니면 본성에 의해 존재하는지(『메논』71b) 말할 수 없다. 더욱이 어떻게 ὅ τι ἔστι(그것이 무엇인지)를 알지 못하고서 ποῖόν τί ἐστι(그것이 어떠한 것인지)를 결정할 수 있겠는가?

8) 이 모든 것은 무엇을 시사하는가? 그것은 οὐσία에 대한 앎이 πάθος에 대한 앎보다 우선한다는 것을 암시한다. 그리고 Mr. 로빈슨이 말한 그 'intimate(사적인)' 의미에서 경건이 무엇인지 알지 못하는 한, 어떤 특성들이 경건과 관련되는지 알 수 없다. 만일 초기 대화편들이 Mr. 로빈슨이 주장하는 이중성을 드러냈다면, 그 대화편들은 안타깝게도 일관되지 못하다. 그러나 사실은 그것들을 다룸에 있어서 이중성이 아니라 우선성이 있는 것이다: 본질이 앎의 핵심이다.

9) πάθος에 대한 οὐσία의 우선성은 꼴들이 표준들이라는 사실로 직접적으로 시사된다. 경건의 어떤 특성이든 그것의 연관의 토대는 경건 자체의 본성이든 경건한 것들에든 자리해야만 한다: 예를 들어 경건을 가르칠 수 있다는 것은 경건의 꼴에 대한 주장이거나 그 안에 경건이 현존하는 그러한 것들에 대한 주장이다. 어느 경우든 경건이 무엇인지 앎이 없이 그 진리치를 가늠할 수 없는데, 어떤 것들이 경건한지 결정할 수 있기에 앞서 경건이 무엇인지 알아야만 하기 때문이다. 이것은 '경건은 무엇인가?'라는 물음이 왜 경건과 관련된 특성들이나 πάθη로 답해질 수 없는지를 설명한다; 그런 종류의 대답은 순환을 포함할 것이다. '경건은 무엇인가?'를 묻는 것은 경건한 것들이 특징지어지는 것을 묻는 것이 아니라 경건이 정의되는 것을 묻는 것이다.

R. E. Allen,
 Plato's 'Euthyphro' and the
 Earlier Theory of Forms, Routledge & Kegan Paul, 1970, 67-79p

-蟲-
배경. 에우튀프론은 종을 죽인 품팔이를 묶어 방치해 죽게 한 아버지에게 소송을 걸었다. 소크라테스는 오래된 신들을 믿지 않고 낯선 신들을 만들어내어 젊은이들을 타락시킨다는 죄목으로 멜레토스에 의해 고소를 당했다. 두 죄목이 모두 바실레우스의 종교적 문제에 대한 재판에 관련된다. 따라서 피고인 에우튀프론 아버지와 소크라테스는 '불경죄'로 피소된 것이다. 소크라테스는 불경에 대해서 고소를 한 자는 경건과 불경건을 잘 알기도 할 것이니 고소를 당한 처지인 자신을 그가 가르쳐주어야 한다고 말한다.

제1 논의. 에우튀프론은 올바르지 못하게 살인을 저지른 아버지에게 소송을 거는 지금 자신의 행위가 경건한 것이라고 말한다. 그리고 그 근거로 크로노스가 우라노스를 거세한 일과 제우스가 크로노스를 처단한 일을 든다. 또한 제우스를 모든 이들이 가장 옳고 훌륭한 신으로 모시기 때문에 그의 일과 같은 일을 하는 것은 경건하다 말한다.
 소크라테스는 어떤 하나의 일이 경건하다는 것 말고 이러저러한 경건한 일들을 경건한 것이도록 만드는 경건함, 하나의 특징으로서의 경건함을 알려달라고 말한다.

제2 논의. 에우튀프론은 이에 대해 '신들에게 사랑받는 것은 경건하고 그렇지 않은 것은 불경하다'라고 말한다.
 그러나 소크라테스는 앞서 크로노스와 우라노스의 싸움, 제우스와 크로노스의 싸움뿐만 아니라 신화 속 많은 이야기들 속에서 신들이 서로 의견을 달리하고 싸운다는 점을 지적한다. 그리고 그들이 의견을 달리하고 싸우는 까닭은 수의 많고 적음이나 어떤 것의 크기나 무게와 달리 '옳고 그름, 미추, 좋고 나쁨'은 하나의 것을 두고도 서로 달리 말하면서 조율하지 못하기 때문이라 말한다.
 이를 근거로 소크라테스는 하나의 행위를 신들 중에 누군가는 사랑하고 또 다른 어떤 신은 미워할 수 있다고 주장한다. 구체적으로 에우튀프론이 부모에게 소송을 건 것은 제우스는 좋아할 일이지만 크로노스와 우라노스가 싫어하고, 헤파이스토스는 좋아하지만 헤라는 싫어할 일이기에 경건하면서도 경건하지 않기도 하다고 반박한다.
제2 논의 후퇴1. 소크라테스는 '신들에게 사랑받는 것이 경건'이라는 정의를 논박하였으나 에우튀프론은 다시금 '자신이 한 행위는 부정의한 살인을 고발한 것이고 이 행위는 다들 동의할 것이다'라고 구체적인 행위를 옹호한다.
 소크라테스는 다시금 '부정의한 살인'이라는 말에 주목한다. 만약에 그것이 애초부터 부정의한 것이라면 그것은 물론 고소되고 처벌받아야 한다. 그러나 그 살인은 에우튀프론에게는 부정의하지만 친족들에게는 부정의한 것은 아닌 것으로, 그리고 소크라테스에게는 그러한지 아닌지 알 수 없는 것으로 남아 있다.
 앞서의 결론으로 되돌아온다. 하나의 행위를 두고 이 신들은 옳다 하고 저 신들은 그르다 할 수 있기 때문에 '신들에게 사랑받는 것이 경건'이라는 말은 경건한 것이 동시에 불경건한 것이 되기도 하므로 적절한 정의일 수 없다.
제2 논의 후퇴2. 신들의 의견(혹은 개인 각자의 의견)으로는 어떤 한 행위가 경건한지 아닌지 판단할 수 없다는 결론을 가지고 소크라테스는 에우튀프론의 행위가 옳다는 '증거'를 요구한다. 여전히 이 증거는 앞서 소크라테스가 요구한 정의내림의 조건 속에서 요구되는 것이다.
 그러나 에우튀프론은 다시금 '그건 아마도 작은 일이 아닐 것이다'라고 말한다. 그러나 소크라테스의 요구대로라면 그것은 선명하고 작고 정확한 일이다. 결국 에우튀프론은 더 많은 사례들을 들어 보이려고 한다는 것을 추측할 수 있다.

제3 논의. 소크라테스는 에우튀프론이 '자신의 행위가 경건하다'는 점에 집착하는 것을 의식하고 논의를 전환시킨다. 모든 신이 '그 행위'를 사랑한다는 것은 '신들에게 사랑받는 것이 경건한 것이다'라는 정의가 옳은지 어쩐지에 대한 증명이 될 수 없기 때문이다. 그는 다시 논의를 정의의 문제로 끌어오기 위해 에우튀프론의 논의를 빌려 '(만일 당신의 행위가 모든 신들이 동의하는 것이기 때문에 경건한 것이라면) 모든 신들이 사랑(동의)하는 것은 경건하고 모든 신들이 미워(반대)하는 것은 불경하다'라는 정의로 수정하겠는지 묻는다.
 그리고 소크라테스는 다시 이 정의를 반박한다.
           (1)무엇으로 되거나 무엇을 겪는 것은 그 '무엇이 됨'이나 '겪음'을 통해 그러한 것이 된다.
           (2)신들이 사랑하는 것도 신들에 의해 사랑을 받음으로써 신들이 사랑하는 것이 된다.
 그러나 (3)경건한 것은 것이기 때문에 신들에게 사랑을 받는다.
    만일 (4)경건한 것과 신들이 사랑하는 것이 동일한 것이라면,
           (2-1)신들이 사랑하는 것은 신들에게 사랑을 받기 때문에 신들이 사랑하는 것이고,
또 한편 (3-1)신들이 사랑하는 것은 신들이 사랑하는 것이기 때문에 신들에게 사랑을 받는다.
그런데  (2-1)과 (3-1)이 서로 어긋나고 또한 (3-1)은 (2)에도 모순되므로 경건한 것과 신들이 사랑하는 것은 서로 전혀 다르다.

제4 논의-1. 소크라테스는 경건한 것이 이러저러한 것을 겪는다는 정의를 피하기 위해 다시 한 번 논의를 전환시킨다. 그는 경건함과 올바름을 견준다.
이 경우 (1) 모든 경건한 것은 올바르고 모든 올바른 것도 경건하다, 이거나
           (2) 모든 경건한 것은 올바르지만 모든 올바른 것이 경건하지는 않다, 이다.
           (경건>올바름, 경건≠올바름, 이 두 경우는 언급되지 않고 있다)
 에우튀프론은 (2)를 선택하고 덧붙여서 신을 섬기는 올바름이 경건이며 나머지는 인간을 보살피는 올바름이라고 말한다.
 소크라테스는 여기에서 다시 '섬김', '보살핌'이 무엇인지 묻는다. '섬김(보살핌)'은 그 대상의 이득을 목표로 한다. 그런데 인간이 신을 보살펴 신이 얻을 '이득'은 없다.
제4 논의-2/제 2논의로의 후퇴1. 이제 에우튀프론은 '섬김(보살핌)'을 '노예가 주인에게 행하는 종류의 섬김'으로 수정한다.
 소크라테스는 주인이 목표하는 바를 이루도록 도와주는 것이 노예가 주인에게 하는 봉사인지 되묻는다. 에우튀프론이 동의하자 그는 조선공, 의사 등의 예를 든다. 그리고 다시 그와 같은 방식으로 신들이 이루어내려는 것이 무엇인지 묻는다.
 에우튀프론은 '좋고도 많은 것'이라고 대답한다. 그리고 '조금 전에도 말씀드렸지만'이란 언급을 통해 이것이 제2논의 후퇴2에서와 같은 종류의 답변임을 알 수 있다. 더불어 신들의 '목적'이 되는 것을 '신들이 만족스러워 하는 것'으로 대치하여 스스로 이전의 논의로 확실하게 후퇴해 버린다.
제4 논의-3/제 2논의로의 후퇴2. 소크라테스는 논의가 후퇴하였음을 간파하고 한탄하지만 '신들한테 제물을 바치고 기원을 하는데 대한 일종의 앎'이라는 에우튀프론의 정의에 대해 더 고찰하고자 한다. 이는 주고 받음이며 나아가 상대가 바라는 것을 주고 원하는 것을 청하여 받는 거래이다. 그러나 제4논의-1에서 언급되었듯 신들이 인간들을 통해 얻을 이득은 없고 '만족'을 얻을 뿐이다. 그리고 '만족스러운 것'이 '사랑받는 것'과 다르냐는 소크라테스의 물음에 에우튀프론은 그 둘이 같다고 답한다. 이로써 논의는 아포리아에 빠진다.

-蟲-
  A digression You will need to learn new ways to use (the Principle of) Mathematical Induction. You are probably familiar with Mathematical Induction as a tool for verifying formulas. We will need to use Mathematical Induction for other purposes. For example, we will need to know whether certain kinds of functions from N to N exist. There may be one function for each n, or maybe our statement will be that there is NO such function, for ANY n. We will then construct a set S := {n ∈ N : a function fn : N → N exists such that “thus and so” is true about fn}.

  Then we will follow the “same” pattern as in school induction:

  Step 0: Showthat 0 is in S. The way we show that 0 is in S will depend on what the criterion is for membership in S. But the objective of Step 0 is always the same: make sure 0 ∈ S.

  Step 1: Let someone choose an arbitrary element n of N, then showthat the statement “n ∈ S ⇒ s(n) ∈ S” is true. This is the objective of Step 1, then: showthat (∀n ∈ N)(n ∈ S ⇒ s(n) ∈ S) is true. Since this is a statement with a universal quantifier, we have to be able to show that every one of the statements n ∈ S ⇒ s(n) ∈ S is true. We knowb y nowthat there is one easy way for this statement to be true: in case n ∈ S is false! We usually don’t mention this possibility; we just assume that n ∈ S is true. Then we try to use the truth of “n ∈ S” to deduce the truth of “s(n) ∈ S.” But we can’t always work that way! Sometimes we have to use the contrapositive of “n ∈ S ⇒ s(n) ∈ S, ” namely “s(n) not ∈ S ⇒ n not ∈ S.” That is, we assume that “s(n) not ∈ S” is true. Then we try to use the truth of “s(n) not ∈ S” to deduce the truth of “n not ∈ S.” This second approach is logically equivalent to the first way, but it is not psychologically the same! Step 1 is the “heart” of an induction proof. But Step 0 is essential!

  Step 3: We apply the Principle of Mathematical Induction by noting that S = N. This step is one we do automatically; it’s like a coda in a piece of music. . . end of this digression

  (04) Example: Let’s prove that s(N) = P. We need to recall that P = N\{0}. We want to show that every n in P is s(m) for some m in N. By Postulate (01C) we know that (∀n ∈ N)(s(n) ≠ 0) is true. Thus s(N) ⊆ N\{0} = P. To showequalit y, it remains to showthat P ⊆ s(N). We do already knowof one element of P that is in s(N), namely 1 = s(0). Thus 1 ∈ {n ∈ P : there exists m ∈ N such that n = s(m)} =: E. We will use induction for P instead of N. We already did Step 0 (or ought it be Step 1, for P?)! We need to showthat, for all n in P, “n ∈ E ⇒ s(n) ∈ E” is true. Well, “n ∈ E ⇒ s(n) ∈ E” is true if “n ∈ E” is false. Since “n ∈ E” has to be true or false, we have to see what happens if “n ∈ E” is true. So, we next suppose that “n ∈ E” is true. Now, “n ∈ E” means that there exists m ∈ N such that n = s(m). But then, s(n) = s(s(m)), by the Substitution Rule. The criterion for membership of s(n) in E is that there exists m ∈ N such that s(n) = s(m). This is confusing at first, because we just saw that n = s(m) because n ∈ E. The important thing to learn here is that variables in quantifiers are “dummy” variables; we can use any convenient letter to denote them! In the present context, we just use “s(m)” as our convenient “letter!” Thus, s(n) ∈ E. Then, by Mathematical Induction (set version, for P), E = P. In other words, every n in P is the successor of some m in N, as desired.

  If you have time, I recommend that you read (listed as [3] in the References at the end of this Note) Foundations of Analysis, by Edmund Landau, Chelsea Publishing Co., NewY ork, 1951. Despite the author’s request, do read the preface for the teacher. Read the preface for the student too, and the book too, a little bit at a time. What Landau does is to carefully develop the properties of the positive integers from the axioms (the version for P). Here are some exercises, adapted from theorems in that book. In these exercises, do not use your prior knowledge, just use the axioms.

(05) Exercise: Showthat, for all n in N, and for all m in N, if n ≠ m then s(n) ≠ s(m).
(06) Problem: Showthat, for all n in N, s(n) ≠ n. You will need induction!
(07) Exercise: Showthat, for all n in N, if n ≠ 0 then there exists exactly one m in N such that n = s(m). You may need induction!
(08) Notation: If n ∈ N and n ≠ 0 we let n−1 denote the unique m in N (provided by Exercise (07)) such that n = s(m). That is, n = s(n − 1). An important point: n − 1 does not exist for all n in N; n − 1 exists only for n 0. This is not subtraction! The meaning of n − 1 is “predecessor of n.”

-작성중-
  The Principle of Mathematical Induction is our principal source of “mathematical energy!”
  수학적 귀납법의 원리는 "수학적 능력"에 있어서 우리의 원칙적 원천이다!
  We usually don’t make axioms in a vacuum - there is usually some thought behind them. But that thought can be hidden! So here is an “explanation” of the axioms. First, let’s imagine that we go into the “back room,” where we don’t have to prove anything - just work on ideas! We “know” the integers exist, positive, zero and negative.
   우리는 보통 진공에서 공리들을 만들지 않는다 - 일반적으로 그것들 이면에 어떤 생각이 있다. 하지만 그 생각은 감추어질 수 있다! 그래서 공리들에 대한 '설명'이 있다. 첫째로, 위가 "무대 뒤"로 들어간다고 상상해 보자, 거기에서 우리는 어떤 것도 증명해야 하지 않다 - 단지 계속 생각하기만 하자! 우리는 정수의 존재, 양수, 0 그리고 음수를 "안다."
  The natural numbers are going to be our model of the non-negative integers. The successor function is “really” the function whose value at n, n ≥ 0, is n + 1 : s(n) = n + 1. Different non-negative integers “should” have different successors. This is what “s” is one–one” means. This is an assumption about the successor function! Saying 0 is not the successor of any non-negative integer means that 0
 n + 1 for any non-negative integer n. The postulate about sets of non-negative integers is not something we usually take for granted. But the idea makes sense: if a set S contains 0, and contains the successor of each of its elements, then it also contains 1, and 2, and 3, and so on. So it ought to contain every non–negative integer, namely S = N.
 자연수는 우리의 비-음수인 정수의 원형이 될 것이다. 후속자 함수는 "실제로" n이 0보다 크거나 같을 때 n 값이
n + 1 : s(n) = n + 1인 함수이다. 상이한 비-음수 정수는 상이한 후속자들을 가져"야 한다." 이것은 "s"가 "일대일 대응"임을 의미한다. 이것은 후속자 함수에 대한 가정이다! 0이 어떠한 비-음수 정수의 후속자도 아니라고 말하는 것은 어떠한 비-음수 정수 n에 대해서도
0 ≠ n + 1임을 의미한다
. 비-음수 정수의 집합들에 대한 그 공준은 우리가 일반적으로 당연시하는 것이 아니다. 그러나 그 생각은 말이 된다: 만일 한 집합 S가 0을 포함하고, 그 집합의 원소들 각각의 후속자를 포함한다면, 그것은 또한 1, 2, 3 등을 포함할 것이다. 그래서 그 집합은 모든 비-음수 정수 각각을 포함해야 한다, 즉 S=N이다.

  In your experience, perhaps N starts with 1, not 0. There is an equivalent version of these axioms for that version of N. We simply replace each “0” in the first version with “1, ” and call the newset P, for positive integers. We will want to use induction “starting with 1.” But, instead of making another set of axioms, we can give the name 1 to s(0), and then we can deduce the proposed axioms, from the Peano Postulates, so they become a Theorem: a set of true mathematical statements whose truth is logically deduced from axioms, instead of being assumed.
 당신의 경험에서, 아마도 N은 0이 아니라 1로 시작할 것이다. N의 그 판에 대한 이러한 공리들에 상응하는 판이 있다. 우리는 간단하게 첫번째 판에서 각각의 "0"을 "1"로 대체하고, 새집합 P라고 부른다, 양의 정수이기 때문이다. 우리는 "1로 시작하는" 귀납법을 사용하기를 바랄 것이다. 그러나, 공리들의 또 다른 집합을 만드는 대신, 우리는 s(0)에 1이란 이름을 줄 수 있고, 그 다음으로 우리는 제시된 공리들을 연역할 수 있다, 페아노의 공준들로부터, 그래서 그것들은 정리가 된다: 참인 수학적 진술들의 한 집합, 그 진술의 참이 논리적으로 공리들로부터 연역된, 가정되는 대신에.
  (03) Theorem and Definition and Notation: There is a non-empty set P, whose elements we will call positive integers, a one-to-one function s : P → P, that we will call the successor function, and an element 1 ∈ P such that 1 is not in the image, s(P), of s. In addition, for all subsets E of P, if E contains 1, and s(E) ⊆ E, then E = P.
  (03) 정리와 정의 그리고 기수법: 비-공집합 P가 있다. P의 원소들을 우리는 양의 정수들이라고 부를 것이다. 그리고 일대일 대응 함수 s : P → P가 있다, 우리가 후속자 함수라고 부를. 그리고 1이 s(P)라는 s의 관념 속에 있는 1이 아닌 한 원소
1 ∈ P이 있다. 덧붙여서, P의 모든 부분집합들에 대해서, 만일 E가 1을 포함하고 s(E)
 ⊆ E 라면, E = P이다.
  The idea of the proof is to remove 0 from N to construct
P: P := N
 {0}
. Then we have to check that each of the postulates can be re–interpreted in the newset. The Induction property is deduced by temporarily putting 0 back, then taking it out after applying the Principle of Mathematical Induction for N. We will not carry out the details of this proof unless you ask.
  그 증명에 대한 생각은
P: P := N
 {0}를 구성하기 위해 N으로부터 0을 제거하는 것이다. 그래서 우리는 새집합에서 재-해석될 수 있는 그 공준들의 각각을 점검해야 한다. 그 귀납법 특성은 주기적으로 0을 되돌려 놓고, 다음으로 그 0을 N에 대한 수학적 귀납의 원리를 적용하는 것 뒤로 끄집어 냄으로써 연역된다.
  You have probably seen Mathematical Induction before, but it looked a little different then.
  당신은 아마도 이전에 수학적 귀납법을 봤을 테지만, 그것을 조금 다르게 봤을 것이다.
  Example (unofficial!): Use Mathematical Induction (school version) to show that for all positive integers n,
1 + 2 + 3 + · · · + n = n(n + 1)/2. In this Example, we will use your prior knowledge about P.
  예시 (비공식!): 수학적 귀납법을 사용하라 모든 양의 정수 n에 대해서
1 + 2 + 3 + · · · + n = n(n + 1)/2 임을 보이기 위해. 이 예시에서, 우리는 P에 대한 여러분의 사전지식을 사용할 것이다.
  The procedure is this:
1. Let P(n) denote the statement to be proved: P(n) must have n as its only free variable.
2. Verify that P(1) is true.
3. Show that, by assuming P(n) is true, you can deduce that P(n + 1) must be true.
  절차는 이러하다:
1. P(n)이 다음과 같이 증명될 진술을 표시하도록 하라: P(n)은 반드시 그것의 자유 변수로서만 n을 가져야 한다.
2. P(1)이 참임을 입증하라.
3. P(n)이 참임을 가정함으로써, 당신이 P(n+1)이 반드시 참이어야만 함을 연역할 수 있다는 것을 보여라.
  That is, prove the quantified mathematical statement “For all n in N, P(n) ⇒ P(n + 1).”
  Then Mathematical Induction assures us that P(n) is true for all n.
  즉, 양화된 수학적 진술 "N에서 모든 n에 대해 P(n)
 ⇒ P(n + 1)"을 증명하라.
  Let’s do it, for review.
  검코를 위해 해 보자.
  Step 1. Let P(n) denote the statement “1 + 2 + 3 + · · · + n = n(n + 1)/2.”
 
Step 2. P(1) is the statement
“1 = 1(1 + 1)/2.” This is true, by a very quick computation.
 
Step 3. Assume that P(n) is true for some n.
Then P(n + 1) is the statement “1 + 2 + 3 + · · · + (n + 1) =
(n+1)(n+2)/2.” We have 1+2+3+· · ·+(n+1) = 1+2+3+· · ·+n+(n+1) = (1+2+3+· · ·+n)+(n+1) = n(n+1)/2 +2(n+1/2) = (n+1)(n+2)/2, by a few basic algebraic manipulations. The equality of the first and last expressions in
this chain of equalities is P(n + 1), shown to be true under the assumption that P(n) is true. The truth of P(n) for all n is now established, by Mathematical Induction.
   
1단계. P(n)이 "
1 + 2 + 3 + · · · + n = n(n + 1)/2"라는 진술을 표시하도록 하라.
   2단계. P(1)은 "
1 = 1(1 + 1)/2"라는 진술이다. 이것은 참이다, 매우 즉각적인 계산에 의해.
   3단계. P(n)이 어떤 n에 대해 참이라고 전제하라. 그래서 P(n + 1) 은 "
1 + 2 + 3 + · · · + (n + 1) =
(n+1)(n+2)/2"이라는 진술이다. 그래서 우리는 약간의 기초적인 대수적 조작들을 통해
1+2+3+· · ·+(n+1) = 1+2+3+· · ·+n+(n+1) = (1+2+3+· · ·+n)+(n+1) = n(n+1)/2 +2(n+1/2) = (n+1)(n+2)/2 를 가진다, P(n)이 참이라는 저제 하에 참이라 보여지는. 모든 n에 대한 P(n)의 참은 이제 확정된다, 수학적 귀납법에 의해서.
 
  Remark: Note that, were P(n) false, the statement P(n) ⇒ P(n + 1) would be true vacuously. Thus we can
eliminate that possibility, and concentrate on what happens if P(n) is true.
   주의: 다음을 주의하라. P(n)이 거짓이라면,
P(n) ⇒ P(n + 1)이라는 진술은 허황된 참일 것이다. 따라서 우리는 그 가능성을 고려하지 않을 수 있고, P(n)이 참이라면 어떤 일이 일어나는지에 집중할 수 있다.
  The way we will often “do” Mathematical Induction differs only a little from this (and you may certainly use the old way in your papers in this class). The set–theoretic version of the argument for this example:
Let E := {n ∈ P : 1 + 2 + 3 + · · · + n = n(n+1)2}. We show 1 ∈ E (just as we just did), and that n ∈ E implies n + 1 ∈ E (just as we just did). Then E = P by Mathematical Induction.
  우리가 자주 '할' 수학적 귀납법의 방식은 이것(그리고 당신이 확실히 이 수업에서 당신의 paper들에서 사용했을지 모르는 그 오래된 방식)과 약간만 다르다. 이 예시에 대한 논증의 집합론적 판:
E := {n ∈ P : 1 + 2 + 3 + · · · + n = n(n+1)2}라고 하자. 우리는
1 ∈ E 임을 보이고(바로 우리가 방금 했던 것처럼),
n ∈ E 이
n + 1 ∈ E을 시사함을 보인다(바로 우리가 방금 했듯이). 그래서 수학적 귀납법에 의해 E = P이다.

-蟲-
출처 : www.math.umn.edu/~jodeit/course/Peano3.pdf

  THE NATURAL NUMBERS
  자연수
 
  We are ready for the official introduction of the natural numbers. I ask you to suspend belief in the truth of the things you already know about the natural numbers. I insist you keep every bit of your knowledge - just stop taking things for granted. “The Natural Numbers” is now a name only, to be used in the following list of mathematical statements, each of which you are asked to regard as true.
  우리는 자연수에 대한 공식적인 입문(개론)의 준비가 되어 있다. 나는 당신에게 당신이 자연수에 대해 이미 알고 있는 것들의 진실성에 대한 믿음을 보류할 것을 요청한다. 나는 당신이 당신 지식의 모든 낱낱을 붙들어 둘 것을 강력히 주장한다 - 그것들을 당연시 하는 것은 확실히 중단하자. "자연수"는 이제 하나의 명칭일 뿐이다. 그 명칭은 아래의 수학적 진술들의 목록에서 사용될 것이며, 당신에게는 그 진술들의 각각을 참으로 간주할 필요가 있다.
  This is where the course really starts. What came before was “preliminary.” We still have some preliminary stuff to do. But the natural numbers will literally be the foundation on which are built the ordinary integers, the rational numbers, and then the real and the complex numbers, which all form the foundations for Calculus.
  그 교육 과정은 실제로 여기서 시작한다. 앞서 나왔던 것은 "예비적인 것"이었다. 우리는 아직 해야할 몇몇 예비적인 것들을 가지고 있다. 그러나 자연수는 문자 그대로 기초가 될 것이다. 그 기초 위에 정수, 유리수, 그리고 실수와 복소수가 세워진다. 그 모든 수들은 계산을 위한 기초를 이루는 것들이다.

(01) The Peano Postulates (or Axioms)
(01) 페아노 공준 (혹은 공리)
  We assume that the four following mathematical statements are true.
  우리는 다음 네 가지 수학적 진술들이 참이라고 가정한다.
(01A) There exists a non-empty set N, whose elements we will call natural numbers.
(01A) 공집합이 아닌 하나의 집합 N이 있다. 우리는 그 집합의 원소들을 자연수라 부를 것이다.
(01B) There exists a one-to-one function s : N → N, that we will call the successor function.
(01B) 하나의 일대일 대응 함수 s : N → N이 있다. 우리는 그 함수를 후속자 함수라고 부를 것이다.
(01C) There exists an element 0 ∈ N such that 0 is not in the image, s(N), of s.
(01C) N에 포함되는(0 ∈ N) 하나의 원소 0이 있다. 그 같은 0은 s집합의 상인 s(N) 내에 없다.
(01D) For all subsets S of N, if S contains 0, and s(S) ⊆ S, then S = N.
(01D) N의 모든 부분집합 S에 대해서, 만일 S가 0을 포함하고, s(S)가 S에 동치이거나 포함된다면(s(S) ⊆ S), S는 N이다.
  Here is a restatement of these axioms, in a less dense form:
  여기 이 공리들에 대한 조금은 덜 난해한 형식의 재진술이 있다:
  (02-1) There exists a non-empty set N, called the set of natural numbers, and a function s on N, called the successor function, whose value, s(n), at any n in N, is called the successor of n, such that (02-2) Different elements of N have different successors (s is one-to-one), (02-3) There is an element, 0, of N that is not the successor of any element of N (0 is not in the image of s), (02-4) Every subset S of N that contains 0 and also contains the successor of each element of S must be equal to N (if S contains 0, and s(S) ⊆ S, then S = N).
  (02-1) 공집합이 아닌 집합 N이 있다. N은 자연수의 집합으로 부른다. 그리고 N에 대해 함수 s가 있다. 함수 s는 후속자 함수라 부른다. 함수 s의 값 s(n)은, N 내에서 어떠한 n에서든, n의 후속자라고 부른다. (02-2) N의 그러한 서로 다른 원소들은 서로 다른 후속자들을 가진다 (s는 일대일 대응이다). (02-3) N의 원소 0이 있다. 0은 N의 어떠한 원소에 대해서도 후속자가 아니다 (0은 s의 상 내에 없다). (02-4) 0을 포함하고 또한 S의 각 원소들에 대한 후속자를 포함하는 모든 부분집합 S는 반드시 N에 동치여야만 한다 (만일 S가 0을 포함하고, s(S) ⊆ S라면, S = N).

  According to the Encyclopedia Britannica, 15th edition, the five Peano postulates are:

  브리트니 백과 15번째 편집본에 따르면, 다섯 가지 페아노 공준들이 있다:
1. 0 is a number.
1. 0은 수이다.
2. The successor of any number is also a number.
2. 어떤 수의 후속자든 또한 수이다.
3. No two distinct numbers have the same successor.
3. 그 어떤 별개의 두 수도 동일한 후속자를 가지지 않는다.
4. 0 is not the successor of any number.
4. 0은 그 어떤 수의 후속자도 아니다.
5. If any property is possessed by 0 and also by the successor of any number having that property, then all numbers have that property.
5. 만일 어떤 특성이든 0에 의해 그리고 또한 그 특성을 지닌 어떤 수의 후속자에 의해 소유된다면, 모든 수는 그 특성을 지닌다.
  Postulate 5, (01D) and (02-4) are certainly different. But all three versions amount to the same thing mathematically, when expressed in terms of set–selector notation. For, “property” has to refer to a mathematical statement P(n) about elements n of N. Thus, {n ∈ N : P(n) is true } is the set of all elements of N that possess the property P(n). If the “property” satisfies the conditions of postulate 5, then 0 ∈ {n ∈ N : P(n) is true }, and s({n ∈ N : P(n) is true }) ⊆ {n ∈ N : P(n) is true }, so {n ∈ N : P(n) is true } = N. On the other hand, let S be a subset of N. We define the “property (of n)” to be that n belongs to S. That is, P(n) is the statement “n ∈ S.” This “property” is then covered by postulate 5.
  공준 5, (01D)와 (02-4)는 확실하게 다르다. 그러나 세 가지 설명들 모두 수학적으로 동일한 것에 이르게 된다, 집합-선택자 기수법이란 용어로 표현될 때. 왜냐하면, "특성"은 N의 원소들 n에 대한
수학적 진술 P(n)를 나타내야 하기 때문이다. 만일 그 "특성"이 공준 5의 조건들을 만족시킨다면,
0 ∈ {n ∈ N : P(n) 가 참 }, 그리고 s({n ∈ N : P(n) 가 참 }) ⊆ {n ∈ N : P(n) 가 참 }, 그래서 {n ∈ N : P(n) 가 참 } = N 이다. 다른 한편, S를 N의 한 부분집합으로 두자. 우리는 그 "(n의) 특성"을 n이 S에 속한다는 것으로 정의한다. 즉, P(n)은 "
n ∈ S"라는 진술이다. 이 "특성"은 그래서 공준 5에 의해 포섭된다.
  Postulate 5 is about sets of natural numbers. You probably did not bring this postulate with you to this course, at least not explicitly. And yet, it will seem natural to you shortly, I hope. Here are the three versions of Postulate 5, collected in one place:
  공준 5는 자연수 집합들에 대한 것이다. 당신은 아마도 이 공준을 갖고 이 과정을 수행하진 않았을 것이다, 최소한 명백하게는 아닐 것이다. 그리고 아직, 그것이 당신에게 곧 자연스러워 보일 것이다, 내가 희망하기로는. 여기 공준 5의 세 판이 있다, 한 자리에 모은:
  For all subsets P of N, if P contains the element 0, and s(P) ⊆ P, then P = N; Every subset of N that contains 0 and also contains the successor of each of its elements must be equal to N; If any property is possessed by 0 and also by the successor of any number having that property, then all numbers
have that property.
  N의 모든 부분집합 P에 대해서, 만일 P가 원소 0을 포함한다면, 그리고
s(P) ⊆ P라면, P = N이다; 0을 포함하고또한 그 원소들의 각각에 대한 후속자를 포함하는 N의 모든 각각의 부분집합은 N과 동치여야만 한다; 만일 어떤 특성이든 0에 의해 그리고 또한 그 특성을 지니는 어떤 수든지 그에 대한 후속자에 의해 소유된다면, 모든 수는 그 특성을 지닌다.
  This Postulate is called The Principle of Mathematical Induction.
  이 공준은 수학적 귀납법의 원리라 부른다.

-蟲-

+ Recent posts